Abhay Kumar Singh I E Irodov Solutions To I.E. Irodov - 'S Problems in General Physics. Volume 1 1998

You might also like

Download as docx, pdf, or txt
Download as docx, pdf, or txt
You are on page 1of 493

Solutions to

LE. Irodev's
Problems in
General Physics
Volume I
Mechanics • Heat • Electrodynamics

SECOND EDITION

ABHAY KUMAR SINGH


Director
Abhay's 1.1. T. Physics Teaching Centre
Patna-6

CBS

CBS PUBLISHERS & DISTRIBUTORS


4596/1A, 11 DARYAGANJ, NEW DELHI -110 002 (INDIA)
Dedicated to
my Teacher
Prof. (Dr.) J. Thakur
(Department of Physics,
Patna University,
Patna-4)

ISBN : 81-239-0399-5

First Edition : 1995


Reprint : 1997
Second Edition : 1998
Reprint : 2000
Reprint : 2001
Reprint : 2002
Reprint : 2003
Reprint : 2004
Reprint : 2005

Copyright © Author & Publisher

All rights reserved. No part of this book may be reproduced or


transmitted in any form or by any means, electronic or mechanical,
including photocopying, recording, or any information storage and
retrieval system without permission, in writing, from the publisher.

Published by S.K. Jain for CBS Publishers & Distributors,


4596/1A, 11 Darya Ganj, New Delhi - 110 002 (India)

Printed at
India Binding House, Delhi - 110 032
FOREWORD

Science, in general, and physics, in particular, have evolved out of man's quest to know beyond
unknowns. Matter, radiation and their mutual interactions are basically studied in physics.
Essentially, this is an experimental science. By observing appropriate phenomena in nature one
arrives at a set of rules which goes to establish some basic fundamental concepts. Entire physics
rests on them. Mere knowledge of them is however not enough. Ability to apply them to real
day-to-day problems is required. Prof. Irodov's book contains one such set of numerical
exercises spread over a wide spectrum of physical disciplines. Some of the problems of the book
long appeared to be notorious to pose serious challenges to students as well as to their teachers. This
book by Prof. Singh on the solutions of problems of Irodov's book, at the outset, seems to remove
the sense of awe which at one time prevailed. Traditionally a difficult exercise to solve continues
to draw the attention of concerned persons over a sufficiently long time. Once a logical solution
for it becomes available, the difficulties associated with its solutions are forgotten very soon.
This statement is not only valid for the solutions of simple physical problems but also to various
physical phenomena.
Nevertheless, Prof. Singh's attempt to write a book of this magnitude deserves an all out
praise. His ways of solving problems are elegant, straight forward, simple and direct. By writing
this book he has definitely contributed to the cause of physics education. A word of advice to its
users is however necessary. The solution to a particular problem as given in this book is never to
be consulted unless an all out effort in solving it independently has been already made. Only by
such judicious uses of this book one would be able to reap better benefits out of it.
As a teacher who has taught physics and who has been in touch with physics curricula at
I.I.T., Delhi for over thirty years, I earnestly feel that this book will certainly be of benefit to
younger students in their formative years.

Dr. Dilip Kumar Roy


Professor of Physics
Indian Institute of Technology, Delhi
New Delhi-110016.
FOREWORD

A. proper understanding of the physical laws and principles that govern nature require
solutions of related problems which exemplify the principle in question and leads to a
better grasp of the principles involved. It is only through experiments or through solutions of
multifarious problem-oriented questions can a student master the intricacies and fall outs of
a physical law. According to Ira M. Freeman, professor of physics of the state university
of new Jersy at Rutgers and author of "physic—principles and Insights" --
"In certain situations mathematical formulation actually promotes intuitive understand-
ing Sometimes a mathematical formulation is not feasible, so that ordinary language must
take the place of mathematics in both roles. However, Mathematics is far more rigorous
and its concepts more precise than those of language. Any science that is able to make
extensive use of mathematical symbolism and procedures is justly called an exact science".
I.E. Irodov's problems in General Physics fulfills such a need. This book originally
published in Russia contains about 1900 problems on mechanics, thermody-
namics, molecular physics, electrodynamics, waves and oscillations, optics, atomic and
nuclear physics. The book Jas survived the test of class room for many years as is evident
from its number of reprint editions, which have appeared since the first English edition of
1981, including an Indian Edition at affordable price for Indian students.
Abhay Kumar Singh's present book containing solutions to Dr. I.E. Irodov's Problems in
General Physics is a welcome attempt to develop a student's problem solving skills. The
book should be very useful for the students studying a general course in physics and also in
developing their skills to answer questions normally encountered in national level entrance
examinations conducted each year by various bodies for admissions to profes-
sional colleges in science and technology.

B.P. PAL
Professor of Physics
I.I.T., Delhi
PREFACE TO THE SECOND EDITION

Nothing succeeds like success, they say. Now, consequent upon the warm
welcome on the part of students and the teaching fraternity this revised and
enlarged edition of this volume is before you. In order to make it more up-to-date and
viable, a large number of problems have been streamlined with special focus on the
complicated and ticklish ones, to cater to the needs of the aspiring students.
I extend my deep sense of gratitude to all those who have directly or
indirectly engineered the cause of its existing status in the book world.

Patna
June 1997 Abhay Kumar Singh
PREFACE TO THE FIRST EDITION

When you invisage to write a book of solutions to problems, one pertinent question crops up in
the mind that—why solution! Is this to prove one's erudition? My only defence against this is
that the solution is a challenge to save the scientific man hours by channelizing thoughts in a right
direction.

The book entitled "Problems in General Physics" authored by I.E. Irodov (a noted
Russian physicist and mathematician) contains 1877 intriguing problems divided into six
chapters.

After the acceptance of my first book "Problems in Physics", published by Wiley


Eastern Limited, I have got the courage to acknowledge the fact that good and honest
ultimately win in the market place. This stimulation provided me insight to come up with my
second attempt—"Solutions to I.E. Irodov's Problems in General Physics."
This first volume encompasses solutions of first three chapters containing 1052
problems. Although a large number of problems can be solved by different methods, I have
adopted standard methods and in many of the problems with helping hints for other methods.

In the solutions of chapter three, the emf of a cell is represented by (xi) in contrast
to the notation used in figures and in the problem book, due to some printing difficulty.

I am thankful to my students Mr. Omprakash, Miss Neera and Miss Punam for their
valuable co-operation even in my hard days while authoring the present book. I am also
thankful to my younger sister Prof. Ranju Singh, my younger brother Mr. Ratan Kumar Singh,
my junior friend Miss Anupama Bharti, other well wishers and friends for their emotional
support. At last and above all I am grateful to my Ma and Pappaji for their blessings and
encouragement.

ABHAY KUMAR SINGH


CONTENTS
Foreword iii
Preface to the second edition
Preface to the first edition vi

PART ONE
PHYSICAL FUNDAMENTALS OF MECHANICS

1.1 Kinematics 1-34


1.2 The Fundamental Equation of Dynamics 35-65
1.3 Laws of Conservation of Energy, Momemtum, and Angular Momentum 66-101
1.4 Universal Gravitation 102-117
1.5 Dynamics of a Solid Body 118-143
1.6 Elastic Deformations of a Solid Body 144-155
1.7 Hydrodynamics 156-167
1.8 Relativistic Mechanics 168-183

PART TWO
THERMODYNAMICS AND MOLECULAR PHYSICS

2.1 Equation of the Gas State. Processes 184-195


2.2 The first Law of Thermodynamics. Heat Capacity 196-212
2.3 Kinetic theory of Gases. Boltzmann's Law and Maxwell's Distribution 213-226
2.4 The Second Law of Thermodynamics. Entropy 227-241
2.5 Liquids. Capillary Effects 242-247
2.6 Phase Transformations 248-256
2.7 Transport Phenomena 257-266

PART THREE
ELECTRODYNAMICS

3.1 Constant Electric Field in Vacuum 267-288


3.2 Conductors and Dielectrics in an Electric Field 289-305
3.3 Electric Capacitance. Energy of an Electric Field 306-324
3.4 Electric Current 325-353
3.5 Constant Magnetic Field. Magnetics 354-379
3.6 Electromagnetic Induction. Maxwell's Equations 380-407
3.7 Motion of Charged Particles in Electric and Magnetic Fields 408-424
PART ONE

PHYSICAL FUNDAMENTALS OF MECHANICS

1.1 KINEMATICS
1.1 Let v
be the stream velocity and v' the velocity of motorboat with respect to water. The
o
motorboat reached point B while going downstream with velocity (vo + v') and then returned
with velocity (v' — vo) and passed the raft at point C. Let t be the time for the raft (which flows
with stream with velocity vo) to move from point A to C, during which the motorboat moves from
A to B and then from B to C.
Therefore
j (vo+ v')-v — I
0) •.s--------(Vat 171)1 - - - -
v 0 v
A
On solving we get v_ 1
2

1.2 Let s be the total distance traversed by the point and t1 the time taken to cover half the
distance. Further let 2t be the time to cover the rest half of the distance.

Therefore —= vo t1 or t = (1)
2 1 2 vo

s
and — (vi + v2) t or a (2)
2 + V2

Hence the sought average velocity


+ )
s s 2 v (v1 v2
<V> mg
2t [s/2 vo ]+ [ s/(vi + v2).] v1 + v2 + 2 v
o
13 As the car starts from rest and finally comes to a stop, and the rate of acceleration and
deceleration are equal, the distances as well as the times taken are same in these phases
of motion.

Let At be the time for which the car moves uniformly. Then the acceleration / deceleration
i .ti—At
t me s each. So,
2
1 — At )1 At )
<v> it = 21 w + At
4 j 2

22 2 4 <v>
or A az Es
w

Hence At = V 1 — 4 <v> = 15s.


W

1.4 (a) Sought average velocity


SRI 1111111111111■
s 200 cm 20 1111111111111111111111SIMIR
<v> t 20s = 10 cm/s 1111111111111111M1101111
ds
1111111111111164 11111110
(b) For the maximum velocity, — should be
dt 111111111115M111111111111111
1111111115111 11111111111111
maximum. From the figure dis maximum for ■11111111■
all points on the line -ac, thus the sought 111111MICUIP
maximum velocity becomes average velocity 11121111MIM EMU
for the line ac and is equal to :
gite:211111111111111111111111■
be= cm
ab 4s 25 cm/s

ds
(c) Time to should be such that corresponding to it the slope should pass through the

ds s
point 0 (origin), to satisfy the relationship — = To-. From figure the tangent at point d
dt
passes through the origin and thus corresponding time t = to = 16 s.
1.5 ,Let the particles collide at the point A (Fig.), whose position vector is 73*(say). If t be the
time taken by each particle to reach at point A, from triangle law of vector addition :
2t
3= 1 +u t— 2+
SO, r1— r2 (v2 — v 1) t (1)

(2)

0
--a. —0
ri r2 vz— v1
or, = , which is the sought relationship.
I r2I I v2 — v 11

1.6 We -have
V = V—V0
(1)

From the vector diagram [of Eq. (1)] and using properties of triangle
3

v' — V v(2) + v2 + 2 vo v cos cc, = 39.7 km/hr (2)

v' v v sin cp
and = or, sin 0 —
sin (it — cp) sm 0 vi
0 . sin_i (v sin q))
or
v'
Using (2) and putting the values of v and d
0 = 19.1° i
1.7 Let one of the swimmer (say 1) cross the river along AB, which is obviously the shortest
path. Time taken to cross the river by the swimmer 1.
d
t1= . 7.7..v.2.._ , (where AB = d is the width of the river) (1)
o

For the other swimmer (say 2), which follows the quickest -path, the time taken to
cross the river.

d
t2 = —,- (2)
A v
B.i „ r . _ . . . _ •. . . . x ______.).0

1
In the time t2, drifting of the swimmer 2, becomes
1 VI
A

vo
x = vo t2 = --i d, (using Eq. 2) (3)
v
If t3 be the time for swimmer 2 to walk the distance x to come from C to B (Fig.), then
X v0 d
1..1 at — -•-
° U V? U (using Eq. 3) (4)
According to the problem ti. = t2 + t3

d d vo d
or, 1 +
V7 . -
V; v Vu
v — 2
On solving we get
vo
u _1
= 3 km/hr.
i
12 - 1
V
4

1.8 Let 1 b e t he d i s t a nc e c ov e r e d b y th e boat A a l o ng the riv e r as w ell as b y th e boa t B ac n

the r i v et Let vo b e the strea m vel oc i t y a nd v ' the vel oc i t y o f e a c h b o a t w ith res pe c t

w a t e r . Th e r e f or e t i me t a k en b y the b oat A in its j ourne y

1 1
tA

VI 4- Vo - V o

1 1 2/
a nd for th e bo a t B t
B
+

V - V2 V VI2 - v o2 V 112
0

to

Henc e , wh e r e i )

tB VI2 - V t) V T12 - 1

On sub s t i t u t i o n tA / t B F8

1 . 9 Le t v o be th e s t r e a m ve l o c i t y a nd v ' th e ve l oc i t y o f bo a t wi t h r e s pe c t t o wa t e r . A

v o

-7 = 2 > 0, s ome d r i f t i n g of boa t is i nev i t a bl e .

Let i 7v m a ke a n a ngl e 0 with l f o w d i r e c t i on (Fig . ) , t hen th e t i me ta ken to c r os s th e ri ve

t (wh e r e d i s t h e w i d th of th e ri v e r )

V s i n 0

In th i s tim e i nt e r v a l , h t e dr i f t i n g o f t he bo a t

x= (v' cos 0 + vo) t

(v' cos 0 + v„) = ( c ot 0 + cos e c 0 ) d


v' sin 0

Fo r x• • (mi ni mu m d r i f t i ng )

d d 0
ot

(c
0 + co s e c 0 ) 0,

1
whi c h

1
y i e l d s

cos 0 -

- 2

0
11

He nc e , 0 = 1 20°

1 . 1 0 Th e so l ut i on o f t hi s probl e m be c o me s si mpl e in the fra me a t t a c h ed with o ne of the bod i e s .

e bo d y t hrown s t r a i ght up be 1 a nd t he o t h e r bo d y be 2, th e n for the b od y 1 in th e


Let th

f r a me of 2 f r o m th e kinema t i c e qu a t i on for c on s t a nt a c c e l e r a t i on :

...► -10 1 -1' 42


r12- ro (12) + VO (12) -2 W12

So , v00 2 ) t , (be c a us e i i c2 = 0 and r 2) = 0)


T12 'a

Of ' ra j = 1 4( 1 2 ) 1 t ( 1 )

But 1 1 7 :1 1 . . 1 v+0 2 1 =
'1 o
So , from proper t i e s o f t ria ngle

2 2
VO(12) Vo Vo — 2 vo vo c os ( 7 t /2 - 0 0 )

H e nc e , the so u gh t dista nc e

1 7 : 2 1 = v0 1 / 2 (1 - sin 0) t= 2 2 m
5

1 . 1 1 Let the velocities of the paricles (say ii17' and v2 ) becomes mutually perpendicular after
time t. Then their velocitis become
' '
gt .
V1 = Vi+gt; V2 it V2
(1) N

V;
{171 72
Hence, t (3)
g
1 -0 2
Now form the Eq. r =r +v t+
12 0(12) 0(12) 2 W I12

= 0 and 02) = 0)
;Tz — a12)j2) I t, (because W12
here
Hence the sought distance
v1 v2 (as I v—0*02) = v1 + v2)
+ v2
r12 I.—
g
1.12 From the symmetry of the problem all the three points are always located at the vertices
of equilateral triangles of varying side length and finally meet at the centriod of the initial
equilateral triangle whose side length is a, in the sought time interval (say t ).

-->
15

Let us consider an arbitrary equilateral triangle of


edge length 1 (say).
Then the rate by which 1 approaches 2, 2 approches 3, and 3 approches 1, becomes :
— dl 2n
V - V COS (
--)
dt 3
0

On integrating : f al = 3v— f dt
2 0
a

2a
3
a= — vt so t= —
2 3v
6

1.13 Let us locate the points A and B at an arbitrary instant of time (Fig.).
If A and B are separated by the distance s at this moment, then the points converge or
— ds
point A approaches B with velocity = v — u cos a where angle a varies with time.
dt
On intergating,

ds = f (v — u cos a) dt,

f
(where T is the sought time.)

or 1= f (v — u cos a) dt
0

As both A and B cover the same distance in x-direction during the sought time interval, so
the other condition which is required, can be obtained by the equation

ex= fx
v dt

So, uT = f v cos a dt (2)

Solving (1) and (2), we get T


—u
One can see that if u = v, or u < v, point A cannot catch B.
1.14 In the reference frame fixed to the train, the distance between the two events is obviously
equal to L Suppose the train starts moving at time t= 0 in the positive x direction and
take the origin ( x = 0) at the head-light of the train at t = 0. Then the coordinate of first
event in the earth's frame is
1 2,
x1= —2 WI

and similarly the coordinate of the second event is


1
x,4 — w(t + —1

The distance between the two events is obviously.


xi —x2= 1— wt (t 4- t/2 ) = 0.242 km
in the reference frame fixed on the earth..
For the two events to occur at the same point in the reference frame K, moving with
constant velocity V relative to the earth, the distance travelled by the frame in the time
interval T must be equal to the above distance.
Thus V'T mg 1 — w t + T/2)

So, —/— w ( t + T/2 ) 4.03 m/s

The frame K must clearly be moving in a direction opposite to the train so that if (for
example) the origin of the frame coincides with the point x1 on the earth at time t it
coincides with the point x2 at time t + T.
7

1.15 (a) One good way to solve the problem is to work in the elevator's frame having the
observer at its bottom (Fig.).
Let us denote the separation between floor and of celing by h = 2.7 m. and the acceleration
the elevator by w = 1.2 m/s2
From the kinematical formula

yo + voy t +1 wy t
Y= 2 (1)

Here y = 0, yo = + h, vg, = 0
and wy watt (y) Wele (y)

(- g) - (w) = (g + w)
= 2 -7 m.
1 e
So, 0= h+ - {--(g+w)}t2 t1.0=1.2 in
2

or, t= =0.7 s.
g+w

(b) At the moment the bolt loses contact with the elevator, it has already aquired the
velocity equal to elevator1given by :
vo = (1.2) (2) = 2.4 m/s
In the reference frame attached with the elevator shaft r
(ground) and pointing the y-axis upward, we have for
the displacement of the bolt., •

= voy t +1 wy t2
T
1 a
= v„t+-., (-g)t22
4 f
or, Ay = (2.4) (0.7) + (- 9-8) (0.7)2 = - 0.7 m.
2

Hence the bolt comes down or displaces downward relative to the point, when it loses
contact with the elevator by the amount 0.7 m (Fig.).
Obviously the total distance covered by the bolt during its free fall time

s= I Ay (2-4) 2
+ 2 (yg) = 0.7 m + m= 1.3m.
(9.8)

1.16 Let the particle 1 and 2 be at points B and A at t = 0 at the distances /1 and 12 from
intersection point 0.
Let us fix the inertial frame with the particle 2. Now the particle 1 moves in relative to
this reference frame with a relative velocity712 = v its trajectory is the straight
line BP. Obviously, the minimum distance between the particles is equal to the length of the
perpendicular AP dropped from point A on to the straight line BP (Fig.).
8

vi
From Fig. (b), v12 vi and tan 0 = V2 (1)

The shortest distance

AP= AM sin 0 = (OA - OM) sin 0 = (12 - /1 cot 0) sin 0

v2 v v1 I2 - V2 11
or AP = (12- 11H _ 1 71 , -, , 7 (using 1)
vi v vi + v-2 v vi + V2

The sought time can be obtained directly from the condition -that (4 - v1 t)2 + (12 - v2 t)2
2 v2
11 v1 + 1
is minimum. This gives t = 2 2 •
Vi + V2

1.17 Let the car turn off the highway at a distance x from the point D.
So, CD = x, and if the speed of the car in the field is v, then the time taken by the car to
cover the distance AC = AD - x on the highway
AD - x
ti= (1)
ri v C c X
and the time taken to travel the distance CB in
the field
‘112 4-X2
t2 - (2)
V

So, the total time elapsed to move the car from point A to B

,, AD - x 12 +
t ti+ t2
1V V

For t to be minimum

dt 1 [ 1
dx 0 Or - - + =0
I, 11 V 12 +x2
12x2. 2 +x2 or
Or 1
9

1.18 To plot x (t), s (t) and wx (t) let us partion the given plot vx (t) into five segments (for
detailed analysis) as shown in the figure.
For the part oa: wx= 1 and vx = t = v
t
t2 VX b
Thus, 0x1 (t) = fvx dt = fdt = — = si (t) 1 e
2
0 0 2 3 4 5 6
1
Putting t = 1, we get, A xi = s = a- unit -1
-2
For the part ab : d
wx = 0 and vx = v = constant = 1

Thus 46a2 = f vx dt = f dt = (t -1) = s2 (t)

Putting t = 3, Ax2 = s2 = 2 unit


For the part b4 : wx = 1 and vx = 1 - (t - 3) = 4 - t) = v

Thus

Putting

For the part 4d :

So,

t2
Thus A x4 (t) = f (1 - t) dt = 4 t - -8
4
Putting t= 6, Ax4= -1
2
t
Similarly s4 (t) = f I vx I dt = f (t - 4) dt = —2 - 4t + 8
4
Putting t = 6, 54 = 2 unit
For the part d 7 : wx= 2 and vx = - 2 + 2 (t - 6) = 2 (t - 7)
for
v= I vx I 2 (7 - t) t 4-- 7
6
2
Now, Ax (t ) = f2 (t - 7) dt = t - 14t + 48

Putting t = 4, bas = -1
6
Similarly s5 = f 2 (7 - t) dt = 14 t - t 2 - 48

Putting t = 7, s5 = 1
On the basis of these obtained expressions wx (t), x (t) and s (t) plots can be easily plotted as
shown in the figure of answersheet.
10

1.19 (a) Mean velocity


Total distance covered
<v> —
Time elapsed
s R

50 cm/s
V
= = =
(1)
(b) Modulus of mean velocity vector
-D. I A Fi = 2R
I<v>I= — = 32 cm/s (2)
At
(c) Let the point moves from i to at f along the half circle (Fig.) and vo and v be the spe
the points respectively.
dv
■■• 1111 w

We have dt

or,

f ( v o + w, t) dt

vo + (vo + wit) vo
+V
So, <v> =
2 2

f fit

So, from (1) and (3)


+
n R

2 (3
Now the modulus of the mean vector of total acceleration
I i(7, I vo + v
< i;7> I I All= (see Fig.)
At

1.20 (a) we have

d
and w - 2 a a
dt

(b) From the equation


51 (1 — a t),
1
71." 0, at t= 0 and also at t= At a
1
So, the sought time At = a
As v - 2 a t)
1
a (1 — 2 a t ) for s t

2a
So, v= 1o= a(2at-1) fort>
1
2a
11

Hence, the sought distance


1/2 a 1/a

Simplifying, we get, s = t
2c
1.21 (a) As the particle leaves the origin at t = 0 So,
Ax = x = f vx dt (1)

As v = Ti;(1 — —,:),

where vo). is directed towards the +ve x-axis


v
So, x = v0 (1 — —t) (2)
t
From (1) and (2),
r

(3)

Hence x coordinate of the particle at t = 6 s.

x= 10 x 6 (1 — — —) = 24 cm = 0.24m
15
2x
Similarly at t = 10 s
10
x= 10 x 10 (1 — )— 0
2 5
and at t= 20s

x= 10 x 20 (1 — ° = —200 cm = — 2 m
2x 5
(b) At the moments the particle is at a distance of 10 cm from the origin, x = ± 10 cm.
Putting x = + 10 in Eq. (3)

10= 10t 1- 01) or, t 2 — 10t + 10 = 0,

10± V 100 — 40
So,
t=t= = 5± NifY s
2
Now putting x = — 10in Eqn (3)

— 10 = 10 (1 — 1
10 '
On solving, t = 5 ± 075 s
As t cannot be negative, so,
t = (5 + ) ss
12

Hence the particle is at a distance of 10 cm from the origin at three moments of time :
t = 5 ± -43- s, 5 + Z-5- s

(c) We have v '70 (I- — 1


x

vo (1 —1 forts x
t
So, v= NI =
vo (I- — 1) for t > T
t
r
So s= f vo (11 — IT) dt for t s t = vo t (1 — fit)
0
Y r

and r>T

(A)
4 4

S=f V0

And for t = 8 s

0

5
(1 8

s= f 10 (1 - i) dt + f 10 (5 - 1) d t
0 5
On integrating and simplifying, we get
s= 34 cm.
On the basis of Eqs. (3) and (4), x (t) and s (t) plots can be drawn as shown in the answer
sheet.
1.22 As particle is in unidirectional motion it is directed along the x-axis all the time. As at
t= 0, x= 0
So,
Therefore,

or,

(1)

As,
r
v
,,,2
a2
On integrating, f dv . f = Or, v= 2 t (2)
dt
2
o o
13

(b) Let s be the time to cover first s m of the path. From the Eq.

s= f v dt

r
a2 , a2 t2
s= f 2 a t = -- -- (using 2)
0 2 2

2 _ f—
or t. vs (3)
a

The mean velocity of particle

2 iS-/a

a2
— t dt
f V (t) dt 2
a 'is-
<v> =
fdt
=
f0 2 'IF/ a
2

1.23 According to the problem


v dv
- .. a y (as v decreases with time)
ds

0 s
or, - f Vv- dv = a f ds
v0 o

2
On integrating we get s = — vo
3a 3/2

Again according to the problem

- cc: = a Nr; or - 41-' = a dt


v

0
Ndc v .
or, a .1* dt
vo

2 V— vo
Thus t=
a
A 3
--+ ..-1.
1.24 (a) As r = att-.. bt2 j
X
So, x= at, y= -bt2

—bx2
and therefore
a2 Y —
14

which is Eq. of a parabola, whose graph is shown in the Fig.

(b) As v atr-bt2j*

--11. dr
= -
d t
= at-rill.2btr (1)

17t11. (art2btn.( -2k1 1


(c) cos a =
vw a2 + 4 b2 t2) 2b

2bt
or, cos a-
Va2+4b2t2

a
so, tan a 2bt

_ a
or, a= tanLan
2bt

(d) The mean velocity vector

f(ait 2 btndt
-+ i7dt
<v >- at-btj
fdt t
a2+
Hence, j<17)>I= V (-bt)2 = V112 +b2 t2

1.25 (a) We have


x= at and y= at(1-at) (1)
Hence, y (x ) becomes,
ax (i-ax)
2
Y= a a = a x (parabola)

(b) Differentiating Eq. (1) we get


vx= a and vy= a(1-2at) (2)
- -
15

So, v= Vv2 +v2 =aV1 +(1 - 2 at)2


x y
Diff. Eq. (2) with respect to time

wx = 0 and w = - 2 a cc
z
So, w= "vi 14/2+ w = 2aa

(c) From Eqs. (2) and (3)


-0
We have v- ai+a(1-2at)i and w = 2aa

7r 1 v•w -a(1 - 2 ato)2 act


So, cos — — ,
4 aV +(1-2ato)2 2aa

On simplifying. 1- 2 a to= *1

1
As, to PI 0 , rf =

t' a
1.26 Differentiating motion law : x= a sin wt, y= a(1- cos w t ), with respect to time,
= a co cos cot , vy = a co sin cot
So, cia.= a w cos cot a w sin coti (1)

and v = aw= Coast. (2)


Differentiating Eq. (1) with respect to time
--10
$4, = dV,11' 2 • 2
—dt = - a w sm cott+aw cos cot) (3)
(a) The distance s traversed by the point during the time t is given by

s = f vdt- facodt- a cot ( using 2 )

Thus, 1711 w, i.e., the angle between velocity vector and acceleration vector equals —71
2
1.27 Accordiing to the problem
w= w(-j)

dvx dvv
So, = = 0 and w = = -w
wx dt Y dt

Differentiating Eq. of trajectory, y = a x - b x 2, with respect to time


adx _
2bx dx
dt dt dt
16

dyl . a dx
So,
dt dt
x-0 x..0

,Again differentiating with respect to time

2
d2y _ ad dx d2X
2/3 (-cii) -2bx dt2
d t2 d r2

d x 2
or,
- w = a (0) - 2 b ( -c Tt -2 b x (0) (using 1)

d x .1177
or,
cI t = 2L (using 1)
(-

117,
Using (3) in (2)
ifdlt I = a 2b
I x- o

Hence, the velocity of the particle at the origin

v=

Hence, v = Viib'- (1 + a2)

1.28 As the body is under gravity of constant accelration r, it's velocity vector and displacemen
vectors are:
--a. --). --0

v = vo + g t (1:
—3.
1 2 ,--). _
r=v = u at t = 0) (2;
and Ar = t+— t r
0 2g k

So, <v; over the first t seconds

--> Ai* r --3IP


--I. gt_
<v> = — = = vo + (3)
At t 2

Hence from Eq. (3), <v> over the first t seconds

---OP ---)
<v>= VA + t (4)
u 2

But we have v = v0 at t = 0 and

Also at t = t (Fig.) (also from energy conservation)


17

Hence using this propety in Eq. (5)

vii = vo + 2 (1.70'•git + g2-c2

2 (1,0 • e
As t * 0, SO, t-
2
g

Putting this value of t in Eq. (4), the average velocity over the time of flight
, „(vo • gi
<v> = vo - g 2
g

t.29 The body thrown in air with velocity vo at an angle a from the horizontal lands at point
P on the Earth's surface at same horizontal level (Fig.). The point of projection is taken
as origin, so, Ax = x and Ay = y
1
(a) From the Eq. Ay = v0 J.+ 2- wy t 2
1
0= vo sin at- gt2
2
2 vo sin a
As t * 0, so, time of motion t =
g
(b) At the maximum height of ascent, vy = 0

so, from the Eq.

2 2
vo sin a
Hence maximum height H =
2g
During the time of motion the net horizontal displacement or horizontal range, will be
obtained by the equation
1
Ax= v, t + - w t2
- 2 x
1 v2,, sin 2 a
Or, R= vo cos a t - - (0) t2 = vo cos a -
2 T- g

when R= H
2 2 2
vo sin a vo sin a
or tan a= 4, so, a= tan-1 4
g 2g

(c) For the body, x (t) and y (t) are


x = vo cos a t (1)
18

and y= vo sin a t - —
t2 (2)
2g
Hence putting the value of t from (1) into (2) we get,
2 2
x 1 x gX
y = vo s a ) = x tan a - „
in (v cos a)co - 2 g (vo cos a 2 vo cos- a'
Which is the sought equation of trajectory i.e. y (x)
(d) As the body thrown in air follows a curve, it has some normal acceleration at all the
moments of time during it's motion in air.
At the initial point (x = 0, y = 0), from the equation :

Wn = R—R , (where R is the radius of curvature)


2 2
Vo Vo
g cos a = — (see Fig.) or Ro =
Ro g cos a
At the peak point vy = 0, v = vx = vo cos a and the angential acceleration is zero.
2
Now from the Eq. wn =
R
2 2
Vo cos a v20 cos2 a
g= R , or R-
g
Note : We may use the formula of curvature radius of a trajectory y (x), to solve part (d),
2 3
r
[1 + (dy / dx) R=
:_ld2Y/dr21
1.30 We have, vx = vo cos cc, vy = vo sm a - gt

Thus

Now,

v
= - —g
V (vo sin cc - g t) . -g TY
t r
.g iv I
Hence I wt I ___ _z_
v

Now w= 1/ IV - w 2 = g2 _ g2 2122.
n t
11?

Vx 1./ 2 2 \
or Wn = g — (where vx = vt - vy
v )
19

A uring time of motion


As 7 I v„ d

v
w = w = - g --L v
t
v
On the basis of obtained expressions or facts the sought plots can be drawn as shown in the
figure of answer sheet.
1.31 The ball strikes the inclined plane (Ox) at point 0 (origin) with velocity vo = Viiii (1)
As the ball elastically rebounds, it recalls with same velocity vo, at the same angle a from the
normal or y axis (Fig.). Let the ball strikes the incline second time at P, which is at a
distance 1 (say) from the point 0, along the incline. From the equation
1

y= voy t + -i wy t2

1
0= vo cos a lc — — g cos a t2
2
where t is the time of motion of ball in air
while moving from 0 to P.
2 vo
As T * 0, SO, T - (2)
g
Now from the equation.
1
2
x= v t+- w t
Ck 2 x

1= vo sin a t 1- -1 g sin a T2
2

vo sin a (2 vo) + i1 _ sn a (2 vo)


SO, 1= )
S 26 g

4 vo2 sm a
(using 2)
g
4 (2 gh) sin a =
Hence the sought distance, 1= 8h sin a (Using Eq. 1)
g
1.32 Total time of motion
2 vo sin a II_ 9.8 t
t— or sin a = (1)
g 2 vo 2 x 240
and horizontal range
R 5100 85
R . vo cos a t or cos a = = = (2)
vo T 2401 4t
From Eqs. (1) and (2)
,T24. (85)2 1
(9.8)2
(
(480)2 4 t2)2 —

On simplifying T4 - 2400 T2 4- 1083750 = 0


20

Solving for r2 we get :

2400 ± V 1425000 2400 ± 1194


I`
T2— 2 2
Thus T= 42.39 s = 0.71 min and

T = 2455 s = 0.41 min depending on the angle a.

1.33 Let the shells collide at the point P (x, y). If the first shell takes t s to collide With second
and At be the time interval between the firings, then
x= vo cos 01 t = vo cos 02 (t — A0 (1)

and y= vo sin 0 t — — gt2


' 2

= vo sin 02 (t — At) — g (t — At)2 (2)


2

Y t

Integrating f dy = vo f dt or y = vo t (1)
o o
dr
And also we have — ay or dx = a y dt = a vot dt (using 1)
dt=
x t
1 a y2
So, f dx = a vo f t dt, or, x= la v t2 = — (using 1)
2 0 2 o
o 0
(b) According to the problem
vy = vo and vx = a y (2)

\rvx277. v; . V v20 + a2 y2
So, v=
dv a2 y dy a2 y
Therefore )4, = =
t dt V q) + a y2 dt 11 1 + (ay I v 0)2

Diff. Eq. (2) with respect to time.


dv d vx dy
14, 0 and =w a -di = a
dtY = Y= x= Vo
dt
So, w= 1%1= a vo
21

a vo
Hence wn = V w2 - q - V a2 v2o — a4
y2
2
1 + (ay I vo)2 . V 1 + (ay 1 vo)

1.35 (a) The velocity vector of the particle


--a. -.-. --1.
v= at+bxj

So, d= a : A = bx
dt

And dy = bx dt = bat dt
Y t
1
Integrating 5 dy = ab f t dt or, y= — ab t
2 2
o o

From Eqs. (2) and (3), we get, y =--- x2

(b) The curvature radius of trajectory y (x) is :


3
2
[ 1 + (dy Idx)2 1
R= (5)
Id2y1dx21
2
Let us differentiate the path Eq. y = b with respect to x,

d2
= b x and y b (6)
dx a dx2 a
From Eqs. (5) and (6), the sought curvature radius :
3
2

1.36 In accordance with the problem


R 7a- 1+ (—b x)
12
-410
= a •"V
v v
= ds d
But wt or v dv = w ds

So, v dv = criti)ds=a+-dr-*
--10.
OT, v dv = ai•d r adx (because a Is directed towards the x-axis)
z

So, fvdv= a fdx


0 0

Hence v2 = 2 ax or, v = V2crx


22

1.37 The velocity of the particle v = at


dv
So, w = a (1)
dt
r
v2 a2t 2
And wn = — in (using v = at) (2)
R R

From s = fvdt
r
1 2
-2/ERTI= a fv d t = —at
2
o

4 x i . t2 _
So, (3)
a R

From Eqs. (2) and (3) wn = 41cari

Hence w =

= V a2 + (4 It a i)2 = a V 1 + 16 x2 ri2 = 0.8 m/s2


1.38 According to the problem
lwrI'm Iwnl
- dv v2
For v (t), .—
dr R

Integrating this equation from vo s vs v and Os ts t

v , dv 1 vo
-= i dt or, v =

—f
(1 +vot)

o
0 R
2
v dv v
NJW for v (s), - —ds = —R , Integrating this equation from vo s vs v and Os ss s

v dv 1
s

v s
SO, -

Iv v R

0
cis or, In TI =
o
- siA
R

Hence
o v= voe (b) The normal acceleration of the point
2 /R (2)
v V2 e-2s
wn =
R R
(using 2)
And as accordance with the problem
and w
I wr 1 = 1 %1
A
1 A
t ut wn Un
2 1,2
Vn
SO, w= a = a -=-e- 2"
= a L-
w ig R R
-▪
23
1.39 From the equation v=a
dy a ds a a2
w vi a -Cs = and
dt 25 dt 2 —2 '
2
v a2 S
w = - =
R R
As wr is a positive constant, the speed of the particle increases with time, and the tangential
acceleration vector and velocity vector coincides in direction.
A
Hence the angle between v aand and a can be found
wis equal to between wt is an w,
1%1 a 2 s/R = 2s
=
by means of the formula : tan a
wrI a2/2 R

1.40 From the equation 1= a sin ou t


dl
— = a co cos co t
dt v
dv 2 •
So, wr —dt – a co sin w t , and

v2 a2 CO2 COS2 CO t
wM
—It at
(a) At the point 1= 0, sin w t = 0 and cos w t = ± 1 so, wt = 0 , n etc.
a2 €02
Hence w= W

R
Similarly at 1= ± a, sin cot = t 1 and cos cot = 0, so, rvn = 0
2
Hence w= I wt I = a0)

1.41 As wt = a and at t = 0, the point is at rest

(1)

Let R be the curvature radius, then


2 2 2
w= v at 2 as (using 1)
R

But according to the problem

bt 4

2 2
alt 2 a a
So, bt4 = or, R = 2= (using 1) (2)
bt 2 bs

Therefore w=1/7w-1- rt = a2 + (2 as / R)2 a2 + ( 4 bs2 / a2)2 (using 2)

Hence w= a1/1 +(4bs2/a3)2


24

41. 2 ax cr cPy . 2a[


dt dt ' dt2
r)2
1.42 (a) Let us differentiate twice the path equation y (x) with respect to time.

dt
d2X
+ x dt2
Since the particle moves uniformly, its acceleration at all points of the path is normal and at
the point x = 0 it coincides with the direction of derivative d2 y/dt 2. Keeping in mind
that at the point x = 0, i 111 v,
-
d 1 c/_21
NE 2 a v2 = wn
dt 2
x- 0
We get w=
2
V 1
2a v2 = —

So, IVn
In or R= —
R' 2a
Note that we can also calculate it from the formula of problem (1.35 b)
(b) Differentiating the equation of the trajectory with respect to time we see that

b2x -d—r + a2y A( = 0 (1)


dt dt
which implies that the vector (b2x r+ a2yri is normal to the velocity vector
--). dx „A. li/ •r•
t +
dt j which, of course, is along the tangent. Thus the former vactor is along
V =
dt
the normal and the normal component of acceleration is clearly

2 d2y
b 2 x d2X a dt2
dt2 dt2
Wn me ( b4x 2 + a4y2 )112

--0 ---,
wn = w •n / l i t l . A t x =
on using 0,y= ± bandsoatx=0

Adl
± dt2 ... 0
Differentiating (1)

=0

Also from (1) II= 0 at x = 0


dt
So ±v (since tangential velocity is constant = v )

Thus ( 4.-2Y- ) = ± b- v2
dt2 a2

bV2 v2
and I Wn I m
-R
2

This gives R = a2/b .


25

1.43 Let us fix the co-ordinate system at the point 0 as shown in the figure, such that the
radius vector rof point A makes an angle 0 with x axis at the moment shown. Note
that the radius vector of the particle A
rotates clockwise and we here take line ox as
reference line, so in this case obviously the
( d0
angular velocity co= - - taking
dt
anticlockwise sense of angular displacement as
positive.
Also from the geometry of the triangle OAC
R = r
or, r= 2 R cos 0.
sin 0 sin ( lt - 2 0 )
Let us write,
-0. -o -0 2 -0 •
r = r cos 0 i + r sm Oi = 2 R cos 0 i + R sm 2 07
Differentiating with respect to time.
d 0 ---*
dcrit or i7':- 2R2cos0(-sin0) — •+ 2R cos20 -4107*
dt 1 dt -1
-0 (-d 0 ) -.--0
or, v = 2R [sm201-cos20j]
dt
-4. --*2 74.
or, v= 2Rw(sin20i-cas 0j )
So, Ivi or v. 2wR=0.4m/s.

dv
As w is constant, v is also constant and w — 0,
t dt
v2 _( 2 (.0 R )2
So, w= wn - . 4 w2 R .. 0.32 m/s2
R R
Alternate : From the Fig. the angular velocity of the point A, with respect to centre of the
circle C becomes
d (d2t0 (-d0
- 2 = 2 w . constant
2. dt
Thus we have the problem of finding the velocity and acceleration of a particle moving
along a circle of radius R with constant angular velocity 2 w.
Hence v= 2wR and

v2 ( 2 w R )2
2
w= wn = ---- S
R -
4w R
R R
1.44 Differentiating q) ( t) with respect to time
d cp
c 2at (1)
dt '1' oz =
For fixed axis rotation, the speed of the point A:
v
v=coR= 2atR or R. (2)
ta t
26

Differentiating with respect to time


dv v
w — = 2 a R = -, (using 1)
dt
2
=RV V2
But w11/4 =2atv (using 2)
v/2at
,
So, wr w 2 n =NI (vIt)2+(2atv)2

=Y-111+4a2t4
t

1.45 The shell acquires a constant angular acceleration at the same time as it accelerates linearly.
The two are related by (assuming both are constant)
w
1 2nn
Where w = linear acceleration and f3 =angular acceleration

2
Then, w= V 2.132 itn.V 2•—w ( 2 n n)
1

But v 2= 2 w 1, hence finally 2


nnv
co.
1

1.46 Let us take the rotation axis as z-axis whose positive direction is associated with the
positive direction of the cordinate p, the rotation angle, in accordance with the right-hand
screw rule (Fig.)
(a) Defferentiating tp ( t) with respect to time.

dcl9= a -3bt2= coz (1) and


dt
d2c1) do)
= fiz -6bt (2)
dt2 - dt

N r
From (1) the solid comes to stop at .6, t = t = —
3b

The angular velocity co = a- 3 bt2, for 0 s t s Nra73T;

ri3b
(a-3bt2)dt

fdt
Va
dt
0
Similarly 13 fiz 6b t for all values of t.
27

3/;17,3b
ff3 dt Jo 6br dr
So, <p>= m V3711-,
fdt i"--
a/$3b
i dt
0
(b) From Eq. (2) pz = - 6b t

So, ( pz ), =vri/rt; = -6b Nrli = -2 11(17,

Hence i3 = I )
(~z t- irdig I= 2-iSiTE

1.47 Angle a is related with I w, I and w,, by means of the fomula :

Wn
tan a = where w,, = w 2 R and I w1 I ... 13R (1)
,
l wri
where R is the radius of the circle which an arbitrary point of the body circumscribes.
dco dco
From the given equation (3 i= — = at (here t3 = dr ' as 13 is positive for all values of t)
dr

2
4
R. at 2 R . a2t R
So, w . r.,2
At -
2 4
and lwrk 13-R= atR
Putting the values of I w, I and w„ in Eq. (1), we get,
113
3
tan a = a
2 R/ 4 = ar
t4
or, t= [ ( 4- ) tan a
atR 4
a

1.48 In accordance with the problem, I3z < 0


dco
Thus - dt k Ira7 , where k is proportionality constant

kt
Or, - 1 da) .- kidtor, 10-)= .1/.0 . - -4- (1)
v 0) ° 2
0

When co = 0, total time of rotation t - x = 2 “1130


k
28

2 I(0 0 / k

Average angular velocity < w >


fwdt
1
f
CO0
4- k
24t
idt 2
0 -kt
1/7
(0)
dt

V
i
T
o
;

/
k

2VEnik
Hence < co > = coo t + k{ 3 k'vrw— t21 - / 2 -legr)° = co0/3
1122 2 ° k
o

1.49 We have co = coo - a q) =


dt
Integratin this Eq. within its limit for (q)) t

d wo -kg)
= f d t or ,, In --kt
too
-10 (00 — icci) 0
Hence cp = ( 1 - el") (1)

(b) From the Eq., w = coc, k cp and Eq. (1) or by differentiating Eq. (1)

-kt
(02. w °6
1.50 Let us choose the positive direction of z-axis (stationary rotation axis) along the vector
. In accordance with the equation
o
O
r
Hence
,

O
r
,
w
4,

f 1
3
2
Wz 0
2
f
coz
c
o
s
c
p
t
h
p

0
= 130 sin
cp

tV2
Po
sin
fp
The plot coz (q)) is shown in the Fig. It can be seen that as the angle cp grows, the vector iiifirst
increases, coinciding with the direction of the vector g ((,)> 0), reaches the maximum at cp q)/2,
then starts decreasing and finally turns into zero at q) = r. After that the body starts rotating
in the opposite direction in a similar fashion (coz < 0). As a result, the body
will oscillate about the position fp = cp/2 with an amplitude equal to n/2.
29
1.51 Rotating disc moves along the x-axis, in plane motion in x - y plane. Plane motion of a
solid can be imagined to be in pure rotation about a point (say I) at a certain instant
known as instantaneous centre of rotation. The instantaneous axis whose positive sense is
directed along (Trof the solid and which passes through the point I, is known as instantaneous
axis of rotation.

Therefore the velocity vector of an arbitrary point (P) of the solid can be represented as :
-IP -■
V 30 (0 X TEr
(1)
On the basis of Eq. (1) for the C. M. (C) of
the disc A
-•
vc x rd
2it CO (2)

According to the problem v t t i and


---)4
ri.e. cTilx -y plane, so to satisy the rcr
Eqn. (2) rc./ is directed along (-n. Hence point I is
IUNIO • >3C
at a distance r 1= y, above the centre of the 0 C
disc along y - axis. Using all these facts in Eq.
(2), we get
Vc
coy or y (3)
CO

(a) From the angular kinematical equation

woz + 13z t (4)

co 80 t.
On the other hand x v t, (where x is the x coordinate of the C.M.)
x
or, t (5)

From Eqs. (4) and (5), w =

V2
Using this value of to in Eq. (3) we get y — ( hyperbola )
f3 x/v px
(b) As centre C moves with constant acceleration w, with zero initial velocity
1 2
So, X= —
Wt and v,= wt
2

Therefore, vc v2 x w

Hence
- ▪

30

132 The plane motion of a solid can be imagined as the combination of translation of the C.M.
and rotation about C.M.
-4.
So, we may write VA = vc + VA

Vc
+ CO X rA (1) and
--• —
W A " wc wA c

2
W + CO ( — r
) + (fix )
(2)
A C rAC

s the position of vector of A with respect to C.


rAC i
In the problem vc = v - constant, and the rolling is without slipping i.e., vc = v. w R ,
So; we = 0 and 13= 0. Using these conditions in Eq. (2)
2
A V A
2
R

Here, uA c is the unit vector directed along rA

2
A
V
Hence WA = — and WA is directed along (-u or directed toward the centre of the
R A C

wheel.
(b) Let the centre of the wheel move toward right (positive x-axis) then for pure tolling on
the rigid horizontal surface, wheel will have to rotate in clockwise sense. If w be the
vc
angular velocity of the wheel then co=
R •

Let the point A touches the horizontal surface at t = at 0, further let us locate the point A
t= t,
When it makes 0 = w t at the centre of the wheel.
-a. -a. -4 -4.
From Eqn. (1) VA- V CO X rAc
c
Vi+03(—iC)X[Rcos0(—j) + R sin 0 (- I

or, vA = v i - R [ cos co t (— i) + sin cot j

7*
au (V — COS W O + V S M COO (as v= caR)

So, VA= V (V — V COS (002 (v sin cot)2-

= v V 2 (I - cos cot) = 2 v sin (cot / 2)


Hence distance covered by the point A during T= 2n/co
2a/au

8
s= f vA dt = f 2 v sin(cot/2) dt = v = 8 R

0
133 Let us fix the co-ordinate axis xyz as shown in the fig. As the ball rolls without slipping
along the rigid surface so, on the basis of the solution of problem 1.52 :
v0= V c 0) X roc u
Thus —4. 7.} (1)
ye wR and (1-0* t (- k) as vc t t

▪▪ •
31

03c + 134°X roc = 0

Vc1WR=VA

Vc
wt w

and — - - an d 13 = — (using 1) V8
R R R

(a) Le t us fix the co-ordinate system wi t h the fram e

attached wi t h the rigid surface as s h o wn in the Fig.

As point 0 is the instantaneous centre of rotation of the ball at the mo me n t s h o wn in Fig.

so,
"I 0,

Now, vA vc + x rAc

Vc + CO (- k) x R ) (v c+ wR) i

So, A = 2 vc 2 wt i (using 1)

-0 -0 -0 --I.
Si mi l a l r y v - vc. + co x rBc v c + (- k ) x R (i )

V c ÷ (DR (- j ). vc. t + vc (- j )

-0 - 0

So, vB = Nr2- vc = wt and v; is at an angle 45° fro m b o t h i and j ( Fi g . )

Wnbc + (02 (— + rx 770.c

‘,2 A W
vc A
az CO2 (""' ) = — ( - u m, ) (using 1)
c'c R
A -10
w here uoc is the unit vector along roc

t2
4)2R B
SO, W0 gm - ( u s i n g 2 ) a n d w0 is
R R
(02R
directed t o wa r d s the centre of the ball

Now WA = We + co2 (-

) rx rAc
= wr+o)2R(—.13+13(—r)xR7

w4 t 4 V ( wt 2 ) 2

So, WA= 4 wh + R2 = 2 1 +
2R

S i mi l a r l y 14r8 im + (.02 (- rx

se W i+0)2R (- 0 + t3 (—k)xR(i)

(I ) (- ji (using 1)
32

— 2 w (-1) (using 2)

w2t 2 2 2
So, WB = r w— +W

1.54 Let us draw the kinematical diagram of the rolling cylinder on the basis of the solutioi
of problem 1.53.

A VA:72Vc we-07.

0
As, an arbitrary point of the cylinder follows a curve, its normal accelerationand
radius of curvature are related by the well known equation

v2
Wn =
1F
2
vA
so, for point A, WA (n)
RA

4 v2
or, RA = 2 ac 4r (because v s (or, for pure rolling)

Similarly for point B,


2
VB

RB

Or, RB los 2 v 2 =2 r
Dr
1.55 The angular velocity is a vector as infinitesimal rotation commute. Then the relative angular
velocity of the body 1 with respect to the body 2 is clearly.
(1 )12 c°2
22 (1 )1

as for relative linear velocity. The relative acceleration of 1 w.r.t 2 is


( )
dt s•
-
33

where S' is a frame corotating with the second body and S is a space fixed frame with
origin coinciding with the point of intersection of the two axes,
( dirii ) (dcTi;) + (7). .. 7.
but 2 x 4-1)1
dt dt
S s,
• -0 d4
Since S' rotates with angular velocity w2 . However — . 0 as the first body rotates
dt s
with constant angular velocity in space, thus
--1. --a.
P12= CO 1 X CO2.

Note that for any vector bthe relation in space forced frame (k) and a frame (k') rotating with
angular velocity (ins
d El d —,
dt dt + co x 6*
K IC

1.56 We have 2 74.


0..) = at 1 + bt j (1)
So, w .. V (at)2 + (t2)2 , thus, colt_ 10.= 7.81 rad/s

Differentiating Eq. (1) with respect to time


,, ,_
p s. —
dt Z.
at+zotj (2)

So, (3 = V a2 + (2 bt)2

and Pit- 10. - 1.3 rad/s2

(b)

Putting the values of (a) and (b) land' taking t = 10s, we get
a fe 17°

1.57 (a) Let the axis o1 the cone (OC) rotates in anticlockwise sense with constant angular
velocity (7.)1' and the cone itself about it's own axis (OC) in clockwise sense with angular
velocity ZO*0 (Fig.). Then the resultant angular velocity of the cone.
-0 --., -.
w 1.1 CO + (0 o
(1)
As the rolling is pure the magnitudes of the
vectors ZIP and (7.:0 can be easily found from Fig.
v
co' = c = v/R (2)
R cot a ' o°

As w 1 Z,70, from Eq. (1) and (2)


34
co . f7772-
.I-wo
V 2
2.3 rad/s
1/ (R cot a )2 "I' ( Rv ) 2 -I R cos a=

(b) Vector of angular acceleration


--IV --1.

d (7;1 d (0 + wo )
ir. --- =
(as w = constant.)
dt dt

--I. --I.,
T about the 00' axis with the angular velocity co , re
h
e
,..4. --00 --1.
Thus p = co x coo

v The magnitude of the vector 'is equal to

e
c
t
o
r

c
o
o

w
h
i
c
h

r
o
t
a
t
e
s
13 = co' coo (as al 1 o—io )

v v v
So, 13 - - 2 tan a = 2.3 rad/s
R cot a R R2

1.58 The axis AB acquired the angular velocity


CO al rt (1)
Using the facts of the solution of 1.57, the
a
--). --*
angular velocity of the body 0' --= .4 t
if2-71"
co = v coo + co
a .402 4. 020 t 2
= 0.6 rad/s
Vo A
And the angular. acceleration.
—3.0
di:1-i. d(0) +GO dol (lioo
fr .
dt
dt dt
+ --wr
dt

di4 .--110 ....IP. d Zir


But . co x coo , and
dt dt = fl: t

So, 13= ( ro t x iiro ) + 137,1

As, gi Z1).0 so, it = 1/ (coo Po t)2 + fill = fill


V 1 + (coo t)2 = 0.2 rad/s2

- 35

THE FUNDAMENTAL EQUATION OF DYNAMICS


139 Let R be the constant upward thurst on the aerostat of mass m, coming down with a
constant acceleration w. Applying Newton's second law of motion for the aerostat in
projection form

F = mw

mg — R mw (1)

Now, if Am be the mass, to be dumped, then using the Eq. F = mw


R — (m — Am) g (m — Am) w, (2)
2 mw
From Eqs. (1) and (2), we get, Am MI
g + w
1.60 Let us write the fundamental equation of dynamics for all the three blocks in terms of
projections, having taken the positive direction of x and y axes as shown in Fig; and using
the fact that kinematical relation between the accelerations is such that the blocks move
with same value of acceleration (say w)

mo g =mow (1) v2
— T2 — g = w (2)

and T2 — km2 g M2 w (3)


>
The simultaneous solution of Eqs. (1), (2) and
frz
(3) yields,
[ mo k (m1+ m2)1
w= g
mo+ m 1+m2

(1 + k) mo V
and T2 at M2 g ?nog
mo+ mi+ m2
As the block m0 moves down with acceleration w, so in vector form
[mo — k (mi + m2)]
W at
m0+ /721 + M2
1.61 Let us indicate the positive direction of x-axis along the incline (Fig.). Figures show the
force diagram for the blocks.
Let, R be the force of interaction between the bars and they are obviously sliding down with
the same constant acceleration w.
36

Newton's second law of motion in projection form along x -axis for the blocks gives :

mi g sin cc - mi g cos a + R mi w
(1)

m2 g sin a - R - k2 m2 g cos a= m2 w (2)

Solving Eqs. (1) and (2) simultaneously, we get

mi + k2 m2
w= gsina-gcosa and
m1 ÷ m2

m1m2(k1 - k2) g cos a


R
(3)
m1+ m2

(b) when the blocks just slide down the plane, w 0 , so from Eqn. (3)

mi + k2 m2
g sin a-gcos a = 0
mi +

Or, -
(M 1 + m 2) sina (ki mi ir-2 m2) cos a
(k1 m1 + k2 m2)
Hence tan a
mi +

1.62 Case 1. When the body is launched up :

the velocity of projection and 1 the distance traversed


Let k be the coefficeint of friction, u
along the incline. Retarding force on the block - mg sin a + k mg cos a and hence the
retardation 7 g sin a + kg cos a.

Using the equation of particle kinematics along the incline,

0= u2- 2(g sin a + k g cos a )1

2
U
or, 1-
2 (g s i n a + k g c o s a)

and 0= u - (g sin a + k g cos a) t

or, u= (g sin a+ kg cos a) t

1 2
Using (2) in (1) 1- - (g sin a + k g cos a) t
2
Case (2). When the block comes downward, the net force on the body =

mg sin a - km g cos a and hence its acceleration = g sin a - k g cos a Let, t be

the time required then,


1
1= - (gsina—kgcosa)tI2 (4)

From Eqs. (3) and (4)

t2 sin a - k cos a
e2 sin a +kc os a

1
But t= - (according to the question),

Hence on solving we get

k= tan a = 016
(r12 +1)
37

1.63 At the initial moment, obviously the tension in the thread connecting m1 and m2 equals
the weight of m2.
(a) For the block m2 to come down or the block m1 to go up, the conditions is
m2g-Tz0 and T-migsina-fra 0

where T is tension and f, is friction which in the limiting case equals km1g cosa. Then

or m2 g - mi sin a > k mi g cos a

m
2
Or - > (k cos a + sin a)
m1
(b) Similarly in the case
mlgsina-m2g>frjlm

or, mi. g sin a - tn2g > kmi g cos a

or, (sin a - k cos a)


-1712- <
m1
(c) For this case, neither kind of motion is possible, and fr need not be limiting.
M2
Hence, (k cos a + sin a) > — > (sin a - k cos ot)
m1

1.64 From the conditions, obtained in the previous problem, first we will check whether the
mass m2 goes up or down.
Here, m2/m1 = -n > sin a + k cos a, (substituting the values). Hence the mass m2 will come down
with an acceleration (say w). From the free body diagram of previous problem,
m2-g m2 w (1)

and T- g sin a - k g cos a w (2)


Adding (1) and (2), we get,
m2 g - g sin a - k mi g cos a (m1+ m2) w

(m2/m1 - sin a - k cos ct) g - sin a - k cos a) g


w-
(1 + m2/m1) 1 + 11
Substituting all the values, w a. 0.048 g .0.05 g
As m2 moves down with acceleration of magnitude w 0.05 g > 0, thus in vector form
acceleration of m2 :
(ri - sin a - k cos a) el
w2 1+11 0.05

1.65 Let us write the Newton's second law in projection form along positive x-axis for the
plank and the bar
fr ml w1, fr m2 w2 (1)
38

At the initial moment, fr represents the static


friction, and as the force F grows so does the
friction force fr, but up to it's limiting value i.e.
fr = fro.) = k N = k m2 g.
Unless this value is reached, both bodies moves as a
single body with equal acceleration. But as soon
as the force fr reaches the limit, the bar starts
sliding over the plank i.e. w2 x w1.

Substituting here the values of w1 and w2 taken from Eq. (1) and taking into account that

f, = k m2 g, we obtain, (at — km2 g)/ m2 a —km2 g, were the sign "=" corresponds to the moment
m1
t = to (say)
k g m2 (mi + m2)
Hence, to =
a m1

- k m2 g
If t s to, then w1=
m1 (constant). and
w2 = (at - km2 g)/ m2
On this basis w1(t) and w2 (t), plots are as shown in the figure of answersheet.

1.66 Let us designate the x-axis (Fig.) and apply Fx = m wx for body A :
mg sina—kmgcosa= mw
or, w= gsina—kgcosa
Now, from kinematical equation :
/ sec a = 0 + (1/2) w t2

Or, t= V 21 sec a/(sin a— k cos a) g

= V 21 / (sin 2 a/2 — k cost a) g


(using Eq. (1)).

( sin 2 a
d — k cost a)
2
for trnin , =0
da

2 s 2a
i.e. "I + 2 k cos a sin a — 0
2

or, tan 2 a = — — et a= 49°

and putting the values of a, k and / in Eq. (2) we get t . - ls.

1.67 Let us fix the x —y co-ordinate system to the wedge, taking the x — axis up, along the
incline and the y — axis perpendicular to it (Fig.).
39

Now, we draw the free body diagram for the


bar.
Let us apply Newton's second law in projection
form along x and y axis for the bar :
T cos 13 — m g sin a — fr = 0 (1)
T sini3+N—mgcosa= 0
or, N= m g cos a— Tsin f3 (2)
But f; = kW and using (2) in (1), we get

T m g sin a+ k mg cos a/(cos 13+ k sin 13) (3)


For T the value of (cos 13 + k sin 13) should be maximum

d (cos 13 + k sin (3) —


So, 0 or tan 13 = k
d13

Putting this value of (3 in Eq. (3) we get,


m g (sin a + k cos a) m g (sin a + k cos a)
mn
hri71c2 + k2/V7+7:2 1 442

1.68 First of all let us draw the free body diagram for the small body of mass m and indicate
x — axis along the horizontal plane and y — axis, perpendicular to it, as shown in the figure.
Let the block breaks off the plane at t = to i.e. N = 0
So, N= m g — ato sin a .. 0

mg
or, to — a sin a (1)
From F. = m wx, for the body under
investigation :
m d N/dt at cos a ; Integrating within the
limits for v (t)

m f dvx= a cos aft dt (using Eq. 1)

ds a cos a 2
So, vg• dt
t
2m
Integrating, Eqn. (2) for s (t)
a cos a t3
2m 3
Using the value of t = to from Eq. (1), into Eqs. (2) and (3)
2
v=m g2 cos a _ m g3 cos a
and s
2 a sin2 a 6 a2 sin3 a
40
1.69 Newton's second law of motion in projection form, along horizontal or x - axis i.e.
Fx = m wx gives.
dv
F c,os (as) my —ds.
as)
or, F cos (asa =
(as) ds m v dv
Integrating, over the limits for v (s)
2 X
v
OS (as) ds=
-fC
2
Or v==
177n .a
ma
= V 2 g sin a/3 a (using F= 21-311 )
which is the sought relationship.
1.70 From the Newton's second law in projection from :
For the bar,
For the motor, T — 2 king = (2m) w
T— long m w'
Now, from the equation of kinematics in/the frame of bar or motor : (3)
1 t2

1= — +
2
From (1), (2) and (3) we get on eliminating T and w'
t= V 21/ (kg + 3 w)
2171

(zr 1/11////////////////////112/ 7 ;7)///17--).


J r fr
1.71 Let us write Newton's second law in vector from 1?= mg, for both the blocks (in the
frame ofjround).
T + mi 1%. mi WI (1)
71' 'n2 r= m2172 (2)
These two equations contain three unknown
quantities iic , ii-r2 and TT . The third equation
is provided by the kinematic relationship
between the accelerations :
3. 3., I. -1. -3.,
wi - wo + w , w2 - wo w (3)
where ;V.. is th acceleration of the mass nt1
with respect to the pulley or elevator car.
Summing up termwise the left hand and the
right-hand sides of these kinematical equations, we get
41

+ = 2 IT,'
W1 W2 (4)

The simultaneous solution of Eqs. (1), (2) and (4) yields


-30
("11 1112)
wi
e+ 2 m2 wo
ml+ m2
Using this result in Eq. (3), we get,

1711 -
m2
--I. --I. -,. 2 m1 m2
(g -wo) and T- (wo
m + M2 M1 + M2

-I., M1 - M2
Using the results in Eq. (3) we get w = m2 wo)
M1 +
(b) obviously the force exerted by the pulley on the celing of the car
4 mi 4 -÷

F= - 2 T - (g wo)
m1 +m2
Note : one could also solve this problem in the frame of elevator car.

1.72 Let us write Newton's second law for both, bar 1 and body 2 in terms of projection having
taken the positive direction of x1 and x2 as shown in the figure and assuming that body 2
starts sliding, say, upward along the incline
T1-mlgsina= m1 w1 (1)

m2 g - T2 M2 W (2)
For the pulley, moving in vertical direction
from the equation Fx = m wx
2 T2 - = (me ) =0
(as mass of the pulley mP = 0)
Or T1= 2 T2 (3)
As the length of the threads are constant, the 1n2
kinematical relationship of accelerations
becomes
w = 2 wi (4)
Simultaneous solutions of all these equations yields :
m2
2 g (2-- SM a
m1

w= 2 g ( 2 ri - sin a)
(41 + 1)
( 4 -m2- + 1)
m1
As 1 > 1, w is directed vertically downward, and hence in vector form
4 + 1 2 e(2 - sin a)
w
42

1.73 Let us write Newton's second law for masses m1 and m2 and moving pully in vertical
direction along positive x - axis (Fig.) :
mig-T= miwa (1)

m2 g m2 wa (2)

T1- 2T = 0 (as m = 0)

or T1= 2 T (3)
Again using Newton's second law in projection
form for mass mo along positive x1 direction
(Fig.), we get
T1= mo wo (4)
The kinematical relationship between the
accelerations of masses gives in terms of
projection on the x - axis
wix + w2x = 2 Ivo (5)
Simultaneous solution of the obtained five equations yields :
[4 m1 m2 + mo (m1 - m2) ] g
w -
4 mi m2 + mo (mi + m2)
In vector form
[4 mi m2 + mo (mi - m2) ] r wi
= 4 mi m2 + mo (mi + m2)
1.74 As the thread is not tied with m, so if there were no friction between the thread and the
ball m, the tension in the thread would be zero and as a result both bodies will have free
fall motion. Obviously in the given problem it is the friction force exerted by the ball on
the thread, which becomes the tension in the thread. From the condition or language of
the problem wm> wn, and as both are directed downward so, relative acceleration of

M = wm - wn, and is directed downward. Kinematical equation for the ball in the frame of
rod in projection form along upward direction gives :
1
1= — (w - ) (1)
2 m w'" t2
NeWton's second law in projection form along T
vertically down direction for both, rod and ball
gives,
Mg-fr= M wm (2)

mg - fr mwm (3)
Multiplying Eq. (2) by m and Eq. (3) by M
and then subtracting Eq. (3) from (2) and after
using Eq. (1) we get
21Mm
(M - m) t2
43

1.75 Suppose, the ball goes up with accleration w1 and the rod comes down with the acceleration w2.
As the length of the thread is constant, 2 w1 = w2 (1)
From Newton's second law in projection form along vertically upward for the ball and
vertically downward for the rod respectively gives,
T—mg= mw1 (2)
and Mg—T'= Mw2 (3)
but T=2T (because pulley is massless) (4)
From Eqs. (1), (2), (3) and (4)
(2M— m) g (2 g
w1 m + 4M (in upward direction)
11 + 4
2 (2 — g
and w2 — (downwards)
+ 4)
From kinematical equation in projection form, we get
1 ( + )t2
1= —2 w1 w2

as, w1 and w2 are in the opposite direction.


Putting the values of w1 and w2, the sought
tune becomes
t= V 2 / (ri + 4) / 3 (2 — g = 1.4s
1.76 Using Newton's second law in projection form along x — axis for the body 1 and along
negative x — axis for the body 2 respectively, we get
m g — T1 = (1) `01111/11,11h
T2 -- M2 g = m2 w2 (2) T2 A
For the pulley lowering in downward direction
T2
from Newton's law along x axis,
T1 — 2 T2 = 0 (as pulley is mass less)
or, T1= 2 T2 (3)
As the length of the thread is constant so,
=
w2 2 wi (4)
The simultaneous solution of above equations yields,
2 (mi — 2m2) g 2 (Ti _ 2) m1
w2 (as 7= YI) (5)
4 m2 + m i +4 2
Obviously during the time interval in which the body 1 comes to the horizontal floor
covering the distance h, the body 2 moves upward the distance 2h. At the moment when
the body 2 is at the height 2h from the floor its velocity is given by the expression :
u2 [ 2 (ri — 2) g 2h 8 h (T) — 2) g
•2 IA,
2 (-h.) =
+4 11+4
After the body m1 touches the floor the thread becomes slack or the tension in the thread
zero, thus as a result body 2 is only under gravity for it's subsequent motion.


44

Owing to the velocity v2 at that moment or at the height 2it from the floor, the body 2
further goes up under gravity by the distance,

L 4h — 2)
= =
2g +4
Thus the sought maximum height attained by the body 2 :
-2)
4h670
If= 2h+h'= 2h+ (TI
(1) + 4) ri + 4
137 Let us draw free body diagram of each body, i.e. of rod A and of wedge B and also draw
the kinemetical diagram for accelerations, after analysing the directions of motion of
A and B. Kinematical relationship of accelarations is :

W
A (1)
tan a =
wB
Let us write Newton's second law for both bodies in terms of projections having taken
positive directions of y and x axes as shown in the figure.
mA g — cos a = mA wA (2)

and N sin a = mB wB (3)


Simultaneous solution of (1), (2) and (3) yields :
mA g sin a
WA 2 and
mA sin a + mB cot a cos a (1 + ri cot a)
W
A
W -
tan a = (tan a + cot a)

'
W8
Note : We may also solve this problem using conservation of mechanical energy instead of
Newton's second law.
1.78 Let us draw free body diagram of each body and fix the coordinate system, as shown in
the figure. After anAlysing the motion of M and m on the basis of force diagrams, let us
draw the kinematical diagram for accelerations (Fig.).

As the length of threads are constant so,


ds.m=dsm and as andd vm do not change their directions that why

I iiram = I iirm I = w (say) and


45

1wmM 1-

As wm = Wnim 1- Wm
so, from the triangle law of vector addition
=VGw
From the Eq. Fx m wx , for the wedge and block :
T—N= Mw,
and N mw
Now, from the Eq. Fy = mwy , for the block
mg — T — kN = mw
Simultaneous solution of Eqs. (2), (3) and (4) yields :
mg
w—
(lan + 2m + M)— (k + 2 +M/m)
Hence using Eq. (1)
g
I=
wm (2 + k +M/m)
1.79 Bodies 1 and 2 will remain at rest with repect to bar A for wmin 5 w 5 w , where wmmis
the sought minimum acceleration of the bar. Beyond these limits there will be a relative
motion between bar and the bodies. For 0 5 w 5 wmin, the tendency of body 1 in relation
to the bar A is to move towards right and is in the opposite sense for w a w . On the
basis of above argument the static friction on 2 by A is directed upward and on 1 by A is
directed towards left for the purpose of calculating wmin.
Let us write Newton's second law for bodies 1 and 2 in terms of projection along positive x —
axis (Fig.).
T—fri. mw or, fri = T—mw (1)
N2 mw (2)
As body 2 has no acceleration in vertical
direction, so
mg
fr2= —T (3)
46

From (4) and (5)


g(1 — k)
m (g — w) s mk (g + w) , or w
(1 + k)
g(1 — k)
Hence wmin (
1 +k)
1.80 On the basis of the initial argument of the solution of 1.79, the tendency of bar 2 with
respect to 1 will be to move up along the plane.
Let us fix (x — y) coordinate system in the frame of ground as shown in the figure.
From second law of motion in projection form along y and x axes :
mgcosa—N= mwsina

Or, N= m ( g cos a — w sin a ) (1)


m g sin ct+fr = mw cos a
or, fr= m(w cos a — g sin a ) (2)

but fr s kN, so from (1) and (2)


( w cos a — g sin a)sk(g cos a + w sin a ) or,
w( cos a —ksin a )sg(kcos a + sin a )

( cos a + sin a )
or, w s g
cos a — k sin a
So, the sought maximum acceleration of the
wedge
( k cos a + sin a ) g ( k cot a + 1) g
wma where cot a > k
X cos a — k sin a cot a — k
1.81 Let us draw the force diagram of each body, and on this basis we observe that the prism
moves towards right say with an acceleration w 1and the bar 2 of mass m.z.moves down
-30
=
the plane with respect to 1, say with acceleration w21 , then , w2 w21 wi (Fig.)
Let us write Newton's second law for both bodies in projection form along positive y2
and x1 axes as shown in the Fig.
m2 g cos ct — N = m2 W2 ( y2 ) M2 [

W
21 ( y2 ) y2 )
M2 [ 0 + Wi sin a I

or, m2 g cos a — N = m2 w1 sin a


and N Sill a = miwi
Solving (1) and (2), we get
m2 g sin a cos a g sin a cos a
• 2
mi 2

U./2
47

1.82 To analyse the kinematic relations between the bodies, sketch the force diagram of each
body as shown in the figure.
On the basis of force diagram, it is obvious that the wedge M will move towards right and
the block will move down along the wedge. As the length of the thread is constant, the
distance travelled by the block on the wedge must be equal to the distance travelled by the
wedge on the floor. Hence d snof - d sm. As i;°' and vmI' do not change their
directions and acceleration that's why 1;,„m t t 1,m and Tirm t t 17m. and wnim= wm= w
(say) and accordingly the diagram of kinematical dependence is shown in figure.

--, -' , --91.

As wn, - wrnm + wm, so from triangle law of vector addition.

wn, = 1/Wm2 1- Wmm - 2 wmm wm cos a = wV2(1 - cos a) (1)

From Fx = m wx , (for the wedge),

T. T cos a +Nsin a . Mw (2)


For the bar m let us fix ( x - y) coordinate system in the frame of ground Newton's law in
projection form along x and y axes (Fig)) gives
mg sina - T - m wn.(x). m {wnaf (x)+ wm (x)]

. m [wmat + w cos (a - a)]. m w (1 - cosa)m (3)

mgcos a-N= m wm(y)= m[w.m(y)+wm(y)1= m [ 0 + wsin a.] (4)

Solving the above Eqs. simultaneously, we get


m g sin a
w-
M+ 2m ( 1 - cos a )
Note : We can study the motion of the block m in the frame of wedge also, alternately we may
solve this problem using conservation of mechanical energy.

1.83 Let us sketch the diagram for the motion of the particle of mass m along the circle of
radius R and indicate x and y axis, as shown in the figure.
-I.
(a) For the particle, change in momentum Ap = my (- i) - m v (j )
-1.
so, I Ap I = 1/2- m v
and time taken in describing quarter of the circle,
4-8

R
At=
2v

Hence, <F > - I AP I mv 2{2- m v 2


At R/2v nR
(b) In this case
--30
pi= 0 and pf= mwtt(- i),
—30

so I Ap I m wt t

Hence, I < F >1 -


t m wt.
1.84 While moving in a loop, normal reaction exerted by the flyer on the loop at different
points and uncompensated weight if any contribute to the weight of flyer at those points.
(a) When the aircraft is at the lowermost point, Newton's second law of motion in projection
form Fn = m wn gives
2
N - mg - R

2
mv
or, N= mg + - 2 09 IN

(b) When it is at the upper most point, again


from F,, = m wn we get

my2
N" + mg - R

2
mv
N"= - m g = 0.71N

(c) When the aircraft is at the middle point of the loop, again from F„= m w,,

mv2
N' - 1.4 kN

The uncompensated weight is mg. Thus effective weight 1IN'2+ m2 g2 =1.56 kN acts
obliquely.
1.85 Let us depict the forces acting on the small sphere m, (at an arbitrary position when the
--0
thread makes an angle 0 from the vertical) and write equation F = m w via projection on
A A

the unit vectors ur and un. From Fr= m wt we have


dv
mg sin Dim —
dt
vdv _ vdv
m
I.

ds m 1 (-d0)
(as vertical is refrence line of angular position)
49

or vdv = - gl sin 0 d 0
Integrating both the sides :

v 0
fvdv= -glf sin0d0
o xr2
2
V
or, —= g 1 cos 0
2
2
V
Hence — = 2 g cos 0 = wn .(1)
1
(Eq. (1) can be easily obtained by the
conservation of mechanical energy).
From Fn = m wn

in V 2
T-mgcos0- 1 •

Using (1) we have


775,
T = 3 mg cos 0 (2)

Again from the Eq. Ft - m wt :

mg sin 0 = m wt or wt = g sin 0 (3)

Hence w = Tv. 2 + wt = V( g sin 0 ) 2 + ( 2g cos 0) 2 ( using 1 and 3 )

= g 1/1 + 3 cos 2 0
(b) Vertical component of velocity, vy = v sin 0

So, v 2g / cos 0 sin 2 0 (using 1)


in
Y2=V2s 20=

2 d (COS 0 sin 2 0)
For maximum v, or Vy , = 0
Y d0

which yields cos 0 = Ty-

v--r mg
Therefore from (2) T = 3 m g - --- = 3

(c) We have w = wt ut + wn un thus wy = wo)+ wn(y)

But in accordance with the problem wy = 0


So, Wo ) + Wn(y) 312 0

or, g sin 0 sin 0 ÷ 2g cos 2 0 (- COS 0) = 0

or, cos 0 = Ts- or, 0 = 54.7°


SO
1.86 The ball has only normal acceleration at the lowest position and only tangential acceleration
at any of the extreme position. Let v be the speed of the ball at its lowest position and 1
be the length of the thread, then according to the problem

2
V
— (1)
sin cc
ill
i g
where a is the maximum deflection angle

From Newton's law in projection form : Ft= mwt

dv
— mg sin 0 . mv
1d0
or, —glsin0d0. vdv
On integrating both the sides within their limits.

Or,
2
V = 2g1 (1 — cos a) (2)
Note : Eq. (2) can easily be obtained by the conservation of mechanical energy of the ball
in the uniform field of gravity.
From Eqs. (1) and (2) with 0 =a
2g1(1 — cos a) = lg cos a
or, cos a = 2- so a = 53°
3 '
1.87 Let us depict the forces acting on the body A (which are the force of gravity mga nd the
normal reaction N ) and write equation F = mw via projection on the unit vectors
A A
lit and un (Fig.)

From F t. mwt
dv
mg sin 0 = m—
dt
vdv vdv
—m .m
ds Rd0
or, gR sin 0 d 0 = vdv
Integrating both side for obtaining v (0)
0 v

R sin 0d0 - fvdv


N
fg
(i)
v2
mg cos 0 —N — m—R ing (2)
At the moment the body loses contact with the surface, N — 0 and therefore the Eq. (2)
becomes
v2= gR cos 0 (3)
51

where v and 0 correspond to the moment when the body loses contact with the surface.

Solving Eqs. (1) and (3) we obtain cos 0 = 3 or, 0 = cos -1(2/3) and v =V2,11?/3
1.88 At first draw the free body diagram of the device as, shown. The forces, acting on the
sleeve are it's weight, acting vertically downward, spring force, along the length of the
spring and normal reaction by the rod, perpendicular to its length.

Let F be the spring force, and Al be the elongation.


From, Fn = mwn :

Nsin 0 +F cos 0 = m to2r (1)


where r cos 0 = + Al).
Similarly from F1= mw,

N cos 0- F sin 0= 0 or, N= F sin 0/cos 0 (2)


From (1) and (2)
F (sin 0/cos 0)•sin 0 + F cos 0 m co2 r
= m w2 (10 + A1)/cos
On putting F = K A 1,
KA/sin20+KA/cos20= moi2(10+A1) on
solving, we get,
10
Al= m (.02 2
K — M 0)- (K/M CO2 — 1)

and it is independent of the direction of rotation.


1.89 According to the question, the cyclist moves along the circular path and the centripetal
force is provided by the frictional force. Thus from the equation F,1= m wn
2 2
Jr m = — v V
or kmg M

2
r
or ko (1 — )g = v — or v2= ko (r - r2 /R) g (1)
R
r2
d(r -
For v., we should have =0
dr
2r
or, 1-
R = v0, so r = R/2
Hence vmax= Nrro TR
1.90 As initial velocity is zero thus
2 (1)
V as 2 wt s

As wt > 0 the speed of the car increases with time or distance. Till the moment, sliding
starts, the static friction provides the required centripetal acceleration to the car.

Thus fr = mw, but fr s king


52

So,

Hence
v (k2 g2 w21 R
max ti
v2 1

V
r
so, from Eqn. (1), the sought distance s — -11 = 60m.
2 wt 2 w tr
1.91 Since the car follows a curve, so the maximum velocity at which it can ride without sliding

2
at the point of minimum radius of curvature is the sought velocity and obviously in this
case the static friction between the car and the road is limiting.

Hence from the equation Fn = mw


m v2
king or v s 1/1/tg

so v =3/1c17-W . (1)
We know that, radius of curvature for a curve at any point (x, y) is given as,
R [ 1 + (dy/dx)2 13/2
(2)
(d2Y) M1'2
For the given curve,
a d2y. -a x
dx cos (—x) and = —2 sin
a a dX2 a
Substituting this value in (2) we get,
2/ 2 (x/a) j
[1 + (a a ) cos2 3/2
R
(a/a2) sin (x/a)
X It
For the minimum R,
and therefore, corresponding radius of curvature
a2
Rmin. (3)
a
Hence from (1) and (2)
vmax - a V kg/a
1.92 The sought tensile stress acts on each element of the chain. Hence divide the chain into
small, similar elements so that each element may be assumed as a particle. We consider
one such element of mass dm, which subtends angle d a at the centre. The chain moves
along a circle of known radius R with a known angular speed w and certain forces act on
it. We have to find one of these forces.

From Newton's second law in projection form, Fx = mwx we get

2 T sin (da/2) - dN cos 0 = dm w2 R


and from Fz = mwz we get
dArsin0 = gdm
Then putting dm = mda/2 n and sin (da/2) = da/2 and solving, we get,
m ((o2R +gcot0)
T=
2n
53

cl-mg-
1.93 Let, us consider a small element of the thread and draw free body diagram for this element.
(a) Applying Newton's second law of motion in projection form, Fn. mwn for this element, (T +
dT) sin (d 0/2) + T sin (d 0/2) — d N = dm (o2 R = 0
Or, 2T sin (d 0/2) = dN, [negelecting the term(dT sin d 0/2) ]

in 0 0
or, TdO= dN, as s =d (1)
2
Also, dfr= kdN= (T+dT)—T= dT (2)

In this case Q = n so,


T2
Or, or, In — = kn (3)
Ti.
T2 1
So, k= -- = _ In 11 0
T1 It

T2 M2 g m2
as = — = rio
g
Ti I. m1 m1
m2
(b) When — az ri, Which is greater than Tio, the blocks will move with same value of
ml
acceleration. (say w) and clearly m2 moves downward. From Newton's second law in
projection form (downward for m2 and upward for m1) we get :

M2 g — T2 = M2 W (4)

and T1 — mi g = mi w (5)
54 -•

T2
Also (6)
Ti = 71°
Simultaneous solution of Eqs. (4), (5) and (6) yields :

(m2 - 110 mi ) g (11 - lio ) m2


Ti
w- g as — = )
(m2+10mi) 01+10) m1
1.94 The force with which the cylinder wall acts
on the particle will provide centripetal force
necessary for the motion of the particle, and
since there is no acceleration acting in the
horizontal direction, horizontal component of
the velocity will remain constant througout
the motion.

So vx . vo cos a
Using, F„ = m wn , for the particle of mass m,
2 2 2
mV M Vo cos a
N- x - ,
R R
which is the required normal force.
1.95 Obviously the radius vector describing the position of the particle relative to the origin of
coordinate is
r= xt+yi= asincotti - bcoscotj
Differentiating twice with respect the time :
2 --D. -3.
-3. r
w = —,, - - (02 (a sinoui+bcoscotj)= -(027.4b (1)
d t`
-)'
Thus F= mw= -mw 2rr
1.96 (a) We have AFA. f ldt
t
= f m rdt = m et (1)
0 -3.-3.
2 (vo.g)
(b) Using the solution of problem 1.28 (b), the total time of motion, t - 2
g
Hence using t = -c in (1)
I
I V =mgt

1.97 From the equation of the given time dependence force F = a t(t-t) at t= t, the force
vanishes,
-t

(a) Thus Ap=p= f Fdt


0
55

--* , ,, 3
or, p= f atut—t j at n
6
0
--• 3
-I. -• --o at
but p. my so v.
6m
(b) Again from the equation F miir

at(t —t)-m--
dt
or, iibttt —t2)dt. mdr
Integrating within the limits for irtt ),
I V

f
2
Gilt T - t ) dt = mf dr
o
--, 2
Or, V is 5+( X t2 t3 ) a t ( "C t
m 2 3 m 2 3

a t2 ( t )
Thus vi• — L for t s t
m 2 3
Hence distance covered during the time interval t - t,
I

S IMI f vdt
0
T
a t2

-f m
t
— (
0 t
d
t

On integrating, a
T
--) — F0 4
2

m
1
2
my - 70- cos cot + C, (where C is integration constant)
H ce, r-
e
When n
4
t - 0, v
- 0, so
C-
—F0 m
cos w
m o) cot + F0
m co

F0
As 1 cos cot s 1 so, v - ( 1 — cos cot )
m co
56

Thus s= f v dt
o
Fo t Fo sin cot F0
= — 2 = in cot )
m co m (0 PI w2 ( Wt — s

(Figure in the answer sheet).


—*
1.99 According to the problem, the force acting on the particle of mass m is, F = Fo cos wt

dr —0 • Fo
So, m -- = Fo cos cot or d v —. — cos cot dt
dt m
Integrating, within the limits.
■II.

v -.. I
, Fo --I. F0
f d v = -77-1 cos wt dt or v — — sin cot
m co
0 0
It is clear from equation (1), that after starting at t — 0, the particle comes to rest fro
7r
the first time at t = co
—3. F0
From Eq. (1), v = I vl = — sin wt for t s —7t (2)
M0) 0)
Thus during the time interval t = 31/co, the sought distance
xlw
2F
s = F° f sin cot dt =
mw m 0)2
0
From Eq. (1)
F0
v .333 as I sin cot I s 1
max M 0)
—1. --, d r -,
1.100 (a) From the problem F = — ry so m —= - ry
dt
dv
Thus m—= - [ as dv ti v]
dt ry
dv r
or, - — dt
v m
r
On integrating In v=-- t+C
m
But at t = 0, v = vo, so, C = In vo
r
or, v r
In — = — —t or, v= vo e me r
vo m
Thus for t --, co, v = 0
dv —r
(b) We have m — = —ry so d v = — ds
dt m
57

Integrating within the given limits to obtain v (s):

Vo
Or, dv
MI f - m (1)
f

d
s

o
r r

a
m

V
o

0
P
i

V
Thus for v 0, s = stow=
r
m dv dv -r,
(c) Let we have - r v or — —at
v v m
vin
dv vo
or, J v =-7-1 dt , or, In —t
m 1V° m
0 0

1/71 ) — M ln( m In n
So t
r r
Now, average velocity over this time interval,
—Inn

rt
voe-7; dt
f v dt dt
<V>
1.101 According to the problem vo(rl 1)
dv
—m In ri In

dv
m dt = -kv2 or, n12""-kdt v

Integrating, withing the limits,


v2
k (vo - v )
f dt or, t m (1)
m vo v
0 o
To fine. the value of k, rewrite
dv dv k
my =-kv2 or, — - —as
ds v
On integrating

ffL. J.c_f ds
J V Mel
0 0
m vo
So, k = — In — (2)
h v
Putting the value of k from (2) in (1), we get
h(vo-v)
t=
Vo
v In —

r
58

1.102 From Newton's second law for the bar in projection from, Fx = m wx along x direction
we get
mg sin a - kmg cos a = mw

Hence, the maximum velocity will be at the distance, x .. tan a/a


Putting this value of x in (1) the maximum velocity,

Al g sin a tan a
V MB

max a
1.103 Since, the applied force is proportional to the time and the frictional force also exists, the
motion does not start just after applying the force. The body starts its motion when F
equals the limiting friction.

Let the motion start after time to , then


mg
F = ato = kmg or, to - k
a
So, for t = s to, the body remains at rest and for t > to obviously
mdv
- a (t - to) or, in dv = a (t - to) dt
dt
Integrating, and noting v = 0 at t = to , we have for t > to
v r

a 12
f m dv = a f (t - to) dt or v= (t - t
2m °'
0 to
I

Thus s= 5 v dt = - 712- f ( t - to)2 dt = (t-


to )3 2 om
Co
59

1.104 While going upward; from Newton's second law in vertical direction :
v dv kv v dv
m — - ( mg + 2 ) or
kv 2 - ds
ds
m

g
At the maximum height h, the speed v = 0, so
0
vdv ds
g + ( kv 2/flt )
0 0

Integrating and solving, we get,


kv2
(1+h= (1)
2k In mg
When the body falls downward, the net force acting on the body in downward direction
equals ( mg k v 2 ),
Hence net acceleration, in downward direction, according to second law of motion
v dv kv 2 v dv
ds = g- --or, kv2 ds
a
a in
v'

f V dV 112 f d s
Thus kv 2/
2/m
o
Integrating and putting the value of h from (1), we get,

v'= Vo 111+kvo2/mg.

1.105 Let us fix x - y co-ordinate system to the given plane, taking x-axis in the direction along
which the force vector was oriented at the moment t = 0, then the fundamental equation
of dynamics expressed via the projection on x and y-axes gives,
d vx
F cos t m (1)
dt
dv
and Fsincot= d (2)

(a) Using the condition v(0) - 0, we obtain vv = sin w t (3)


m

and

F
V (1 -coswt) (4)
Y mw

Hence, v
1I v2 + v 2x
M 2 )1
60

(b) It is seen from this that the velocity v


turns into zero after the time interval A t,
which can be found from the relation,
At
co — = 1E. Consequentely,
2 F
the sought distance, is
At 0 =wt
8F
s v dt
771 0) 2
•••
••••

f v dt
Average velocity, < v > =
0 X

2n/au
fdt
2F ._ co t , / ,, 4F
So, <v> = f — sm — ) at / k Z 31 0) ) —
in 0) 2 /I M 0)
0

1.106 The acceleration of the disc along the plane is determined by the projection of the force
of gravity on this plane Fx. mg sin a and the friction force fr . kmg cos a. In our case
k = tan a and therefore
fr= Fx mg sin a,
Let us find the projection of the acceleration on
the derection of the tangent to the trajectory and
on the x-axis :
m = Fx cos cp — fr = mg sin a, ( cos q) — 1 )

m = — fr cos cp = mg sin a ( 1— cos cp )


It is seen fromthis that wt = — wx, which means
that the velocity v and its projection vx differ
only by a constant value C which does not
change with time, i.e.
v = vx + C,
where vx = v cos cp. The constant C is found from the initial condition v = v0, whence

C = v0 since (Ft — —initially. Finally we obtain


2
v= vo ( 1 + cos tp ).
In the cource of time p 0 and v v0/2. (Motion then is unaccelerated.)
1.107 Let us consider an element of length ds at an angle p from the vertical diameter. As the
speed of this element is zero at initial instant of time, it's centripetal acceleration is zero,
and hence, d N — ds cos p = 0, where k is the linear mass density of the chain Let
T and T + dT be the tension at the upper and the lower ends of ds. we have from,
Ft = m wt
61

If we sum the above equation for all elements, the


term f dT = 0 because there is no tension at the
free ends, so
1/R

k gR f sin cp d cp = Xwt f ds =XI wt


o

Hence wt = ( 1 — cos
R
As wn = a at initial moment

, ei ,
So, w = I w 1=
' (I — cosR1
1.108 In the problem, we require the velocity of the body, realtive to the sphere, which itself
moves with an acceleration wo in horizontal direction (say towards left). Hence it is advisible
to solve the problem in the frame of sphere (non-inertial frame).

At an arbitary moment, when the body is at an angle 0 with the vertical, we sketch the
force diagram for the body and write the second law of motion in projection form Fn =
mwn

mv 2
or, mg cos 0 — N — mwo sin 0 = R (1)

At the break off point, N = 0, 0 = 00 and let


v = voso the Eq. (1) becomes,
2
V0
(2)
—r e g cos 00 - wo sin 00 From,
Ft = mwt
v dv
v dv
mg sin 0 - mwo cos 0 - m
ds 21 m R d 0
or, v dv = R ( g sin 0 + wo cos 0 ) d 0
v0 o0

Integrating, f v dv = 5 R (g sine + w0 cos0) d 0


o o
2
V0= g(1 — cos00) + w0 sin 00 (3)
2R
Note that the Eq. (3) can also be obtained by the work-energy theorem A = A T (in the
frame of sphere)
1
therefore, mgR (1 — cos 00) + mwo R sin 00 = .f mvo2
[here mwo ) sin 00 is the work done by the pseudoforce (— mi.)]
2
V0
Or, —2if = g ( 1 - cos 00) + wo sin 00
62

Hence

1.109 This is not central force problem unless the path is a circle about the said point. Rather
here Ft (tangential force) vanishes. Thus equation of motion becomes,

vt = v0= constant

2
MVO A
and,
r2 for r = ro
r 22

We can consider the latter equation as the equilibrium under two forces. When the motion
is perturbed, we write r= r0+ x and the net force acting on the particle is,

2 2 2
A mvo —A ( nx ) m vo x m vo
— + = 1— + 1— )= — (1—n )x
( r0 + x)n To+ x r; ro To ( ro r0 2

2
mv„
This is opposite to the displacement x, if n < F (--r-1--is an outward directed centrifugul

- A
force while -A is th8 inward directed external force).
r
1.110 There are two forces on the sleeve, the weight F1 and the centrifugal force F2. We resolve

both forces into tangential and normal component then the net downward tangential force on
the sleeve is,
2
mg sin() (1 —
co R cos 0)
g
This vanishes for 0,= 0 and for

0 = 00 = cos -1 (--g--, w hic h is re al if


(0.4R

2
2 2 W
R
(.0 R > g. If w R < g, then 1 — cos 0
g
is always positive for small values of 0 and
hence the net tangential force near 0 = 0 0'
opposes any displacement away from it.
0 = 0 is then stable.
2
2 R
If w R > g, 1 — (° cos() is negative for small

0 near 0 = 0 and 0=0 is then unstable.

However 0 = 00 is stable because the force tends to bring the sleeve near the equilibrium
position 0 = 00.

If w2R = g, the two positions coincide and becomes a stableequilibrium point.


63

1.111 De f i n e the axes as s h o wn wi t h z along the local vertical, x d u e e a s t a n d y d u e n o r t h . ( We

assum e w e are in the northern hem i sphere). Th e n the Co r i o l i s force has the c o mp o n e n t s .
--11.,

Fc0r = - 2 m (cTi x il

:-• ---, 70
= 2 mw [ vy cos0 - vz sin0) t - vx cos0 j + vx cos0 k 1 = 2mw(vy cos() — vz sin0) t
since vx is s ma l l wh e n the direction in wh i c h de gun is fired is due north. Th u s the

equation of mo t i o n (neglecting centrifugal forces) are

x = 2 mw (v sing) - vz coscp), y
Y • = 0 and z = —g It) _NO Trh
- gt
Integrating we get Y = v (constant), i =
Z -Ve r t j c al
and x= 203 v silly t + wg t 2 c o s c p
x_ E a S t

Finally,

2_ 1
X =
r2 singe + 3
cogt 3 CORI)

Now v » gt in the present case. so,

2 2
s
x= w v sincp
v
= w sing) I-

7 cm (to the east).

1.112 Th e disc exerts three forces wh i c h are mu t u a l l y perpendicular. Th e y are the reaction of

the w eight, mg, v e r t i c a l l y u p wa r d , the Co r i o l i s force 2 mv ' w perpendicular to the plane of

the vertical and along the d i a me t e r , and mw 2 r outward along the dia me t e r . Th e resultant

force is,

1.113 Th e sleeve is free to slide along the rod A.B. Th u s only the centrifugal force acts on it.

Th e equation is,

dr
my = mw2 r wh e r e v =
at

dv d 1 2
B ut I., = v
dr = dr 2 v

1 22
SO, 1
V2 = —2 w r + constant

2 2 2
Or, V = Vo+ C.0 r2
vo being the initial velocity w hen r = 0 . Th e Co r i o l i s force is then,

2m col/v02 + co2 r2 = 2=1)2 r 111 + vo2/(o2 r2

= 2.83 N on putting the values.


64

1.114 The disc OBAC is rotating with angular


velocity w about the axis 00' passing through
the edge point 0. The equation of motion in
rotating frame is,

mw = F + mco2 R+2mv xw= F+Fy

where Fin is the resultant inertial forc,.., kpseudo


force) which is the vector sum of centrifugal and
Coriolis forces.
(a) At A, Fin vanishes. Thus 0 = — 2mco2 R n + 2mt, co nA

where n is the inward drawn unit vector to the centre from the point in question,here A.
Thus, wg (OR
►2
_V,2 _V (02 R.
so,
p R
(b) At B Fin= mw2 OC + mw2 BC

its magnitude is mw2 V4R2 — r2 , where r = OB.


1.115 The equation of motion in the rotating coordinate system is,
mw = F + mco2 R + 2m x (1.

Now, ' v R e 4+ R sin()


--a ,
and w w' cos 0 er — w sin 0 ee

e,, 4
1 ev
= 0 R0 R sin 0 ip
2m F'
co cos 0 — co sin A 0

= Fr°((oR sin20 4)) + UDR sin 0 cos 0 ip coR 0 cos 0 4


Now on the sphere,
i7= (— R 62 - R sin2 0 cp2) + (R
6. — R sin cos cp. 2) + (R sin
04). + 2R cos 0 c.p )
Thus the equation of motion are,
m (— R 62 R sin2 0 c.p 2 ) = N — mg cos 0 + mw2 R sin2 0 + 2mw R sin2 0 4
m (R O. — R sin 0 cos 0 cp 2 ) = mg sin 0 + R sin 0 cos 0 + 2mco R sin 0 cos 0 c.p m (R
sin 0 cp.. + 2R cos 0 ci)) = — 2 mai R 0 cos 0
From the third equation, we get, q) = —
A result that is easy to understant by considering the motion in non-rotating frame. The
we get,
eliminating 'p
mR 62 = mg cos 0 — N m R
0. = mg sin 0
Integrating the last equation,
1 •2
- m R 0 = mg (1 — cos 0)
2
65

H e nce
N= (3 - 2 co s 0) mg
So t he b o d y m u st fl y o ff fo r 0 = 0 0 = co s - 1 e x a ct l y as if th e sphere w ere n o nr o t a t i n g

N o w, at t h is po i nt Fc 1 = c e n tr i fu g a l fo r ce =
mw2 R si n 0 0 =
mco2 R
Fror V 0)2 R2 02 0 + (0)2 R2)2
cost sine0 x 2m

_
5 (0)2 R + 0)2 R2 x _4 x 2g R I +
V
x 2m m c o2

2
9 3R 3

3 co R
nl y th e

1.116
(a) Wh e n th e tr a in is m o v in g al o ng a mer i dian o C o r i ol is fo r ce h as a lat e r al

co m po ne nt and i ts m a g ni t u d e (see t he p r ev i o u s p r ob l e m) i s,

2 m co v co s 0 = 2m o) si n k

(He r e w e have p ut R v)

S o , Fi a t erd = 2 x 2 00 0
x
1 0
3
x 8 676
2 7t

, x
5 40 0 0 Irs
u 3 6 00 2

= 3 7 7 kN , (w e w r it e k fo r th e l a ti tu
de)

(b ) T he res u l ta n t of th e i n ert i al fo r ce s a c t i ng

o n t he trai n is ,

+ (mo)2 R sin 0 co s 0 + 2m
_ 2„ R
R 0 cos 0 e:
s i n 0 co s 0 Ci9 )
For

(mo)2 R sin2 + 2m co R s in 2 Ci) )

Th i s v a n is h e s i f = 0, - co

Th u s
v = v

cp
e = " R s in 0 - - R co s k

(4) v
' 2

We w r it e fo r t he l at it u d e here)

Th u s t h
e tr a i n m u st m o v e fr o m th e east t o w es t al o n g t he 60th p ar a ll el w it h a s pee d ,

1 -4
as
6

2
co

R cos A. = — x —21( x 10 4 8 64
x 6 -3 7 x
1 0 1 1 5 8 m /s 4 1 7 km / h r

1.117 We g o to th e
e
q u at i o n g ive n in 1 1 1 1 H ere vy = 0 so w e can ta k e y = 0, t hus w e get fo r

t he m o tio n in t he x plane

co vz cos 0
x = - 2

and
= - g
1 2
I n te gr a t i ng , z = -
—gt
2

= g cos cpt 2
3/2
3
= —
2h

x=
1
1

- co g c os cp t g cos cp
So

3 3

2 co h

cos cp /27ji
3

t to th e e a s t o f
Th e r e i s t h us a displ a ce m e n

3 x
2 7t

-8- 6 4 x 5 00 x 1 x
\F0- 0

2 6 cm
66

1.3 Laws of Conservation of Energy, Momentum and Angular Momentum.

1.118 As A s constant so the sought work done

A= (7:2b-

--10
or, A= (3 i+4j )•[(2i-3j )-(i+2j )].. (3 i+4j )•(i-5j 17J
1.119 Differentating v (s) with respect to time

dy a ds a _r- a2
we — CIVS w
dt 2 Ifs dt 2 is- 2

(As locomotive is in unidrectional motion)

ma2
Hence force acting on the locomotive F=mw=
2

Let, at v = 0 at t = 0 then the distance covered during the first t seconds


, 2
1 2 a 2 aa t 2
S= wt = —t =
z 2 2
4
ma2 0721.2) m aat 2
Hence the sought work, A = Fs -
2 4 8

1.120 We have

1 2 2 2 2 as2
T- —my = as or, v
2
Differentating Eq. (1) with respect to time

2 as
2 vw = —
4 as v or, Wr =
r
m

Hence net acceleration of the particle

Hence the sought force, F = mw = 2as \/1 + (s/R)2

1.121 Let F makes an angle 0 with the horizontal at any instant of time (Fig.). Newton's second
law in projection form along the direction of the force, gives :

F = kmg cos 0 + mg sin 0 (because there is no


acceleration of the body.)

As F tt d rthe differential work done by the force F ,

dA=I•dr" Fds, (where ds

= long ds (cos 0) + mg ds sin 0 =


kmg dx + mg dy.
1

Hence, A long dx + mg J dy

= lcmg 1 + mgh= mg(k 1+ h).


67

1.122 Let s be the distance covered by the disc along the incline, from the Eq. of increment of
M.E. of the disc in the field of gravity : AT + AU = Afr
0 + (- mgs sin a) = - lcmg cos a s - lang 1
k
I
or, s
sin a - k cos a (1)
Hence the sought work
Afr = - kmg [s cos a + / ]

k 1mg
Afr = [Using the Eqn. (1)]
1 - k cot a
On puting the values Afros -0.05 J
1.123 Let x be the compression in the spring when the bar m2 is about to shift. Therefore at this
moment spring force on m2 is equal to the limiting friction between the bar m2 and horizontal
floor. Hence
K.Xlm k m2 g [where K is the spring constant (say)] (1)
For the block m1 from work-energy theorem : A = AT = 0 for minimum force. (A here
indudes the work done in stretching the spring.)
so, Fx - - - long x = 0 or K - km g (2)
2 2 F
From (1) and (2),

F = kg (nt +n12)
1 •
1.124 From the initial condition of the problem the limiting fricition between the chain lying on
the horizontal table equals the weight of the over hanging part of the chain, i.e.
X rl lg = k (1 - lg (where X, is the linear N
mass density of the chain)

So, k= (1)
1 -II
fr
Let (at an arbitrary moment of time) the length . 'of
the chain on the table is x. So the net friction
force between the chain and the table, at this
moment :
kN= kkxg (2)
The differential work done by the friction forces :

dA = r. • dr= - f,. ds- (--1—)x dr


- kkxg (- dx) = kg (3)
1
(Note that here we have written ds = - dx., because ds is essentially a positive term and as
the length of the chain decreases with time, dx is negative)
Hence, the sought work done
0
1
( - )
A=f 1 1 1-1 1
x dx - (1 - i) rl = - 1.3 J
2
68

1125 The velocity of the body, t seconds after the begining of the motion becomes
v = vo + g t. Thy power developed by the gravity (m g) at that moment, is

P ( T;+ g2t) mg g -vo a) (1)


As me is a constant force, so the average power
A mg - Or
<P> = =
where 67 is the net displacement of the body during time of flight.
As, mei. di* so <P> = 0
2
v
1.126 We have Wn = — am at 2, or, v = wiT? t,

t is defined to start from the begining of motion from rest.


dv
So, IV iV
dt A
Instantaneous power, P = F • v m (wt A +w ) • KIT? tA ),
A A

(where ut and ut are unit vectors along the direction of tangent (velocity) and normal
respectively)
So, P = mw IfirK t = ma Rt
Hence the sought average power
t t

f P dt f ma Rt dt
0 0
t

dt
0
ma R t 2 ma Rt
Hence <P>
2t
2
1.127 Let the body m acquire the horizontal velocity vc, along positive x - axis at the point 0.
(a) Velocity of the body t seconds after the begining of the motion,
—1, —10
V ni Vo "I' w t . (v0-kgt) (1)

Instantaneous power P = F • 17. (- long ) • (vo - kg t) i = - long (vo - kgt )


From Eq. (1), the time of motion i = v0/ kg
Hence sought average power during the time of motion

king vc,
= - 2 W (On substitution) 2

dvx
- fang = mwx = mvx
dx
or, dvx = - kg dx = - ag x dx
69

To fmd v (x), let us integrate the above equation

J vxdvx. - agf x dx0 or, v2 vii-agx2 (1)


0
Now, P= F• i7= -maxg1/4-agx2 (2)
vo
For maximum power, dt (142 °x2 - kgx4 = 0 which yields x. Nr2TIg
Putting this value of x, in Eq. (2) we get,
1 2
P - — m vo licrg-
2
1.128 Centrifugal force of inertia is directed outward along radial line, thus the sought work
r2

A = f mw2 r dr = 12 mw2 - = 0.20 T (On substitution)

1.129 Since the springs are connected in series, the combination may be treated as a single spring
of spring constant.
K1 K2
K
+K2
K1
From the equation of increment of M.E., AT+AU..

1 2 1 K K2 2
0 + — K AI = A, or, A= M
2 1C 1 K2 )
1.130 First, let us fmd the total height of ascent. At the beginning and the end of the path of
velocity of the body is equal to zero, and therefore the increment of the kinetic energy of
the body is also equal to zero. On the other hand, in according with work-energy theorem
AT is equal to the algebraic sum of the works A performed by all the forces, i.e. by the
force F and gravity, over this path. However, since AT = 0 then A = 0. Taking into
account that the upward direction is assumed to coincide with the positive direction of the

y — axis, we can write

A= f + d r f (Fy - mg) dy
0 0
h

sr. mg f (1 - 2 ay) dy mgh (1 - ah) = 0.

whence h = 1/a.
The work performed by the force F over the first half of the ascent is
h/2 h/2

AF Fy dy = 2mg (1 - ay) dy = 3 mg/4a.

The corresponding increment of the potential energy is


A U= mgh/2 = mg/2a.
70

dU 2a b
1.131 From the equation Fr= - we get Fr= [- ---i- + --i.
dr r r
2a
(a) we have at r = r0, the particle is in equilibrium position. i.e. Fr is 0 so, ro =
b
To check, whether the position is steady (the position of stable equilibrium), we have to
satisfy
d2 U>
dr
0

d2U 6a _ 2b
We have -
dr2 r4 r3
2a
Putting the value of r = ill= 7, , we get
d2 U b4
- — (as a and b are positive constant)
dr2 8a3 '
d2U b2 n
So,
dr2 8a3 >
which indicates that the potential energy of the system is minimum, hence this position is
steady.
dU [- 2a +
(b) We have
Frm - dr 2. r3 r2
dF
For F, to be maximum,
drr= 0
a b3
SO r -L and then F ma
b r( x) INE '
27a
As Fr is negative, the force is attractive.
1.132 (a) We have
Fx= - au - 2ax and F
-au -2py
Y dy
So, F= 2axi-2 13yi and, F= 2ila2x2+f12y2 (1)

For a central force, r x F= 0


Here, rxF= (xi+y) )x(-2ctxT:213yr)
= -213xyic-2axy(r)• 0
Hence the force is not a central force.
(b) As U= ax2 +13y2
OU - au
So, = — = - 2 a x and F - f3 y.
ax Y ay

2 y2
So, F=111!+F;= 4a2x2+4 f3
According to the problem
F = 2 a2 x2 + 12 y2 = C (constant)
71

c2
Or, a2 X2 + 02 y2=
2

2 2 2
Or, x y C k (say) (2)
2+ 2 = 22
13 a 2a
Therefore the surfaces for which F is constant is an ellipse.
For an equipotential surface U is constant.
So, a x2 + 13 y2 = Co (constant)

2 2
x y CO
or, +
Vi3T. a 13 Ko (constant)
Hence the equipotential surface is also an ellipse.
k.133 Let us calculate the work performed by the forces of each field over the path from a
certain point 1 (x1, y1) to another certain point 2 (x2, y2)
x2

(i) d A . F cli:1 ay r d F"t aydx or, Ax a f y dx


x1

(ii) d A + byil.) • dit axcbc + bydy


X2 y2

Hence A= f a xdx + f bydy


xi yl
In the first case, the integral depends on the function of type y (x), i.e. on the shape of the
path. Consequently, the first field of force is not potential. In the second case, both the
integrals do not depend on the shape of the path. They are defined only by the coordinate of the
initial and final points of the path, therefore the second field of force is potential.

1.134 Let s be the sought distance, then from the equation of increment of M.E.
r
AT+ AU= Af

0- -a niq + (+ mg s sina) = - lang cos a s

v,2i
or, s / (sin a + k cos a)
2g

- km 4,
Hence Afr = -king cos as-
2 (k + tan a)

1.135 Velocity of the body at height h, vh = V 2g (H - h), horizontally (from the figure given in
the problem). Time taken in falling through the distance h.

t= (as initial vertical component of the velocity is zero.)

Now s = vh t = 1/2g (H + h) x = 114 (Hh - h2)


72

For sue, --- (Hh — h2) = 0, which yields h =


2
Putting this value of h in the expression obtained for s, we get,
=H
smax

1.136 To complete a smooth vertical track of radius R, the minimum height at which a particle
5
starts, must be equal to —R (one can proved it from energy conservation). Thus in our
2
problem body could not reach the upper most point of the vertical track of radius R/2. Let
the particle A leave the track at some point 0 with speed v (Fig.). Now from energy
conservation for the body A in the field of gravity :

mg [h — (1 + sin 0) 1= —1 mv2
2 , 2

Of, v2 = gh (1 — sin 0) (1)


From Newton's second laW for the particle at the
point 0; FR = mw, ,

my
N + mg sin 0 —
(h/2)
But, at the point 0 the normal reaction N = 0
eg
So, V2 = sin 0 (2)

2
From (3) and (4), sin 0 = — and v
3 3
After leaving the track at 0, the particle A comes in air and further goes up and at maximum
height of it's trajectory in air, it's velocity (say v') becomes horizontal (Fig.). Hence, the
sought velocity of A at this point.

1.137 Let, the point of suspension be shifted with velocity vA in the horizontal direction towards
left then in the rest frame of point of suspension the ball starts with same velocity horizontally
towards right. Let us work in this, frame. From Newton's second law in projection form
towards the point of suspension at the upper most point (say B) :

2 2
MVB MVB
mg (1)
mg + T = I or, T-
1
Condition required, to complete the vertical circle is that T2 0. But (2)

mv2 sw mg (21) +1 mv2 So, v2= v2 — 4 gl (3)


2 A 2 13 3 A
73

From (1), (2) and (3)


m (vd2i 4 gl)
T= - mg k 0 or, vA 2 Arcir
VB B
1 1:.--"'----.,,
1 \
Thus vA (min) .. 1157gi / vT \
I I 112.
From the equation Fn = mivn at point C I
I V 771g
1
1 <, C
mv 2 1 T• ;
c %
T= (4) ■ /
/
\ t . /
Again from energy conservation • ■
----
mv2 = 1 mv2 + mg1 VA (5)
2 A 2
From (4) and (5)
T= 3 mg

1.138 Since the tension is always perpendicular to the velocity vector, the work done by the
tension force will be zero. Hence, according to the work energy theorem, the kinetic energy
or velocity of the disc will remain constant during it's motion. Hence, the sought time

t = —,, where s is the total distance traversed by the small disc during it's motion.

Now, at an arbitary position (Fig.)


ds (10 - R 0) d 0,
1/R

So, s= f (4)- R 0) d 0
0
/26 i? /1 to
or, s=
R 2R2 2R
72

Hence, the required time, t=


2 R v0
It should be clearly understood that the only uncompensated force acting on the disc A in this
case is the tension T, of the thread. It is easy to see that there is no point here, relative to
which the moment of force T is invarible in the process of motion. Hence conservation of
angular momentum is not applicable here.

1.139 Suppose that Al is the elongation of the nibbler cord. Then from energy conservation,
Mir AUd = 0 (as AT= 0)

or, - mg (1 + Al) +1- Al2 .• 0

2
or, 2 K Al - mg Al - mg I = 0
74

2 K
mg ± V (mg) + 4 x -„, mg/ K 271A 2 K/
Or, A1= L x = 1. + V i t -
K 2 K mg
2x —
2

Since the value of 1 + -4c-i- is certainly greater than 1, hence negative sign is avoided.
mg
1 4.1/1 4. 2 id
So, = 2"8.
mg
1.140 When the thread PA is burnt, obviously the speed of the bars will be equal at any instant
of time until it breaks off. Let v be the speed of each block and 0 be the angle, which
the elongated spring makes with the vertical at the moment, when the bar A breaks off the
plane. At this stage the elongation in the spring.

Al= 10 sec 0 — /0 = /0 (sec 0 — 1) (1)

Since the problem is concerned with position and there are no forces other than conservative
forces, the mechanical energy of the system (both bars + spring) in the field of gravity is
conserved, i.e. AT + AU = 0
2 141v2 +
So, K to 0 - 1)2 - mglo tan 0 = 0 (2)
2 2 °
From Newton's second law in projection form
along vertical direction :
mg = N + K 10 (sec 0 — 1) cos 0 (5e40-9
But, at the moment of break off, N 6 0.
Hence, K 10 (sec 0 — 1) cos 0 = mg
K - mg
Or, COS 0 - (3)
K /0
5mg III
Taking K simultaneous solution\of (2)- and (3) *Ids :
0
11-179gT
v= = 1.7 m/s.
32

1.141 Obviously the elongation in the cord, Al = 10 (sec 0 — 1), at the moment the sliding first

starts and at the moment horizontal projection of spring force equals the limiting friction.

So, K 1Al sin 0 = kN (1)


(where K1 is the elastic constant). KAI,
From Newton's law in projection form along
vertical direction :

K1Alcos0+N= mg.

Or, N= mg - K1 Al cos 0 (2)


From (1) and (2),
Ki Al sin 0 = k (mg - Ki Al cos 0)
75

king
or K1 =
Al sin 0 + k Al cos 0
From the equation of the increment of
mechanical energy : AU + AT= Air

Or, 1 A 2
Ki /at 1.3 Air

kmg 2
Or,
=
2 Al (sin 0 + k cos 0) Air

kmg 10 (sec 0 - 1)
Thus A in c - 0.09 J (on substitution)
fr =
2 (s 0 k os 0)
1.142 Let the deformation in the spring be Al, when the rod AB has attained the angular velocity w.
From the second law of motion in projection form F.. mw. .
2 /
MOD to
K A/ = m (02 00 4- Ai) or, Al= 2
K - M(0
1
From the energy equation, A = mv2 + K Al2
' -2
1 2 2 1 2
= - MO (10 + Al) + - K Al
2
2 2
inc02 /02
_1__ .m(02 , . MC°2 10 I
= go /- + K
2 K - 171(.02 2 K — mw2
%

K 11 (1 + )
i m(02
On solving Aext where .. _7' ,
2 (1 - T)- K
1.143 We know that acceleration of centre of mass of the system is given by the expression.
M1 W1 + M2 W2
WC "I
MI + /112

Since - W2
W1

(M1 M2) W1
we = (1)
+ M2
Now from Newton's second law F mw, for the
bodies m1 and m2 reSpectively.
- 4,
"JO

T + m1 g= mi (2)

and T + m2 g M2 w2 Ma m2 w1 (3)

Solving (2) and (3)

(mi - m2)
W, = (4)
mi +
76

Thus from (1), (2) and (4),


2 --/P
(M 1 M2) g

1—V: 2
kin 1 4. M2)
1.144 As the closed system consisting two particles
mi and of m2 is initially at rest the C.M. of
the system will remain at rest. Further as
m2 as m1/2, the C.M. of the system divides the

line joining m1 and m2 at all the moments of


time in the ratio 1 : 2. In addition to it the total
linear momentum of the system at all the
times is zero. So, and therefore the
velocities of ml and m2 are also directed in
opposite sense. Bearing in mind all these thing, the
sought trajectory is as shown in the figure.
1.145 First of all, it is clear that the chain does not
move in the vertical direction during the
uniform rotation. This means that the vertical
component of the tension T balances gravity.
As for the horizontal component of the tension
T, it is constant in magnitude and permanently
directed toward the rotation axis. It follows from
this that the C.M. of the chain, the point C,
travels along horizontal circle of radius p (say).
Therefore we have,

T cos 0 = mg and T sin 0 = mo)2 p


g tan 0 mg_
Thus p 2 0.8 cm
0)

and T= 21g— - 5 N
cos 0

1.146 (a) Let us draw free body diagram and write Newton's
second law in terms of projection along vertical and
horizontal direction respectively.
77

2
c I
So, fr = mg sin a + o( cos a ) = 6N (3)
g

(b) For rolling, without sliding,


fr s kN

but, N = mg cos a - m co2 1 sin a

01) 2 1
mg (sin a + cos a s k ( mg cos a — m co 2 / sm. a ) [Using OA
g
Rearranging, we get,

m co 2 1 ( COS a, + k sin a ) s ( k mg cos a — mg sin a )

Thus co sVg(k—tana)/(1+ktana )1= 2 rad/s

1.147 (a) Total kinetic energy in frame K' is

-4' 2 1
T= lm ( r-V ) +-m (r-V )2
2 1 1 2 2 2
--a.
This is minimum with respect to variation in V, when
--3. —3.
0, i.e. mi ( v—'1 — V1 2 + m2 ( v2 — V) = 0
6V
--lb
---P M I Vi + M2 V2 _,,„
or V= .= v
m1+7712 c

Hence, it is the frame of C.M. in which kinetic energy of a system is minimum.

(b) Linear momentum of the particle 1 in the K' or C frame


c_...), . , ___,, _...
M1 M2 --3. -4.
-P1 - mi ( vi - vc) - ( v1- v2)
mi + M2

here, tx — M1 M2
or, Pi= t.i. ( vi - v2), w — reduced mass
mi + m2
-
Similarly, p — -)
= µ(

So, 1;71 = 1.7i; I = /5 = g v,,,, where,vrd = I vi. — v2 I (3)

Now the total kinetic energy of the system in the C frame is

T= 7.1 + 72 ma 1- + 1- = - 2 - -
2m
1 2sn2 g

Hence T= -} IA vZ/ = 1- IA I
1717— ii; 12
78

1.148 To f i n d t h e r e l a t i o n s h i p b e t we e n the values of the me c h a n i c a l energy of a syste m in the

K and C r e f e r e n c e f ra me s , let us begin wi t h the kinetic energy T of the system Th e

—a. mi • —1.
velocity of the i-th particle in the K frame may be represented as vi = vi + vc Now we

c a n wr i t e

Since in the C frame E mi v 0, the previous expression takes the form

2 •-•• 1 2
T=T+—
2
m v- c T+—
2
mV (since according to the problem
v c - V) (1)

Since the internal potential energy U of a system depends only on its configuration,

the magnitude U is the s a me in all refrence f r a me s . Ad d i n g U to the left and right

hand sides o f Eq . ( 1 ) , we obtain the sought relationship

1
2
E- E + — m V
2

1.149 As initially U = U = 0, so, E=


F r o m the solution of 1.147 (b)

1
T la —2 IV1 v21,

As
1' 7 1 172

1 m m2
Th u s (v2, + )
2 m i. + m2 J. 4

1.150 Ve l o c i t y of m asses m1 and m2 , after t seconds are respectively.

—10 ••••11■ --110


111 Vi+gt and v2 = v2+ g t
He n c e the f i n a l mo me n t u m of the s y s t e m,

10 I
= M 1 1 + M2 2 = M1 + m2 1 7 ; + ( M 1 + M2 )

:4 + m rt , (wh e r e , Ty ; m2 i , 7 + m2 a n d m mi + m2 )

--1) 2
An d radius vector, rc Vc t + —2 t

( m i t 1 2
1 + m2
+— t
(mi + m2) 2

—00

1 -"No 2 MI + m2 V2
= Vc, t + — gt , wh e r e vc,
2 M1 4. m2
79
1.151 After releasing the bar 2 acquires the velocity v2, obtained by the energy, conservation :

1 2 1 .2 1 / 7 7
—2 m2 v2 K or, 1,2 is X — (1)
M2

Thus the sought velocity of C.M.

0 + M2X V:12 xNrI7ciC


V IS

4- M2 ( M1 + M2 )

1.152 Let us consider both blocks and spring as the physical system. The centre of mass of the

system moves with acceleration a - towards right. Let us work in the frame of
m + 2
1 m
centre of mass. As this frame is a non-inertial frame (accelerated with respect to the
ground) we have to apply a pseudo force m1 a towards left on the block mi and m2 a
towards left on the block m2.
As the center of mass is at rest in this frame, the
blocks move in opposite directions and
come to instantaneous rest at some instant. The rn2 a
elongation of the spring will be maximum or m a
minimum at this instant. Assume that the block [1711 M --- f
m1 is displaced by the distance xi and the block
//////i///////////////
m2 through a distance x2 from the initial
positions.
From the energy equation in the frame of C.M.
AT+U-A,„, ,
(where Aca also includes the work done by the pseudo forces)
Here,
1
A T. 0, U. k (xi+ x2)2 and

( F — M2F) m1F (xl+x2)


W —ca X2 + x=
+
M1 + M2 M1 + M2 M1 m2

1 M1 (x1 + x2) F
Or, —k ( )2 + 2
2 x1 + x2 = m m

x + 2mF
So, X1 + 2 0 or, x 1 x2 ms
k (mi + m2)
2 mi F
Hence the maximum separation between the blocks equals : 10 +
k (ml + m2)

Obviously the minimum sepation corresponds to zero elongation and is equal to /0


1.153 (a) The initial compression in the spring Al must be such that after burning of the thread,
the upper cube rises to a height that produces a tension in the spring that is atleast equal to
the weight of the lower cube. Actually, the spring will first go from its compressed
80

state to its natural length and then get elongated beyond this natural length. Let 1 be the
maximum elongation produced under these circumstances.
Then
K 1 .. mg (1)
Now, from energy conservation,
2 1
2 K A/ i• mg (AI + I) + — 12 (2)
2
(Because at maximum elongation of the spring, the speed of upper cube becomes zero) From
(1) and (2),
2 2mg Al 3 m2 g2
Al — K2 ... 0 or, A/ - -3111g - —1711
K k ' K
3mg
Therefore, acceptable solution of Al equals K
(b) Let v the velocity of upper cube at the position (say, at C ) when the lower block
breaks off the floor, then from energy conservation.
1 2 1 2 2
MV ag —2 K (A/ — / )— mg (1 + Al)

(where I., mg/K and A 1 - 7 III-)


K
2
Or, 172 si .,.1 211._ (2)
2 K
mv + 0 v
- —Let, the C.M. of the system
At the position C, the velocity of C.M; vc. . 2m
(spring+ two cubes) further rises up to A yc2. B
Now, from energy conservation,

2 (2m) v2c ' (2m) g Arc 2


1
Ay_ vc. v2 . 4 mg
Or, ' a
2g 8g K
But, uptil position C, the C.M. of the system has
already elevated by,
(A/ + /)m + 0 4 mg
AYciz' 2m = K
Hence, the net, displacement of the C.M. of the
system, in Upward direction
8 mg
A Yc= AYcl"Yc2 1` K
1.154 Due to .ejection of mass from a moving system (which moves due to inertia) in a direction
perpendicular to it, the velocity of moving system does not change. The momentum change
being adjusted by the forces on the rails. Hence in our problem velocities of buggies
change only due to the entrance of the man coming from the other buggy. From the
81

Solving (1) and (2), we get


My
v1=1
m and vv2 hf m
As 1 7: t i7and 2tt

- My
So, v and v 2 a z of- no
m)
1 -

1.155 From momentum conservation, for the system "rear buggy with man"
(M + m) i(7. m( + R) + M R (1)
From momentum conservation, for the system (front buggy + man coming from rear buggy)
M ir;+ (ti+ v;4 ) (M + m) y4;

M
So, V = + m v;4)
F Al+M M+M
Putting—the value of frfroomm (1), we get

—0 --• mM —0
yr vn U
+ my,

1.156 (i) Let 2 be the velocity of the buggy after both man jump off simultaneously. For the
closed system (two men + buggy), from the conservation of linear momentum,
M + 2m (ir + -

- 2/m1.
or, (1)
v1 M+ 2m
(ii) Let v be the velocity of buggy with man, when one man jump off the buggy. For the
closed system (buggy with one man + other man) from the conservation of linear
momentum :
0- (M + m)17-'1+ m (2)
LetiT;be the sought velocity of the buggy when the second man jump off the buggy; then from
conservation of linear momentum of the system (buggy + one man) :

(M + m) M +m + ) (3)

Solving equations (2) and (3) we get

--p m (2M+ 3m) IT'


v2= (M + m)(M + 2m) (4)
From (1) and (4)
V2 rri
71- >1
+
1 2 (M + m)
Hence y2> v1
1.157 The descending part of the chain is in free fall, it has speed v Vi-ift at the instant, all
its points have descended a distance y. The length of the chain which lands on the floor
during the differential time interval dt following this instant is vdt.
82

For the incoming chain element on the floor :


From dpy = F,, dt (where y - axis is directed down) Y
0- (A. vdt) v = F dt
Y
Or -kv2= - 2 kgy
FY =
Hence, the force exerted on the falling chain
equals X, v2 and is directed upward. Therefore
from third law the force exerted by the falling
chain on the table at the same instant of
time becomes X v2 and is directed downward.
Since a length of chain of weight (Xyg) already lies on the table the total force on the
f oor is (24g) + (Ayg) . (3A.yg) or the weight of a length 3y of chain.
l
1.158 Velocity of the ball, with which it hits the slab, v = V2 7:1 I
v 1
After first impact, v' = ev (upward) but according to the problem v' = — , so e = (1)
1 11
and momentum, imparted to the slab,
- mv - (-mv' ). mv (1 + e)
Similarly, velocity of the ball after second impact,
2
v"= ev' = e v

And momentum imparted - m (v',+ v" ) . m (1 + e) ev


Again, momentum imparted during third impact,
- m (1 + e) e2 v, and so on,
Hence, net momentum, imparted = my (1 + e) + mve (1 + e) + mve2 (1 + e) + ...

= mv (1 + e) (1 + e + e2 + ...)
(1 + e)
(from summation of G.P.)
I. my (1 - e)

(1 + 1)
11
- Vigil 1 - m 1,12W1 / (r) + 1) / (Ti - 1) (Using Eq. 1)
(1 -;1-1-

= 0.2 kg m/s. (On substitution)


1.159 (a) Since the resistance of water is negligibly small, the resultant of all external forces
acting on the system "a man and a raft" is equal to zero. This means that the position of the
C.M. of the given system does not change in the process of motion.
i.e. Fc ..... constant or, ATc*.. 0 i.e. 2 „Aria. 0

or, m (AF,:m + Arm ) + M bil' = 0

r
Thus, m (T+ + TT + M /4:: 0, or, I=- m
m+M
(b) As net external force on "man-raft" system is equal to zero, therefore the momentum of
this system does not change,
So, 0- m [ i7' (r) + 172" (t) ] + M -V; (t)
83

1.159 (a) Since the resistance of water is negligibly small, the resultant of all external forces
acting on the system "a man and a raft" is equal to hero. This means that the position of the
C.M. of the given system does not change in the process of motion.

i.e. r-c* = constant or, Art = 0 i.e. mi Air= 0


or, m -„* + Ar;) + M 6,4 ="0

Thus, +/)+Mi= 0, or, 1= — mr


m+M
(b) As net external force on "man-raft" system is equal to zero, therefore the momentum of
this system does not change,
So, 0= m [ (t) + i727(t) ] + MIT; (t)

m
or, i;+(t) (1)
2
M M
As i;* (t) or 4 (0 is along horizontal direction, thus the sought force on the raft
Md (t) Mm d T7' (t)
dt m+M dt
Note : we may get the result of part (a), if we integrate Eq. (1) over the time of motion of man
or raft.

1.160 In the refrence frame fixed to the pulley axis


the location of C.M. of the given system is
described by the radius vector
M AT; + (M — m) A 7;4m_ + m Arm
A i"Z4.
2M

But A F:f =

A = A r: m-m)÷,in
and m _ + A 77,41_ )

m T"
Thus A rc 2m

Note : one may also solve this problem using momentum conservation.
1.161 Velocity of cannon as well as that of shell equals 1/2 gl sin a down the inclined plane
taken as the positive x — axis. From the linear impulse momentum theorem in projection
form along x — axis for the system (connon + shell) i.e. Apx- Fx At :
p cos a — M V 2 gl sin a = Mg sin a A t (as mass of the shell is neligible)

p cos a — MV 2 gl sin
or, At= a•
Mg sin a
1.162 From conservation of momentum, for the system (bullet + body) along the initial direction
of bullet
M
VO
M Vo &IC (M + V, or, v=
m +M
84

1.163 When the disc breaks off the body M, its velocity towards right (along x-axis) equals the
velocity of the body M, and let the disc's velocity/in upward direction (along y-axis) at
that moment be v'
Y
From conservation of momentum, along x-axis for the system (disc + body)
or v' my
my - (m + M) Vx x = m +m (1)

And from energy conservation, for the same system in the field of gravity :
1 r2
1 MV2 = 1 (M + M) Vr2 + - v + mgh' ,
2 2 x 2 Y
where h' is the height of break off point from initial level. So,
2 1
2 2
V 2
1 MV = - (M + M) m v + mgh' , using (1)
2 (M + m) + 2 ,Y

r2 2 MV2
Or, V = V - — 2 gh'
(M + M)

Also, if h" is the height of the disc, from the break-off point,
then, v'2 = 2 gh"
Y

M V2
So, 2g (h" + h' ) - y2 —
(M + m)
Hence, the total height, raised from the initial level
M V2
= h' + h" —
2g (M + m)

1.164 (a) When the disc slides and comes to a plank, it has a velocity equal to v = Nriiii. Due
to friction between the disc and the plank the disc slows down and after some time the disc
moves in one piece with the plank with velocity v' (say).
From the momentum conservation for the system (disc + plank) along horizontal towards
right :
My
mv = (m + M) v' or v' —
m +M
Now from the equation of the increment of total mechanical energy of a system :
1
f (M + m) 1/2 — —1 mv2 = Air
2
1
or, — 1
2 (M + m) MV2on + m)2 2 mv2 =4-' A,fr
2
1 v2 [ 7m72
SO,
2 M+m—mim Afri

MM
Hence, Air = — (m .4. Al) gh = — Rgh

(where IA, = = reduced mass


M
m

+ M
85

(b) We look at the problem from a frame in which the hill is moving (together with the disc
on it) to the right with speed u. Then in this frame the speed of the disc when it just gets onto
the plank is, by the law of addition of velocities, v = u +11F. Similarly the common speed
of the plank and the disc when they move together is

V = +
V-2-111-.
m+M
1 -2 1 2
Then as above ; 1
ii. 2 (m +M);2 - i rrtv - iMu
fr
1 12 m2 1 , 1
2171
se - (m + M) u + u V2gh + m) 20} - -(m + Al) it- - m 2u V2gh - mgh
2 m+M 2
(m + 2 2
We see that ;fi, is independent of u and is in fact just - µ g h as in (a). Thus the result
obtained does not depend on the choice of reference frame.
Do note however that it will be in correct to apply "conservation of enegy" formula in the
frame in which the hill is moving. The energy carried by the hill is not negligible in this
frame. See also the next problem.

1.165 In a frame moving relative to the earth, one has to include the kinetic energy of the earth
as well as earth's acceleration to be able to apply conservation of energy to the problem.
In a reference frame falling to the earth with velocity v0, the stone is initially going up with
velocity vo and so is the earth. The final velocity of the stone is 0 = vo - gt and

that of the earth is vo + m—m gt (M is the mass of the earth), from Newton's third law,

where t = time of fall. From conservation of energy


2
1 mv2 + 1M 2+ mgh = -1 m )
-2 0 -2 v0 M vo + — vo
2
1 2 m2
Hence - = mgh
m+—
2 v°
Negecting mi1 in comparison with 1, we get
M
2
vo = 2gh or vo = V2gh
m
The point is this in earth's rest frame the effect of earth's accleration is of order — and
can be neglected but in a frame moving with respect to the earth the effect of earth's
acceleration must be kept because it is of order one (i.e. large).

1.166 From conservation of momentum, for the closed system "both colliding particles"
mivi m2 7 = (ml + m2) v
-• -•
117+ m2v2 1 (3 i - 2j ) + 2 (4 j - 6 k ) v
or, m = + 2j 4k
m2
mi+ 3

Hence Ii7,1= V 1 + 4 + 16 m/s = 4.6 m/s


86

1.167 For perfectly inelastic collision, in the C.M. frame, fmal kinetic energy of the colliding
system (both spheres) becomes zero. Hence initial kinetic energy of the system in C.M.
frame completely turns into the internal energy (Q) of the formed body. Hence
1 2

Q II v2 1
Now from energy conservation A T— —Q— 2 v212 ,
2
In lab frame the same result is obtained as
(mii717+ 2 1
AT— v1 m2
2 mt I 2 + 12 12
2 mi + m2
1 ••• •10 4, 2
— 11,1 —21
2

1.168 (a) Let the initial and final velocities of ml and m2 are 4, u2 and v v2 respectively.
Then from conservation of momentum along horizontal and vertical directions, we get :
mi ui = m2 v2 cos 0 (1)
and mivi = m2v2 sin 0 (2)
Squaring (1) and (2) and then adding them,
2 2 2 2 2
m2v2 = m 1 Vi

m 1) 2 (i m1
or, 1 1 • p„._
—2

2
"12 — M1
or, (4)
M1 + M2

So, fraction of kinetic energy lost by the particle 1,


1 1 2
-2mi vi 2
2m1u1 l 1 Vi
— 2
1 2 ul
—2 ml u1
2 mi.
ws 1
m +m2 m2 —ml [Using (4)]
1 mi + m2
(b) When the collision occurs head on,
+ m2v2
= m1v1
and from conservation of kinetic energy,
87

1 2 1 2 1
—m, = — + —m11/12
2 2 ... 2

„ 2
1 _2 1 i. (U1 — V)" I
ml T m2 M [Using (5)]
M2

M1
Or, V (1 + -"11 se U1 ( — 1)
1
M2 m2

Vi (mi/m2— 1)
Or, (6)
u1 (1 + mi / m2)

Fraction of kinetic energy, lost


2 2
Vi (mi — m2 ) 4 mi m2
= 1 — ---f = 1— [Using (6)]
mo2
M1 + M2) (mi
u1

1.169 (a) When the particles fly apart in opposite direction with equal velocities (say v), then
from conservatin of momentum,
mi u + 0 = (m2 — mi) v (1)
and from conservation of kinetic energy,

1 2 1 2 1 v2
— U —M +—m
2 1 2 1 2 2

or, m1 u2 = (m1 + m2) v2 (2)


From Eq. (1) and (2),
2
/7/2 U1
m + m2)
(m2 — M1)2
0=60°
2
Or, M —3m =0
2

1 1
Hence - as m2 r0
m 3
2
(b) When they fly apart symmetrically relative to the initial motion direction with the
angle of divergence 0 = 600,

From conservation of momentum, along horizontal and vertical direction,


m1 u1 = mi v1 cos (0/2) + m2 v2 cos (0/2) (1)

and m1 v1 sin (0/2) = m2 v2 sin (0/2)


or, mi v1 mi. M2 V2 (2)

Now, from conservation of kinetic energy,

1 2 11 1 m v2 + v2
2 mi ui + —2 2 in2 2 (3)

From (1) and (2),

m
mi u1= cos (0/2) mi vi +( .i. vin2 m2) = 2 mi vi. cos (0/2)
88

2 „2
2 M2 M21 vl
4 mi cos2 (0/2)1,1 = mi +
M2

m1
or, 4 cost (0/2) = 1 +
m2

ml 2
Or, = 4 cos — — 1
M2 2

m1
and putting the value of 0, we get, = 2
m2

1.170 If (v1x, vly ) are the instantaneous velocity components of the incident ball and

( )
2y are the velocity components of the struck ball at the same moment, then since
there are no external impulsive forces (i.e. other than the mutual interaction of the balls)

We have u sins = v ), = 0
ly v2
m u cosa = m +mV
2x

The impulsive force of mutual interaction satisfies

d F d,
= kv
d
t m dt 2x
( F is along the x axis as
the balls are smooth. Thus
Y component of momentum
is not
1 2 1 2
transferred.) Since loss of
K.E. is stored as
deformation energy D, we
have
D = — m u -—171 v)i.1 e —--MV
2 2 2 2
1 2 1 2 1 2
so — mu cos ta — -rnv --rnva
2 2 lx 2

1 2
MU
m2u2cos2a — m2V1x — ( mucosa— mvia) 2
2m

1 2 2
2m ucosavi.— 2m 2V1x = m ( vaucosa — v1x2)
2m ]
2
u2cos2a u cosa _
=m
4 2
We see that D is maximum when

u cosa
= V a
2

mu2cos2a
and D
4
max 1
Then 2 = 2a c o s t= 4-
—mu 2

On substiuting a = 45°
89

1.171 From the conservation of linear momentum of the shell just before and after its fragmentation
3i 17,7 + + 17; (1)

where i-,7,172' and IT; are the velocities of its fragments.

vi + v 22 .4. v23
From the energy conservation 3rp/2 = (2)
14.'10 —IP —IP —IP —IP —IP
Now Vi Or Vi.c 21, Vc Vi
(3)
where v 17.'= velocity of the C.M. of the fragments the velocity of the shell. Obviously
in the C.M. frame the linear momentum of a system is equal to zero, so

V1 + V2 + V3 NE 0 (4)

Using (3) and (4) in (2), we get


2 2 —II' 2 --11. 21 * =a* 2 2 •••• 2 =.11
3riv- = (v + ) + (v + v2 ) + (v - - v2 ) 3v2 + 2v + 2v 2+2 vi . 2

Or, 2i-; + 21; 112 COSO + 22 + 3 (1 - ri)v2 = 0 (5)


If we have had used 2 = - vi - 3 then Eq. 5 were contain v3 instead of i72 and so on.
The problem being symmetrical we can look for the maximum of any one. Obviously it
will be the same for each.

6 (1 -)
So, v2 s v 2 or v20.) - V2 (ri - 1)
4 - cos 0

Hence v2 .) = I i7+ 2( = v + V2 - 1) v v (1 + 1/2 (Ti — 1) )= 1 km/s


(

Thus owing to the symmetry


v10.) = v2 .) = v3(.) = v (1 + V2(ri - 1)) = 1 lun/s
(
1.172 Since, the collision is head on, the particle 1 will continue moving along the same line
as before the collision, but there will be a change in the magnitude of it's velocity vector. Let it
starts moving with velocity v1 and particle 2 with v2 after collision, then from the
conservation of momentum
mu- mvi + mv2 or, u= + V2 (1)
And from the condition, given,
2 2
1 2 (1 MV + MV
2 mu — 2 1 2 2 V2 + V2
1 2
= 1—
1 2

2mu
- 2 2
or, vi 2 v 2 = (1 - 1) u (2)
From (1) and (2),
v2i (u v1)2 . (1 _ u2

2
or, + U2 — 2 uvi. + v2i = (1 -
90

or, 2v2 - 2v u + u2 0

V4u2 - u2
So, 1/1 in La ±
4

= [ut
u2-T'Is2 I = lu(1
2 2
Positive sign gives the velocity of the 2nd particle which lies ahead. The negative sign is
correct for v1 .

1
So, v1= — u (1 - = 5 m/s will continue moving in the same direction.

Note that v1= 0 if = 0 as it must.

1.173 Since, no external impulsive force is effective on the system "M + m", its total momentum
along any direction will remain conserved.
So from px = const.
U
MU Mv1 cos 0 or, V1 = — (1)
M cos 0
and from py= const

mv2 = My, sin 0 or, v2 = — sin 0 = u tan 0, [using (1)]


m
Final kinetic energy of the system
2 1
T r .f mv2 + Mvi

1
And initial kinetic energy of the system= 2- mu2

Tc T.
So, % change - x 100

2 2
1 2 2 1 177 U 1 2
m u tan 0 + M - mu
2 m2 cost 0 2
x 100
1
12 mug

1 2 2 1 ni 2 1
— u tan 0 + — — 2 u sec 0 - — u2
2
2M 2 x 100
1 2
2u

(tan2 0 4: M sec2 0 -1 x 100

and putting the values of 0 and —111 we get % of change in kinetic energy= - 40 %
M
1.174 (a) Let the particles m1 and m2 move with velocities iv and i72' respectively. On the basis
of solution of problem 1.147 (b)
= IA Vr ei =
91

As TA-. 1
2

M1 m2
So, p Im IA1471
1. v2 where IA - m1 +

(b) Again from 1.147 (b)

tivei- 11r-17;12
So, 1 (v2 + v22)
2 1
1.175 From conservation of momentum

, 2
► 2 =
SO (T27 P21
2P1 Pli CaS01 Pl P2 2
From conservation of energy
P12 2 r2 „ r2
F2
2m1 2m1 2m2
Eliminating p2' we get

,2 M2 2
0 an pi ( + cos01+p12
m1 — 2p 1'p1 - )
m1

This quadratic equation for p1' has a real solution in terms of p1 and cos 01 only if
mi

4 cos201 a 4(1 — --.2=


ml
m2
2
or sine 01 S 2
M1

m2 M2
or sin01s+ or sin 01
m1 m
This clearly implies (since only + sign makes sense) that
M2
sin °1 max " m •
1.176 From the symmetry of the problem, the velocity of the disc A will be directed either in
the initial direction or opposite to it just after the impact. Let the velocity of the disc A
after the collision be v' and be directed towards right after the collision. It is also clear
from the symmetry of problem that the discs B and C have equal speed (say v") in the
directions, shown. From the condition of the problem,

cos 0 2= so, sin 0 = 4:7712 / 2 (1)


d 2
For the three discs, system, from the conservation of linear momentum in the symmetry
direction (towards right)
my. 2m v" sin 0 + m V or, v= 2 v" sin 0 + (2)
92

From the definition of the coefficeint of restitution, we have for the discs A and B (or C)

v" — v' sin 0


e—
v sin 0 — 0

But e = 1, for perfectly elastic collision,

So, v sin 0 = v" — v' sin 0 (3)

{using (1)}

Hence we have,
, v 2 — 2)
v — (1
6 — ri2
Therefore, the disc A will recoil if II < VI and stop if ri =
Note : One can write the equations of momentum conservation along the direction per-
pendicular to the initial direction of disc A and the consevation of kinetic energy instead of
the equation of restitution.
1.177 (a) Let a molecule comes with velocity i77. to strike another stationary molecule and just
after collision their velocities become v 1 and v 2 respectively. As the mass of the each
molecule is same, conservation of linear momentum and conservation of kinetic energy for
the system (both molecules) respectively gives
--30
V1= V + V 2

2 2
and v1= v, + v' 2
From the property of vector addition it is obvious from the obtained Eqs. that
0
v 1 1v 2 or v 1 •v 2 =
a
(b) Due to the loss of kinetic energy in inelastic collision vi2 > va + v 2

SO, V1 • V 2 > 0 and therefore angle of divergence < 90°.


1.178 Suppose that at time- t, the rocket has the mass m and the velocity relative to the
reference frame, employed. Now consider the inertial frame moving with the velocity that the
rocket has at the given moment. In this reference frame, the momentum increament that the
rocket & ejected gas system acquires during time dt is,
4;4=. mdv+µdtu F dt
d —30
or, M = r - u
dt
Of, mw== F —
93

1.179 According to the question, F = 0 and IA, = - dm/dt so the equation for this system becomes,
dv dm
— m
dt dt
As d c7t 4, Tt so, m dv = - u dm.
Integrating within the limits :
m

I f ch, = f dm mo
u - — or = In —
mo m u m

mo
Thus, v = u In —
m
-• -• mo
As d u, so in vector form v u In
m

1.180 According to the question, F (external force) = 0


d i7 dm
So, m —u
dt dt
As
so, in scalar form, m dv = u dm
wdt dm
or,

Integrating within the limits for m (t)


m

wt dm v m
= or, -= - In
mo
mo
- (swim )
Hence, m = mo e

1.181 As is= 0, from the equation of dynamics of a body with variable mass;
-. m
m u or, dv U d — (1)
dt dt
Now Iriland Once must have I = vo d a (because vo is constant)
vihere d a is the angle by which the spaceship ,turns in time dt.
dm u dm
So, -u — vod a or, d a
m= vo m
m
- u f dm u In
or, a_
vo m vo mil
m0 m
94

dm
1.182 We have = - .t or, dm = - dt
dt

Integrating f dm = - µ f dt or, m= m 0 - µt

m0
As it= 0 so, from the equation of variable mass system :
,d dv „
(mo - tit) — dt = r or, — w = r kmo - Lit)
dt
-. t
v
dt
or, dill. If
J (mo - 110
0 0
F mo
Hence v — in
mo
1.183 Let the car be moving in a reference frame to which the hopper is fixed and at any instant
of time, let its mass be m and velocity
Then from the general equation, for variable mass system.
di; * -4.dm
m r+u
dt dt
We write the equation, for our system as,
dv -,dm
m— - v dt as, u v (1)
dt

d IT"
Thus the sought acceleration, 147.
2
dt Nx
mo +
mo

1.184 Let the length of the chain inside the smooth horizontal tube at an arbitrary instant is x.
From the equation,
--.dm
mw r + u
dt
95

as u = 0, F T t w, for the chain inside the tube

kxw= T where =m
1
Similarly for the overhanging part, T
0
lx;;F;C;;;C i:.
Thus mw= F

or khw=khg-T (2)
From (1) and (2),
dv
(x + h)w= khg or, (x + h)v— = hg
ds
v
Or, (x + h) v d = gh,

[As the length of the chain inside the tube decreases with time, ds = -thc.]
dr
or, vdv= -gh
x+h
V 0

Integrating, vdv. -gh

0
I
(1- h )
xa +h
2
V
or, -2-- gh In (Tl) or v = /\2gh In (-)
h

1.185 Force moment relative to point 0 ;

,
= — = 2 vt
dt
Let the angle between M and N ,
h
Ao. —
a= 45° at t = to

1)% )•(2 E070)


Then -
Vat + b2 to 2bto
T h 2 to3
b to
Va2 + 1,2 t4 t
0 2bt0 a2 + b2 t o4 gt
2 4 2 2 4
So, 2 b to = a +b to or, to = V (as to cannot be negative)

It is also obvious from the figure that the angle a is equal to 45° at the moment to,

when a= b to2 i.e. to = viTir, and Xir% 2 A - b.


mvo g t 2 cos a
Thus M (t) —
2
Thus angular momentum at maximum height
Vo sin a
i.e. at t — —
2 g
3
I) sin2 a cos a 37 kg — m2/s
2 2g

M(
Alternate :
M(0)= 0 so, rf(t)= f .17dt= f (ii(mr)
0 0

2
+— )
= [ o g x M 1 dt. (Txmi)—

1.187 (a) The disc experiences gravity, the force of


reaction of the horizontal surface, and the force
R of reaction of the wall at the moment of the
impact against it. The first two forces
counter-balance each other, leaving only the
force R. It's moment relative to any point of
the line along which the vector R acts or along
normal to the wall is equal to zero and therefore
the angular momentum of the disc relative to on
any of these points does not change in the given
process.
(b) During the course of collision with wall
the position of disc is same and is equal to
rod Obviously the increment in linear
A
momentum of the ball 2mv cos a n
A

Here, AM = rot; x Aii = 2 my 1 cos a n and directed normally emerging from the plane of figure
Thus I AMI= 2 my / cos a
1.188 (a) The ball is under the influence of forces T and m eat all the moments of time, while
moving along a horizontal circle. Obviously the vertical component of T balance m rand
97

so the net moment of these two about any point becoems zero. The horizontal component of T,
which provides the centripetal acceleration to ball is already directed toward the centre
(C) of the horizontal circle, thus its moment about the point C equals zero at all the
moments of time. Hence the net moment of the force acting on the ball about point C equals
zero and that's why the angular mommetum of the ball is conserved about the horizontal
circle.
(b) Let a be the angle which the thread forms
with the vertical.
Now from equation of particle dynamics : KM
T cos a - mg and T sin a - moi2 1 sin a

Hence on solving cos a - --g2-- (1)


-IP
/
As I M I is constant in magnitude so from figure.
I t> M I= 2M cos a Where

M- lMfI

rboxmv-10 I- mvl(as r60.1.17)


I

ThusI /Ti = 2 mv / cos a = 2 mu) / 2 sin a cos a

2 mgl. 2
V 1 - 2 ) (using 1).
t

1.189 During the free fall time t — , the reference point 0 moves in hoizontal direction

(say towards right) by the distance VT. In the translating frame as Vf (0)=0, so
• 7.
ANM = rff > (V)

Hence

1.190 The Coriolis force is. (2m 1.7*1 x (7p).


Here (7.iis along the z-axis (vertical). The moving disc is moving with velocity vo which is
constant. The motion is along the x-axis say. Then the Coriolis force is along y-axis and has
the magnitude 2m vo w. At time t, the distance of the centre of moving disc from
0 is vot (along x-axis). Thus the torque N due to the coriolis force is N-
2m v0 co•vot along the z-axis.
98

dM
Hence equating this to
dt
dM 2 2 2
dt = 2m vo cot or M = mo w + constant.
The constant is irrelevant and may be put equal to zero if the disc is originally set in
motion from the point 0.
This discussion is approximate. The Coriolis force will cause the disc to swerve from
straight line motion and thus cause deviation from the above formula which will be substantial for
large t.

1.191 If r = radial velocity of the particle then the total energy of the particle at any instant is
1 • M2 2
2 + 2inr2 + kr E (1)
im r

where the second term is the kinetic energy of angular motion about the centre 0. Then the

extreme values of r are determined by r = 0 and solving the resulting quadratic equation
_Er2 + 2 0
k(r2)2
2m
we get

E E2 —2Mnizic
r2 —
2k
From this we see that
E 41.21+ r22)
(2)
where r1 is the minimum distance from 0 and r2 is the maximum distance. Then
1 2
+ k( + r2 2)
2 2
2kr2
Hence, m=
V2

•Note : Eq. (1) can be derived from the standard expression for kinetic energy and angular
momentum in plane poler coordinates :
1 1
T=— 2
+ — m r2 0
2m 2 2

M = angular momentum = mr2 0


1.192 The swinging sphere experiences two forces : The gravitational force and the tension of
the thread. Now, it is clear from the condition, given in the problem, that the moment of
these forces about the vertical axis, passing through the point of suspension Nz = 0. Con-
sequently, the angular momentum /1/z of the sphere relative to the given axis (z) is constant.

Thus mvo (1 sin 0) = my 1 (1)


where m is the mass of the sphere and v is it s velocity in the position, when the thread
TE
forms an angle — with the vertical. Mechanical energy is also conserved, as the sphere is
2
99

under the influence if only one other force, i.e. tension, which does not perform any work, as it
is always perpendicular to the velocity.
1 1
So, — m + mg 1 cos 0 = mv (2)
2 v° —2 2
From (1) and (2), we get,
vo = V2g1/cos 0
1.193 Forces, acting on the mass m are shown m the figure. As 1-:". mg, the net torque of these
two forces about any fixed point must be equal to zero. Tension T, acting on the mass m
is a central force, which is always directed towards the centre 0. Hence the moment of
force T is also zero about the point 0 and therefore the angular momentum of the particle
m is conserved about 0.

Let, the angular velocity of the particle be co, when the separation between hole and
particle m is r, then from the conservation of momentum about the point 0, :
(
m w0 ro) ro m (c° r)

co r2o o
or c,0 -
, I
Now, from the second law of motion for m, 1 %
, 1 1
T = F = m co2r \
\ / r
i
Hence the sought tension;
\
,
, ,/
2 4 2 4
M C00 ro r M CO0 ro
F- =
r4 r3 p
1.194 On the given system the weight of the body m is the only force whose moment is effective
about the axis of pulley. Let us take the sense of C-ti of the pulley at an arbitrary instant
as the positive sense of axis of rotation (z-axis)

As Mz (0) = 0, so, AMZ .- Mz (t) - f Nzdt


t

So, Mz (t) = f mg R dt = mg Rt
0
1.195 Let the point of contact of sphere at initial
moment (t = 0) be at 0. At an arbitrary
moment, the forces acting on the sphere are
shown in the figure. We have normal reaction
Nr = mg sin a and both pass through same line

and the force of static friction passes through


the point 0, thus the moment about point 0
becomes zero. Hence mg sin a is the only force
which has effective torque about point 0, and is
given by IN I. mg R sin a normally
emerging from the plane of figure.

As ill (t = 0) = 0, so, AlI f = rf (t) = f Isidt


Hence, M (t) = Nt = mg R sin at
- -
100

1.1 96 Let position vectors of the particles of the system be Frand Fr with respect to the points
0 and 0' respectively. Then we have,
-0 -1.,
+ro (1)

where r04. is the radius vector of 0' with respect to 0.


Now, the angular momentum of the system relative to the point 0 can be written as follows;

E (i-rx 2 (Fr X /.37) E (7,7. [using (1)]

or, M= M + (ro x / , where, /5.= (2)

From (2), if the total linear momentum of the system, Fi= 0, then its angular momen-
tum does not depend on the choice of the point 0.
Note that in the C.M. frame, the system of particles, as a whole is at rest.

1.197 On the basis of solutio' n of problem 1.196, we have concluded that; "in the C.M. frame,
the angular momentum of system of particles is independent of the choice of the point,
relative to which it is determined" and in accordance with the problem, this is denoted
by M.

We denote the angular momentum of the system of particles, relative to the point 0, by
M. Since the internal and proper angular momentum in the C.M. frame, does not depend on
the choice of the point 0', this point may be taken coincident with the point 0 of the
K-frame, at a given moment of time. Then at that moment, the radius vectors of all the

particles, in both reference frames, are equal (r = in and the velocities are related by

the equation,
-10
vi= v , (1)
where IT: is the velocity of C.M. frame, relative to the K-frame. Consequently, we may
write,
m= mir. x = mi( ri x vi + vc)
Or, M= ft+ m(rx ), as 2 miri= mrc, where m= 2 mi.
or, M= 1t+ ( xm ) = ft+ ( xp)

1.198 From conservation of linear momentum along the direction of incident ball for the system
consists with colliding ball and phhere
my° = my' + —771 (1)
2 v1
where v' and v1 are the velocities of ball and sphere 1 respectively after collision. (Remember that
the collision is head on).
As the collision is perfectly elastic, from the definition of co-efficeint of restitution,

1= or, - v1 = - yo (2)
0 — vo
101

Solving (1) and (2), we get,

4 v0
(30
v1 3 ) directed towards right.

In the C.M. frame of spheres 1 and 2 (Fig.)

pl = -74 and 17):1=1;2I

Also,717c = - r42c. , thus la = 2 [1c x

74 , so, 14.2 [1 m / 2 4 vo AI
AS 717c. 2 2 3 n

(where n is the unit vector in the sense of ric x pi )

- m v0 / z
Hence M =
3
1.199 In the C.M. frame of the system (both the discs + spring), the linear momentum of the

discs are related by the relation, p1= -p2, at all the moments of time.

where, P 1= P2 313 P = vrel

And the total kinetic energy of the system,

1 2
T=— ti vr„
, [See solution of 1.147 (b)]
2
Bearing in mind that at the moment of maximum deformation of the spring, the projection of v
along the length of the spring becomes zero, i.e. vrej(x) = 0.

The conservation of mechanical energy of the considered system in the C.M. frame gives.

1 (m) 2 1 2 1 (M)
(1)
2 v°3rA 2KX +2 2 vrei 0)

Now from the conservation of angular momentum of the system about the C.M.,

1ro) (m ) . 2 (10 +X) m


v
2 2 ° 2 2 rel (Y)

-1
0 10 x x
or, = v0 1+— v0 (1 - — a s x « 10 (2)
vrei (Y) 00 + 10 10

Using (2) in (1), 1 /7/1/1 1--(1- 12 = K X2


2 ° To

2 )2
1 2 ., (. -2x+ X
or, — m vo 1- 1 K X2
2 / /2
0 0
2
11 vo X

or,
1 ,
g0
aK X2, [neglecting x2 I gi
2
M v°
x pg 0, thus x=
As
K
102

1.4 UNIVERSAL GRAVITATION


1.200 We have
Mv2 Y Mms Y ins
or r =
r r2 v2

V V V3
Th us o) II =
2 21

r yin,' v y Ms

(Here ms is the mass of the Sun.)

2-31Y ms 231 x 6.67 x 10 -11 x 1•97 x 103°


So T= = = 1.94 x 107 sec= 225 days.
V3 (34.9 x 103)3
(The answer is incorrectly written in terms of the planetary mass M)

1.201 For any planet

MM Y Ms
MR w2= Y s Or 03=
R2 R3

27 (
So, T= — - 2nR3/21V:Fin;
o)
3/2
Tj (Rj
(a) Thus —= -1
TE RE

R1
So ii- = (T. 1 / TE )2/3 = (12)2/3= 5 • 24.
E
2/3

(b)

So

where T= 12 years. ms= mass of the Sun.


Putting the values we get V, = 12.97 km/s

Acceleration =

= 215 x 10-4 km/s2


103

1.202 Semi-major axis= (r + R)/2


r +R
It is sufficient to consider the motion be along a circle of semi-major axis for T
2
does not depend on eccentricity.
3/2
r +R
2 it( 2
Hence T= = It V (r + R)3/2 y ms
V

(again ms is the mass of the Sun)

1.203 We can think of the body as moving in a very elongated orbit of maximum distance R
and minimum distance 0 so semi major axis = R/2. Hence if i is the time of fall then
2 3
( 2 ) . ( R/2 ) 2 2
or T = T /32
T R
or i = T/ 4V-2- = 365 / 4V-2- = 64.5 days.

1.204 T= 21tR 3/2 / V51Trirs


If the distances are scaled doWn, R3/2 decreases by a factor rj3/2and so does ms . Hence T
does not change.

M1 m2
1.205 The double star can be replaced by a single star of mass moving about the centre
mi +
of mass subjected to the force y m1m2 / r2. Then

3/2
2 21 r3/2 2 it r
T=
ffFif
mi m2 / nil m2 + M2
r3/2 3/2 T
So
2/3 vy m
2 31
or, r = (L) (yM)1/3 VryM (T/2302
2

1.206 (a) The gravitational potential due to m1 at the point of location of m2

r r

m1 "12
So, U21- M2 V2 °I -

Similarly U - Y m1 m2
104

Hence
nif
Y in2 • •
U12= U21 is U=
r
r x
'ix
(b) Choose the location of the point mass as the origin. Then the potential ertergy dU of an

element of mass dM = --dr of the rod in the field of the point mass is

M
dU = — ym —dx —
1 x

where x is the distance between the element and the point (Note that the rod and the point
mass are on a straight line.) If then a is the distance of the nearer end of the rod from the
point mass.

-- --- - -• DC
k >in
d x

a+►

mMi dr 1 )
U=— — = — ymT In( 1 + —
a a

The force of interaction is

aU
F=—
as

Minus sign means attraction.

1.207 As the planet is under central force (gravitational interaction), its angular momentum is

conserved about the Sun (which is situated at one of the focii of the ellipse)

So, m vt rl = m V2 r2 or, vi = (1)


ri
From the conservation of mechanical energy of the system (Sun + planet),

Y ms m 1 2 y ms m 1 2
+ at
2 v1 r2 + m v2
2
2
t Ms 1 2 r2
or, — + — v — = —(-1' 1 + 1 v2 [Using (1)]
2 2 r2i r2 2 2
rt

Thus, v2= V2 y ms ri / r2 (r 1 + r2) (2)

Hence M = m v2 r2 = m V2 y ms r1 r2 / (r1 + r2)


105

1.208 From the previous problem, if r1 , r2 are the maximum and minimum distances from the
sun to the planet and v1 , v2 are the corresponding velocities, then, say,

1 2 runs
E —mv —
2 2 r2
?Mins ri ymms ymms ymms
r — — [Using Eq. (2) of 1.207]
ri + r2 r2 2 ri + r2 2a

where 24 = major axis = r i + r2. The same result can also be obtained directly by writing

an equation analogous to Eq (1) of problem 1.191.


1 .2 M2 'Minis
E = —m r 2 + — —
2 2mr2 r
(Here M is angular momentum of the planet and m is its mass). For extreme position

r 0 and we get the quadratic


m2
Er2 + ymmsr — —2m = 0

The sum of the two roots of this equation are


ymms
+ r2 — E =2a

ymms
Thus E — constant
24

1.209 From the conservtion of angular momentum about the Sun.


m vo ro sin cc = mviri= m v2 r2 or, vi = v2 r2 = vo r2 sin a (1)
From conservation of mechanical energy,

1 2 If Ms M 1 2 I/ Ms M
MV am M Vt
ro 2 r1
2
vo y m3 v20 rt2,1 sin2 a y ms
or, —— = (Using 1)
2 ro 24 rt
2 y ms)
2 2 2
or, (v 0 — 1.0 ri + 2 y m3 r1 — vo2 ro sin a = 0

2 2 2
— 2 yms± 4y + 4 (v20 r20 sine a) vo 2 yroms)
So, 1 'a
2 (1702 — 2 YroMs)

2 n 2 / 2\
vo ro s i a 2 VO ro [ 1 ± V 1 — (2 — sin2 a I
— r
y ms \ro ms

(2 vo (2 —11)
ro y ms
where ri = 1/( /Y ms, ms is the mass of the Sun).
106

1.210 At the minimum separation with the Sun, the cosmic body's velocity is perpendicular to
e the sought minimum distance, from con-
its position vector relative to the Sun. If rnunb
servation of angular momentum about the Sun (C).
vol
mvol= mvrmin or, v = (1)
rmin

From conservation of mechanical energy of the system (sun + cosmic body),


1 2 Y ms m 1 2
MVn MV
2 nun 2

v2o Y ms vo
So, -- ... + 2r 2 (using 1)
2 rim

2 2 vo 2 2 0
or, Vo Train + 2y ms n

2 2 2 _yms±V,,2m2+ v412
— 2y ms ±1/4y 2 M 2 + 4V0 Vo s 0
So, rmin 2
21/02 Vo

Hence, taking positive root


mass m (say -
( point P) (Fig.).1As all the points of the band are
y 1
located at the distance 1 from the point
1.211 Suppose
P, so,
that thei sphere
n
has a radius
equal tos a. We
may imagine
that the/ sphere
is madev
up of i
concentric thin
spherical shells
)
(layers) with
{
radii ranging
1
from 0 to a,
1
and each1
spherical layer
is made+up of ac —Ya m
elementry
bands (rings).
(
Let us first
1
calculate
4
/ due
potential
to an y
elementry band
m
of a spherical
layer atsthe
point of location
)
of the point
2
(where mass of the (1)
band)

do
am-(d M (aasin0)(a )
47t a 2

= sin0 d0 (2)

2 2
And 12= a 4. r —2 arcos0 (3)

Differentiating Eq. (3), we get

ldl = ar sin() d() (4)

Hence using above equations

/y
2a (5)
a (4)— )di
107

Now integrating this Eq. over the whole spherical layer


F+ a
y dM
dcp..f
tar
r-a
-ydM
So d (6)
y- r
Equation (6) demonstrates that the potential produced by a thin uniform spherical layer
outside the layer is such as if the whole mass of the layer were concentrated at it's centre;
Hence the potential due to the sphere at point

f r (7)
r
This expression is similar to that of Eq. (6)
Hence thte sought potential energy of gravitational interaction of the particle m and the
sphere,

(b) Using the Eq., Gr


ar
M
- (using Eq. 7)

So G= - r and F= m YmAl r _ (8)


r3 r
1.212 (The problem has already a dear hint in the answer sheet of the problem book). Here we
adopt a different method.
Let m be the mass of the spherical layer, wich is
imagined to be made up of rings. At a point
inside the spherical layer at distance r from the
centre, the gravitational potential due to a ring
element of radius a equals,

dy ... dl (see Eq. (5) of solution of 1.211)

a+r

So, = fdy f dl -E-1 (1)


2ar a
a- r

Hence Gr = - 0
ar •
Hence gravitational field strength as well as field force becomes zero, inside a thin sphereical
layer.

1.213 One can imagine that the uniform hemisphere is made up of thin hemispherical layers of
radii ranging from 0 to R. Let us consider such a layer (Fig.). Potential at point 0, due
to this layer is,
108

= —ydm — —3yM rdr, where dm — 4itr 2 )


dq dr
r R (2/3 ) R 3 ( 2
(This is because all points of each hemispherical shell are equidistant from 0.)
R
3yM
Hence, = f dcp f rdr =
Rs' R

Hence, the work done by the gravitational field


force on the particle of mass m, to remove it to
infinity is given by the formula
A = mcp, since q) = 0 at infinity.
Hence the sought work,

Ao = — 37 mjil
2R
(The work done by the external agent is — A.)
and G
1.214 In the solution of problem 1.211, we have obtained q due to a uniform shpere, at
a distance r from it's centre outside it. We have from Eqs. (7) and (8) of 1.211,
yM
0:p = and G = 3 r (A)
r r
Accordance with the Eq. (1) of the solution of 1.212, potential due to a spherical shell of
radius a, at any point, inside it becomes

q) = = Const. and Gr.. — =0 (B)


a ar
For a point (say II) which lies inside the uniform solid sphere, the potential q at that point may
be represented as a sum.

CPinsid
e= CP1 + CP2
where (pi is the potential of a solid sphere having radius r and q)2 is the potential of the
layer of radii r and R. In accordance with equation (A)
y( M 4 nr3). _YM r2
q)1
r (4/3) 7( R 3 3 R3
produced by the layer (thick shell) is the same at all points inside it. The
The potential p2
potential q)2 is easiest to calculate, for the point positioned at the layer's centre. Using Eq.
(B)
R

dM
P2- f (R 2 — r
r — — 2 R3
r

M 2 (3 M ) 2
where d M — , 4 it r dr = r dr
(4/3) ic R - R
is the mass of a thin layer between the radii r and r + dr.
(yM)(3_r2)
Thus 12 (C)
(I) ide= cP1+ CP2 = R 1 1' R=
109

From the Eq.


ar
yM r
Gr. R3

Or G. -4.1-= -y3npr

M
(where p = A , is the density of the sphere) (D)
-t 3
- 7tR
3
The plots (p (r) and G (r) for a uniform sphere of radius R are shown in figure of answersheet.
Alternate : Like Gauss's theorem of electrostatics, one can derive Gauss's theorem for
G•
gravitation in the form dS.
-4
7t y
f
mu
wio
sed
.
Fo
r
cal
cul
ati
on
of
G
at
a
poi
nt
inside the sphere at a distance r from its centre, let us consider a Gaussian surface of
radius r, Then,
M) 3
G, 4;cr2= -47cy (—3 r or, Gr. - 1:2t311 r
R
4
Hence, G. -14 y - 7t pr p=
R." 3 _ co
(4/3) R 3 )
CO

So, Pc = f G, d r = f - Y-111 r dr +f
R3
_ T2
dr
Integrating and summing up, we get,
LA1(3_ 2
2R 2

And from Gauss's theorem for outside it :


G, 4 3t r 2 = - 47c y M or G,= M2
Thus q) f G, dr = -YM
r
1.215 Treating the cavity as negative mass of density - p in a uniform sphere density + p and
using the superposition principle, the sought field strength is :
-IP -10 -IP
G. G1+ G2

4 4
or G. - - t y p + (- p)
3i
(where i: and aarree the position vectors of
an orbitrary point P inside the cavity with
respect to centre of sphere and cavity
respectively.)
4 4
Thus G= -- 7typ( - r_) = - - nyp/
3 3
110
1.216 We partition the solid sphere into thin spherical layers and consider a layer of thickness
dr lying at a distance r from the centre of the ball. Each spherical layer presses on the
layers within it. The considered layer is attracted to the part of the sphere lying within it
(the outer part does not act on the layer). Hence for the considered layer

dp47cr 2= dF

3
Jtr p
(41gr2drp)
Or, dP43Ir2=
r2
(where p is the mean density of sphere)
4
or, dp= — n y p2 rdr
3

23t
Thus p= fdp= y p2 (R2 — r2 )
r

(The pressure.must vanish at r = R.)

3 -2 4 •
or, P- (1 — (r /R))yM2/ rcI R , Putting p = M/(4/3) nR 3
Putting r= 0, we have the pressure at sphere's centre, and treating it as the Earth where
mean density is equal to p= 5.5 x 103 kg/m3 and R= 64 x 1021cm
we have, p= 1.73 x 1011 Pa or 1.72x 106 atms.

1.217 (a) Since the potential at each point of a spherical surface (shell) is constant and is equal

to cp. — R, [as we have in Eq. (1) of solution of problem 1.212]

We obtain in accordance with the equation

U=
fdmrp= pf dm
2 c

_Y_Lt1_2
2R
(The factor is needed otherwise contribution of different mass elements is counted twice.)
2
(b) In this case the potential inside the sphere depends only on r (see Eq. (C) of the
solution of problem 1.214)

m(1— r23R2
32R
Here dm is the mass of an elementry spherical layer confined between the radii r
and r+dr:

dm= (4nr2drp). (4;7. )r2dr


- ▪
111

U= ill dm cp

R
2
3
if m-) r 2 dr {-
R3 2Rm 1 - 3rR 2
0
After integrating, we get

- 2
R

y ME
1.218 Let w = —3 = circular frequency of the satellite in the outer orbit,
r
Y ME ug circular frequency of the satellite in the inner orbit.
(130.1 3
(r - A r)

So, relative angular velocity = coo ± ou where - sign is to be taken when the satellites are
moving in the same sense and + sign if they are moving in opposite sense.
Hence, time between closest approaches
2n 2n 1 4.5 days (6 = 0)
wo 6° leyr- 3/2 3 A r 0.80 hour (6 = 2)
r ME °
where 6 is 0 in the first case and 2 in the second case.

yM 6.67 x 10-11 x 5.96 x 1024


1.219 wi = = 9.8 m/s2
(6.37 x 106)2

2 2
(27[ 2 x 22
2R n R 6.37 x 106 = 0.034 m/s2
(i)2 T (24 x 3600 x 7

Y Ms 6.67 x 10-11 x 1.97 x 103° 2


and = 5- 9'x 10-3 /s
6°3 R;, (149.50 x 106 x 103)2 m

Then wl : (02: ai3 = 1 : 0.0034: 0.0006

1.220 Let h be the sought height in the first case. so


99 yM
g=
100 + h)2

YM
112

-2
99 h
or
100 1+

From the statement of the problem, it is obvious that in this case h «


99 ( 2h R (6400
Thus - 1 - R or h = = 200 ) km = 32 km
100
In the other case if h' be the sought height, than
I.' -2 LI -2
g g(1+11 or 1 (1+1
2 R R
From the language of the problem, in this case h' is not very small in comparision with R.
Therefore in this case we cannot use the approximation adopted in the previous case.
2
hi
Here , (1 + —/e) =2 So , — = -1
R
As - ye sign is not acceptable
h' (Nri - 1) R (12- - 1) 6400 km = 2650 km

1.221 Let the mass of the body be m and let it go upto a height h.
From conservation of mechanical energy of the system
yMm 1 2 -yMm
+ m voo +0
R
2 (R + h)
Using YM - g, in above equation and on solving we get,

R v(2)'
h=
2gR-v(2)
1.222 Gravitational pull provides the required centripetal acceleration to the satelite. Thus if h
be the sought distance, we have
2
mv y Mm
so, or, (R + h) v2 = y M
(R + h) h)2

Or, Rv2+hv2= gR2, as g= YM


R2
R2-Rv2
Hence h g R BR - 11
v2

1.223 A satellite that hovers above the earth's equator and corotates with it moving from the
west to east with the diurnal angular velocity of the earth appears stationary to an observer
on the earth. It is called geostationary. For this calculation we may neglect the annual
motion of the earth as well as all other influences. Then, by Newton's law,

2
ffi !-71-30 M( 271 r
r2
113

where M = mass of the earth, T = 86400 seconds = period of daily rotation of the earth and r
= distance of the satellite from the centre of the earth. Then

r= 3 ym( T )2

Substitution of M = 5.96 x 1024 kg gives

r • . . 4.220 x 104 km
The instantaneous velocity with respect to an inertial frame fixed to the centre of the earth at
that moment will be

T
( r i . 3.07 km/s
-2 -31

and the acceleration will be the centripetal acceleration.


2

) r = 0.223 m/s2

1.224 We know from the previous problem that a satellite moving west to east at a distance
R- 2.00 x 104 km from the centre of the earth will be revolving round the earth with an
angular velocity faster than the earth's diurnal angualr velocity. Let
co = angular velocity of the satellite
221
WO = anuglar velocity of the earth. Then
T=
w 27t
- (‘)0 ' T
as the relative angular velocity with respect to earth. Now by Newton's law
ILI1
to2R
=
R2
2. 2n
So, M=
—+

Substitution gives
t
. 4 it R3
Y

Y 72
T
)
M= 6.27 x 1024 kg

l i LY
The velocity of the satellite in the inertial space fixed frame is east to west. With
1.225 R
respect to the Earth fixed frame, from the 1;7' = 171:- (i; x Fr the velocity is
2 7t R V a r"
v' T + R = 7.03 km/s

Here M is the mass of the earth and T is its period of rotation about its own axis.
114

2 nR
It would be — + , i f the satellite were moving from west to east.
T R
To fmd the acceleration we note the formula

m H—r-F+2n70-1-wx(7))+inco2R

Here F — 1-11.12-n- R and 1 iand x aPis directed towards the centre of the Earth.
R3

w ,. y_t _4. 2( 27ER + 1 / 7 31/ 2n-


Thus
R2 T R T- T
toward the earth's rotation axis

yM [2 nR ic 5 7 1
+2 ..,
R2 + T T + R z . 4.94 m/s2 on substitution.

1.226 From the well known relationship between the velocities of a particle w.r.t a space fixed
—•
frame (K) rotating frame (K ) v-v +(wxil

v 1= v— 2 nR

,
Thus kinetic energy of the satellite in the earth's frame
)
2
1 ,2 2n R)
/ al rtV i n' v T

Obviously when the satellite moves in opposite sense comared to the rotation of the Earth
its velocity, relative to the same frame would be

v2' v+ 2 1 R

And kinetic energy

1 12 l m (v+ 231R)2
T T2 -m v (2)
2 2= 2 T
From (1) and (2)
2
27tR)

T'
2 (3)
27LR)
V- T
Now from Newton's second law

yMm m v2

R2
r o = FR- (4)
R
Using (4) and (3) -
2
( vik + 2 n R )

T21
2 10, 1.27 nearly (Using Appendices)
Ti
— 2 xTR
115

1.227 For a satellite in a circular orbit about any massive body, the following relation holds
between kinetic, potential & total energy :

T.—E,U=2E (1)

Thus since total mechanical energy must decrease due to resistance of the cosmic dust, the
kintetic energy will increase and the satellite will 'fall', We see then, by work energy theorm

dT = — dE — dA
fr
adt dv
So, mvdv av2vdt or, 2
m Is v

Now from Netow's law at an arbitray radius r from the moon's centre.

v2 yM Ary:.L—1
or v=
r r2

(M is the mass of the moon.) Then

of = LM
R

where R= moon's radius. So

of

f dv a at
dt
n;
v1 0

T 1 1)
or, - 1)
a vi 1/1 a
Jul a VgR

where g is moon's gravity. The averaging implied by Eq. (1) (for noncircular orbits) makes the
result approximate.

1.228 From Newton's second law


m 2
yMm vo kn
or vo = 1.67 vs (1)
R2 R V R

From conservation of mechanical energy

1 2 yMm
--M V — 0 or, ve VT—LAT = 2.37 km/s 2 (2)
2 e R
116

In Eq. (1) and (2), M and R are the mass of the moon and its radius. In Eq. (1) if M and R
represent the mass of the earth and its radius, then, using appendices, we can easily get

v0 = 7.9 km/s and vc= 112 km/s.

1.229 In a parabolic orbit, E = 0

So i m -0 or, vi 171-1-
2 y i2 -
R
where M. mass of the Moon, R. its radius. (This is just the escape velocity.) On
the other hand in orbit

YMIn
mv 2R - or vf=
f 2 R
R

Thus Ay = ( 1 - {2- ) - 0.70 km/s.

1.230 From 1.228 for the Earth surface

larig
vo and v e=

Thus the sought additional velocity

lrEt r-
Av v. - v0 ( v2 - 1 ) gR ( -1)
R

This 'kick' in velocity must be given along the direction of motion of the satellite in its orbit.

1.231 Let r be the sought distance, then

YTIM YM )2
or Tir 2 IN (nR-r
(nR-r)2 r2

1.232 Between the earth and the moon, the potential energy of the spaceship will have a maximum
at the point where the attractions of the earth and the moon balance each other. This
maximum P.E. is approximately zero. We can also neglect the contribution of either body
to the p.E. of the spaceship sufficiently near the other body. Then the minimum energy
that must be imparted to the spaceship to cross the maximum of the P.E. is clearly (using
E to denote the earth)
117

?MEM
RE

With this energy the spaceship will cross over the hump in the P.E. and coast down the hill
of p.E. towards the moon and crashland on it. What the problem seeks is the minimum energy
required for softlanding. That reguies the use of rockets to loving about the braking of the
spaceship and since the kinetic energy of the gases ejected from the rocket will always be
positive, the total energy required for softlanding is greater than that required for
crashlanding. To calculate this energy we assume that the rockets are used fairly close to the
moon when the spaceship has nealy attained its terminal velocity on the moon

ZyMo
where Mo is the mass of the moon and Ro is its radius. In general
R0
dE = vdp and since the speed of the ejected gases is not less than the speed of the rocket, and
momentum transfered to the ejected gases must equal the momentum of the spaceship the
energy E of the Bass ejected is not less than the kinetic energy of spaceship
?Mom
Ro

Addding the two we get the minimum work done on the ejected gases to bring about the
softlanding.

On substitution we get
A. gym RE

13 x 108 kJ.
+
m-)
0
R 0

1.233 Assume first that the attraction of the earth can be neglected. Then the minimum velocity,
that must be imparted to the body to escape from the Sun's pull, is, as in 1230, equal to
( V-2- — 1 ) vi

where vi /r, r - radius of the earth's orbit, Ms — mass of the Sun.


In the actual case near the earth, the pull of the Sun is small and does not change much over
distances, which are several times the radius of the Earth. The velocity v3 in question
is that which overcomes the earth's pull with sufficient velocity to escape the Sun's pull. Thus

2 MV 32 -?TMcE es -21- M(112--1)2

where R = radius of the earth, M E = mass of the earth.

Writing yME /R, we get

v3= V 2 + (112- — 1 )2v1 se 16.6 km/s


118

1.5 DYNAMICS OF A SOLID BODY


1.234 Since,motion of the rod is purely translational, net torque about the
C.M. of the rod
should be equal to zero. 2 a

Thus 1 // (1)
For the translational motion of rod.
F2 _
F1 niwc

or 1 -
F1= mwc (2)
From (1) and (2) F2 IM F2

a mw , 2aF2 =

1/2
or, — m F2
1 =
niw

1.235 Sought moment x B.1;(ac + W)


aBr+Ab(—r). (aB—Ab)r

and arm of the force 1=


N
F
aB — Ab
,

A2 + B2

1.236 Relative to point the net moment of force :


0,

N-r xF +r xF
1 1 2 2 (aixAj)+(BjxBi)

abr+ AB ( ( ab — AB )1c (1)


Resultant of the external force + Br (2)
As if- 0 ( as /71 so the sought arm of the force 1
ab — AB

1C427 B2
1— N/F

1.237 For coplanar forces, about any point in the same plane, ri. x r x
-0 -IP -OD
Fn et

(where Fnet Fi resultant force) or,


1-
rx Fn er

Thus length of the Nnet Fne


arm,
Here obviously v. 2F and it is directed toward right Take the origin at C. Then
AC.

about C, along
directed normally into the plane of figure.
-{2-aF+ wa F—NrilaF)

(Here a = side of the square.)


Thus N.. a directed into the plane of the figure.
F
a

2F 2 r--- —2 sin 45*


F(
Hence 1=
.

..v2
Thus the point of application of force is at the mid point of the side
BC.
119

1.238 (a) Consider a strip of length dx at a perpendicular distance x from the axis about which
we have to find the moment of inertia of the rod. The elemental mass of the rod equals
m
dm - — dx
1
Moment of inertia of this element about the axis

dl= dm x2= M drx2


Thus, moment of inertia of the rod, as a whole
about the given axis
1
m /2
f 111-.X2CL
3 1
0
(b) Let us imagine the plane of plate as xy
plane taking the origin at the intersection point
of the sides of the plate (Fig.). 0 VA I
Obviously f dm y 2
a

—In b dy )y2
ab
f ma
3

mb2
Similarly / 3
.Y
Hence from perpendicular axis theorem
Ln_ / a 2 + k 2)
3\
z = Ix +

which is the sought ,moment of inertia.


1.239 (a) Consider an elementry disc of thickness dr. Moment of inertia of this element about
the z —axis, passing through its C.M.
(dm )R2 R2 4*••■••■•......................................................... OINNI■MINI•■■••■••■•42I

diz- 2 " Sdx 2


where p density of the material of the plate
and S area of cross section of the plate.
Thus the sought moment of inertia
b

f ax — R2p Sb
2 2
0
n 4
- (asS.. nR2)
120

putting all the vallues we get, l = 2• gm.m2


(b) Consider an element disc of radius r and
thickness dx at a distance x from the point
0. Then r = x tana and volume of the disc
= 3rx 2 tan 2 a cbc
Hence, its mass dm = n x 2 tana.dx•p (where

p = density of the cone = m/3 2


h) b
3 d
Moment of inertia of this element, about the
axis OA,
r2
dI = dm T

2 2
2 2 -1- tan a
k/ x tan a ux X A
2

p
= x4 tan 4Cit
2

p 4 4
Thus the sought moment of inertia I 2 tan f a x cfr
0
pR4.h 5 (
as tana = —
10h4 h

3m
Hence /= 3mR2 putting p = 2
10 5r.R h
[.240 (a) Let us consider a lamina of an arbitrary shape and indicate by 1,2 and 3, three axes
coinciding with x, y and z — axes and the plane of lamina as x — y plane. Now,
moment of inertia of a point mass about
x — axis, di = dm y 2
Thus moment of inertia of the lamina about
this axis, IX = f dmy 2

Similarly; /y = f dmx 2

and Iz = f dmr 2

=f dm (x2+y2) as r= 1,1x 2 + y2

Thus, /z + /y or, I3= + /2

(b) Let us take the plane of the disc as x —y plane and origin to the centre of the disc (Fig.)
From the symmetry Ix = Iy. Let us consider a ring element of radius r and thickness
dr, then the moment of inertia of the ring element about the y — axis.
121

m 2
dIz 2= (2.7trdr ) r
31R2

Thus the moment of inertia of the disc about z


— axis

mR 2
2ni f 3
iz= r dr
R2 2
o

But we have 1 = + ly = 2/x

mR 2
Thus Iz = =
2 4
1.241 For simplicity let us use a mathematical trick. We consider the portion of the given disc
as the superposition of two. complete discs (without holes), one of positive density and
radius R and other of negative density but of same magnitude and radius R/2.
As ( ar e a) a (mass), the respective masses of the considered discs are
(4m / 3) and (— m / 3) respectively, and these masses can be imagined to be situated at
their respective centers (C.M). Let us take point 0 as origin and point x — axis towards
right. Obviously the C.M. of the shaded position of given shape lies on the x — axis. Hence
the C.M. (C) of the shaded portion is given by

( —m/3 )(—R/2)+( 4m/3)0 R


x =
( —m/3)+4m/3 -1 6

Thus C.M. of the shape is at a distance R/6


from point 0 toward x — axis

Using parallel axis theorem and bearing in mind

that the moment of inertia of a complete


homogeneous disc of radius mo and radius 7.0

1 2
equals — mo ro . The moment of inetia of the
2
small disc of mass (— m / 3) and radius R/2
about the axis passing through point C and
perpendicular to the plane of the disc
2
11 m \(R \ m\(R R 2
+ F1 ) -2-1-6-)

mR 2 4 ...E• 2

24 27

1 (4m 2 4m )(R )2
Similarly Iic = —3— R + 6-

2
2 mR
= mR +
3 27

Thus the sought moment of inertia,


122

1.242 Moment of inertia of the shaded portion, about the axis passing through it's certre,
2(4 3 ) 2 2 ( 4 3 ) 3
1 = -5- p R - 5- 7cr p r

2
4 5)
= 3- p (R5- r

Now, if R = r + dr, the shaded portion becomes a


shell, which is the required shape to calculate
the moment of inertia. ( ik A
2 4
Now, 1= - - - np I ( r+dr)s-r5}
5 3
24
= —5 -3 n p ( r 5 + 5r 4 dr + - rs ) 4/8/ 1

Neglecting higher terms.

2 ,7 2
= -s ( 4n r 2 drp)r- = - mr2
3

1.243 (a) Net force which is effective on the system (cylinder M + body m) is the weight of
the body m in a uniform gravititional field, which is a constant. Thus the initial acceleration of
the body m is also constant.

From the conservation of mechanical energy of the said system in the uniform field of

gravity at time t = A t : AT+ AU= 0


A itp 2
1
or mil (1)2- mg Ali = 0
- mv2 +
2 2 2
1
or, - ( 2m +M) v2- mg Ah 0 [ as v wR at all times ] (1)
4

But v 21•8 2w Ah
Hence using it in Eq. (1), we get
1 2mg
- (2m + M )2w Ah - mg Ah = 0 or w=
4 (2m+M)
2mg
From the kinematical relationship, 13= =
R ( 2m + M ) R

Thus the sought angular velocity of the cylinder


2mg gt
( t ) = fit= t
(2m+M)R (1+M/2m)R

(b) Sought kinetic energy.


1 2 1 MI 2 2
T(t). - mv + -— w 2 = 4 (2m+M)R 2W
2 2 2
123

1.244 For equilibrium of the disc and axle


21'.. mg or l'.. mg/2
As the disc unwinds, it has an angular acceleration 13 given by
2Tr gr
/13 = 2Tr or fi - —1. . m

The corresponding linear acceleration is


2
mgr
r 13 = w-
/
Since the disc remains stationary under the
combined action of this acceleration and the
acceleration (- w) of the bar which is
transmitted to the axle, we must have
2
mgr
w.
/
1.245 Let the rod be deviated through an angle p'from its initial position at an arbitrary instant
of time, measured relative to the initial position in the positive direction. From the equation
of the increment of the mechanical energy of the system.

AT- A,„a

1 2
Or, i KO = fN2dp

'q
1 1111 2 2
Or, 2 --T w = fF/ cosy dy = Fl situp
ll

6F sing)
Thus, GO =
MI
1.246 First of all, let us sketch free body diagram of each body. Since the cylinder is rotating
and massive, the tension will be different in both the sections- of threads. From Newton's
law in projection form for the bodies mi and m2 and noting that w1 = w2 = w = [IR, (as

no thread slipping), we have (m1 > m2)

mi g - 1.1 = mi w = mi PR

and T2 - M2 g= M2 W (1)

Now from the equation of rotational dynamics of


a solid about stationary axis of rotation. i.e. Nz =
11z, for the cylinder.
or, ( T1 - T2 )R - 113 = mR2 13/2 (2)

Similtaneous solution of the above equations yields :

Ti mi ( m + 4m2 )
( mt - m2 )g and -
0- T2 M2 ( M + 4Mi )
m, + m,+—m)
I Ao
R (
124
1.247 As the system- ( m + m1 + m2) is under constant forces, the acceleration of body m1 an
m2 is constant. In addition to it the velocities and accelerations of bodies m1 and m2 al
equal in magnitude (say v and w) because the length of the thread is constant.
From the equation of increament of mechanical energy i.e. AT + AU= Afr, at time t whe
block m1 is distance h below from initial position corresponding to t = 0,

2
1 2 1 ( mR 2
2 ( mi + )v + v (1
2 2 R 2 - M2 gh km1 gh
(as angular velocity co = v/R for no slipping of thread.) But
2
V = 2wh
So using it in (1), we get
2 ( m2- lani ) g
w= (2,
m + 2 ( mi + m2 ) -
Thus the work done by the friction force on mi

Afr= - km igh - kmig 1


2 Wt 2

mi /ani g 2 t 2
(using 2).
m + 2 ( m1 + m2 )
1.248 In the problem, the rigid body is in translation equlibrium but there is an angular retardation.
We first sketch the free body diagram of the cylinder. Obviously the friction forces, acting
on the cylinder, are kinetic. From the condition of translational equlibrium for the cylinder,
mg - + kN2 N2- kNi

mg „7. , mg
Hence, Ni ; 111 2 I . it

k 1 1+k2
+

For pure rotation of the cylinder about its


rotation axis, Nz= Ifz

mR2
or, kNi R kN2 - 2

kmgR (1 + k) mR 2 ft
or,
1+k2 2 rz
21c (1.14)g
or, -
z (1+ k-9)R

Now, from the kinematical equation,


2
(I) = (00 2I3z A (pi we have,
2
(Do ( 1 + k ) R
= because co - 0

4k(1+k)g '
125

Hence, the sought number of turns,


AT, W02 ( 1 + k 2) R
n II
23t 831k(1+ k)g

1.249 It is the moment of friction force which brings the disc to rest. The force of friction is
applied to each section of the disc, and since these sections lie at different distances from
the axis, the moments of the forces of friction differ from section to section.
To find Nz, where z is the axis of rotation of the disc let us partition the disc into thin rings
(Fig.). The force of friction acting on the considered element
dfr = k (2n r dr a ) g, (where a is the density of the disc)
The moment of this force of friction is
dNz = - r dfr = - 2n k g r2 dr
Integrating with respect to r from zero to R, we get
R

2
Nz= -27ciccrgfr2dr= -- irkagR3.
3

For the rotation of the disc about the stationary


axis z, from the equation Nz =

2 nR2a)R2 4kg
-- akagR3 - ( 13z or 13 z= -.
2 3R
Thus from the angular kinematical equation
coz = cooz +13z t
3R coo
0= coot - —41c1) t or t=
3R 4kg
1.250 According to the question,
rdco dco
= -kV or, I = IfiTo = - k dt
kt
Integrating, ViTo = - —+
2/

2/
Let the flywheel stops at t = to then from Eq. (1), to = k
Hence sought average angular velocity
211ATO
k

dt
k2 t2
2 ir° ° kt + co- o
41

fo(
I
oco
<w>=
“171-30 3

f dt
126

d Mz
1.251 Let us use the equation .. Nz relative to the axis through 0 (1)
dt
For this purpose, let us find the angular momentum of the system Mz about the given
rotation axis and the corresponding torque N. The angular momentum is

Mz= + mvR = +m)R2o3


2

2
[where I = and v = w R (no cord slipping)]

dMz (M R2 + D2 )41
So, PZ (2)
dt 2
The downward pull of gravity on the overhanging part is the only external force,
which exerts a torque about the z —axis, passing through 0 and is given by,

xgR

dMz
Hence from the equation dt — Nz

2
m R 2 ) 13z= Tm xgR
2
M2 2mgx
Thus, >0
13z — 1R (M + 2m)
Note : We may solve this problem using conservation of mechanical energy of the system'
(cylinder + thread) in the uniform field of gravity.

1.252 (a) Let us indicate the forces acting on the sphere and their points of application. Choose
positive direction of x and ozp (rotation angle) along the incline in downward direction and in
the sense of (Ti(for undirectional rotation) respectively. Now from equations of dynamics
of rigid body i.e. Fz mwa and Ncz pz we get :
mg sin a — mw (1)

2 2
and fr R= —5 mR p (2)

But fr s kmg cosa (3)


In addition, the absence of slipping provides
the kinematical realtionship between the
accelerations :
w = 13R (4)
The simultaneous solution of all the four
equations yields :
2 2
k cos a z — sin a, or k — tan a
7
(b) Solving Eqs. (1) and (2) [of part (a)], we get :
127

5
w,= 7 g sin a.

As the sphere starts at t = 0 along positive x


axis, for pure rolling

(5)

Hence the sought kinetic energy


1 2 1 2 2
T . — mv + — — mR w2 = —7 mv2 (as co = vc/R )io c
2 c 2 5
2

s• 7 m ( Igsinat = 5mg2sin2at2
10 7 14

1.253 (a) Let us indicate the forces and their points of application for the cylinder. Choosing
as shown the figure, we write the equation of motion
the positive direction for x and q in of the
cylinder axis and the equation of moments in the C.M. frame relative to that axis
i.e. from equation Fx mwc and N Oz.

mR 2
mg — 2T = mwc; 2TR . 2

As there is no slipping of thread on the cylinder


wc = 13R
From these three equations

T- = 13N, 13 = 2 = 5 x 102 rad/s2


6 R
(b) we have 13 = fi

2
So, wc = —3 g > 0 or, in vector form c 3 i*

P= F • IT!. F • (T41: t)

= mg — gtt —

S 3g 3
mg
2t

1.254 Let us depict the forces and their points of application corresponding to the cylinder attached
with the elevator. Newton's second law for solid in vector form in the frame of elevator,
gives
2T+ mr+ m (— i-v+0) (1) T
The equation of moment in the C.M. frame
relative to the cylinder axis i.e. from
NZ = 4 (3z. -

mR '
2TR —mR 2 - 2w
2 2 R mwa
[as thread does not slip on the cylinder, w' = 13R I
128

mw'
or, T=
4

As (1) Ff.t i-v+

so in vector form
--*
MK,
7+'. (2)
4

Solving Eqs. (1) and (2), Tif = 2: (r— w0) and sought force
3

i= 2T= 1 m (r— 4).


3
1.255 Let us depict the forces and their points of application for the spool. Choosing the positive
direction for x and cp. as shown in the fig., we apply Fx = mwc, and Nd= 4 fsz and get

mg sin a. — T = mw ; Tr = II-1
"Notice that if a point of a solid in plane motion
is connected with a thread, the projection of
velocity vector of the solid's point of contact
along the length of the thread equals the velocity
of the other end of the thread (if it is not
slacked)"
Thus in our problem, vp = vo but v0 = 0,
hence point P is the instantaneous centre of
rotation of zero velocity for the spool. Therefore vc
= or and subsequently we = Pr.
Solving the equations simultaneously, we get
g sin a
w= — 1.6 m/s2
1+
2
mr
1.256 Let us sketch the force diagram for solid cylinder and apply Newton's second law in
projection form along x and y axes (Fig.) :
fri + fr2 = my, (1)

and N1 +N2 — mg — F = 0

or Ni+N2= mg+F (2)


Now choosing positive direction of q) as shown
in the figure and using Ncz— 413z,
we get

i+ fr2) R .= R2 mR 2 Wc
FR — Ur (3)
m22 — 2 R
[as for pure rolling we = OR J. In addition to,
f r 1 + fr2 s k (Ni+ N2) (4)
129

Solving the Eqs., we get


3kmg 3kmg
F s or Fm. =
(2 — 3k) ' 2—3k
k (N1 + N2)
and wc (max) ml m

k r 1 k [ 3kmg 1 2 kg
ail ;?1 ling + FMaX j m ;71 M g + 2 — 3k - 2 — 3k

1.257 (a) Let us choose the positive direction of the rotation angle cp, such that wce and 13z have
identical signs (Fig.). Equation of motion, Fx = mwex and Nez = 4 f3z gives :

F cos a — fr = mwcx : fr R — Fr = Icfz= ymR 2 13x

In the absence of the slipping of the spool wex = f3zR

F [cos a — (r/Ril c r
From the three equations wex = we = , where os a > — (1)
m (1 + y) R
AI

(b) As static friction (f r) does not work on


the spool, from the equation of the increment of
mechanical energy Aeu = AT.
2
1 2 1 2 Vc 1
ilea = f nwe + 1y mR i—z-- I. f M (1 + y) v!

1 1
= 1 m (1 + y) 2w x = —c (- w t2 m (1 + y) 2 w
' 2
2

F2 Cosa — 1 t2
R

2 m (1 + y) 777g

Notethat at cos a = r/R , there is no rolling and for cos a < r/R , wcz< 0, i.e. the spool

will move towards negative x-axis and rotate in anticlockwise sense.

1.258 For the cylinder from the equation /sic = 113z about its stationary axis of rotation.

mr2 T
2 Tr = —2 13 or 13. L (1)
mr

For the rotation of the lower cylinder from the


equation Ncz= 4 pz
2
mr , , p, = 4T

2Tr = — p or,
2 mr

Now for the translational motion of lower


cylinder from the Eq. Fx = mwex :
mg — 2 T = mwc (2)
As there is no slipping of threads on the ?I
cylinders :
-we_ 13' r + f3r = 2 f3r (3)
130

Simultaneous solution of (1), (2) and (3) yields

T-
10'

1.259 Last us depict the forces acting on the pulley


and weight A, and indicate positive direction
for x and 0:p as shown in the figure. For the
cylinder from the equation Fx. m IN and

N = (3z, we get
Mg + TA — M we (1)

I Iv,
and 2TR + TA (2 R) R (2)
For the weight A from the equation
Fx mwx
7114
mg — TA MWA (3) T
As there is no slipping of the threads on the
pulleys.
-
wc + 2 PR wc + wc 3 wc (4)
Simultaneous solutions of above four equations
gives :
3 (M + 3 m)g
WA =
- (m+9m+-)
R2

1.260 (a) For the translational motion of the system (m1+ m2), from the equation : Fx mwcx
F = (mi+ m2) we or, we = F/(mi + m2) (1)
Now for the rotational motion of cylinder from the equation : Ncx (3z

2F
mir2
Fr — 13 or Or = (2)
2 ml
But , So
wx = w, + p r

= F 2F F(3m1+2m2)
C + 1 — (3)
M1+ M2 m M1(721+ M2)
(b) From the equation of increment of mechanical energy : AT= A e,t
Here AT. T (t) , so, T (t) Aeu
As force F is constant and is directed along x-axis the sought work done.
Aar = Fx
(where x is the displacement of the point of application of the force F during time interval t )
131

2) t2 (3 mi + 2 no
=F( 1w t . F2 = T (t)
2 K 2 mi (mi + m2)
(using Eq. (3)
t
Alternate : T (t) = Tfr ansla ( ) + Trotanot (t)

ir 2 —2
1 ( Ft )2 +1 m e 2Ft r- t 2 (3 mi 4' 2 m2)
= — (ml + m)2 (mi + m2) 2 2 mir 2 mi (mi, m2)

1,261 Choosing the positive direction for x and cp as shown in Fig, let us we write the equation
of motion for the sphere Fx = mwcx and Ar cz- I, 13z.

2
fr = m2 w2; fr r = 5 m2r2 13

(w2 is the acceleration of the C.M. of sphere.) For


the plank from the Eq. Fx = mwx
F - fr= m1 w1

In addition, the condition for the absence of


slipping of the sphere yields the kinematical
relation between the accelerations :
w1 ' wz + 13 r
Simultaneous solution of the four equations yields :
F 2
W1 — and w2 = -7- 4,
1
2 m
mi + —7 ,L)
1.262 (a) Let us depict the forces acting on the cylinder and their point of applications for the
cylinder and indicate positive direction of x and fp as shown in the figure. From the
equations for the plane motion of a solid Fx - mwcx and Na = Ic 13z :
k mg = m wcx or wcx = kg (1)

niR 2
- king R 3. --- . if or fiz = - 2 Icg. (2)
2 R
Let the cylinder starts pure rolling at t ... to after
releasing on the horizontal floor at t = 0.
From the angular kinematical equation
wz ' woz +13z t,
sIl
or co = (40 - 2 R t (3)

From the equation of the linear kinematics,

vcz 111 VOCX + WCX


or vc = 0 + kg to (4)
132

But at the moment t = to, when pure rolling starts v, = wR

so, kg to -(coo 2 1/1 RR

wo R
Thus to '11 3 kg

(b) As the cylinder pick, up speed till it starts rolling, the point of contact has a purely
1 ti in the forward directions but there is also a backward
translatory movement equal to —2 wc

1 2
movement of the point of contact of magnitude (coo To — to) R. Because of slipping
the net displacement is backwards. The total work done is then,

A fr = kmg [12-wc - (00t0 13tbRi

woR {cooR cooR mcoo2R


kmg
3kg 6 + 3 w°R — 6
The same result can also be obtained by the work-energy theorem, Aft. = A T.

1.263 Let us write the equation of motion for the centre of the sphere at the moment of breaking-off:
mv2/(R + r) = mg cos 0,
where v is the velocity of the centre of the sphere at that moment, and 0 is the corresponding
angle (Fig.). The velocity v can be found from the energy conservation law :

mgh mv2 + / (02,

where I is the moment of inertia of the sphere


relative to the axis passing through the sphere's
2
centre. i.e. I = — mr2. In addition,
5
v = (or ; h = (R + r) (1 — cos 0).

From these four equations we obtain

= V10 g (R + r) 17 r2
1.264 Since the cylinder moves without sliding, the centre of the cylinder rotates about the point
0, while passing through the common edge of the planes. In other words, the point 0
becomes the foot of the instantaneous axis of rotation of the cylinder.

It at any instant during this motion the velocity of the C.M. is vi when the angle (shown in
the figure) is 13, we have
2
m vi
R — mg cos 3 — N ,
133

where N is the normal reaction of the edge


R
or, vi2 gR cos 13 — N 77 , (1)

From the energy conservation law,


2 2
1 1
V 1 vo
—2 / —0 R2 — —2 mg R — cos p)
R2

3
mR 2
But 10- 2 + m R2 mR2 ,
(from the parallel axis theorem)
2 2 4
Thus, vo + — g R (1 — cos p) (2)
3
From (1) and (2)
2 g NR
vo = 3 (7 cos 13 — 4) — m
in this equation is clearly smaller than or equal to a so putting
The angle I 1 = a we get
NoR
vo2 —— gA3 (7 cos a — 4) —
M
where No is the corresponding reaction. Note that N a No. No jumping occurs during this
turning if No > 0. Hence, vo must be less than

g_LZ
vmax =V (7 cos a — 4)3

1.265 Clearly the tendency of bouncing of the hoop will be maximum when the small body A,
will be at the highest point of the hoop during its rolling motion. Let the velocity of C.M.
of the hoop equal v at this position. The static friction does no work on the hoop, so from
conservation of mechanical energy; E1 - E2

2 2
0 + mv2 + _1 m R2 _vo_ _
or, mg R = 1 m (2v)2 + 1 m v2 + 1 m R2 (-R1 + mg R
2 ° 2 R 2 2 2
2
or, 3 vo2— 2 gR v2 - (1)
From the equation Fn - mwn for body A at final position 2 :
2

mg + = m w2 R = m (tt) R (2)
134

As the hoop has no acceleration in vertical direction, so for the hoop,


N+N' = mg (3)
From Eqs. (2) and (3),
m v2
N= 2 mg — (4)
R
As the hoop does not bounce, N 0 (5)
So from Eqs. (1), (4) and (5), 2
8gR—vo
k 0 or 8gR
3R
Hence ve, s Virg TC
1.266 Since the lower part of the belt is in contact with the rigid floor, velocity of this part
becomes zero. The crawler moves with velocity v, hence the velocity of upper part of the
belt becomes 2v by the rolling condition and kinetic energy of upper part

1 (m 2
= — — (202 = MV , which iis also the sought kinetic energy, assuming that the length of
2 2
the belt is much larger than the radius of the wheels.

1.267 The sphere has two types of motion, one is the rotation about its own axis and the other
is motion in a circle of radius R. Hence the sought kinetic energy
1 ( 2+ 1 2
T= 01 —2/2 CO2 (1)
—2/1

where I, is the moment of inertia about its own axis, and /2 is the moment of inertia about
the vertical axis, passing through 0,
2 2 2 2
But, 3- mr2 and 12 mu mr + mR (using parallel axis theorem,) (2)

In addition to

an (1)2. (3)
d
7 v2 2r2

Using (2) and (3) in (1), we get T' io + 7R2

1.268 For a point mass of mass dm, looked at from C rotating frame, the equation is

dm w = f + dm CO2 i-w + 2 dm (i7' 61.)


where r = radius vector in the rotating frame with respect to rotation axis and v =
velocity in the same frame. The total centrifugal force is clearly

Fcf = dm w2 r = mw2
Rc. is the radius vector of the C.M. of the body with respect to rotation axis, also

= 2mv x
where we have used the definitions

mRc= 2 dmr and my= 2 dm i-Tw'


135
1.269 Consider a small element of length dx at a distance x from the point C, which is rotating
in a circle of radius r x sin 0
Now, mass of the element = I dx 0'
So, centrifugal force acting on this element
- PI dx w2 x sin 0 and moment of this force(
/
about C,
‘1'
C
I d/Sr I = (11) dr (02 x sin 0•x cos 0

//I (0 2
2/ sin 20 x2 dx
and hence, total moment
1/2

N=2 — sin 2 0 x2 dx .,?:(02/2sin20,


f 2/ 24
0
1.270 Let us consider the system in a frame rotating with the rod. In this frame, the rod is at
rest and experiences not only the gravitational force m rand the reaction force R, but also
the centrifugal force Fcf.
In the considered frame, from the condition of equilibrium i.e. No, = 0
or, Ncf = mg 2 sin 0 4 (1)
where Ncf is the moment of centrifugal force
about 0. To calculate Ncf , let us consider an j0
element of length dx, situated at a distance x - ... —__ 1I._ ...
from the point 0. This element is subjected to t- i
273 - _ r---
a horizontal pseudo force (T dx o.)2x sin 0.
The moment of this pseudo force about the 1
I
axis of rotation through the point 0 is i
dNcf . (7m) dxco 2 x si• n 0 x cos0

2
m CO
= / sin 0 cos 0 x2 dx
1
2 mco2/2
So Ncf = ml
CO
sin 0 cos 0 x2 dr - smOcos0
3
0
It follows from Eqs. (1) and (2) that,
3g
cos 0 = —1P2— ) or 0= cos 1
2 Eu2 I 2w21/
136
1.271 When the cube is given an initial velocity on
the table in some direction (as shown) it
acquires an angular momentum about an axis
on the table perpendicular to the initial velocity
and (say) just below the C.G.. This angular
momentum will disappear when the cube stops
and this can only by due to a torque. Frictional
forces cannot do this by themselves because
they act in the plain containing the axis. But
if the force of normal reaction act eccentrically
(as shown), their torque can bring about the
vanishing of the angular momentum. We can
calculate the distance Ax between the point of
application of the normal reaction and the C.G.
of the cube as follows. Take the moment about

C.G. of all the forces. This must vanish because Initial


the cube does not turn or tumble on the table. angular
Then if the force of friction is fr momentum
fr a Ax
— =N 2
But N = mg and fr = kmg, so
Ax = ka/2
1.272 In the process of motion of the given system the kinetic energy and the angular momentum
relative to rotation axis do not vary. Hence, it follows that
12 2 1
1 M/2 2
M i m ( c 0 2 / 2 + 1 / 2 ) 4. 0
)
2 3
(w is the final angular velocity of the rod)

Ml 2 /
and 3 coo co + mi2 w

From these equations we obtain


3M\
= w°/ (1 + and
M

v' = cool / 1/1 +3m/M


1.273 Due to hitting of the ball, the angular impulse received by the rod about the C.M. is equal
1
to p — If w is the angular velocity acquired by the rod, we have
2
m/2
— 4 or w (1)
12 2 ml
In the frame of C.M., the rod is rotating about an axis passing through its mid point with
the angular velocity w. Hence the force exerted by one half on the other = mass of one
half x acceleration of C.M. of that part, in the frame of C.M.
m (c02 I ). mco 2/ 9p 2_ 9N
2 4 8 2m1
137

1.274 (a) In the process of motion of the given system the kinetic energy and the angular
momentum relative to rotation axis do not vary. Hence it follows that
2
1 2 1 , 2 1 ( M1 2
-mv = - mv + - —
2 2 2 3
2
1 , 1 M1
and mv +
2= 2 3
From these equations we obtain
( 3m — 4M ) 4v
— v and co =
3m + 4M 1 (1 + 4 m/3M)
# --310 -1■0 ( 3m — 4M ) -•
As v -30 t T v, so in vector form v =
3m + 4M
(b) Obviously the sought force provides the centripetal acceleration to the C.M. of the rod
and is
Fn= mwcn
2/ 8Mv 2
M0) = 2
2 / ( 1 + 4M/3m )

1.275 (a) About the axis of rotation of the rod, the angular momentum of the system is conserved.
Thus if the velocity of the flying bullet is v.

my1 M12 +

my 3my
as m « M (1)
(m + )1
3
Now from the conservation of mechanical energy of _the system (rod with bullet) in the
uniform field of gravity
to 2
1m ( 12 + M1 0) 2 = (M+m)g-1 (1— cosa ) (2)
2 3 2

[because C.M. of rod raises by the height 1( 1 — cosa ) ]

Solving (1) and (2), we get

v= ) g1 s in -c1 and co VF sin


m 3 2

/
(b) Sought 4 .. [ m(wI)+M wy —mv

/
where.co/ is the veloccity of the bullet and co - equals the velocity of C.M. of the rod
2
after the impact. Putting the value of v and co we get

-1 my— m sin -ct


2 6 2

This is caused by the reaction at the hinge on the upper end.


138

(c) Let the rod starts swinging with angular velocity w', in this case. Then, like part (a)
2
MI 3mvx
mvx = ( — + mx 2 ) (.0 r Or CO , a. Alp
3
Final momentum is
m m
3
Pf mxwi+fyco'Tdymyw'l=imv-i-

So,

This vanishes for

1.276 (a) As force F on the body is radial so its angular momentum about the axis becomes
zero and the angular momentum of the system about the given axis is conserved. Thus

MR' MR 2 2m )
wo+mwoR2 = T-- or co coo 1 + —m
2
(b) From the equation of the increment of the mechanical energy of the system :
AT= ilext

1 MR2 2 ( MR2 2 2

w + lfi R = A'

Putting the value of w from part (a) and solving we get

m co:R 2 ( 2m
2 1+ m

1.277 (a) Let z be the rotation axis of disc and p be its rotation angle in accordance with
right-hand screw rule (Fig.). (p and p' are to be measured in the same sense algebraically.)
As Mz of the system (disc + man) is conserved and Mz( )in 0, we have at any instant,

m R 2d
2 _ 1
0—
2 +m1 Cldit R R
dt
Or, d = m1 I d (pi
( m2/2 )

cp'
mi
On integrating fdcp= - f )4'
mi+(m2/2)
0 o(
mi
or, P( = (1)
m2
+
m1 2
This gives the total angle of rotation of the disc.
139

(b) From Eq. (1)


d cp ) tni v' (t)
dpi _
dt m2 dt m2 R
mi.+ —2 +

Differentiating with respect to time

d2cp ldv'(t)
d t2 = M2 R dt
m1+ —2

m1m2R dvi (t)


Hence Nz
2m1+ m2 dt
1.278 (a) Frome the law of conservation of angular momentum of the system relative to vertical
axis z, it follows that:
11 0) z + 12 0)2z ( + 12) (.0z

Hence coz= (iico1z+/2co2z)/(/1+/2) (1)

Not that for coz > 0, the corresponding vector Fo.coincides with the poitive direction to the z
axis, and vice versa. As both discs rotates about the same vertical axis z, thus in vector form.
--11, --• /
(.0m li(01÷.12W2i(li+/2)

-1111■
However, the problem makes sense only if col 11 co2 or wlT �w2

(b) From the equation of increment of mechanical energy of a system: Afr. AT.

1
(il+ ) (01 - W i2z /2 CO 22z

Using Eq. (1)

1.279 For the closed system (disc + rod), the angular momentum is conserved about any axis.
Thus from the conservation of angular momentum of the system about the rotation axis
of rod passing through its C.M. gives :
2
M1
mv —
, /
my CO (1)
2 2 12
140

(v' is the final velocity of the disc and w angular velocity of the rod) For
the closed system linear momentum is also
conserved. Hence
my =
my' 4 1 1 ni Vc (2)

(where 1,, is the velocity of C.M. of the rod)


From Eqs (1) and (2) we get
1
vc = 3 aandnu v— = rive
Applying conservation of kinetic energy, as the collision is elastic
1 2 1 2 1 2 1 2
—MV = —MV, + —Timv +— w (3)
2 2 2 2 12
2
or v — I/2 = 41vc2 and hence v + 4vc
Then
4 i 12 v
v1— 4 v and w= (4 + TO / C
Ti •

Vectorially, noting that we have taken 17* parallel to ii*


(4
4+

So, u= 0 for rl = 4 and /7* 1l t 17* for ri > 4 0-+


1.280 See the diagram in the book (Fig. 1.72)
(a) When the shaft BB' is turned through 90° the platform must start turning with angular
velocity S2 so that the angular momentum remains constant. Here
/o o
( / + /0 ) C2 = I° w° or, Q= 1+ 4

The work performed by the motor is therefore


I2 1 0 w2wo
(/+ 4 ) S22 =
2 2 / + /0
If the shaft is turned through 180°, angular velocity of the sphere changes sign. Thus from
conservation of angular momentum,
Q —10 wo = 10 coo

(Here — 4, coo is the complete angular momentum of the sphere i. e. we assume that the

angular velocity of the sphere is just — coo). Then

Q IT(1)0

and the work done must be,


r2 2
1 1 2 L'10
— / + — / co / —
2 2 o o 2 0 o
141

(b) In the case (a), first part, the angular momentum vector of the sphere is precessing with
angular velocity Q. Thus a torque,
2 2
/0
c Q I 4.10 Wo is needed.
Io o0 =

1.281 The total centrifugal force can be calculated by,


to

_i_ i 2
4-' 0.)1 Xtu- = m i co
2 2 °
J
0 -
Then for equilibrium,
1 10
(T2—T1)2= mg
t

1 2
and, T2+ T1= — m 100)

Thus Ti vanishes, when

( a (0. IF,
0 2= = 6 rad/s
/ ' /

Then T2 = mg— . 25 N
10 0
/

1.282 See the diagram in the book (Fig. 1.71).

(a) The angular velocity Oihabout 00' can be resolved into a component parallel to the rod
and a component co sin() perpendicular to the rod through C. The component parallel to the
rod does not contribute so the angular momentum

M= I w sin() = —1 m /2 co sin()
12

mi2cosin20
Also, Mz = Msin0 = —121

This can be obtained directly also,


(b) The modulus of M does not change but the
modulus of the change of M is I A M I.

I Aril = 2Msin(90-0)= im/2cosin20


12

(c) Here M1= M cosO = 1 w sin° cos°

(0 dt 1 2 2 • 2
Now
dt — — — ml o) sm 0
= /co sin() cos0
dt 24
as M precesses with angular velocity w.
142

1.283 Here M = Ro is along the symmetry axis. It has two components, the part /co cos0 is
constant and the part Af.c. /co sine presesses, then

dM
= / co sine a = mgl sin() dt

or, co' = precession frequency = jn1g013 = 0.7 rad/s

(b) This force is the centripetal force due to precession. It acts inward and has the magnitude

--* /2 M CO' 2 / sin() = 12 mN.


IF' .. 12 Mi W 1 =

517 is the distance of the i th element from the axis. This is the force that the table will
exert on the top. See the diagram in the answer sheet

1.284 See the diagram in the book (Fig. 1.73).


1
The moment of inertia of the disc about its symmentry axis is — m R 2. If the angular
2
1
velocity of the disc is w then the angular momentum is — m R . The precession frequency
2 2w
being 2n n,

dM 1
we have
dt = — mR 2 co x 2nn

This must equal m ( g + w ) 1, the effective gravitational torques g + (g being replaced by


w in the elevator). Thus,

co = ( g + w )1 — 300 rad/s
7rR2n
143

1.285 The effective g is 11g 2 + w2 inclined at angle tan- I Iv with the vertical. Then with reference

to the new " vertical" we proceed as in problem F283. Thus

niv g2T. w2
w= = 0.8 rad/s.
/co

The vector cTi' forms an angle 0 = tan-i —w - 6° with the normal vertical.

2 2
1.286 The moment of inertia of the sphere is --mR and hence the value of angular momentum
5
2
is - mR 2w. Since it precesses at speed w' the torque required is
5

2
- R co = 1
5m

2 2
So, F' = - mR coin = 300 N
5
(The force F must be vertical.)

1 2 1 2
1.287 The moment of inertia is -mr and angular momentum iss - mr co. The axle oscillates
2 2
about a horizontal axis making an instantaneous angle.
27tt
cip = (Pm sin T
dc
This means that there is a variable precession with a rate of precession
dt . The maximum
2.7tcp „,
value of this is . When the angle between the axle and the axis is at its maximum

value, a torque Ico Q

1 2 2a(1). 7t mr 2
= - mr T mo'" acts on it.
2

mr 2 coy,.
The corresponding gyroscopic force will be - 90 N
1T
1.288 The revolutions per minute of the flywheel being n, the angular momentum of the flywheel

is 1 x 27tn. The rate of precession is R1'•


Thus N = 2 nINV/R = 5.97 1(1■1. m.

1.289 As in the previous problem a couple 27rInv/R must come in play. This can be done if a
23t/nv
force, acts on the rails in opposite directions in addition to the centrifugal and other
RI
forces. The force on the outer rail is increased and that on the inner rail decreased.
The additional force in this case has the magnitude 1.4 kN. m.
144

1.6 ELASTIC DEFORMATIONS OF A SOLID BODY

1.290 Variation of length with temperature is given by


A/
4= /0( + aAt ) or — = a At = e (1)
to

But e=
E,
Thus a = ccAtE, which is the sought stress of pressure.
Putting the value of a and E from Appendix and taking At = 100°C, we get

= 2.2x 103 atm.

1.291 (a) Consider a transverse section of the tube and concentrate on an element which subtends
an angle AT at the centre. The forces acting on a portion of length Al on the element are
(1) tensile forces side ways of magnitude aArAL
The resultant of these is

2aArA/sin — aArA/Aq5
2
radially towards the cente.
(2) The force due to fluid pressure — pr&pAl
Ar
Since these balance, we get pma. = am7

where am is the maximum tensile force.

Putting the values we get pma. = 19.7 atinos.

(b) Consider an element of area c1.5 it ( r A0/2 )2 about z —axis chosen arbitrarily. There
are tangential tensile forces all around the ring of the cap. Their resultant is

AO AO
a [ 27t r Ar sin
2

Hence in the limit


2
rA0 ) =
am n rA0 Ar AO
Pm 2 2

2a. Ar
Or pm = = 396 atmos.

1.292 Let us consider an element of rod at a distance x from its rotation axis (Fig.). From
Newton's second law in projection form directed towards the rotation axis

— dr = (dm) CO 2 X gm -M W 2x cLx

On integrating
2 2
M O) X
—T — — + C ( constant )
1 2
145

/
But at x= ± — or free end, T= 0
2
2 2
MO) 1
Thus 0- +C or C . MO) 2i
2 4 _m8

mw2 (1 x2)
Hence T—
2 4 1

MCO 2 /
Thus T — (at mid point)
8
Condition required for the problem is
Tom= Sam
2
MCO / 2 7213 r
S — S am or co = — —
O' 8 / P
Hence the sought number of r p s

a) 1 1/2a,„
n= —= v [using the table n . 0.8 x 102rps ]
2n n/ • p

1.293 Let us consider an element of the ring (Fig.). From Newton's law Fn = mw„ for this
element, we get,

Td0 = (—m dO)co 2 r [see solution of 1.93or 1.921


2n
nl 2
So, AT =Mg .-.-- '€0
r
Lit
Condition for the problem is :
2
T MO) r
2 S 0m o
r,
nr It 2
r 2 S aamm

2
2Jt 2 an, r am
or, wymix =
nr 2 ( 27trp ) Pre

Thus sought number of rps


comax 1
n= _2y
2n 2nr p
Using the table of appendices n - 23rps
1.294 Let the point 0 desend by the distance x (Fig.). From the condition of equilibrium of point
0.

2T sine . mg or T= --1-1111--= — V( 1/2 )2 + x 2


2sin0 2x

T el ez d2
Now, a - or T. n —
n ( d/2 )2 4

( a here is stress and E is strain.)


146

In addition to it,

V(//2)2+x2
E_ 1/1 + g-X - 1 (3)
1/2

From Eqs. (1), (2) and (3)

x 32f21_
x- as x « /
T )2 7tEd 2
V 2x

4x3 mgl
So, —
212 nEd 2
m 1/3

or, x= 1 ( g = 2.5 cm
2irEd-
1.295 Let us consider an element of the rod at a distance x from the free end (Fig.). For the
considered element 'T - T ' are internal restoring forces which produce elongation and
dT provides the acceleration to the element. For the element from Newton's law :

F
dT = (dm) w (-171d,r)F1 dx
1 m 1
As free end has zero tension, on integrating the above expression,

F
dr = T dx or T= °x
0 0
Elongation in the considered element of lenght dx :
T
a" EL (x) —SE dx SEL

F.1
Thus total elengation =
SE1 xdx=2SE
T Tilfr
Hence the sought strain
z F.
0
1 2SE
1.296 Let-us consider an element of the rod at a distance r from it's rotation axis. As the element
rotates in a horizontal circle of radius r, we have from Newton's second law in projection
form directed toward the axis of rotation :
T - (T + dn. (dm) w2 r

or, - dr = dr) w2 r = w2 r dr
1 1
147

At the free end tension becomes zero. Integrating the above experession we get, thus
0 1

— f dT = L?i 0)2 f r dr
T r

(02 - r2 ) . m (02 / (1 _ r2
12
Thus T= m
1 2 2 /2
Elongation in elemental length dr is given by :

a = al.l dr ..• --L dr


E SE
(where S is the cross sectional area of the rod and T is the tension in the rod at the
considered element)
m (o2 1 2
Or, a = SE 1- )dr
2 12
Thus the sought elongation
I
mw2ir, r
2\
dr

-. .fd-- 2SE J ri-2)


0
m w21 21 (S/ p) 2
or, - = (1) 13
2SE 3 3 SE
1 2 3
' - =''' --- (where p is the density of the copper.)
m 3 E

1.297 Volume of a solid cylinder


V = TE r2 1

AV it2rArl xr2 Al 2Ar A/


. 4. (1)
So,
V ic n r2 1 + I t r2 1 r 1

But longitudinal strain A1/1 and accompanying lateral strain A r/r are related as

Ar Al
r 2E —II 7-

Al — Fh t r2
But
1 E

(Because the increment in the length of cylinder Al is negative)

AV —F
So, , (1 — 2 ii)
v it r2 E
148

- Fl
Thus, AV (1 - 2 it)

Negative sign means that the volume of the cylinder has decreased.

1.298 (a) As free end has zero tension, thus the tension in the rod at a vestical distance y from
its lower end

-7.gy (1)

Let al be the elongation of the element of length dy, then

a 1)
al - 0 dy

Td
= SE 1"-Y = SlE - p gydy/E (where p is the density of the copper)

Thus the sought elongation

p ra ydy. ln ai2/E
Al= (2)
f 'J E 2"

A/
(b) If the longitudinal (tensile) strain is e = — the accompanying lateral (compressive)

strain is given by
, Ar
E - (3)

Then since V = n r21 we have


AV 2A r A/
V +—
r /

A
where — is given in part (a), it is the Poisson ratio for copper.
/
1.299 Consider a cube of unit length before pressure is applied. The pressure acts on each face.

The pressures on the opposite faces constitute a tensile stress producing longitudianl com-

pression and lateral extension. The compressions is aa nndd the lateral extension is it, E.

The net result is a compression

2-- (1 - 20 in each side.

AV AV Al
Hence =- (1 - 2 .t) because from symmetry v =3T
V E
149

(b) Let us consider a cube under an equal compressive stress a, acting on all its faces.
AV
Then, volume strain = - — = (1)
V k'

where k is the bulk modulus of elasticity.

a 3a
So - 2 IA)
3
or E= 3 k (1 - = -(1 - 21.4)(as k=
,
1
s - i if E and 3 are both to remain positive.
2
1.300 A beam clamped at one end and supporting an applied load at the free end is called a
cantilever. The theory of cantilevers is discussed in advanced text book on mechanics. The
key result is that elastic forces in the beam generate a couple, whose moment, called the
moment of resistances, balances the external bending moment due to weight of the beam,
load etc. The moment of resistance, also called internal bending moment (I.B.M) is given
by

I.B.M. = BUR
Here R is the radius of curvature of the beam at the representative point (x, y)./ is called the
geometrical moment of inertia

fz2ds
of the cross section relative to the axis passing through the netural layer which remains
unstretched. (Fig.1.). The section of the beam beyond P exerts the bending moment N (x)
and we have,
El
— - N(x)
R
If there is no load other than that due to the
weight of the beam, then
1 TIEDIFI:12727:011.=
N (x) = - p g (1 - x)2 bh
2
t

Here b = width of the beam perpendicular to paper.


h/2
3
If ttiz bh
Also,
12
-h/2

(1) 6 p g12
Hence, - (0.121 km) -I-
-
k)(1 Eh2
150

1.301 We use the equation given above and use the result that when y is small
d 2y d2y N (x)
Thus
R dx2 ' dx2 E
(a) Here N (x) = No is a constant. Then integration gives,

dy No r
dr E + s-1

But = 0 for x = 0, so C1 - 0. Integrating again,

2
No x
Y = 2 EI
where we have used y = 0 for x = 0 to set the constant of integration at zero. This is the
equation of a parabola. The sag of the free end is
No /2
X —
y (x In

2EI
(b) In this case N (x) = F (1— x) because the load F at the extremity is balanced by a
similar force at F directed upward and they constitute a couple. Then
d2y F (1 — x)
dr2 EI

F (lx — x2/2)
dy
Integrating, + L-1
dx EI
As before C1 = 0. Integrating again, using y = 0 for x = 0
/x2 x3\
2 6
F/3
y=
F( EI
here k
3 EI

Here for a square cross section


a/2
I= f 2 a dz = a4/12.
- a/2

1302 One can think of it as analogous to the previous


case but with a beam of length 1/2 loaded
upward by a force F/2. F/2
T
F /3
•••
Thus k 48E1'
1.
` ... s' ... ••• ... ..•
.........' ...,

.... .1•1• MEM ...............

On using the last result of the previous problem.


1
1.303 (a) In this case N (x) = pg b h (1 — x)2 where b = width of the girder.

Also I = b h3/12. Then,


151

Ebh2d2y.pgbh
(12 21x +x2).
12 dx2 2

3
6 pg(g x_ix2+ 1
Integrating,
dr E h2 3

using -c1X .. 0 for x = 0. Again integrating


dr
6 pg (/ 2 X2 1X3 x4)
Eh2 2 3 +12

6pg14(1 1 1
Thus X -+
- Eh2 2 3 12

4 4
6 pg1 3 3 pg/

3w —Eh2—.1 3g —2E7r
2
d y
(b) As before, EI N (x) where N (x) is the bending moment due to section PB.
cbc2
This bending moment is clearly

N= f w dg - - (2/ -x)

2
W 212 - + -x12 wl (21 - x)= w - xl

(Here w= p g b h is weight of the beam per unit length)

dy x2 /
Now integrating, El —dx w ( T -x3 co

or since = 0 for x = 1, co = w 13/3

x4 x3 /) w/3 x
Interggrating again, EIy=w (— - + +C1
24 3

As y = 0 for x = 0, c1 = 0. From this we find

" Y(x- 5"4 /EI 5Pg14


24 1 7

1.304 The deflection of the plate can be noticed by going to a co - rotating frame. In this frame
each element of the plate experiences a pseudo force proportional to its mass. These

forces have a moment which constitutes the bending moment of the problem. To calculate
this moment we note that the acceleration of an element at a distance from the axis is
a = and the moment of the forces exerted by the section between x and 1 is
152

From the fundamental equation

E/5j= tp 1 h (13 — x3).

+1./2
1 h3
The moment of inertia / f z21 d z-
12.
-
Note that the neutral surface (i.e. the surface which contains lines which are neither
stretched nor compressed) is a vertical plane here and z is perpendicular to it.
d 2y 4 P13 (j3 — x3). Integrating
dx2 Eh2
4
ay 4 p (3 (13x x ) +c
dX E h2

Since A= 0 , for x = 0, c1= 0. Integrating again,

13x2
xs
Eh 2 20 +c2

Thus
5E h2

1.305 (a) Consider a hollow cylinder of length 1, outer radius r + Ar inner radius r, fixed at one
end and twisted at the other by means of a couple of moment N. The angular displacement p, at
a distance 1 from the fixed end, is proportional to both 1 and N. Consider an element of length
dx at the twisted end. It is moved by an angle q as shown. A vertical section is also shown
and the twisting of the parallelopipe of length 1 and area Ar dx under the action of the
twisting couple can be discussed by elementary means. If f is the tangential force generated
then shearing stress is f/Ardx and this must equal

GO= G ?1. since 0=


1' 1•

Hence, f G Ar dx
/•
The force f has moment fr about the axis and so the total moment is

23tr3 Ar
N— G Ar r dx = G
1 1
153

(b) For a solid cylinder we must integrate over r. Thus


r

dx f 2 a r3 dr cp G
N= 7( r4 G (1)
1 21
0

d2/2

3
f2nr dr G it
1306 Clearly N = 9) G (p (d24
32 1 -4)
d1/2

oN
using G= 81 G Pa = 84 x 101
m2

d2= 5 x 10 -21111, d1= 3 x 10 -2 M

ozp = 2.0° = radians, 1= 3 m


90
n x 8.1 x it ( ,)c
N- - 81) x 102 N•m
32 x 3 x 90 "--'

= 0.5033 x 103 N•m =, 0.5 k N•m

1.307 The maximum power that can be transmitted by means of a shaft rotating about its axis
is clearly N w where N is the moment of the couple producing the maximum permissible
torsion, cp. Thus
itr4G cp
P. co = 16.9 kw
2/

1.308 Consider an elementary ring of width dr at a distant r from the axis. The part outside
d
exerts a couple N + N—
dr on this ring while the part inside exerts a couple N in the
dr
opposite direction. We have for equilibrium
dN
dr dr = - dip
154

where dl is the moment of inertia of the elementary ring, 3 is the angular acceleration
and minus sign is needed because the couple N (r) decreases, with distance vanshing at
the outer radius, N (r2) = 0. Now
m 2'
dl = 2ardr r
ri)

2m 13 3
Thus dN r dr
(r2i — r2i)

Or,
N=2 1
m (r1 — r4), on integration

N-i-del
dr

1.309 We assume that the deformation is wholly due to external load, neglecting the effect of
the weight of the rod (see next problem). Then a well known formula says,
elastic energy per unit volume
1
— stress x strain = —
2 2
1 m
This gives — — 2 c. 0.04 kJ for the total deformation energy.
2 p Ee

1310 When a rod is deformed by its own weight the stress increases as one moves up, the
stretching force being the weight of the portion below the element considered.
The stress on the element dx is
p Jtr2 (1 - x) g/n r2 = p g (1 - x) The
extension of the element is
A dx = d & = p g (I - x) dx/E
1
Integrating A/ = — s the extension of
2 p g 12/E i
the whole rod. The elastic energy of the element is
1 2
g(1--X)
Pg(1 - X ) P
E r cfr
Integrating
Ai 2
2g2 /3/E _2 r)
AU= —1 70.2 n r21 E
6 r 3
155

1.311 The work done to make a loop out of a steel band appears as the elastic energy of the
loop and may be calculated from the same.

If the length of the band is 1, the radius of the loop R =


/7t. Now consider an element
2
ABCD of the loop. The elastic energy of this element can be calculated by the same sort of
arguments as used to derive the formula for internal bending moment. Consider a fibre at a
distance z from the neutral surface PQ. This fibre experiences a force p and undergoes
ds Z
an extension ds where ds = Z d p, while PQ = s = R d cp. Thus strain — = R. If a is the
s
cross sectional area of the fibre, the elastic energy associated with it is

_1 E (_Z)2 R dcp a
2 R
Summing over all the fibres we get
EIS I E 1 d (ID
2R
Z-_ 2R ds
For the whole loop this gives,

using f d cp = 21t, 21

n it 2E/7t 2

6/2

h 63
Now I = f Z2 hdz
12
-6/2

1 TE2Eh 83
So the energy is k
1 — 0.08 kJ
6
1.312 When the rod is twisted through an angle 0, a couple

N(0) —
then
r4 G
2/
0 appears to resist this. Work done in twisting the rod by an angle pis

7 J on putting the values. 4


IN(0)d0= / r
4

7cr 3 dr
0
G

1.313 The energy between radii r and r + dr is, liy differentiation,


/ G
7C r3 dr
G cp2 1 G cp2 r2
Its density is
27crdrl 1 2 12

1.314 The energy density is as usual 1/2 stress x strain. Stress is the pressure p g h. Strain is
px p gh by defination of (I. Thus

= p (p gh)2 = 23.5 kJ/m3 on putting the values.


156

1.7 HYDRODYNAMICS
1.315 Between 1 and 2 fluid particles are in nearly circular motion and therefore have centripetal
acceleration. The force for this acceleration, like for any other situation in an ideal fluid,
can only come from the pressure variation along the line joining 1 and 2. This requires
that pressure at 1 should be greater than the pressure at 2 i.e.

Pi >P2
so that the fluid particles can have required acceleration. If there is no turbulence, the
motion can be taken as irrotational. Then by considering

f 17t dr= 0

along the circuit shown we infer that


i
V2 > V1

(The portion of the circuit near 1 and 2 are


streamlines while the other two arms are at )

._-.-,
right angle to streamlines)
In an incompressible liquid we also have div i7=

0
By electrostatic analogy we then find that the density of streamlines is proportional to the
velocity at that point.
1.316 From the conservation of mass
V1 S1 V2 S2 (1)
But S1 < S2 as shown in the figure of the problem, therefore

vi > V2
As every streamline is horizontal between 1 & 2, Bernoull's theorem becomes
1
p+— v2 = constant, which gives
2
Pi < p2 as vi > v2
As the difference in height of the water column is Ah, therefore

P2 -P1 m P Oh (2)

From Bernoull's theorem between points 1 and 2 of a streamline


1 2 1 2
P1 + 2 P vi =P2 + 2 PV2
1 p (v2, vi)
Of, P2 - P1 m 2 (v1 -

or pgAh . p (vi — v22) (3) (using Eq. 2)


2
using (1) in (3), we get
_____> 1I
2gAh —
... S
Vi i 2 4 _ 4
-- - , ' i

Hence the sought volume of water flowing per see 2

2gAh
157

1.317 Applying Bernoulli's theorem for the point A and B,

pA = pB v2 as, vA = 0
2p
1 2
or, P v = PA -PB A h Po g

.1 /2 Ah po g A
So, v=v

V2Ahpog
Thus, rate of flow of gas, Q = S v = S

The gas flows over the tube past it at B. But at A the gas becomes stationary as the gas will
move into the tube which already contains gas.
1
In applying Bernoulli's theorem we should remember that p + v2 + gz is constant along
p -2
a streamline. In the present case, we are really applying Bernoulli's theorem somewhat
indirectly. The streamline at A is not the streamline at B. Nevertheless the result is correct. To
be convinced of this, we need only apply Bernoull's theorem to the streamline that goes
through A by comparing the situation at A with that above B on the same level. In steady
conditions, this agrees with the result derived because there cannot be a transverse pressure
differential.
1.318 Since, the density of water is greater than that of kerosene oil, it will collect at the bottom.
Now, pressure due to water level equals h 1 p1 g and pressure due to kerosene oil level
equals h2 p2 g. So, net pressure becomes h1 p1 g + h2 p2 g.
From Bernoulli's theorem, this pressure energy
will be converted into kinetic energy while - - La■-

maL

lfowing through the whole A. - ro s e n Oi


1 --
i.e. hi pi g + h2 p2 g = 1 2
2 pi v

P2
Hence v = 2 (h1 + h2 —) g = 3 m/s
Pi A
1.319 Let, H be the total height of water column and the hole is made at a height It from the
bottom.
_
Then from Bernoulli's theorem -
1 2
- pv (H - h) pg
2 r ."1". •••••

•••••

or, v = - h) 2g , which is direFted horizontally. ow/ HL

For the horizontal range, 1= v t

• \r"h _
= 1/2 g (H — h) 21I (Hh — h2)
158

d (H h — h2)
Now, for maximum 1, dh =0

which yields h = 2= 25 cm.

1.320 Let the velocity of the water jet, near the orifice be v', then applying Bernoullis theorem,

1 1 2
p v 2 = ho p g + p v

Or, = V v2 — 2g ho (1)
Here the pressure term on both sides is the same and equal to atmospheric pressure. (In the
problem book Fig. should be more clear.)
Now, if it rises upto a height h, then at this height, whole of its kinetic energy will be
converted into potential energy. So,
1 V12
— va = p gh or h
2 2g
V2
— ho g• 20 cm, [using Eq. (1)]
1321 Water flows through the small clearance into the orifice. Let d be the clearance. Then
from the equation of continuity
( 27( Rid ) vi = ( r d)v = (2arR2d) v2
R R
or v1 1 = v r = v2 2 (1)
where v1 , v2 and v are respectively the inward
radial velocities of the fluid at 1, 2 and 3.
Now by Bernoulli's theorem just before 2 and
•we
■•••■ ••■■■■
just after it in the clearance 1.11 mmi•

••■• •■•• ••■• •••••• ■••


■■11.

sum. ..■■■••

1 2 m• •• • ••■■•0
..m.on .6* ...•
•••■■■••

PO h Pg 132 + PV2 (2) ••••• ma /

•■•••••■•

■■■••

Applying the same theorem at 3 and 1 we find


that this also equals
1 2 1
p + — pv p0 + — pvi2 (3)
2 2
(since the pressure in the orifice is pc, )

From Eqs. (2) and (3) we also hence

vi= (4)

and po +
1 2
pvi

= p0 + hpg (1 — [Using (1) and (4)]


159

1311 Let the tome acting on the *ton be F and the length of the cylinder be 1.
Then, work done = Fl (1)
iso441NAny„ lat.mealli's N)attittill for points
2 ,__
A and B,p . -1- p v where p is the density / - ■
...ma. ■■•• •••

2
and v is the velocity at point B. Now, force on - V - ---- s
the piston, - -----.*:
- -__ --_- -z-- A-- .---a=
F - pA. p v2A (2) ..._, --
-
...,
_ _ --_ .... - -,. a
. . . w o w ,a ........................
■■•■••
.www.
..■...•
_ ..........

2
-........... 1 ...................- - - - - 1
where A is the cross section area of piston. 4. ..---..
Also, discharge through the orifice during time
interval t = Svt and this is equal to the volume
of the cylinder, i.e.,
V
V. Svt or v= -,-.,- (3)
at
From Eq. (1), (2) and (3) work done
1 1 A V2 /- /pv3/s31.2
i. -i pv 2Al- (as Al - V)
i P (St)2 2
1323 Let at any moment of time, water level in the vessel be H then speed of flow of water
through the orifice, at that moment will be
v -112-111- (1)
In the time interval di, the volume of water ejected through orifice,
dV - svdt (2)
On the other hand, the volume of water in the vessel at time t equals
V.. S H
Differentiating (3) with respect to time,
dV e, dH
-a —
dt or dV .. S d H (4)
Eqs. (2) and (4) dt
S dH
SdH= svdt or dt= ,from (2)
s Nrig1 7
r 0
Integrating,
fdt-
0
S
siih— f VII
k
dh

S VT;
Thus, t=
s g
1.324 In a rotating frame (with constant angular velocity) the Eulerian equation is
,
d v"
- V p + p g'+ 2 p 67' x (7.)') + p co 2 F..= p
dt
In the frame of rotating tube the liquid in the "column" is practically static because the
orifice is sufficiently small. Thus the Eulerian Eq. in projection form along 7*(which is
160

the position vector of an arbitrary liquid element of Tenth dr relative to the rotation axis)
reduces to

Thus p (r) = po +P-(2L) —2{r2 — (1— h)21 (1)

Hence the pressure at the end B just before the orifice i.e.

(2)

Then applying Bernoull's theorem at the orifice for the points just inside and outside of the
end B
1 2
Po + p (o2 (21 h — h2) = Po + p v ( where v is the sought velocity)
2

\r2 / —
So, v = wit Ti- — 1

ctiT' --a --,


1.325 The Euler's equation is V (p + p gz), where z is vertically upwards.
P Tit- — f — VP I. —
dv av , --1 „b, —,..
Now --. — + k . v j V. (1)
dt at v•

But V)v V (1 v2 rx Curl r (2)


2
we consider the steady (i.e. a,",at = 0) flow of an incompressible fluid then p = constant.
and as the motion is irrotational Curl v 0

So from (1) and (2) p V v2) = — V (p + p gz)

or, V p+Z pv2+pgz =0

2
Hence p + —1 p v p gz = constant.
2

1.326 Let the velocity of water, flowing through A be vA and that through B be vB, then discharging
rate through A = QA = S VA and similarly through B = S vB.
Now, force of reaction at A,
161

Hence, the net force,

F= pS (vA2 - vB2) as F A11 Fp (1)


Applying Bernoulli's theorem to the liquid
flowing out of A we get

1 2 •■•■••
amIND

.1••■•
em• •••••

■•••

PO + Pgh PO + PvA ••■ ea .


=WO

and similarly at B
1
= pvB
Po + Pg (11+ Ah) Po +
B
•mw

Hence (v8 - vd2i)


2 =Ahpg •■•••

••■

Thus F = 2pgSAh = 0.50 N

1327 Consider an element of height dy at a distance y from the top. The velocity of the fluid
coming out of the element is

v 1/2-Ty
The force of reaction dF due to this is dF = p dA v2, as in the previous problem,
= p (b dy) 2 gy
h

Integrating F = p gb f 2y dy
h-1

= p gb
[h2 - (h - 1)2]= p gbl (2h - 1)
(The slit runs from a depth h -1 to a depth h from the top.)

1.328 Let the velocity of water flowing through the tube at a certain instant of time be u, then

u. --2-, where Q is the rate of flow of water and n r2 is the cross section area of the tube.
r

pQui-pQuj. AN•1■0111. ■•■=1. al•■•

■N.

Hence, the sought moment of force about 0 ••••■• ■••

mama ■•

becomes
2
1 (-1)x(pQui-pQui)= pQulr= 2-2— 1i°
2
r

and I/71 P r2
Q21 0.70 Alin
▪ ▪ ▪
162

1329 Suppose the radius at A is R and it decreases uniformaly to r at B where S = 7r.R2 and
s = 7tr2. Assume also that the semi vectical angle at 0 is a. Then

L2 Li X

R-r
So Y r + (x - L 1)
L2 - Li
where y is the radius at the point P distant x from the vertex 0. Suppose the velocity with
which the liquid flows out is V at A, v at B and u at P. Then by the equation of continuity
3tR2v . 7tr2v . ty2u
7
The velocity v of efflux is given by
..... - _
v = 112 i _ _ ...
I-_
and Bernoulli's theorem gives - -_
1 2 1 _ _2 h .. _
pp + -f put = po + -2 pv2 _-.. 0
"P.
B
where pp is the pressure at P and pc, is the _
-
atmospheric pressure which is the pressure just ...-
outside of B. The force on the nozzle tending to
pull it out is then
A Bt 0
F = f (pp - pa) sin() Ityds
We have subtracted pc, which is the force due to atmosphenic pressure the factor sin 0
gives horizontal component of the force and ds is the length of the element of nozzle
surface, ds = dx sec 0 and
-r
tan 0 = R
L2-L1
Thus
L2

dx

4
Ttp f v2 (I - y dy

4 n(R2
r2)2
= 71 pv2 1 (R2 - rr2 +—r - r =
2 p gh ( 2
R2

= pgh (S - s)2/S = 6-02N on putting the values.


Note : If we try to calculate F from the momentum change of the liquid flowing out wl will
be wrong even as regards the sign of the force.
There is of course the effect of pressure at S and s but quantitative derivation of F fron
Newton's law is difficult.
163

-4,
1.330 The Euler's equation is p — f - V p in the space fixed frame where f = - p g k
dt
downward. We assume incompressible fluid so p is constant.
Then f = - V (pg z) where z is the height vertically upwards from some fixed origin. We go to
rotating frame where the equation becomes

2 -11
-V(p+ pgz) +pco r + 2p (i/ x ro)
the additional terms on the right are the well known coriolis and centrifugal forces. In the
frame rotating with the liquid v = 0 so
1 2
7 p + pg z - -2 p co 2
r- =0
or p+ pgz- p co2r2 = constant
2
On the free surface p = constant, thus

z r2 + constant
2g
If we choose the origin at point r 0 (i.e. the axis) of the free surface then "cosntant" = 0 and
f.12
z= r2 (The paraboloid of revolution)
2g
At the bottom z = constant
So p=p (1)2 r2 + constant
2
If p = Po on the axis at the bottom, then
1 2 _2
Po + -2 P
1.331 When the disc rotates the fuild in contact with, corotates but the fluid in contact with the
walls of the cavity does not rotate. A velocity gradient is then set up leading to viscous forces.
At a distance r from the axis the linear velocity is w r so there is a velocity gradient
co r
both in the upper and lower clearance. The corresponding force on the element whose
radial width is dr is
cor dv
Ti 2nrdr — (from the formular F = iiA )
h
The torque due to this force is
tor
23rrdr —r
h
ind the net torque considering both the upper and lower clearance is

2f T12 itr3 dr 51-)


o
= iciewri/h
So power developed is
ecozpii
P v •n = 9.05 W (on putting the values).
(As instructed end effects i.e. rotation of fluid in the clearance r > R has been neglected.)
164

1.332 Let us consider a coaxial cylinder of radius r and thickness dr, then force of friction or
viscous force on this elemental layer, F = 2 It r 1 1 —dv.
dr'

This force must be constant from layer to layer so that steady motion may be possible.
F dr
or, = 2n 1 dv. (1)

Integrating,
r

F f = 27( I rl f dv

R2

or, F In = v (2)

Putting r = R1, we get


R1
Fin-- 2nlrl vo
1.2

From (2) by (3) we get,


In r/R2
v = vo
In R1/R2
Note : The force F is supplied by the agency which tries to carry the inner cylinder with
velocity vo .
1.333 (a) Let
us consider an elemental cylinder of radius r and thickness dr then from Newton's
formula
dw d(O
F= 2nrli r = 2n/Tir 2
dr dr

and moment of this force acting on the element,


do) 3 d
N= 2nr 2lrl — r= 2nr 11 w
dr
dr

dr
or, 2 n / do) = N --3- (2)

As in the previous problem N is constant when


conditions are steady
r
0)

dr3r
Integrating, 2 n 1 ri fd w = N f—

Rt

[ 1 11
or, 23t/lw= N (3)

Putting r R2, CO = w2 , we get

[ 1
2yr/yi (02= -2- - 11 (4)
R1 R2
From (3) and (4),
1 1

(b) From Eq. (4),


D2 D2
N 1 %1 '2
N1 = —1 = 4 n ri (o2 2 2
R R
1.334 (a) Let dV be the lfowing per second through the cylindrical shell of thickness
dr then,
r2 r3
dV = — (2 71 r dr) vo(1 — v?i). 2 n vo (r — dr
R2
R

( r3 R2 31
V= 2nvo r — --2-) dr = 2 n v ...
R ° —4 —2 R v°
o

Fr
(b) Let, dE be the kinetic energy, within the above cylindrical shell. Then
1 1
dT = — (dm) v= — (27t r 1 d rp) v2
2 2
1 • 2 r2 2r
R R

(r 2r3 nR2plvo
T= nip v20 — — +R4 = 6

Here frictional force is the shearing force on the tube, exerted


dv

dt.
r2

dv r
2 vo R2-
dr —

dv 2 vo
And at r= AR = -
' dr R
166

dv
Then, viscous force is given by, F - - Ti (2.7t RI) (
dr r . B

2v,,
= -27cRi/(- = 47cTivo/
R

(d) Taking a cylindrical shell of thickness dr and radius r viscous force,


dv
F= - Ti (231 rl) —
dr '

Let Ap be the pressure difference, then net force on the element = Ap it r2 + 2 7( 11 r —

But, since the flow is steady, Fn et= 0


dr
dr

r
dv
- 2 1 t 1 i r ( - 2 v o . 71
- 2nhir— 2
or, Ap- dr = 4 .ri vo /A?
nr2 n r2

1.335 The loss of pressure head in travelling a distance I is seen from the middle section to be
h2 - h1= 10 cm. Since h2 - h1 = h1 in our problem and h3 - h2 = 15 cm = 5 + h2 - h1 ,
we see that a pressure head of 5 cm remains incompensated and must be converted into
kinetic energy, the liquid flowing out. Thus
i,2
-P- - = pg A h where A h = h3 - h2

Thus v = 15T h a 1 m/s


1.336 We know that, Reynold's number (Re) is defined as, Re.. p v Iii, where v is the velocity
1 is the characteristic length and Ti the coefficient of viscosity. In the case of circular cross
section the chracteristic length is the diameter of cross-section d, and v is taken as average
velocity of flow of liquid.
Pdi v1
Now, Ret (Reynold's number at x1 from the pipe end) - where v1 is the velocity
m1
at distance x1

p d2 V2 Re, 1 v1
and similarly, .. SO - d V2
Re ,
1 Re, d2
From equation of continuity, Al v1= A2 v2
2
or, it ri vi = it r22 v2 or di vi ri .. d2 v2 r2
- ax
2

div 1 r2 r e e- a Ar (as x2 - x1= Ax)
0
au :
d2 V2 r1 roe - axt
Re,
a ex e.
Thus - e - ,
Re,
1.337 We know that Reynold's number for turbulent flow is greater than that on laminar flow:
pvd . 2 pi vi ri 2 p2 v2r2
Now, (Re)i- and (RA :.
TI 11 1
167

But, (ROt a (R e)1


pi viri 12
SO v2 mu at 5 it m/s on putting the values.
21 P2 r2

v d
1.3311 We have R = and v is given by
1
4 it
67rrirv=
r2 (p - po) g
3
(P = density of lead, Po = density of glycerine.)
2 1
v a. 11- (1) P°) g r2 UTI (P 13°) g d2

1 1
Thus = 2 (13 Po) g Po d3
4 18
1/3 5.2 nun on putting the values.
and d = [9 ri2/pc, (p - Po) g ] =

dv
1339 m— mg - 6 n ry
dt
dv 63trir
or —+ m v = g
dr
+kv. g,k.. Ltil
or dv
dt m
ict (IV kt
kt kt kt
or e dT fice v- ge or -c/- e v - ge
dt
la g g - kt
Or ve .. - e il + C or v- (where C is const.)
Ce
k -k +

Since v. 0 for t= 0, 0= 1+C

The steady state velocity is fc.

v differs from by n where e- n


1
or t = - ln n
k
4n
r3 P
1= 3 4r2p d2p
Thus - --
k- 6 n II r 18 r)
We have neglected buoyancy in olive oil. 18 1
168

1.8 RELATIVISTIC MECHANICS

1.340 From the formula for length contraction

(10-10117 )- fi lo
2
V
So, 1 - -2- = (1 - 1)2 or v - c Itri (2 - ri)
C
1.341 (a) In the frame in which the triangle is at rest the space coordinates of the vertices are

/3- a
(000), a( — , + -, 0 a
a NIT
— , - -0 , all measured at the same time t. In the moving
2 2'
frame the corresponding coordinates at time t' are

A : (ve, 0, 0), B :( g-v3-.N r1 --.p2+ ve 1( 0) and C :: (lf- V31/1 - 132 + Vti , - g-, , 0
2 2' 2 2
The perimeter P is then
in

P= a + 2a (14 (1 -132) + 14) = a (1 +114-737 )

(b) The coordinates in the first frame are shown at time t. The coordinates in the moving
frame are,

B 8
L. oafs \
2 ) —2- ,Cy

0)0,0) )
C ca,o,o

A : (vt', 0,0), B : (11117112 + vti , a _s_o), c: (a VITT + ve , 0 , 0)

The perimeter P is then


1/2 V
P= a11 -72- +1[1-132 +3] x 2- a ( 71 71 +"4 - 1-32- ) here 13= c

proper length /0
1.342 In the rest frame, the coordinates of the ends of the rod in terms of
A : (0,0,0) B : (l0 cos00 , /0 sin00 , 0)

at time t. In the laboratory frame the coordinates at time t' are

A : (ve, 0, 0), B :(10cos00\117-1-32 + ve ,10 sin00 , 0)


169

Therefore we can write,


1
1 cos 00 = 10 cos00 117 2- and / sin 0 = 10 sin00

cost 0 + (1 - 132 ) sine 0


Hence q . (12) 1-12
or, - 132 sin2 0
-132 A
1.343 In the frame K in which the cone is at rest the coordinates of A are (0,0,0) and of B are
(h, h tan 0, 0). In the frame K', which is moving with velocity v along the axis of the cone, the
coordinates of A and B at time t' are

Vt _A2 x'B - x'A

and the lateral surface area is,


S = n h'2 secO' tan0'

h2 (1 ti2) tan° tan2 /32 0


50 11 cost

N :

Here S0 - n h2 sec° tan° is the lateral surface area in the rest frame and

h'= h111771 , 13- vie.

1.344 Because of time dilation, a moving clock reads less time. We write,

t- At= rili17 (32,


2
2At (At 1 .112
Thus, 1- + =
t t

or, v c At- 2- A—f)


t t
1.345 In the frame K the length 1 of the rod is related to the time of flight At by
1= v At

In the reference frame fixed to the rod (frame K')the proper length 10 of the rod is
given by

But
170

v At
Thus, v At' =
V-1-17

and

1.346 The distance travelled in the laboratory frame of reference is vA t where v is the velocity
of the particle. But by time dilation
Ato
At= So v = c / /1 ( At 0 / 6■0 2
V
1 v 2 /c 2

Thus the distance traversed is


cAtV1 - (Ato/At)2

1.347 (a) If to is the proper life time of the muon the life time in the moving frame is
VT
TO o
and hence 1..
V — y2/c2 111-v2/c2
1
Thus to- V 1 — V2/C2
V

(The words "from the muon's stand point" are not part of any standard terminology)

1.348 In the frame K in which the particles are at rest, their positions are A and B whose
coordinates may be taken as,
A : (0,0,0),B = (10,0,0)
In the frame K' with respect to which K is moving with a velocity v the coordinates of A and
B at time t' in the moving frame are

A = (ve, 0,0)B = (10111.-7 +ve,o,o), p =

Suppose B hits a stationary target in K after


time t'B while A hits it after time tB + At. Then,

/A B
v At
So,
1° — y2/c2
1.349 In the reference frame fixed to the ruler the rod is moving with a velocity v and suffers
Lorentz contraction. If 10 is the proper length of the rod, its measured length will be
171

In the reference frame fixed to the rod the ruler suffers Lorentz contraction and we must
have

A x 2 NirT3' - /0 thus l0 = rAirA—x-2

Ax,
and 1 — ir --- or V= c 1—
AX2 6`x2

1350 The coordinates of the ends of the rods in the frame fixed to the left rod are shown.
The points B and D coincides when
Cl — 10
10= c1 — vt0 or to =

The points A and E coincide when

ci + /0
0= cl + 10 V1 7 — vti , tt
V

From this

1351 In K0, the rest frame of the particles, the events corresponding to the decay of the particles
are,
A : (0,0,0,0) and (0,10, 0,0) = B
In the reference frame K, the corresponding coordintes are by Lorentz transformation

Now

by Lorentz Fitzgerald contraction formula. Thus the time lag of the decay time of B is
v/0 vI vl
At

B decays later (B is the forward particle in the direction of motion)

1.352 (a) In the reference frame K with respect to which the rod is moving with velocity v, the
coordinates of A and B are

A: t, xA + v (t — tA ), 0, 0 B :

t, xB+ v (t — ta),O, 0
172

Thus 1= xA - x B - v (tA- tB) =

XA XB - V OA tB)

B A

1353 At the instant the picture is taken the coordintes of A, B, , B' in the rest frame of A B
are
A: (0, 0, 0, 0) B:

(0, 10, 0, 0) B':

(0, 0, 0, 0)
A
A' : (01 - /0117 , 0, 0)
In this frame the coordinates of B' at other times are B' : (t, vt, 0, 0). So B' is opposite to B at
10
time t (B) — . In the frame in which B' , A' is at rest the time corresponding this is by

Lorentz tranformation.
1 to vio
t° (B' ) =
v 2 v

V
( c
Similarly in the rest frame of A, B, to coordinates of A at other times are

A' :(t, - /0 V17722 + vt, 0, 0


c
/0
A' is opposite t.,A at time t (A) .=
The corresponding time in the frame in which A' , B' are at rest is
/0
t yt(A).. —
v
1 ( vx
t
1.354 By Lorentz transformation t' =
c
1- -7
173

, vx 1
So at time t= 0, t
c2

Ifx > 0 t' < 0, if x < 0, > 0 and we get the diagram given below "in terms of the K-clock".

K:

K:
The situation in terms of the K' clock is reversed.

1355 Suppose x (t) is the locus of points in the frame K at which the readings of the clocks of

both reference system are permanently identical, then by Lorentz transformation

1 (t Vx (t))
t' = = t
1/1 - V2/c2 c2

C2( A nr72:. V
So differentiating x (t) = 1- 1- = (1 - VF7)

Let = tan h0 , 0 s 0 < oo , Then

cos h0
x (t) -111 - tan h2 0) =c 1
tan h0 sin cosh0

cosh0-1 Vc o s h 0 - 1 e
c c = ctanh— s v
s in h 0 cosh0+1 2

-(tan h 0 is a monotonically increasing function of 0)

1356 We can take the coordinates of the two events to be

A: (0, 0, 0, 0) B: (At, a, 0, 0)

191
For B to be the effect and A to be cause we must have At >

In the moving frame the coordinates of A and B become


1
A : (0, 0, 0, 0), B :[ y (At - , y (a - V At) , 0, 01 where y

Since
J. (Vc
2

2
2
(At, ai: 112 1 ai27) 2
At - (a - V At)2 = >0
(&)
C2

a'
we must have At' >
174

1357 (a) The four-dimensional interval between A and B (assuming Ay = Az = 0) is :


52
- 32 = 16 units
Therefore the time interval between these two events in the reference frame in which the
events occurred at the same place is
c (t'B - t'A ) Ct
= 4m i
7 B
or t'B - t'A = 4= -8 6 •
x 10 S
c
3 5
(b) The four dimensional interval between A 4 4
C
and C is (assuming Ay Az = 0) 3
32 _ 52
-16 2
So the distance between the two events in the frame
in which they are simultaneous is 4 units = 4m. 0 1 23 4 6' 6

1358 By the velocity addition formula


-V
vx v Nil -V2/c2
vx' = V = y
V vx ' Y v V
1 - ----5- 1- x
C- C2

il(vx - v )2 + (1 - y2/c2 )
and =
v' =
x y
1— vx 2V
C
1359 (a) By definition the velocity of apporach is
dxl
- (- v2) = vi + v2
v ch = — - dx2 v1
aPPma dt dt
in the reference frame K .
(b) The relative velocity is obtained by the transformation law
1,1 -(-V2) v1+ v2
V =
1/1 (- V2) Vi V2
1 1+
C2
rod is
1360 The velocity of one of the rods in the reference frame fixed to the other
v+ v 2v
V= -7r =
11` 1+13
1+-2-

The length of the moving rod in this frame is

V 4 1/2/c2 1
lo (14.r2)2 = -1+ p2

1.361 The approach velocity is defined by


di+ di+1 v
2 " v2
VaPPrcach = dt dt 2
in the laboratory frame. So Vappmach - v vi + v2
175

On the other hand, the relative velocity can be obtained by using the velocity addition
formula and has the components

1/2 __2 2 Vi 2
[ - v1 , V2 V 1 -— SO
2
21 V• = VI+ V2 - C
C
1.362 The components of the velocity of the unstable particle in the frame K are

(V, V 1-- -,0


c
so the velocity relative to K is
r 2 T72
Vv2 +vr2_V v
2
C
The life time in this frame dilates to
V2 vr2 v,2 V2
AtO / V 1 4
v-'
C2 C2 c
and the distance traversed is
Vv2 + v/2 (vr2 v2 ) c2
'64
V 1 - V2/ c2 1/1 - V12 / C2
1.363 In the frame K the components of the velocity of the particle are
v cos 0 - V
Vix
1 V V COS 0
2
C

v sin 011 -1.772 2-


v'y =
VV
1 - -T cos 0

v' v sin 0
Hence, n 0' - - 1/(1 - V2 )/c2
ta v'Y v cos 0 - V
1.364 In K' the coordinates of A and B are
A: (t', 0, - v' t', 0); B : (t',1,- v' t', 0)
After performing Lorentz transformation to the frame K we get
,V1
A:t= B:t=y(t

yVe\x= y(1+Vt')
y= t' y= - t'
z=0 z=0
„ V/
By translating t t - -T, we can write

the coordinates of B as B : t = y t'


176

V1
y= - v' (t, - --I) , z= 0
c

Thus Ax = /1F 11 , Ay = V
yi O
c
Hence tan 0' -
2

VI V

2 \ F-1717
C 1 - -7_
-

t I + dt
1.365
-4. -ii. --.).
v v + w dt

--0 -0 c
In K the velocities at time t and t + dt are respectively v and v + wdt along x - axis which
is parallel to the vector V. In the frame K' moving with velocity V with respect to K, the
velocities are respectively,
v V v + w dt - V
and
vV V
1- , 1 - (v + wdt) --f
c"- c
The latter velocity is written as

i V2
wdt (A - 2)
v- V w dt v- V wV „. v- V c
+
V v + —1- "" = ,+
(1 -Y1 c 4 V 2
1-v— 1-vV 2 I - V-- (1 _ V;)
C2 C2 C 2
C

Also by Lorentz transformation

dt - V / c2 1 - vV/c2
de = dx = dt
Vi_v2/c2 111 - V2/c2
Thus the acceleration in the K' frame is

, dv' w
w = --= 3 1-
de V 712)3/2

(b) In the K frame the velocities of the particle at the time t and t + di are repectively
(0, v, 0) and (0, v + wdt, 0)

where V is along x-axis. In the K frame the velocities are

(- V, v1/1 - V2/c2 , 0)

and (- V, (v + wdt)11171 , 0) respectively


177

Thus the acceleration


wdt V(1 — V2/c2) V2
= • w 1 — — along the y—axis.
2
dt
We have used dt' =
— V2/c2
1.366 In the instantaneous rest frame v = V and
3/2 (from 1.365a)

(1 -

dv
So, w' di
3/2
( V2
1—
C2
WI is constant by assumption. Thus integration gives
w' t
V 1°

( t ) 2

2
( C+ (WI 12 1
Integrating once again x —,
w
1.367 The boost time to in the reference frame fixed to the rocket is related to the time t elapsed
on the earth by
.1/2

0 = 1 — -c72
1 2
dt

0
dt f 0
1+ (-"Lt-)

1 1
For 46 1,
mo V2 1-11T Arrii
1.369 We define the density p in the frame K in such a way that p dx dy dz is the rest mass
dm0 of the element. That is p dr dy dz = Po dxo dyo dzo , where po is the proper density
thco , dyo , dzo are the dimensions of the element in the rest frame K. Now
2„
/
dy= dyo , dz= dzo, dx= ciao _
1 C2
178

if the frame K is moving with velocity, v relative to the frame Ko. Thus
Po
P=
c

or v= C 1- -4- i) . c irl (T#- '1l)


a + To' 1 +1
1.370 We have
Pri v 1 2.--
m0 2
"` P or, un M
O+ c
V -1- -712
c
2

Or

c
P C
Or v= S

2
1ip2 + mo, c2 V 1 4. (mo 7
P

So c — v .• 1— 1+ (-m 95)
1 (mo c
lx 100 % • 7_. - -) x 100 %
c P 2 p

1.371 By definition of TI,


v 2
m0 v 1
= I) mo v or 1— i = 2
c 1
141-7
1— —
C2

or v.c 1 — 2= .L.V1771
n 11
1372 The work done is equal to change in kinetic energy which is different in the two cases
Classically i.e. in nonrelativistic mechanics, the change in kinetic energy is
?nco c2 ((0.8)2 - (0.6)2) s,— 0.28 = 0.14 mo c2
2 2 m° 2 c
Relativistically it is, .
2 2 2
mo c2 m0 c mo c mo c
= m c (1.666 — 1.250)
2
-
1/1 — (0.8)2 V11 — (0.6)2
=
0-6 — 0.8 °
= 0416 mo c2 = 042 mo c2
179

2
mo C
1373 2
' 2 m0 C
2

2
1
Or or 1 — V
g- : = 4
c 2
2
c
v V-3- V3-
or —.. i.e. v .• c
2 2
1374 Relativistically

T 1 1 2 3 4
—1 = +80
2 m1 2 I(
mo C (V177
2
2T 02 \ 2
3 2T 3 ( 2T )
So PL lig ? 2 -di (Prd) .I. - 4 2
no C /fto Cc2 - Mo C

1/2 1 3 T
[ 2T _ 3 T2 I . \ Fr (1 — 4 mo c)
Thus —Pro 'I 2 2
MoC Mo C4 m0 C 2

{27-.-- f3rei — Pet 3 T


But Classically, f3d= - 2 SO a i• 4 1 E
mo c Pct mo c
T 4
Hence if <
2 i£
MO c

the velocity f3 is given by the classical \formula with an error leas than E.

1.375 From the formula

E_

2 2 2 2 2
we fmd E = C2 P 2 + mo c2 c4 or (m0 cc2 + 7) = cp + ma cc4

1J
or T (2 mo c2 + 7) = c2 p2 i.e. p = — V T (2)710 c2 + 7)
c

1376 Let the total force exerted by the beam on the target surface be F and the power liberated
there be P. Then, using the result of the previous problem we see

F = Np = :1-V- VT (T + 2m0 c2) =


c
L VT (T + 2m0 c2)
since I= Ne, N being the number of particles striking the target per second. Also,
2
MO ( 1 I
P= N
1I 1 _ v2 / cZ - M° C = -e- T

These will be, respectively, equal to the pressure and power developed per unit area of
the target if I is current density.
180

1377 In the frame fixed to the sphere :- The momentum transferred to the eastically scatterred
particle is
2 my

NT:7
1
The density of the moving element is, from 1.369, n

and the momentum transferred per unit time per unit area is
2 my 1 v 2 mn v2
p= the pressure = n =
v7,_2 v2
v2 1_ --2-
c2 c2 c
In the frame fixed to the gas :- When the sphere hits a stationary particle, the latter recoils
with a velocity
v+v 2v

1+ 1

m•2v
2
1 + v2/C 2 my
The momentum transferred is 2
2 v
4v2/c
1 - - 2
(1 - V2/C2 )2

2 my 2 mnv2
and the pressure iis ---2- • n • v =
1— 1—
c c
1.378 The equation of motion is
d mo v
=F

c 2) 22 2
m 2 rn0
or using x — 0 at t — 0, we get, x = c2 t 2' + F
F
181

-/ 2 2 2 C2 t
1.379 x= va +c t , so x . v —
a2 + c2 t 2
2 2
v C t mo c
or, is Thus d ( mo v ME F
iiT. 152. a dt 1 7 2— a

1.380 F=

Thus
K. ,vrw=iii _. ..,_. __. __.
w= v, wiv
, —a, :-.

--o•
W
0 3/2 P W ' V , WI V
:i.*1 - mo (1 _

1.381 By definition,

Vdx
(dt — c i)
3 dt1 3
E — V px ,
E' - MO C Tcs — C MO Y ds
1.17:77
c2
1382 For a photon moving in the x direction
E = cpx , py = pz= 0,

1 E) Nr1 -77C
In the moving frame, e' = E—V = E
ifi 7F2- C 1 1- V/C

E . 3 3c
Note that Ei = — i or 11.. — V=
2 f' 4= 1+13 5' —5 •

1.383 As before
3 dt cbc
E= moc Ts , Px= 'floc ds •
182

ft dz
Similarly py . mo c ds , PZ mo c ds
°I

Then E2 _ c2p2 . E2 _ c2. (px2 +py2 +px)

2 4 (c2 — —
=F Pft0 C dt2 dx 2ds2 dye — d22) = mc4 is invariant

1384 (b) & (a) In the CM frame, the total momentum is zero, Thus

V cPix VT(T+ 2J/10 C2 ) 1/ T


2 2
c E1+ E2 T + 2m0 c V T + 2m0 c

where we have used the result of problem (1.375)


Then
2
1 1 .V7. + 2M0 C
m
V1— V2/c2 .1 / T 2M0 C2
V1—
T + 2m0 c2
Total energy in the CM frame is

2M0
c2
— 2mo c
2 VT+ 2m0 c
c2
2
— V 2 mo c2 (T + 2m0 c2) = 7' + 2,710 c2
Vi — V2/c2 2mo

So T= 2m0 c2 ( 1+ ---I- 1)
2m0 c`

Also 2 Vc2/3/2 + 4 c4 = V2m0 c2 (T + 2m0 c2) , 4 c2;2 = 2m0 c2 T, or /5 = 1 mo T

1.385 Mo c2 = VE2 — C2p2

V(2M0 c2 + 7)2 — T (2m0 c2 + 7) = 11 2m0 c2 (2m0 c2 + 7) = c112m0 (2m0 c2 + 7)

11---7
Also cp = VT (T + 2m0 c2) , v = C?-= c
T + 2m0 c2

1386 Let T = kinetic energy of a proton striking another stationary particle of the same rest
mass. Then, combined kinetic energy in the CM frame
183

1.387 We have
E1 + E2 + E3 i +p2 +p3 = 0
ma mo C.2,
P
2
Hence 2
("70 C2 — E1)2 — c2/1 = (E2 + E3) — ()P2 +P3 C

The L.H.S. = (M02 C4 — E1)2 — C214 = (74 + M21) C4 — 2M0 c2 Ei


The RH.S. is an invariant. We can evaluate it in any frame. Choose the CM frame of the
particles 2 and 3.
. (m2 m3)2 c4
In this frame RH.S. (E'2 + E'3)2 = i

(m20 + m2i) c4 2m0 c2 El (m2 + m3)2 c4


Thus — =

mo2 + m2i _
— (m2 + m3)2
c2 E1 s 1 mo2 + m2i m3)2 1 c4 ,
or 2 mo — (m2 + or E1 s c2
2 mo
1.388 The velocity of ejected gases is u realtive to the rocket. In an earth centred frame it is
V—U

vu
1 — —2"

in the direction of the rocket. The momentum conservation equation then reads
v-u
(m + dm) (v + dv) + (- dm) = my
, u
i -v
c2
or mdv-(v-14 -v dm= 0
1
1 —u 2
c
Here — dm is the mass of the ejected gases. so
uv2
—U+ 2
C V2
m dv — dm = 0, or mdv + u 1- u dm= 0
1 —UV 7 c2
c
uv
(neglecting 1— —i- since u is non- relativistic.)
c

Integrating ((3 . 1-1 , - i1 P — + -c •1 11 1 — 0, In Lf- E + u4 1 n m = constant


c 1 — 112 c m 1 — 13 c
u
The constant = — In mo since (3 = 0 initially.
c

Thus Or
PART TWO

THERMODYNAMICS AND MOLECULAR PHYSICS

2.1 EQUATION OF THE GAS STATE • PROCESSES


2.1 Let mi and m2 be the masses of the gas in the vessel before and after the gas is released.
Hence mass of the gas released,
Ms= mi - m2
Now from ideal gas equation
R R
pi V= m1;v7 To and p2 V= m2 117 To

as V and T are same before and after the release of the gas.

m1- R R
SO, (P1 -PO V= ( Am -A1 To

(P1 -P2) VM Ap VM
or, AemM= (1)
R To R To

R M p
We also know p= p :iv T so, I TIT = p—0 (2)
M
(where po = standard atmospheric pressure and To = 273 K)
From Eqs. (1) and (2) we get
.78
Am= p11•. .= 1-3 x 30 x 2— = 30g
Po 1

2.2 Let m1 be the mass of the gas enclosed.


Then, pi V= vi R Ti
When heated, some gas, passes into the evacuated vessel till pressure difference becomes Ap.
Let p'1 and p'2 be the pressure on the two sides of the valve. Then
p'1 V= v'iR T2 and
185

v P1 V P11 V)
112 r = (R Ti—R T2

But, Pii -Pi2 = AP

P1 P12 +.OP 7,
So, P'2
= Ti - T2 )A 2

or, P2'

2.3 Let the mixture contain v1 and v2 moles of H2 and He respectively. If molecular weights
of H2 and H. are M1 and M2, then respective masses in the mixture are equal to
m1= v1Mi and m2 = v2 M2
Therefore, for the total mass of the mixture we get,
m MI MI + M2 or m= vi Mi + V2 M2 (1)
Also, if v is the total number of moles of the mixture in the vessels, then we know,
v = vi + v2 (2)
Solving (1) •and (2) for v1 and v2, we get,
(v M2 - in) m — iiMi.
vi. v —
M2—M1 ' 2 M2—M1

m1 Mi (v M2 — m)
or,
m2 M2 On — V M1)

One can also express the above result in terms of the effective molecular weight M of the
mixture, defined as,
m RT
M= —=m
v pV

mi Mi M2— M 1 — M/M2
Thus, _ /Mi
m2 M2 M — M1 M —I

Using the data and table, we get :


m1
M = 3.0 g and, = 0.50
m2
186

2.4 We know, for the mixture, N2 and CO2 (being regarded as ideal gases, their mixture too
behaves like an ideal gas)
p17- vR T, so po V— vRT
where, v is the total number of moles of the gases (mixture) present and V is the volume of the
vessel. If v1 and v2 are number of moles of N2 and CO2 respectively present in
the mixture, then
v a• Vi + V2
Now number of moles of N2 and CO2 is, by definition, given by
m
1 m2
3/1 = wi. and, v2= m2
is the mass
where, m1 is the mass of N2 (Moleculer weight = M1) in the mixture and m2 of CO2
(Molecular weight = M2) in the mixture.
Therefore density of the mixture is given by

mi + m2 mi + m2
P—
V = (vRT/P0)

Po ml + m2 _ Po (mi + m2) 1/111/12


— RT v1 + v2 RT (1711 M2 + M2 Mi)

= 1.5 kg/m3 on substitution

2.5 (a) The mixture contains v1 ,.v2 and v3 moles of 02, N2 and CO2 respectively. Then the
total number of moles of the mixture
v . vi + v2 + v3
We know, ideal gas equation for the mixture
RT
pV= vRT or p— v v

(v1 + v2 + v3)RT
or, P= = 1 968 atm on substitution
V
(b) Mass of oxygen (02) present in the mixture : m1= v1M1
Mass of nitrogen (N2) present in the mixture : m2 = v2 M2
. Mass of carbon dioxide (CO2) present in the mixture : m3 = v3 M3 So,
mass of the mixture
m= mi+ m2+ m3= v iM 1+ V2 M2 -1- V3 M3

mass of the mixture


Moleculer mass of the mixture : M —
total number of moles
+ +
V1 M1 V2 M2 V3 M3
= + + = 36.7 g/mol. on substitution
v1 v2 v3
187

2.6 Let pi and p2 be the pressure in the upper and lower part of the cylinder respectively at
temperature To. At the equilibrium position for the piston :
mg
= p2 (m is the mass of the piston.)
pi S + mg = p2S or, pi + s
R To
But pi = Vo (where Vo is the initial volume of the lower part)

RT0 RTo 2ng, RT0 1 - 1


So, T V0+ S or, (1)
I Vo S Vo
Let T be the sought temperature and at this temperature the volume of the lower part
becomes V', then according to the problem the volume of the upper part becomes V'
mg RT (1 1\
S — (2)
Hence, 111 )
From (1) and (2).

Or, T—

As, the total volume must be constant,


Vo (1 + Ti)
V0(1 +1) = V' (1 ) or, V' .
(1+11 )
Putting the value of V' in Eq. (3), we get
(1 +TO
To
VA
v (1 +
T
1
Vo 1—ii,

To (ri2 — 1)
— 0.42kK
0112 — 1)1

2.7 Let pi be the density after the first stroke. The the mass remains constant
Vp
V p = (V + AV) pi , or, PI= (V + AV)
i=
Similarly, if p2 is the density after second stroke

V
V 2
Po
V pi = (V + AV) p2 or, In P2 (V + A V) Pi (V + AV o

this way after nth stroke.


n
V
Pn (V + AV) Po
Since pressure a density,
188

V
Pn - (V+ AV) po (because temperature is constant.)

Pn 1
It is required by — to be
Po rl

1 V 17 + AV\n
so, or,
ri V+ AV V

In
Hence n
In (1 + A V
—17

m RT
2.8 From the ideal gas equation p = —
M
LIE RT dm
(1)
dt MV dt
In each stroke, volume v of the gas is ejected, where v is given by
V
v = — [MN _1- MN]
MN

In case of continuous ejection, if (mN_ i) corresponds to mass of gas in the vessel at time t, then
mN is the mass at time t + At, where At, is the time in which volume v of the gas

has come out. The rate of evacuation is therefore —ii.e.


AtAt
V m (t + At) - m(t)
C- - -
At m (t + At) At
In the limit At - 0, we get
V dm
C — (2)
m dt
From (1) and (2)
C mRT C
or -f-dt
dt V MV 17 P p V
Po 0
f gp_ c c
Integrating = - — f dt or In E. - - t
P V Po V
P t
Thus -ct/v
P Po e
2.9 Let p be the instantaneous density, then instantaneous mass = Vp. In a short interval dt
the volume is increased by Cdt.
So, V p = (V + Cdt) (p + dp)
(because mass remains constant in a short interval dt)
189

lcE C
SO, — dt
P V
ce_ C,
Since pressure a density -— at
P V
P2

Or
f _ Lip. . C-. t
P V'
Pt
V Pi V
or t = — In — = In 1— 1.0 min
C p2 C fi
2.10 The physical system consists of one mole of gas confined in the smooth vertical tube. Let
m1 and m2 be the masses of upper and lower pistons and S1 and S2 are their respective
areas.
For the lower piston
P 32+ m2 g "` Po 32 + T,
or, T = (p - po) S2 + m2 g (1)
Similarly for the upper piston
Si + T + ms = p51,
Po
or, T= (p - po) S1- m1 g (2)
From (1) and (2)
(S1- S2) =
(P - Po) (mi+ m2) g

or, (13 -Po) .6` S = mg

so, p = j+ po = constant
AS
From the gas law, p V= v R T
p AV= vR A T (because p is constant)

SI= RAT,
So, (13a + I6s A/

1
Hence, AT= ii (p0AS+mg)1= 0.9K

D 2
po - a V2 =
2.11 (a) p= po - a (9
P
(as, V = RT/p for one mole of gas)

Thus, T= v--
R V■:1-13 PO - P - RVii V17; 9271713 (1)

/
d 1)13132 - P3 )
For Ti.„ must be zero
dp
190

2
which yields, P' 3 Po (2)

1 2 2 2 (// Po
Hence, Tmax.= R 3Po Po 3Po "' R 3a

(b) Poe-Pv 'Poe— fl RT/p


RT Po
so
In &- and T -2--- In — (1)
R p
- dT
For T. the condition is wij-, 0 , which yields

Po P
Hence using this value of p in Eq. (1), we get
P.
Tmax e 13 R

2 T2
2.12 T= To + a V2 T0+a R_—7—
P
(as, V = RT/p for one mole of gas)

So, p= VC; RT (T - T0 )1/2 (1)

c12.
For 0 , which gives
prain dT =
T = 2 To (2)

From (1) and (2), we get,


prnin R 2T0 (2T0 - To) -( 2R Va To

re on
2.13 Consider a thin layer at a height h and thickness dh. Let p and dp + p be the pressu
nward
the two sides of the layer. The mass of the layer is Sdhp. Equating vertical dow
force to the upward force acting on the layer.
Sdhpg+(p+dp)S. pS

So, .11 p g (1)


dh (P+dp)s
d
But p, RT, we have dp = T,
1 sdh .R5
or, - eldT.Ipgdh 1
So,
dT
dh- R
34 Kam 4
above bottom.
That means, temperature of air drops by 34°C at a height of 1 km
191

It
2.14 We have, = - pg (See 2.13) (1)
dh

dp
But, from p .. Cp" (where C is, a const) 613— Cn p"- 1 (2)

R
We have from gas low p = p ..1ii T, so using (2)

Thus, (3)

But,

dr M 1 —Mg (n - 1)
So, = R- C (n - 1) ID " -2 (... p p) =

dh ' nR
Cnp"-1

M
2.15 We have, dp = - p g dh and from gas law p = (1)
RT P

Thus -c1P-. - ---1k dh


p RT

Integrating, we get
P h

-42' = - f dh or, 1 n -2-- = — h

Or, f
P.
P RT
0

(where po is the pressure at the surface of the Earth.)


Po R7' '

- M gh/RT
P= Poe ,

[Under standard condition, po = 1 atm, T = 273 K

28 x 9.81 x 5000/8314 x 273


Pressure at a height of 5 atm = 1 xe- = 0.5 atm.
-28 x 9.81 x (- 5000/8314 x 273
Pressure in a mine at a depth of 5 km = 1 x e = 2 atm.]

2.16 We have dp = - pg dh but from gas law p = mP RT,

Thus dp = CI RT at const. temperature


M

42. ELV_f dh
So, _
p RT

P h

Integrating within limits


f
p0
P
0
RT
dh
192

or, In P= -
po RT

RT p
So, p = po e-Mgh/RT and h
Mg Po
Po
(a) Given T 273°K,

RT _ 1 k
Thus h=- — ln e = 8 m.
Mg
(b) T = 273° K and

-p
p0 = 0.01 or -P- 0.99
Po Po
, RT In
Thus h 0.09 km on substitution
Mg Po

2.17 From the Barometric formula, we have


- Mg h/RT
Po e

and from gas law P=


RT
So, at constant temperature from these two Eqs.

MP0 - Mg h/RT - Mg ii/RT


p= e Po e (1)
RT
Eq. (1) shows that density varies with height in the same manner as pressure. Let us
consider the mass element of the gas contained in the colbmn.

dm = p (Sdh) = —MP° Mg h/RT Sdh


RTe
Hence the sought mass,
h

1f _( _ e- Mg h/RT
m-M POS Mg h/RT dh= p0S
RT g
0
118 As the gravitational field is constant the centre of gravity and the centre of mass are same.
The location of C.M.
h- °
f h p dh
f h dm
o

dm f p dh

-
But from Barometric formula and gas law p p0 e Mg h/RT
193

f h (e- Mg hi") dh
o RT
So, h- -
a
Mg
(e- Mg h/R7) dh
J
o

2.19 (a) We know that the variation of pressure with height of a fluid is given by :
dp= - p g dh

But from gas law p = -2- R T or, p = ' -11


M RT
From these two Eqs.

or,

P h

lca . jJk dh
Integrating,

f In P— = In (1 - ah)meaRTo
Po

Hence, p = po (1 - ah)mg/aRT0. Obvionsly h < 1


a

(b) Proceed up to Eq. (1) of part (a), and then put T = To (1 + a h) and proceed further in
the same fashion to get
Po
P1-
(1 + ah)meaRTo
2.20 Let us consider the mass element of the gas
(thin layer) in the cylinder at a distance r from
its open end as shown in the figure.

Using Newton's second law for the element


Fn = mwn:

(p + dp) S - pS = (p S dr) co2 r

or, dp= p w2 r dr =2111 w2 r dr


RT
194

P r

So,
€0 2rdr r, 2_ . m (02 j,

p = R7' a o 4 r dr,

, p M co2 f Po
RT
0
hi w2 r2 I 2 RT
Thus, in pc, = 2 RT r2 or p wi Po e
2.21 For an ideal gas law
p= -e-RT
M
00
So, 5
p - 0.082 x 300 x — atms = 279.5 atmosphere
44
For Vander Waal gas Eq.
v2a

(p-i:-v-2-)(V-vb). v R T, where V . vVm

av2
or, P- vRT mRT/M amt
V-vb 7f12- mb
I. 17271712
v-

M
R __p_.
a 2 n. 79.2 atm
- pb M2
MMR

2.22 (a) P=

(The pressure is less for a Vander Waal gas than for an ideal gas)

ri Vm + b
OT, a(1+1)=1 RTr1+1-+1 = RT
Vm (Vm - b)
VA2( VM Vm-b a
(1 + 1) (Vm - b)
or T- RVm(riVm + b) ' (here VM is the molar volume.)

1.35 x 1.1 x (1 - 0.039) 25


= K .1
0.082 x (0139)
(b) The corresponding pressure is
RT a a (1 + TO a
P
P = Vm - b vs?, = Vm (1 Vm + b) 1,:f
. a (Vm + ri Vm - i Vm - b) a (Vm - b)
Vh2f (II Vm + b) V‘2,f (Vm + b)

F35 0.961
= 1 x 0.139 = 9.3 atm
195

1 a 1 a
2.23 pi = RT, v _ b - iiy p p2 =R T2 v.. b - 71 2-
R (T2- T1)
So, P2 -P1'
V-b '

R (T2- T1) R (T2- T1)


or. V - b ... or, b= V
P2 -121 P2 -P1

T P2 -121 a
Also, 4 1 7 . 2 ........... Ti. .." yr
111-

a T1 (P 2 -P1) T1p2 -pi. T2


Til- T2 - Ti P1= T2 - Ti

T1P2 -121 T2
a - y2
or
T2 - T1

Using T1= 300 K, p1= 90 aims, T2 = 350 K, p2= 110 atm, V= 0.250 litre

a = F87 atm. litre2/mole2, b is 0.045 litre/mole

RT a .... v(81 RTV 2a


2.24 P
- al V - b Tx av T (V- b)22 — V2
lc. -1 (a
or,
V ap T

[R V3 - 2a (V - b)2 -1 V2 (V - b)
an T
V2 (V - b)2 [RTV3 - 2a (V - b)2}

2.25 For an ideal gas xo =


RT
-1 2 2 -1
b)2 11 - (V — b)2}
Now x = (V — 3 = KO (1 - 1 1 - la— (1 - /
RTV . RTV V RTV V

2a 2b
' Ko {... '-7 +
R , to leading order in a, b
TV
;e 2a 2h 7,
Now K> K or
O RTV > V bR
4 <

If a, b do not vary much with temperature, then the effect at high temperature is clearly
determined by b and its effect is repulsive so compressibility is less.
196

2.2 THE FIRST LAW OF THERMODYNAMICS. HEAT CAPACITY


2.26 Internal energy of air, treating as an ideal gas
T
U— 2-11 Cy T = L?i R — PV (1)
M My—1 y—1
R C
Using CV = y ... 1 since Cp — C v . R and ---L = y
Cv
Thus at constant pressure U = constant, because the volume of the room is a constant.
and V in Eq. (1), we get U = 10 MJ.
Puting the value of p =paten
2.27 From energy conservation
1
Ut + — (vM) V2 = Uf
2
1
or, AU = —vMv2 (1)
2
RT vR
But from U = v AU — AT (from the previous problem) (2)
y —1 y—1
Hence from Eqs. (1) and (2).
mv 2 ty ................. 1 )

AT — `
2R
2.28 On opening the valve, the air will flow from the vessel at heigher pressure to the vessel
at lower pressure till both vessels have the same air pressure. If this air pressure is p, the
total volume of the air in the two vessels will be (V1 + V2). Also if v1 and v2 be the

number of moles of air initially in the two vessels, we have


p V1= vi RT1 and p2 V2 = V2 R T2
1 (1)
After the air is mixed up, the total number of moles are (v1 + v2) and the mixture is at
temperature T.
Hence p (V1 + V2) = (v1 + v2) RT (2)

Let us look at the two portions of air as one single system. Since this system is contained in a
thermally insulated vessel, no heat exchange is involved in the process. That is, total heat
transfer for the combined system Q = 0
Moreover, this combined system does not perform mechanical work either. The walls of the
containers are rigid and there are no pistons etc to be pushed, looking at the total system,
we know A = 0.
Hence, internal energy of the combined system does not change in the process. Initially
energy of the combined system is equal to the sum of internal energies of the two portions of air
:
viR Ti. v 2R T2
Ui = Ul + U2 — 1 (3)
y-1 + y — 1
197

Final internal energy of (n1 + n2) moles of air at temperature T is given by


(v1 + v2) RT
U (4)
f— Y —1
Therefore, U1— Uf implies :

T1+ v2T2 p1 + p2 V2
T— v1 T T 2 pip1 V1 +P2 V2
vi + v2 (pi VI/T1) + (p2 V2/T2) • T2 p2 V2 T2

From (2), therefore, final pressure is given by :


vi + v2 R p1 V1+p2 V2
RT = +
(v, Ti+ V2 T2) —
P + V2 V1 V2 + V2
This process in an example of free adiabatic expansion of ideal gas.

2.29 By the first law of thermodynamics,


Q.= A U + A
Here A = 0, as the volume remains constant,
vR ,r,
So, Q AU — I
y—1
From gas law, po V = v R To

po V A T
So, AU— — — 0.25 kJ
To (y — 1)

Hence amount of heat lost = — AU = 0.25 kJ

2.30 By the first law of thermodynamics Q = AU +A


D AV A
But AU= — (as p is constant)
y—1 y—1
A A F4 x .7j
Q- 1
Y— y— 1 1-4 — 1
2.31 Under isobaric process A = pAV = R AT (as v = 1) = 0.6 kJ
From the first law of thermodynamics
AU= Q—A= Q—R AT= 1 kJ
AT
Again increment in internal energy AU= R , for v = 1
y-1
AT
Thus Q —R AT= R or y= — 1.6
y—1 Q—R AT
2.32 Let v = 2 moles of the gas. In the first phase, under isochoric process, Al = 0, therefore
from gas law if pressure is reduced n times so that temperature i.e. new temperature
becomes To/n.
Now from first law of thermodynamics
vR AT
Q1= AU, =
y—1
198

vR (T0 T) v RT0 (1 — n)
y—1 n ° n (y — 1)
During the second phase (under isobaric process),
A2 = pAV = v R AT
Thus from first law of thermodynamics :
V R AT
Q2 111 U2 4 . A ••■ + vRAT
y—1

vR To—
n Y vRT0(n — 1) y
Y-1 n (y — 1)
Hence the total amount of heat absorbed
vRT0 (1 n) vRT0(n — 1) y
n(Y— 1) n (y — 1)

vRT0(n — 1) y
(— 1 + y) v RT0 —
n(y— 1)

2.33 Total no. of moles of the mixture v vi + v2


At a certain temperature, U. U1 + U2 or v Cy v1 Cy + v2 Cy
2

R R
1 1+ v2
+ *Y1— Y2
V1 CV V2 CV
Thus Cy. 2
V V
Cp + C
V1 1 V2 p2
Similarly C
V

( Y1 R Y2 R
+
Cv + v2 y2 Cv V1 yi —1 V2 y2 — 1
2

Y1 Y2
C +v2v 1R
Thus vl yi 1R 12 —
Cy R R
Y v1 1 + V2 v 1
11-1 I2—

134 -Flom Vat peNious problem


R R
+v2
11-1
cv. v1+ V2
15.23/mole.1(
199

Y1R Y2 R
1-
v1 -1 2 y2- 1
and = 23.85 J/mole. K
V2

Total mass = 20 + 7
Now molar mass of the mixture (M) = = 36
Total number of moles 1+ 1
2 4

Cv
Hence cV = — = 0.42 J/g •K and c = = 0.66 J/g•K
P M

2.35 Let S be the area of the piston and F be the force exerted by the external agent.
Then, F + p S po S (Fig.) at an arbitrary instant of time. Here p is the pressure at the
instant the volume is V. (Initially the pressure inside is po)
Vo

A (Work done by the agent)• f F dx

0
o
Vo 1V

p)s • dx= f (pop)dv


= J 0
ivo r, vo
= po(ri- 1) Vo -f pdV = po - 1) Vo - f vRT • "v
0 v0

= - 1)po Vo - IIRT In -1) vRT - vRT In I"

= vRT - 1 - In = RT (IT- 1 - Ini) (For v= 1 mole)

2.36 Let the agent move the piston to the right by x. In equilibirium position,
pi S + F agent= p2 S, or, Fa gent= (p2 - p1) S

Work done by the agent in an :nfinitesmal change dx is


Fa ge„, • dx = (p2 - p) &Ix = (p2 - pi) dV

By applying pV= constant, for the two parts,

Pi (Vo + Sx) = Po Vo and P2 (Vo - Sx) = Po Vo

po Vo 2Sx 2p0 Vo V
So, P2 -P1 2 2 (where Sx = V)
2 2
In
Vo - Vo - V
When the volume of the left end is T1 times the volume of the right end

(Vo + = (Vo - V), or, V +- 1V0


200

A = f

2.37 In the isothermal process, heat transfer to the gas is given by


V2
V2 P1
Ql. v RT0 ln Tr = v RT0 ln T1 For i = -7- = —
V

1 v 1 P2
In the isochoric process, A = 0
Thus heat transfer to the gas is given by

Q
2 g"
vR
4U = v Cy AT = Ai (for CC y= y
R l

y-1
P2 TO „, Pi Pi

But --- - or T= To for rt = -

Pi T' P2 P2

Q2 = 1,11_R i ( i _ 1) To

O f, AT = rl To - To = -1) To. so,

Thus, net heat transfer to the gas


vR
Q= vRT0 In + - 1) To
Y-1
-1
O f,

vRT0 In O or, f -2— - In


y -1 ' vRT0 Y -1
1 6-1
or, y=1+ =1+ = 1.4
( 80 x 103
vRT0 - ln 11 - In 6
3x8.314x273
vRvR
2.38 (a) From ideal gas law p = (— T= kT where k=
V V
For isochoric process, obviously k = constant, thusp = kT, represents a straight line passing
through the origin and its slope becomes k.
For isobaric process p constant, thus onp - T curve, it is a horizontal straight line parallel to T
- axis, if T is along horizontal (or x - axis)
For isothermal process, T = constant, thus on p - T curve, it represents a vertical straight line
if T is taken along horizontal (or x - axis)
For adiabatic process TY pl" = constant
After diffrentiating, we get (1 - y)p-Y dp • TY + yp 1 - Y • TY - 1 • dr = 0
201

(_y__)(2_2 TY -1). (._y__)2_


dT 1-y p-Y y -1 T

The approximate plots of isochoric, isobaric, isothermal, and adiabatic processess are drawn in
the answersheet.
(b) As p is not considered as variable, we have from ideal gas law
vR _vR
ki T where le

On V - T co-ordinate system let us, take T along x - axis.


For isochoric process V = constant, thus k' = constant and V. k'T obviously represents a
straight line pasing through the origin of the co-ordinate system and k' is its slope.
For isothermal process T = constant. Thus on the stated co- ordinate system it represents a
straight line parallel to the V- axis.
For adiabatic process TVY -1= constant
After differentiating, we get (y -1) VY-2 dV • T +1/1- 1 dT
dV ( 1 ) V
dT y - 1 .T
The approximate plots of isochoric, isobaric, isothermal and adiabatic processess are drawn
in the answer sheet.
2.39 According to T - p relation in adiabatic proces kpy -1
s, TY (where k = constant)
(NY (6' r y-1 P2
and SO, —= for =
.1)1) Po Pi

Hence T = To- TO -1 = 290x10'4 '14= 0-56 kIC


Y
(b) Using the solution of part (a), sought work done
v R AT v RT0 (_ ,„, )
-
A- = ri 1 1" -1 5.61 kJ (on substitution)
Y -1 y -1 (

2.40 Let (p0, Vo, To) be the initial state of the gas.
— v RAT
We know Aadia (work done by the gas)
Y-1
Y
But from the equation TV = constant, we get AT = To -1- 1)

- vRT0 (Tr' -1)


Thus Aadia gli
y-1
1
On the other hand, we know Aiso = vRT0 In

Aadia 11 Y - 1 - 1
(

5 64 - 1
= - v R To In i (work done by the gas)

Thus 1.4
Aiso °R (y - 1) In ri al 0-4 x In 5 r2
202

2.41 Since here the piston is conducting and it is moved slowly the temperature on the two
sides increases and maintained at the same value.

Elementary work done by the agent = Work done in compression - Work done in expansion
i.e.
dA '1 P2 " — Pi " ' (P2 —P1 ) 47
where p1 and p2 are pressures at any instant of the gas on expansion and compression side

respectively.

From the gas law pi (V0 + Sx) = vRT and p2 (V0 — Sx) = vRT, for each section (x is

the displacement of the piston towards section 2)

2 Sx 2V
So, p2 —pi = vRT = vRT 2 2 (as Sx .. V)
Vo -V
VO—S22

2V
So dA = vRT V 2
02 v2 dV

Also, from the first law of thermodynamics

R
dA = — dU = — 2v dT (as dQ = 0)
Y—1
R
So, work done on the gas = — dA = 2v dT
y—1

dT V dV
Or,
T — I"- l ---/--Vo — /-
RS
When the left end is 11 times the volume of
the right end.
c.----p2s
T1 1
(Vo + V) = i (V0 — V) or V- ii V
Fagent
+ °

T V

f dT f VdV
On integrating (y — 1)
T 11 — V2
T o
o
V

Or In T = (y — 1)[— 1 lit (11 — V2 )


To 2
lo

= - I = LII in (vs, — v2 ) — in K; — v2 ) — in 1/ 1
2 i

(11:_ 1) 1 L-1 in (r) + 1)2


= 1--- -1 1 2 2 1—
2 n Vo — 1n Vo 1 + 1 j 2 4 T1

y-1
fyi + 1)2 ) 2
Hence T = To(' ' 411
203

2.42 From energy conservation as in the derivation of Bernoullis theorem it reads

+ —1 v2 + gz + u + Qd = constant (1)
P
In the Eq. (1) u is the internal energy per unit mass and in this case is the thermal energy per
unit mass of the gas. As the gas vessel is thermally insulated Qd = 0, also in our case.
CM T RT RT
Just inside the vessel u = = also = — Inside the vessel v = 0 also. Just P
M (y — 1) M
outside p = 0, and u = 0. Ingeneral gz is not very significant for gases.
Thus applying Eq. (1) just inside and outside the hole, we get
1 2 E
4
—v = +u
2
RT RT y RT
M M (y — 1) M(y — 1)

2y RT 2Y
Hence v2 = r, v RT 3.22 km/s.
M (y — 1) o M (y — 1)
Note : The velocity here is the velocity of hydrodynamic flow of the gas into vaccum. This
requires that the diameter of the hole is not too small (D > mean free path 1). In the opposite
case (D « I) the flow is called effusion. Then the above result does not apply and kinetic
theory methods are needed.

2.43 The differential work done by the gas


dA pd vRT2 ( a
= = — —) dT —vRdT
a T2

(as pV = v RT and V =

T + AT

So, A = -f vRdT = - vRAT

From the first law of thermodynamics


vR
Q= AU + A mg - AT-vRAT
y-1

= vRAT • 1:11 - R AT • (for v = 1 mole)


Y-1 Y—1

2.44 According to the problem :AaU or dA = aU (where a is proportionality constant)


avRdT
or pdV = (1)
Y-1
From ideal gas law, pV = v R T, on differentiating
pdV + Vdp = v RdT (2)
204

Thus from (1) and (2)


a
pdV = — (pdV + Vdp)
y -1

or, pdV (y.(_11 -1 )+ -y-2iVdp= 0

or, pdV (k - 1) + kVdp -, 0 (where k = another constant)

k - 1
or, pdV k + Vdp = 0

or, pdVn + Vdp = 0 (where k i 1 .. n = ratio)

Dividing both the sides by pV


dV k
n— + =0
V p
On integrating n In V+ in p = In C (where C is constant)

or, In (pV" ) = In C or, pV" .. C (const.)

2.45 In the polytropic process work done by the gas


vR [ Ti - T11
A=
n-1
(where Ti and T1 are initial and final temperature of the gas like in adiabatic process)
v
and AU= Ry _ 1 (Tf - Ti )

By the first law of thermodynamics Q = AU + A


- vR („,f _ vR
y 1 \i Td+ n 1 (Ti-Tf)

According to definition of molar heat capacity when number of moles


v = 1 and AT = 1 then Q = Molar heat capacity.
R (n - y)
Here, = <0 for 1 < n <y
Cn (n 1) (y y) 1 1) )
2.46 Let the process be polytropic according to the law pV" = constant
Pi
Thus, pf Vfn = pi Vi" or, --) an f3
(Pif

So, an = 13 or In f3 = n In a or n=
Ina
In the polytropic process molar heat capacity is given by
205

R (n-y) R R
.=
(n -1)(y - 1) y-1 n-1

Rln a In
=R where n= i
y -1 In p
-Ina' Ina
8.314 8.314 In 4
So, cle . 42 J/mol.K
1-66 - 1 - In 8 - In 4 = -

2.47 (a) Increment of internal energy for AT, becomes


v RAT RAT
AU- - = - 324 J (as v = 1 mole)
y-1 y-1
From first law of thermodynamics
R AT RAT
Q= A U +A = - = 011 kJ
y-1 n-1
of

(b) Sought work done, An - f pdV =f —v1cTidV


v;
(where pV" = k= piVi"= pf V; )

(pf V; Vfl-n - pi Vin 1711-n)


k (1 vii-n).

= V -n
- 1 - n
1 -n f
. pf Vf - piVi .vR(Tf -T,)
1-n 1-n
vRA T - RAT - 0.43 kJ (as v = 1 mole)
n-1 n-1
2.48 Law of the process is p = a V or pV-1 = a
so the process is polytropic of index n -- 1
As p= aV so, pi= a Vo and pf = a i Vo

(a) Increment of the internal energy is given by

vR P CVf .. p iVi
AU= 71 T..= [Tf- Ti] -
Y- 1
(b) Work done by the gas is given by
piVi-pfVf . a q) - all Vo • Ti Vo
A=
n-1 -1-1

= -2
(c) Molar heat capacity is given by
R(n-y) . R(-1-y) .Ry+1
C"
I. (n - 1) (y - 1) (- 1 - 1) (y - 1) 2y-1
206

v
2.49 (a) AU = RAT and Q = vC . AT
Y-1
where Cn is the molar heat capacity in the process. It is given that Q -. - AU
R
So, C. AT - — AT, or C. = - R
Y-1 y-1
(b) By the first law of thermodynamics, dQ = dU + dA,
or, 2 dQ = dA (as dQ = - dU )
2Rv
dT + pdV = 0
2vC ,, dT = pdV , or, y - 1
2RV 2 dT dV
So, vRT 0
Y-1 dT + dV 0, or, (y - 1) T + V =
V
dr Y - 1 dV .
Of, T+ 2 0, or, TO 1)/2 = constant
(n R
(c) We know C.=
(n -1) (y 1)
But from part (a), we have C. = - yR_ 1

R (n - y) R
Thus which yields
Y-1 (n - 1) (y - 1)
1+y
n-
2
From part (b); we know TO- 1)/2 = constant
8 - 1)/2
To V )6 ii(y - 1Y2 .
So, In (where T is the final temperature)
T (Vo
Work done by the gas for one mole is given by
(To - T) 2 R To [ 1 - 4 1 - Y )/2 ]
A- R
n-1 = y-1

2.50 Given p= a Ta (for one mole of gas)


-a
So, pT- " = a or, p = a,
(RV)
1- a= -a
or, P a V- - aR or, pVai(a-1)= constant
a
Here polytropic exponent n =
a-1
(a) In the polytropic process for one mole of gas :
RAT RAT
A .71--=
1: -n (1 _ aa_ 1) = RAT (1 - a)

(b) Molar heat capacity is given by


R R R R R
C= — = + R (1 — a)
y - 1 n -1 al y - 1 a Y-1
-1
a -1
207

2.51 Given U = aVa


,., pV
Or, vCvT . a Va , or, vc... — IS a Va
v vR

or,
V a "• 1 . p 1 am CV
Ra

pv 1 ... a ... Ra . R
or constant = a (y -1)[as Cv= 1, = 7
Cv

So polytropric index n = 1 — a.

(a) Work done by the gas is given by

- v R AT v RAT
A= and AU=
n—1 y—1

— AU (y — 1) AU (y — 1)
Hence A = = (as n = 1 — a)
n—1 a

By the first law of thermodynamics, Q = AU + A

= AU + AU (Y - 1) -
a
Au [1 + Y1 cc

(b) Molar heat capacity is given by

R =
R R
C R
y-1 n-1 y-1 1—a-1

R R
= — +a
— (as n .. 1- a)
y- 1
2.52 (a) By the first law .of thermodynamics

dQ = dU + dA = vCvdT + pdV
Molar specific heat according to definition

dQ Cs, dT + pdV
C= —
vdT vdT

vCv dr + vTi7 dV
RT dV
= = +
vdT Cv V dT '
a'
We have T= To e

After differentiating, we get dT = a To eav • dV


dV 1
So,
DT a Toeav

R R
RT 1 To e a V
Hence C = C + v MI C /7 + - C +
V v a V
a Toe"
a VTo e a v
poeav
(b) Process is p =

R7' aV
p
= -f = P°e
208

Po aV
or, T= —
Re
RT dV a V
So, C = Cv+ — C +
V Tlf= Cv+P°e poect va+av) v 1+ aV

2.53 Using 2.52

(a) C = Cv RvT ddVr Cv + 1 (for one mole of gas)

a RT a
We have p = po+ v, or, v po + v , or, RT = po V+ a
dV
Therefore RdT = Po dV , So, R
dT Po

a
Hence C = Cv+ (Po + • i= R + + v)R
Po Po
R \ aR _ aR

(1? +7 Y:T PoV Y -1* PoV


(b) Work done is given by

(112 V2
AU = Cv(T 2— T1) = Cv (for one mole)
R R

(P2 V2 P
(y - 1) R

a (V2 - V1)
1 + aV2) 2 — po + Po
=y (Po 17 y_1

By the first law of thermodynamics Q = AU +A


V2 Po (72 - VI)
= po (V2 - Vi) + a In + _
1 (Y )
YPo (V2 - V1) V2
y-1 + a ln —
V1
2.54 (a) Heat capacity is given by
RT dV
C=C + (see solution of 2.52)
v V dT
T To
We have T To+ aV or, V - ———
a a
dV 1
After differentiating, we get , —
dT a
209

RT 1 R (To + a V) 1
Hence C=Cv+ a =?_R
14-
RT
R
Y R +RT0 +
R (T° + 1)
y-1 aV y-1 aV v (XV P aV

(b) Given T = To + aV

As T = PI for one mole of gas

RT
p=y(To+aV)= v aR

v2 V2

(RT0
Now A- f pdV=.1 —v +aR)dV (for one mole)

V2
= RTo In —+ a ( V2 - 171)

AU = Cv (T2- Ti)
Cv[To + a - To a Vi] = aCv (V2 - Vi)
By the first law of thermodynamics Q = AU + A
aR V2
(V2 - Vi) + RT0 ln — + aR (V2 — V1)
r-1 v1

1 V2
=aR (V2- Vi.) [1 + 1+ RToln —
y -1 v,
V2
a Cp (V2 - + R To n —
V1

V
a Cp (V2 - Vi) + R To n vi

2.55 RT dV
Heat capacity is given by C Cv + —
V dT
(a) Given C = Cy+ aT
RT dV a , dV
So, C
V
+ — — or, — cri Mg
--

V dT R
Integrating both sides, we get iT = In V + In Co = In VC0 , Co is a constant.

aT/R
Or, V • Co e or V- ea " constant
CO
(b) C = Cv + fiV
210

RT dV RT dV
and C = Cv + —
V dT CVV ;TI C V + PV k dT
RT dV dV „._ 2. dT
or, 11V or, -7, or, v
v d7, v R T T
1 R V
Integrating both sides, we get IT 07... i- - In T+ In Co . In T•C0

So, In T • Co = -
R 11/0V or, T e -143v = — =
ov T • Co= e-
,
constant
Co

(c) C- Cv + ap and C . Cv + R ...dV


v dT

RT dV RT dV
So, Cv + ap . Cv+
V dT so, al V dT

RT RT dV
or,
a 17 V dr P V for one mole of gas)

dV dV
or, or, dV = adT or, dT = —
dT a a

V
So,T = - + constant or V - aT = constant
a

2.56 (a) By the first law of thermodynamics A = Q - AU

or, = CdT - C v dT = (C - C v) dT (for one mole)

Given C=

T.

So, A= - Cv) dT = a 1n115- - Cv (-9 To - To)


To
Ta

RT
a In - Cv To(rt- 1) = a In + - 1)
y - 1

dQ RT dV
(b) C +
v
dT Is
V dT + C
a T dV a
Given C=— so Cv + R—
T Vd r 7
R 1 dV . a dT
or,
y -1 R T+ V RT 7
dV_ 1 dT
or, dT --
V Re Y-1 T

dV a (y - 1) dT
or, (y - 1) R7
- T

Integrating both sides, we get


211

(y-1)
or (y-1)1nV= - a -In T + In K
RT
T-
or, ln VY -1 - a(Y - 1)
K RT
in vy -1 2_1_7 . - a (y - 1)
RK pV

or, PVY = e- a (y - 1YpV


RK
. tr - I)/p V
Or, pVT ' - RK - constant
2.57 The work done is
V
V 2
2

A j pdv.f(vRTb_ va2 dv

Vi i vvi

= RT ln — -
V1 -b +a V2 VI.

2.58 (a) The increment in the internal energy is


v1

(am
AU = f dV
av
Vi IT

But from second law


(au) . r as
T C)
av A (av) ° r v—P
T T- P 1
On the other hand P"' RT a
V — b V2
a
or, T Cl = RT and (au).
ar v v— b av V2
:
a 1 _ 1
So, AU.
VI, V2
(b) From the first law
V2 — b
Q = A + AU = RT ln v1_ b

2.59 (a) From the first law for an adiabatic


dQ = dU +pdV = 0
From the previous problem

dU - (au) dT + (au) dV = C v dT + 2- dV
2
ar av V
V T

RT dV
So, 0 - C dT +
v V-b
212

dU = C v dT + zdV
V

Now, dQ= C v dr +
V b"
r RT aV
So along constant p,
cps. %' V+ V - b aT
RT (av RT a
Thus CI,- C v = —
V - b aT) ' But p = v - b - V2
P
(_. RT + 2a)(av) + R
On differentiating, 0 =
(V - b)2 v2 aT p V-b

r(av) RTIV-b V-b


Or, -
aT = RT 2a , 2a (V - b)2
P 2 3 I
(V - b) V 3
RT V
R
and Cp - CV =
1 2a (V - b)2
RT V3
2.60 From the first law
Q - Uf—Ui+A= 0, as the vessels are themally insulated. As
this is free expansion, A = 0, so, Uf = Ui
av2
But vCvT- v

So, Cv(Tf-Ti)=

or, AT=
RV]. (V1 + V2)

Substitution gives A T —3K


2.61 Q= Uf—Ui+A= Uf—Ui, (as A = 0 in free expansion).
So at constant temperature.
— av2 ( av2) 2 V2- Vi
1.1 aV
•2 V
V1 V1 2

- 0.33 kJ from the given data.


213

2.3 KINETIC THEORY OF GASES. BOLTZMANN'S LAW AND


MAXWELL'S DISTRIBUTION

2.62 From the formula p= n k T

p 4 x 10 -15 X 1•01 X 105 3


n —= per m
kT 1-38 x 10- 23 X 300

= 1 x 1011 per m3 = 105 per c.c


Mean distance between molecules
(10-5 c.c.)113 = 10 /13 X 10-2 CM in 0.2 mm.

2.63 After dissociation each N2 molecule becomes two N-atoms and so contributes, 2 x 3 degrees
of freedom. Thus the number of moles becomes
mR T
+ ri) and p -mv ( 1 + ri)
M (1

Here M is the molecular weight in grams of N2.

2.64 Let n1 - number density of H e atoms, n2 = number density of N2 molecules


When p= ni mi + n2 m2
where m1 = mass of He ato m , m 2 - mass of N2 molecule also p (n 1 + n2 ) kT

From these two equations we get

n1 - -P-)/ (1-17'
kT m 2 m
2

nv x2 my cos 0 x dA cos 0
2.65 p dA

=2 m 0n cos2

2.66 From the formula

v ,y

If i = number of degrees of freedom of the gas then

and C =
Cp= CV
+ R T R T

v
2
2 2 2
y 1 + 7 or i
cv y -1

SO,
214

(a) For monoatomic gases i = 3

Vsoundd IF
= — =0.75
v 9
rms

(b) For rigid diatomic molecules i= 5


wound {1—
v — = 0.68
1721S
15
2.68 For a general noncollinear, nonplanar molecule
3
mean energy = - k T (translational) + -3 kT (rotational) + ( 3 N - 6) kT (vibrational
2 )2
= (3N - 3) kT per molecule
3
For linear molecules, mean energy = - kT (translational)
2
+ kT (rotational) + ( 3N- 5) kT (vibrational)
5
= (3 N - -
) kT per molecule
2
1 1
Translational energy is a fraction and in . the two cases.
2 (N - 1) 5
2N - 3

2.69 (a) A diatomic molecule has 2 translational, 2 rotational and one vibrational degrees of
freedom. The corresponding energy per mole is
1
RT (for translational) + 2 x -
2 ' 2 RT (for rotational)
7
+ 1 x RT, (for vibrational) = -RT
2
7 C
Thus, Cv= i R, and y = -c1,,-=

(b) For linear N- atomic molecules energy per mole

= (3N- 9 RT as before
2
5
So, Cv= (3N- i) R and y ..
6N- 5
(c) For noncollinear N- atomic molecules
3N - 2 N - 2/3
Cv = 3 (N - 1) R as betore (2.68) y =
3N-3 N-1

2.70 In the isobaric process, work done is


A = pdv = RdT per mole.
On the other hand heat transferred Q = CP dT
3
Now C = (3N - 2) R for non-collinear molecules and C = 3N - - R for linear molecules
P P 2
215

1
non collinear
3Ar - 2
A
Thus 1
Q 3 linear
3N— —
2
5 A 2
For monoatomic gases, = — and
2 —=
Q 5
2.71 Given specific heats cp, c„ (per unit mass)
R
M(cp — c,,) = R or, M =
C - C
p v

CP
2 2 2c;
Also Y ,+ 1, OS, I
Cy I C C - C
p v
-1 1
cv
29
2.72 (a) C1, =
= R
29 °K mole 8.3
203 29 7
cv. 1.4 .
y= 207
i= 5
(b) In the process pT = const.
2 dT dV
y = const, So
2 7— Vr" °
R2' 2RT
Thus CdT= Cvdr+pdV= CvdT+TidV=CvdT+ 7, dr

(29 12.4 3
Of C lo, Cv + 2R = ici- R So Cv im, 8.3 R= V?
Hence i = 3 (monoatomic)
2.73 Obviously
1 ,
=
R v 2 '1 + l2y
3 5
(Since a monoatomic gas has Cv= —2 R and a diatomic gas has Cv = R. [The diatomic

molecule is rigid so no vibration])


1 3 5
— p —y, +2 —2 ye, + y, + y2
R
5 Cp y2
Hence s
C = 3 yi + 5 y2
p

2.74 The internal energy of the molecules are

U= mN < ( 1-1 2 > az —


1 <112 — v2>
2 m
2
216

where t7t. velocity of the vessel, N = number of molecules, each of mass m. When the
1
vessel is stopped, internal energy becomes - mN <u2>
2
1 2
So there is an increase in internal energy of AU = - m N v
2 . This will give rise to a rise
in temperature of
2
2mNv
AT
R
2
m N v2
iR
there being no flow of heat. This change of temperature will lead to an excess pressure
ROT _ m N v2
"' V iV

Ap M v2
and finally = 2.2 %
p tRT
where M = molecular weight of N2 , i = number of degrees of freedom of N2

2.75 (a) From the equipartition theorem

3 113k T 3RT ,, ,
T - — k T ... 6 x 10-21 J ; and v
rms
-
M
= U'f4 / KM/ S

2
(b) In equilibrium the mean kinetic energy of the droplet will be equal to that of a molecule.

.17 2 p vm2,s. ik T or 3
2kT
. 0.15 m/s
26 vans m
rt d3 p
7
2.76 Here i = 5, CV = 52 R, y - - given
5
, 3RTI . 1 3RT 1 ,
V
rms E
R
'M

11
Of T= .1
vansmu i2
—11
Now in an adiabatic process

TVY -1= T Vvi .. constant or V T 2 = constant
4 i/2
. T
V ( 1 1 V /2 Or *17' 11 — i = V or V' = li V
' re A

The gas must be expanded ii times, i.e 7.6 times.

5m
2.77 Here Cv = -2 m—R (i 5 here)
increases times, the temperature will
m= mass of the gas, M= molecular weight. If vans Ti
have increased ri2times. This will require (neglecting expansion of the vessels) a heat flow
of amount
5m 2
iTiR — 1) T = 10 kJ.
217

2.78 The root mean square angular velocity is given by


1 2 1
it = 2x—
k T (2 degrees of rotations)
2 2kT
or = - 6.3 x 1012 rad/s

2.79 Under compression, the temperature will rise


TV7-1= constant, TV2fi = constant
)2/
or, r (Tr 1 K ' = Tovr or, r=
So mean kinetic energy of rotation per molecule in the compressed state
= kr = k Toe = 0.72 x 10-20 I

1
2.80 No. of collisions = — n <v> v
4
v' n' <1,> 1
Now,
v = n <v> 11 T
(When the gas is expanded i times, n decreases by a factor T). Also
( vf/i vi 1
T T) or r = 21i T so, i
V 7]
i+1
i.e. collisions decrease by a factor /I i ,i 5 here.

2.81 In a polytropic process pVn = constant., where n is called the polytropic index. For this
process

Then dQ= CdT= dU+pdV= CvdT+pdV

1
= -
—dV=
RdT+RT -RdT- RdT= (-} )RdT
2 V 2 n-1
i 1
Now C = R so — - 1
2 n -1=
1 i i-2
or, = -1- 2 or n= .
n - 1 —2 1- 2
n-1
vi n' <11> 2
Now
v = n <v>
218

2.82 If a is the polytropic index then


pVa 'IC constant, TVa - 1 = constant.

v' n'
Now <v' > V 7-"- V T- 112
V n <v> vr-1/2
=
v
1 1
Hence a -1 - or a -1

R
Then =3R

2.83 v = 0.45 km/s,


P=

=
and vrms = 0.55 km/s

2.84 (a) The formula is


4 -u2
df (u). u2 e du, where u = —
v
1+O1

(111-=12 < e r r i ) . f
Now Prob df (u)
vp
) 1-61

4 -1 8
= --e x2oil= oi = 0.0166
Vie
( —V V vans 11V
(b) Prob <
<S 211
V vp vp
1711S

Prob \ S
<

2
4 2 -U
f
u e du
Vn

-3/2 \F 12 VT
4 x 2 e x2
T o il = e 3/2 o = 0.0185
Nirt V2 7c
219

v (v3- ) 1 f i rT_ . Ay,


-v
2.85 (a) ems p m
2
T. m Ay
IIK 384K
k (13--VT)
(b) Clearly v is the most probable speed at this temperature. So
mv2
VITT
v or T — - 342 K
2k

2
(Vi 2 2 2
v
or ev1-
V2 2/ vP
2 ,
m (v1- v9
So T= = 330 K
2

V2

dv
Vp

2
ntv (1. _1) 1
e 2cT
0
11 2g
,...3/2 or "11/2 (1 — 11, In
t 2kTo 11 2
/3 kTo .1 In 11
Thus v V
m V 1 -1/Ti
VTRT
2.87 vpte. V —mN v=
V MN Mo

v. - = Av. NITA11,—7. (1- V 21


AN 0 "N 0

Atr, 002 MN (AV)2


T= 2 - 363 K
/75—)
2R (1_ 2k (1 -
Mo
— v2/V 2 3/2
v
2
e 2PH lie V2 — v2/V viz Imif)a (171He
2.88 PH. or e
V3 V mH
PHe
PH
ln /mH
v2. 3kT , Putting the values we get v = 1.60 km/s
•••
mHe • H
220

2
N 4v2 dv _ v2,„
2.89 dN (v) - - e r
Vn v
For a given range v to v+ dv (i.e. given v and dv ) this is maximum when

dN (v) 0 v_ 4 + 2v 2 _v2/v2
P e
vp 171-V 7ir V
V
2
or, v2 - v2 3kT
Thus 7 MV
2P m 3 k
2.90 d3 v = 2 it vl dvi dvx
3/2 m „
Thus d n (v) = N n e 2kT + dvx2nvldvl
2 nik T

2.91 <vx > = 0 by symmetry


OD

2 2 2 2
MV
I f -
2kT dvx= f vxe - mv - MV
< I Vxl > = f lv e 2kT dvx e 2kT dvx e 2kT dvx
0 0 0

00 2 co 2
nt nity
2.92 2 2 v
<vx > = f vx e -2ki dvx f e 2k1 dvx
0 0
00
f x dx I f x dr
— xe e-
kT 2 Vx- 2 ix-
0 0

2kT r (3) r (i) _ kT


m 2I 2 m

2.93 Here vdA = No. of molecules hitting an area dA of the wall per second
221

CO
2
1/2 my
_
or, V soar n( M ) e 2kT Vx dvx
2 n kr
0
m
v2
2
f n 2k e, udu

Nf T r m
0

1 l i ff c T ric—T 1
sr —n M n im — n <v>,
2 MC 2m 7C 4

(where <v> .

2
2
( m )1/
2.94 Let, do (vx) . n e - my/2kT dvx
2 IC kr

be the number of molecules per unit volume with x component of velocity in the range vx to
vx+ dv
X
m

Then p ... f 2 mvx • vxdn (vx)

co
1/2
2
•) 1,„„2 „ m
se e - mv 2 kr dv

f - - - . - (2
0
7t kT) x

0.
2mn 1 2kT 1 u2 e- u2 du

NITE m J
0
CCI

= —4 — kr • f x e' thc,-- nkT


V7t n 2Vx I.
0
CO

2.95 3/2 2
in ) my 1
<1 >= e - Tif 4 n v2dv —
v f (2 n kT v
0
OD

3/2
( m ) A 1 2kT f _ x _,
us 4 Jt — — e ax
2 n IcT 2 m
0
112 112 2
2m . ( ni62 87; 1 1 4
2 (2 :ikT ) 1. (7( kr) 7C <V>
222
3/2 2

T dN (E)
2.96 dN (v) = N ( 27:4 - my r2k 4 it dv = dN (E) dE de
e
3/2
dN (E) - 2RT 2 dv
or, N e mv2/ 4nv—
de (2 nmk dE
dv 1
Now, E= 1 my2 so — = —
2 dE my
3/2
dN (g) = N m c/kT 117 1
or, e- 4n — —
dE 2nkT m m
2 - 3/2 - e/kT 1
=N (ki ) e E
Wt
i.e. dN (E) = N —2 (kT)- 3/2 e PAT E 1/2 dE
NiTt
The most probable kinetic energy is given from
1/2
d dN (e). , 1 - 1/2 e- e/kT E 1
u or, — E la' . Ep r
de dE 2 EAT= 0 or = --i
kT e E
kT
The corresponding velocity is v = m oe VP,

2.97 The mean kinetic energy is


ce
3/2 e- eller E I/2 e- e/kT dE
<e> E dE f
J
0 0
Thus
3
-(1+ On) kT
2

6N 2 ..)- - EAT 1/2


(0 3/2 e e de

3 kT (1- 6q)
2
3/2
2 fr 3/2
e- 3/2 (3 ) 26 - 3 \ —7t e 6r)
Vn 2
If 6 = 1% this gives 0.9 %

OD

2 - 3/2 f e- c/kT d E
2.98° =
N — (kT)
NITE
Bo co

2 -3/2 - - kT
(kT) v Eo f e e d e (E0 » kT)
\ITT-
0
223

2 _ 3/2 f-
(kT) e-
v Eo kT e- %/kr = 2 \ E° elkT
V 7t 7t kT
(In evaluating the integral, we have taken out Nri as 1E; since the integral is dominated by
the lower limit.)

2.99 (a) F (v) = Av 3 e— nsv2/ 2kT


For the most probable value of the velocity
(v) 2 my v2/2kT
dF = 0 or 3A v2 e- mv2 /2 kT -Av3 — e-m =0
dv 21cT

So, V =
Pr

2kT
This should be compared with the value v = — for the Maxwellian distribution.
Pr m

(b) In terms of energy, E = —1 mv2


2
dv
F (E) = Av3 e- mv/2 kr
dE
3/2
. A (1 e- EAT 1 2E -Ear
m 11-2-m7 = A m
--i. e

From this the probable energy comes out as follows : F' (E) - 0 implies
24 ( - EAT)
2 e EAT_ kT e — = 0, or,, E pr = kT
m

2.100 The number of molecules reaching a unit area of wall at angle between 0 and 0 + d0 to
its normal per unit time is
v-
,,d
dv f do v.) — v cos 0
4n
v- 0
00
3/2
- mv
.fn( m e 2/2kr v3 dv sin 0 cos 0 d 0 x 21t
2 n kT)
0
co
1/2 1/2
T
. n (2/1 fe'xcirsinecosOd0=n - -2k sin 0 cos 0 d 0
mn MIL
0
2.101 Similarly the number of molecules reaching the wall (per unit area of the wall with velocities
in the interval v to v + dv per unit time is
0-n/2
S2
dv = f do (v) c1— v cos 0
o o
224

0 Ic/2
3/2
m - mv 3
e 2/2" v dv MO cos° dO x 2n

-.1.
0 0
n ( 2 7r kT)

3/2
( m mv2/2kT
nn e v3 i(iv
2 7r.kr)

2.102 If the force exerted is F then the law of variation of concentration with height reads
/kT kT In ri
n (Z ) . no e- F7 So, 1 'e
F MAT
or F = = 9 x 10-20 N
Ah

6 R TinTi
2.103 Here F= -31- d3 AP g " R Tini or Na=
6 Ac h nct3gAph

In the problem, a = 1.39 here


110

T = 290K, i = 2, h = 4 x 10 - 5 m, d = 4 x 10 -7 M, g = 9.8 m/s2 , A p = 0.2 x 103 kg/m3 and R =


8.31 J/k
6 x 8.31 x 290 x ln 2 1 n26
Hence, Na 6.36 x 1023 mole -1
- 7t x 64 x 9.8 200 x 4 x "A"ti =

concetration of H2 RT
e- Mil2gh/ (M - M
2.104 1-1 - _ Af _ ;via - 110 e 142 112 gh/RT
concentration of N
= 11°
2 e N2 g
So more N2 at the bottom, (a = 1.39 here
Tlo
2.105 n1 (h) -m gh/kT
=
2
n1 e- all gill/a , 112 (h) .. n2e
ni on nty, hr
They are equal at a height h where — - e
n2
kT ln n1 - ln n2
or h = 1-m2
g m

2.106 At a temperature T the concentration n (( ) varies with height according to


- kT
n (( ) = no e ingz/
ec.

This means that the cylinder contains f n (i )de


o
CO

mgekT ci z - no kT
= f no e- mg
0
particles per unit area of the base. Clearly this cannot change. Thus no kT - Po - pressure at
the bottom of the cylinder must not change with change of temperature.
225

2.107
<U>
gz
e
f
m xe dx
-
Pn gz
= kT 12-21 = kT
° T (1)
act
e- x

!
dz f

k
T

°
.

m
g
z
/
k
T

d
z

f
dx
e
0 0
When there are many kinds of molecules, this formula holds for each kind and the
average energy
kT
<U> 16 kT
E fl

where fi a fractional concentration of each kind at the ground level.


2.108 The constant acceleration is equivalent to a pseudo force wherein a concentration gradient
is set up. Then
e- MA w 1/RT 1 _1.1

RT In (1 - 70 g
or w= -
I
MA1 MA
2.109 In a centrifuge rotating with angular velocity co about an axis, there is a centrifugal acceleration
co2 r where r is the radial distance from the axis. In a fluid if there are suspended colloidal
particles they experience an additional force. If m is the mass of each particle then its

m
volume is — and the excess force on this particle is
m 2
— (p - p0) w2 r outward corresponding to a potential energy - —(p - p0) w r2
2p
This gives rise to a concentration variation

n (r) = no exp n (r2)


Thus exp
4. 2 pmkT (P P°) w2 r2 - r2i)
n (r 1) + (p - Po) co2
2 p RT

M M
= NA m is the molecular weight
where —=—
k R '
2 pRT1nri
Thus
(1) - Po) (02(rz- ?I)
1
2.110 The potential energy associated with each molecule is : - - m w2 r2
2
and there is a concentration variation
(m 0)2 r2) (41 (0272)
n (r) = n0 exp - no exp
2kT 2RT
(111 (02 12)

Thus exp r co = ln
2R T o m
226

Using M . 12 + 32 = 44 gm, 1= 100 cm, R = 8.31 x T = 300,

we get w = 280 radians per second.

ar
2.111 Here n (r) = no exp (- 2)
kT
(a) The number of molecules located at the distance between r and r + dr is
ar2)
4 n r2 dr n (r) = 4 n no exp -
rr2 ddr
kT
3
d 2ar -
(b) r is given by — r2n (r)
Pr dr
0 or, 2r - —
kT
= 0 or r v7
Pr
(c) The fraction of molecules lying between r and r + dr is
dN 4 n r2 dr no exp (- ar2/kT)
N

f4 It r2 dr no exp (- ar2/kT)

02

3
(I
f 4 It r2 dr exp C4nfx ex
= T p
0 ) o
0 / (-
_ar2) 2 k x)
kT a 2V
t 3/2
3/2
(-9 2n r
a 2 a
3/2 2
dN ar
Thus L—1--) 4 it r2 dr exp
N kr kT
3/2 2
- ar
(d) N 4 n r2 dr exp (
kTk kT)'
1/2
(- ar2)
So n (r) = N (-19 exp
n kT kT
When T decreases ri times n(0) = no will increase ri3/2 times.
2.112 VT/ V1— dU dU
Write U. ar2 or r= — , so dr =
a a 2 VD-. 2 wriTI

N 4 it U dU (U)
so d = no 2 exp
a j kT

3/ 2
= 2 it no a - 2 U 1/ exp dU
kT
The most probable value of U is given by

d (dN). 0 ( 1 U112 )
exp U\ or, U r = 1 kT
dU dU 2 kT kT P 2
From 2.111 (b), the potential energy at the most probable distance is kT.
227

2.4 THE SECOND LAW OF THERMODYNAMICS. ENTROPY


2.113 The efficiency is given by
-
- T , T1>T2
1

Now in the two cases the efficiencies are


+ AT- T2
T1 increased
+ AT

Tl - T2 + AT
iT! = , T2 decreased
T1

Thus rih < ni


7
2.114 For H2, V• -
5
p2 V2, p p4 V4
p1V1= 3 V3=

P2 VI p
3 V1 , pi V ? = p4 Pez
Define n by V3 = n V2 adiabatic
Then p3 = p2 n-7 so Pu Vu
1

P4 V4= P3 V3= P2 V2 n 1-7


= p1 V
1
n 1-7 CI P3,1/5
l y? so 17,41 = 7 ni-Y or V4 mg nVi
= p

P4 V41
2 1 -y 3
Also Q1 = p2 V2 In — p3 V In —3 n p2 v2 In —
V1' Q'2 3 V4

Q2
Finally = 1 --„ = 1 - n = 0242

(b) Define n by p3=


n
P2 y
p2 V2 71 V3 or V3 sm n 1/7 V
2
So we get the formulae here by n -0 ni/Y in the previous case.
(1/7)- l
=1-n 1-n 018

2.115 Used as a refrigerator, the refrigerating


efficiency of a heat engine is given by
Q21 Q2' Q2v°—1 1-
E — = 9 here,
A Q1-Q2 1-Q2 ' -
Qi
where ri is the efficiency of the heat engine.
228

2.116 Given V2 = n V1, V4 = n V3


Q1 = Heat taken at the upper temperature
= Rri ln n+ R T2 In n= R (Ti+ T2) In n

1
(Ti)i - 1
Now Ti V1-1 = T2 V; - 1 or I/3 = V2
T2
1 1
TT' 1 Ti)Y - 1
Similarly Vs = (-- V4 , V6 = (-- V1
T3 T3
= heat ejected at the lower temperature = V6
Thus Q2 - RT In —
3 v5
1 1
V1 -1
= - R T3 In 77,'1 I - 1 v ize - R 7.3 in T1 'I V2
i2 If 4 T2 n2 V3
1 1
TiY- 1 1 Ti I - 1
= - R T3 ln Er-, = 2R T3 In n
12 2 T
n 2
27'
3
Thus T1= 1 Ti + T2
Cv
2.117 0 C (T T
Ar., '2 = V N- 2 - - 3,1 = T V2 (P2 -PO
Cy
Q1 = (P1 -P4)
R
J2 -
V2 (/ p3)
Thus TI = 1 -
Vi (pi -p4)
On the other hand,
6 V? = p2 VI , p3 VI = p4 lq also V2= nV1
p1
= p2 ny, p4 = p3 n Y
Thus p1

and 1= 1 - n 1- ?, with y = il for N2 this is ri = 0.602

C
2.118 Q1= R pi (V2 - Vi) , Q'2 = -CE p2 (V3 - V4)

p2 (V3 - V4)
So ri = 1 -pi (v2 - vi) eY/
PIA
1
Now pi= np2, pilq or V3= n y V2
1
n Y Vi
p2 V,1 = pi 17;1 or V4=
1 1
SO 1= 1 - -1 • •n = 1 -n?
-1.-
n ;
-1 Pgy÷
229

2.119 Since the absolute temperature of the gas rises n times both in the isochoric heating and
in the isobaric expansion
p1= np2 and V2 = n V1. Heat taken is
Q1 = Qii + Q12
where Q11 = Cp (n - 1) T1 and Q12 = Cv Tl(1 - ni) Heat
rejected is
= Q'21 Qt22 where 1

Q'21 = Cv (n 1) , V22= Cp (1
n1)
1 1

/ Q1
C V 1 ) 1' Cp (L T1 )

Thus 11 = -Q = 1-
: cp (n - 1) + C v(1 -
19214 Q' P2 ;1/9_
(47n) 22 V
n - 1 + y 1-1 1+Y
=1- - 1- n ,
y(n-1)-1-(1--;)
1 1 ny
y+—

2.120 (a) Here p2= n p1, piVi=


np1 Vi = Po Vo
V
Q'2 = RT0 In — Q1= v To n - 1)
V1' C (
-1
But n Vj" 1 or, V1= Vo n 1
RT
Q'2= RT0 ln n 1= Inn
y-1
Thus 1 = 1 - In n on using Cv=
n-1, -1
(b) Here V2 = nV1 , pi Vi= Po 170
and p1 (n Po Vj
_7_
i.e. nY 1-4-1 or Vi n / 1 Vo
170
Also Q1= C,, (n - 1) , Q' 2= RT In Vl
/Pi, V2, nTo
PIO adiabatic
Ry
or Q12 = RT0 ln n y-l= T ln n = C In n To
y-1 ° p TO
Thus In n isothermal Poly°
11 86
1n-1 cm
230

2.121 Here the isothermal process proceeds at the maximum temperature instead of at the minimum
temperature of the cycle as in 2.120.

P
adiabatic

P2)Vi,Tofri
(a) V
fib)
Pi
(a) Here pi Vi = po Vo , P2 = n
V?' Vj or pi VI = npo Vj
P2 = Po
1
-1
i.e. Vi'' = nVj -1 or Vi = Vo n y - 1

Vi. RT0
Q'2= Cy T (1 -1)Ton , Qi= RToln — ln n - C v Toln n-
V0 y-1

4 Qi 2 4 n1
Thus 1= 1 — = .1. - n In n
Qi
Vi
(b) Here V2 = —n , Po Vo = Pi V1

V or Vi = n (1 /' 1 -1) Vo
Po VO = Pi VI = Pi n - Y /71' " o-1 n-i TY -1
V1 Ry
Q'2= Cp To (1 —1 , Q1= RToln — = To in n = CpTo ln n
n Vo y-1
n-1
Thus '11 1
= n in n
of index a. We shall
2.122 The section from (iii, Vi, To) to (p2, V2, To/n) is a polytropic process
assume that the corresponding specific heat C is + ye.
Here, dQ = CdT = C v dT + pdV
ovo,To) (21
Now pr = constant or TVa- 1 = constant. PP
isothermal
RT R
so pdV = 17 dV = - a 1 dT 13,14,Th
R 1 1
Then C= Cv - a _ 1 = R
y-1 -a-1
diabalic
RT0 pi Vi
We have pi Vi = RT0 = p2 V2 . —
n n216

poV0= P 1 Vi = n p2V2, p0VJ = P2 V2,


231

1
r1- y -1 or V2= Vo n y - 1
Pi Via - pilq or V v
n 2
1 1 1
Vi - 1 . 1 vo t -1 or V1= n a-1 V2= ny-i-a-1 Vo
n 2

1 V, 1 1
Now Q'2 = CTo (1 - -), Q1= RToln= = RT ( - )1n n = CTo ln n
n Vo ° y- 1 a 1

n-1
Thus T1 '' 1
n In n

2.123 g,V0,To 13 vo 7;
77
13,V0,7Tri
621
6121
adiaba& .....**•- ..............

Po) Vo,To

Ti 1 T,
(a) Here Q'2 = Cp (Ti - —n ) = Cp T1 (1 --) Q1 - CV (T0 - --:-)
n 1 n
Along the adiabatic line

To Vj - 1 = Ti (n Vo) Y - 1 Or, To= T1nY-1-

Ti y (n-1)
SO Qi = Cv — (n Y - 1). Thus ri = 1
n n Y-
(b) Here V2 = Cv (n Ti -T0), Qi= Cp•Ti(n- 1)

Along the adiabatic line WI - 1 = constant


y-1

vo
Toi,J-1.T1H n
or Ti = nY - 1 To

Thus ni - 1
2.124 11 = 1- 1
po,V0,To y /1Y - (n-1)

Po 71 Vo 7-
P7°7 1 V0 ).T.n.o 2 77-, ,10
(a) (b)
232

(a) Q '2= CpT0(1 --


Q1= Q'1 +Q„1
n
Q"1= RToln n, Q'1= CvTo(1 -1),
1)'
1 1
n
Cp (

(2'2
So r i= 1— — = 1 -
Q1 Cv(1 — Li)-I - R In n

y (n - 1)
=1- y =
1- 1+
R nlnn n-1+(y-1)nlnn
Cvn-1
(b) Q1= CP To (n - 1), Qu2= CvTo(n -1) Q"12= RT0 In n , Q'2= Q"2+ Qw2

_V2 - 1 + (y - 1) In n
So
Q1 y (n - 1)

2.125 We have
and
Q'1 = T Rro In v, Q"2 = Cv To(t— 1) Q1 = Q'1 Q"1
Q"'2

= RTo In v, Q”, = Cv To (-r - 1) as

well as Qi = Q1' + Q1" and


Q2' = Q2" + Q2m

, Q'2 Cv (t — 1) +Rinv
So ri= - +1=
Cv(t-1)+TR1nv
t- 1
y -1 + In v (-r - 1) In v
T—1 T—1
T In v t In v +
y-1 y-1

2.126 Here Q1" = CpTo - 1) , Q11" = tRTolnn and

Q 2" = CP T0 (t — 1) , Q " = RT0Inn


in addition to we have
Qi = + Q1" and

Q2 = Q211 + Q2111 0
(-r - 1) +R In n
So =1- =
Cp(T-1)+TRInn

+I1-yJlnn
t-1
=1-
t-1-1-(1 - - 1)-r1nn
Y

T_i+ 1— ,' In n
= 1-
(
t - 1 + ( 1 - - 1)tlnn
Tin n
(t - 1) In n

Y y-1
233

2.127 Because of the linearity of the section


BC whose equation is
vV ,
= ki•• p = aV)

pY 0
We have — v or v=
v=

Here Q"2 = Cv To (Nrt — 1), A


Po,Vo,To
Q"2 C
To
( r —1)
= C
P P \Fr
T
Thus Q12= Q" 2+ Qm 2= I? 1 )
(1
Y

Along BC, the specific heat C is given by

CdT= Cvdr+pdV= CvdT+d(locV 2) = (Cv+1R)dT

-1
Thus Qi = 12R To T
y— vt

Q'2 'CT + y 1 — 1) Orr — 1)


Finally rl = 1 — = 1 — 2 ,—
VT+1Y+1 — (y+1)(V—T+1)

2.128 We write Claussius inequality in the form

f1Qf dQs 0
where d-Q is the heat transeferred to the system but ct2 Q is heat rejected by the system,

both are +ve and this explains the minus sign before et2 Q,
In this inequality T > T> T.n and we can write

<0

f f"
Q1
&Q Tmax T. ►.

Thus
Q/2 T.. V2
or
Tmax T.. Tmax< Q1

or TI = 1 —Q2 < 1 — T x — carncg


Qi

2.129 We consider an infinitesimal carnot cycle with isothermal process at temperatures


T +dT and T.

Let &A be the work done in the cycle and SQ, be the heat received at the higher temperature. Then
by Carnot's theorem
234

6A dT P+dP
°Q12' T
P
On the other hand SA = dp dV = (-1 dT dV 1 3
8T v 1
1
u I
while SQ1 = dU12 + pdV = [a av + p dV 1
T 1
1
Hence i
V Vi-dV
2.130 (a) In an isochoric process the entropy change will be
T
f

f CvdT T Rlnn
AS= - C ln f - C lnn-
T v Ti v y 1
T,

For carbon dioxide y = 1.30


so, AS= 19.2 Joule/°K - mole
(b) For an isobaric process,
y Rlin ',., isotherm
AS= C ln :7.L = C inn=
P T, y1 ,

= 25 Joule/°k -mole
\.,
2.131 In an isothermal expansion
V
AS= vR1n--'-
Vi

_L 2.132
s
o' V .- eA S/VR = 2.0 times

2.132 The entropy change depends on the final & initial states only, so we can calculate it
directly along the isotherm, it is AS = 2R In n = 20 J/°J(
(assuming that the final volume is n times the inital volume)

2.133 If the initial temperature is To and volume is Vo then in adiabatic expansion.

TV?-1 = ToVJ-1

V,
so, T= T n1-11= T1 where n =0 1 -
170
V1 being the volume at the end of the adiabatic process. There is no entropy change in
this process. Next the gas is compressed isobarically and the net entropy change is

es 2. (—m c )in -TI


P T
235

Vi . Vo Vo
But or T . - . T an Y
Ti Tf ' 1 T1 V1
( ) 1 m Ry
So AS = m — C In - = - m— C In n - Inn - - 9.7 J/K
MP
n MP -if y - 1
2.134 The entropy change depends on the initial and • Po 11/o To
final state only so can be calculated for any
process whatsoever.
We choose to evaluate the entropy change along
the pair of lines shown above. Then
a
To To
Ii 13 •
f vCvdT dT
AS = +fvC Pa ,Vo T°
T PT P° 4\io 4to
T
o
• To _ 1p 1 ..■
)
A
I
= (- Cv ln 13 + Cp In a) v = vR (y In a - In f3) .. JJouleo
-11 o K
Y -1
2.135 To calculate the required entropy difference we only have to calculate the entropy difference
for a process in which the state of the gas in vessel 1 is changed to that in vessel 2.
Ti
T1
—ciTi Ti

f dT f dr
6S = v Cv-f- + Cp T
T T1
1
all
pi,v1,T7
= v (Cp In ot - C yin aft)
In
= v Et in a - R— In 1-3) = v R ( Ina-
y-1 y-1
5
With y= i a= 2 and 13= 1.5,v = 1.2,
PI I )
T1
this gives AS -0.85 Joule/°K 0( 1 3 . 1 1 1 1 , 0 C

Ri--E1111431;
2.136 For the polytropic process with index n GM 0
p V" . constant
Along this process (See 2.122)
C = R (--1- - 1 1 -= n -y R
y-1 n-1 (y - 1) (n - 1)
t To

As, . dT
So C - RInt
T (y- 1)(n- 1)
236

2.137 The process in question may be written as


p V
a —
PO Vo
where a is a constant and po, Vo are some reference values. For this process (see 2.127) the
specific heat is

C= C + 1R = R +1)= I R 12..1-
2v y-1 2 2 y-1
Along the line volume increases a times then so does the pressure. The temperature must
then increase a2times. Thus
2
a To

dT y+1 a2
AS= vC — = vR In a
T 2y-1 y-1
T

if v= 2, y= a= 2, AS= 46.1 Joule/°K


3'
2.138 Let (p1, V1) be a reference point on the line

p= pc, - aV
and let (p, V) be any other point.
The entropy difference
AS= S (p, V) - S (pi , V1 )

P V V
2ECy In p—1 In = Cy 1n19° V +C In
p P
+ V1 Pl
For an exetremum of AS
a As -acv cn
o
av po - ccV+ V.
or C (po - a V) - ccV Cy= 0

YPo
or y (po - a V) - a V = 0 or V= V. = a (y 1)

a2 AS
This gives a maximum of AS because <0
av
(Note :- a maximum of AS is a maximum of S (p, V) )

2.139 Along the process line : S = aT + C In T

or the specific heat is : C = T —dS= aT + C v


dT
On the other hand : dQ CdT = C v dT + pdV for an ideal gas.

pdv. RT dv
Thus, V ciT dT
237

dV R ,
or —
a V al dT or,
-ln v + constant - T
a
R V
Using T = To when V = V0, we get, T = To + -c-i In 170-

2.140 For a Vander Waal gas

(p + a2) (V - b) - RT
V
The entropy change along an isotherm can be calculated from
v2
as
A dV
S ' f Vii)
., T
vt
It follows from (2.129) that
(as) . (al . R
aV 7, aT V-b
v
assuming a, b to be known constants.
V2- b
Thus AS= R ln
171 - b
7
'2
V2' T2 V
2

2.141
We use, AS. f dS' (V, 7) - f(aa1TS) dT + f (fv5) dV
V1 T., T2
Vi' Ti T1 I V
t
T2 V2
C vdT f R T2 V2 - b
j T + V - b dV = C v in T1 +R ln V1- b
Tt V1
assuming Cv, a, b to be known constants.

2.142 We can take S-0, 0 as T -0 0 Then


T T

S=1 o
C -d2: . f aT2 dT . -1 aT3
T
o

2.143
T
238
1
n
2.144 Here T= a r or S= (-,j7
1-1
1Tn S
Then C= T
n avn n
Clearly C<0 if n<0.

2.145 We know,
T

CdT
S -So = C ln
T To
To
assuming C to be a known constant.
S -S0)
Then T = To exp C

dS cc
2.146 (a) C =
T dT
T2

T
(b) Q = CdT = a In
2
Tt

(c) W = AQ - AU = a In + Cy (Ti T2)


I2

Since for an ideal gas Cy is constant


and AU= Cy (T2- T1)
(U does not depend on 17)

2.147 (a) We have from the definition

Q = f TdS = area under the curve

Q1 = To (Si - So)

Q'2= (To + Ti) (Si - So)


S
To
Thus, using T1 = — , SoiTo
n

+ n n - 1
TO T1
1= 1 - 1 - 1+ -1
2 To 2 2n

(b) Here Q1= (S1 - So) (T1 + To)


2
Q'2 = TI (S1 - S0)
2T1 To 7.1 n-1 So Ti
=1 Q2
+ To To-i1 n+1 S
239

2.148 In this case, called free expansion no work is done and no heat is exchanged. So internal
energy must remain unchanged Uf - Ui. For an ideal gas this implies constant temperature
Tf = Ti. The process is irreversible but the entropy change can be calculated by considering a
reversible isothermal process: Then, as before
V2

As=f 42-fPdv
T T
.vR In n = 20.1 J/K

vi
2.149 The process consists of two parts. The first part is free expansion in which Uf = 14 The
second part is adiabatic compression in which work done results in change of internal
energy. Obviously,
Vo

0= OF—Uf+f pdV, Vf = 2 Vo
V
I
1
Now in the first part p1 = -i RI, Vf = 2V0, because there is no change of temperature.

2Y- 1po Vj
In the second part, p VY = .iyo (2V0)Y -
vo
vo
v
f pdv. f 2Y-ipo KT dv.[ 2Y-1p0V1) vi_yl °
VT [ —y+1
2; .12. Vo
2 Ve

The entropy change AS = AS1+ AS11


AS1= R In 2 and LiSH = 0 as the process is reversible adiabatic. Thus AS = R In 2.
2.150 In all adiabatic processes
Q= Uf — Ui + A = 0
by virtue of the first law of thermodynamics. Thus,
Uf = Ui — A
v

For a slow process, A' =f pdV where for a quasistatic adiabatic process pITY = constant.
v0
On the other hand for a fast process the external work done is A" <A`. In fact A" = 0
for free expansion. Thus U'f (slow) <U"f (fast)
Since U depends on temperature only, Tf<rf
Consequently, p"f > p'1
(From the ideal gas equation pV = RT )
240

2.151 Let Vo, V2 = n Vo


Since the temperature is the same, the required entropy change can be calculated by con-
sidering isothermal expansion of the gas in either parts into the whole vessel.
+ V2 R ln + V2
Thus AS = AS, + AS„ = v1RIn + V2
Vl V2

1+n
= R In (1 + n) + v2 R In = 5.1 J/K
n
2.152 Let c1= specific heat of copper specific heat of water = c2
To 97 + 273

T
f C2 M2 d f micidT To 370
Then AS T = m2 c2 In — m1 ci In
280 o
7 + 273 Ta

To is found from
280 m2 c2 + 370 m1 c1
C2 M2 + m1 C1

using c1 = 0.39 J/g °K, c2 = 4.18 J/g °K,

To• 300°K and AS= 28-4 — 24-5 III 3-9 J /


2.153 For an ideal gas the internal energy depends on temperature only. We can consider the
process in question to be one of simultaneous free expansion. Then the total energy
U= U1 + U2. Since
T1 +T2
U1= Cy T1, U2 = Cy T2, U= 2C„ 2 and (T1 + T2) / 2 is the final temperature. The

entropy change is obtained by considering isochoric processes because in effect, the gas
remains confined to its vessel.
(Ti + T2)/2
T2

f cvdT f dr (Ti + T2)2


AS= — Cy = C iny
4 Ti T2
(Ti+T2)/2
i

Since (Ti + T2)2. (T1 — T2)2 + 4 Ti T2 , AS > 0


1
2.154 (a) Each atom has a probability -2- to be in either campartment. Thus

p = 2 —141
(b) Typical atomic velocity at room temperature is 105 cm/s so it takes an atom 10-5
sec to cross the vessel. This is the relevant time scale for our problem. Let
T = 10 5 sec, then in time t there will be t/T crossing or arrangements of the atoms. This will
be large enough to produce the given arrangement if
t -N In T
—2 , 1 or N In 2 — 75
241

2.155 The statistical weight is


N! 10x9x8x7x6.
&gra-. 252
N 8x 4x3 x 2
N/2 !

The probability distribution is


Nc 2-N = 252 x 2-10= 246 %
N/2

2.156 The probabilites that the half A contains n molecules is


N! N
N 2- =
CC n ! (N - n) !
2.157 The probability of one molecule being confined to the marked volume is
_V
P = T0-
We can choose this molecule in many (Nci) ways. The probability that n molecules get
confined to the marked volume is cearly
pn(1-pf-"- !
'1 n ! (N - n) !Pn (1 n
2.158 In a sphere of diameter d there are
nd3
N= 6 no molecules
where no = Loschmidi's number = No. of molecules per unit volume (1 cc) under NTP.
The relative fluctuation in this number is
aN Nriv 1
N N IS =

1 n _0 6
or — a no or 2= or d - ( 6,, ) = 0.41 1AM
11 = 6 It no ri2 n 71" no
1 6
The average number of molecules in this sphere I' s—2- = 10
li

3
2.159 For a monoatomic gas Cy= —R per mole
2
The entropy change in the process is
To+ AX

dT 3 +A
AS= S -So= f Cy— = — R ln(
T 2 To
To
Now from the Boltzmann equation
S- kin SI
3N
A ) 3 x6 x 10232
C e(S-So)/k. (1+ AT) 2
- (1 + - 1013 X 1021
S20 To .1 300

Thus the statistical weight increases by this factor.


242

2.5 LIQUIDS. CAPILLARY EFFECTS


2160 A_ ( 1 1 ) . 4a
(a) '-"P '1 a V1/2+ d/2) d
—3
4 x 490 x 10 N . 1.307 x 10 6 N
se 2
= 13 atmosphere
1.5 x 10 -6 m2 m
(b) The soap bubble has two surfaces
i 1 1 ) 8a
SO
AP= A- a (d/2 + c1/2 = d
8 x 45
= 2 x 10 —3 = 1.2 x 10 —3 atomsphere.
3 x 10 -"
2.161 The pressure just inside the hole will be less than the outside pressure by 4 a/d. This can
support a height h of Hg where
4a 4a
pgh= 7 or h = p g d

4 x 490 x 10-3 200


• -21 m of Hg
13.6 x 103 x 9-8 x 70 x 10 -6 -13-6 x 70

2.162 By Boyle's law

or

Thus

2.163 The pressure has terms due to hydrostatic pressure and capillarity and they add
4a
P= Po+ Pgh + 7
5 x 9.8 103 4 x-73 x 10-3
= (1+ I-
105 X 10- 5) atoms = 2.22 atom.
4 x 10 —
2.164 By Boyle's law

(po+hgp+ -4cSr :d3m (Po+ 14n1 -16 n3e13

(n3 - 1)] =
Or [hgp -Po 4a (n2 - 1)
d

or h = [po (n3 - 1) + -4111- (n2 - 1) } / g p = 4.98 meter of water


d

-

243

2.165 Clearly

Ahpg= 4alcos01 -+11,;) ■••••••


•••

cow.

4 a Icos 0 I (d2 - di) a■■.


Ah= = 11 mm """ . . . . • ■• • • •
• ■•

di d2 pg ONO

••••
•■•••

■••
.■••• •■•• ••••••
■■•• ••••••• ••••••

2.166 In a capillary with diameter d = 0.5 mm water


will rise to a height
2a 4a
pgr pgd

4 x 73 x 10 -3 •
3= 59.6 mm *ft/6

10'3 x 9.8 x 0.5 x 10 - •■•

•■■

Since this is greater than the height


( = 25 mm) of the tube, a meniscus of radius
R will be formed at the top of the tube, where
•-•• = =

2a 2 x 73 x 10 -3 .Ie==

MM.•

R= 0.6 nun
P gh 103 x 9.8 x 25 x 10 -3 ■Imb 0= = =
=.•

2.167 Initially the pressure of air in the capillary is po and it's length is 1. When submerged

under water, the pressure of air in the portion above water must be po + 4— since the
d 2

level of water inside the capillary is the same as the level outside. Thus by Boyle's law
4a ,
o+ d (1- x) = po /

4a (
Or 1- x) = pox or x -
Pod
1+
4a

2.168 We have by Boyle's law


(po pgh + 4 a dcos 0) (/ Po

4 a cos 0 po h
or,
d = p gh +
1-h
po h d
Hence, a= (PO+ 1-h 4cos 0

2.169 Suppose the liquid rises to a height h. Then the total energy of the liquid in the capillary
is

E (h) = - h x pg x - (d2 - di) ah

Minimising E we get
4ct
h= 6 cm.
pg (d2 - di)
244

2.170 Let h be the height of the water level at a


distance x from the edge. Then the total energy
1s
of water in the wedge above the level outside is.

E=f xbcp• • •
2
-22 J f doc•h•acos0
2 2 a cos 0 •

= f pg5 (h2 - h
dx x x P gb rP
2
(h, 2 a cos 0 4 a2 COS2 0
xf d x 6 -
2 Pg xpg6cp x2 p2 g2 cp2
2 a cos 0
This is minimum when h =
xpgorp
2.171 From the equation of continuity
A2

d2 • = 11 • V or V= n2 V.
4 4 n
We then apply Bernoulli's theorem

+ v2 + = constant
P 2
The pressure p differs from the atmospheric pressure by capillary effects. At the upper
section
2a
P Po+ d

neglecting the curvature in the vertical plane. Thus,


n
2a 2 n ct
Po +
ro d 1 “d 1 4 2
+ v2 + gl =
+ v
2 2n

4
v= 2 g1-— (n- 1)
Or Pd
n4 -1
Finally, the liquid coming out per second is,

4a
V- gl - — (n - 1)
17w/2 pd
4
1 n4 -
2.172 The radius of curvature of the drop is R1 at the upper end of the drop and R2 at the lower
2a
2a
at the top end and po + -15-- at the bottom
end. Then the pressure inside the drop is po + -6- /12
end. Hence
2 a (R2 - R1)
+ 2a = po + 2a,
Po — + p gh or - p gh
Ri R2 R1 R2
h 1
To a first approximation RI..., R2 .• -2 so R2 - RI g%, -8 p gh 3/a. ,%, 0.20 mm

if h= 2.3 mm, u = 73 inN /in


245

2.173 We must first calculate the pressure difference inside the film from that outside. This is
1 1
p. ri r2

Here 2 r1 'cos 0 I h and r2 —R the radius


of the tablet and can be neglected. Thus the
total force exerted by mercury drop on the upper
glass plate is
2 nR2 a lcos 01
typically
h
We should put hl n for h because the tablet is compresed n times. Then since Hg is nearly,
incompressible, me h = constants so R Wit . Thus,
2nR2alcos 01
total force = n2
Part of the force is needed to keep the Hg in the shape of a table rather than in the shape
of infinitely thin sheet. This part can be calculated being putting n 1 above. Thus
2 2
2 TC R a. I cos 01 2 It R a I cos 0 I
mg+ n2
2nR 2 a 'cos 01 2
or M- (n - 1 ) 0.7 kg
hg
1
2.174 The pressure inside the film is less than that outside by an amount a (1 + — where
r1 r2
r1 and r2 are the principal radii of curvature of the meniscus. One of these is small being
given by h .. 2 ri cos 0 while the other is large and will be ignored. Then
2A cos 0
F h a where A - area of the water film between the plates.
2ma
Now A = — so F when 0 (the angle of contact) = 0
Ph p h2
2.175 This is analogous to the previous problem except that : A = n R 2
2nR2 C
So 0-6 IcN ": •
2.176 The energy of the liquid between the plates is

E= 1dhpg
1 h

•—••••—
pgldh2 -2alh
_

2
-2a1h.
2 • ••

...memo ..emw •■• • • • • . •

•••

1 2 a )2 2 a2 / -- - -
-

•••■•
" •
= 2 n-- ....me mwome

ere. .
••• ••
em■emill me.

Pgid( pgd pgd ■■•• •

2a emaawole
mem..
em

This energy is minimum when, h.. pg d and


2 0.2 /
the minimum potential energy is then Enun—
pg d
The force of attraction between the plates can be obtained from this as
- a E.. 2 a2 1
F. (miinnuuss sign means the force is attractive.)
ad 187
alh
Thus F= — 13N
246

2.177 Suppose the radius of the bubble is x at some instant. Then the pressure inside is
4
Po + a. The flow through the capillary is by Poiseuille's equation,

x r4 4ct A 2 dr
Q —
8 yi 1 x dt
r4 a 4
Integrating ic (R — x4) where we have used the fact tthhat,t = 0 where x= R.
2
211 1R4
This gives t = as the life time of the bubble corresponding to x=
cc r4 0
2.178 If the liquid rises to a height 11, the energy of the liquid column becomes
2

E= pgnr2h•1-2nrhoc= pgn(rh-2--L-) -241 (1'2


2 2 Pg Pg
This is minimum when rh = 2 a' that is relevant height to which water must rise.
Pg
2 it a2
At this point, min —
pg
2 It CIL2
Since E 0 in the absence of surface tension a heat Q must have been liberated.
Pg
2.179 (a) The free energy per unit area being a,
F= n a d2 = 3µJ
(b) F. 2 it a d2 because the soap bubble has two surfaces. Substitution gives F - 10
ti J

2.180 When two mercury drops each of diameter d merge, the resulting drop has diameter d1

where d x 2 or, di. - 2" d


6 I 6
The increase in free energy is
AF= 7t2213d2a-27cd2a= 2Icd2a(2-v3- 1) - -1.43µJ
2.181 Work must be done to stretch the soap film and compress the air inside. The former is
simply 2 a x 4 nR 2 ms 8 It R 2a, there being two sides of the film. To get the latter we
note that the compression is isothermal and work done is
v- V

4a 4 7t
V --3-- R
f pdV where VoPo (Po + R , V- 3
v,- vo
py_ 4a
or Va= , p po +
Po
and minus sign is needed becaue we are calculating work done on the system. Thus since pV
remains constants, the work done is
Vo
pVln pV ln
Po
So A'= 8 icR2 a +p Vln
Po
247
2.182 When heat is given to a soap bubble the temperature of the air inside rises and the bubble
expands but unless the bubble bursts, the amount of air inside does not change. Further
we shall neglect the variation of the surface tension with temperature. Then from the gas
equations

) 43 3
(po + 4a n r Re V R T, v = Constant

Differentiating

(Po+ 8a)4ar2dr= vRdT


3r
vRdT
Or
dV = 4 a r 2 dr - 8a
Po + 3r
Now from the first law
v R dT
tt Q= v CdT . v Cv dT + 8a (Po + 4a)
Po + 3r
4a
Po + r
or C = Cv+R
8a
Po 3r
1
R
2
using Cp = C V + R) C = C p+
3 POr
1 +
8a

2.183 Consider an infinitesimal Carnot cycle with isotherms at T - dT and T. Let A be the work
done during the cycle. Then
r) 7) da A
A= [a (T - d - a ( I •5 a = - — dT oa
dT
Where So is the change in the area of film T-dr
(we are considering only one surface).

Then q
= 21= —dT by Carnot therom. oc
Qi- T
da
- — dT o a da
Or dT dT T
rq q86
goo. - T O =- dT
es
2.184 As before we can calculate the heat required. It, is taking into account two sides of the
soap film
da
oq = - T— 8 a x 2
dT
oq da o a
Thus AS= . -2
T dT
da
Now AF= 2 a o a so, AU= AF + T AS = 2 (a.-T — So
dT
248

2.6 PHASE TRANSFORMATIONS


2.185 The condensation takes place at constant pressure and temperature and the work done is
pAV
where AV is the volume of the condensed vapour in the vapour phase. It is

p AV=.62-1-n—RT = 120.67
M
where M = 18 gm is the molecular weight of water.

2.186 The specific volume of water (the liquid) will be written as Yr Since 1/1„ » V'1, most of
the weight is due to water. Thus if m1 is mass of the liquid and m, that of the vapour then
m= tilt + m,

V= mi Vi + m, V„ or V— ml/' 1 = m v (V v — V 1) V —

mV 1
So m — V,1= 20 gm in the present case. Its volume is m, V', = 1-01
v 17%, — i

2.187 The volume of the condensed vapour was originally Vo — V at temperature T = 373 K.
Its mass will be given by
mn„, M p (Vo — V)
p(V0 — V) —-- or m = R7, — 2 gin where p = atmospheric pressure
Mill
2.188 We let V 1= specific volume of liquid. V%) = N V'1= specific volume of vapour.
Let V= Original volume of the vapour. Then
, 7 pV V V
.1,2 M
= v, r — = (ml + Nmv) V'1
RT m1
a. +
v N o1 n

1) V mi VI n—1
So (N — 1) mi V I= V (1 — — = — (n — 1) or Tp.
n n V/n N—1
In the case when the final volume of the substance corresponds to the midpoint of a
horizontal portion of the isothermal line in the p, v diagram, the final volume must be
VI
(1 +N) —
per unit mass of the substance. Of this the volume of the liquid is V1/2 per unit
2 total
mass of the substance.
Thus

2.189 From the first law of thermodynamics


AU+A = Q= m q
where q is the specific latent heat of vaporization
RT
Now A = p (V v — V 1) m = m m

R
Thus AU= m(4? — T m

For water this gives = 2.08 x 106 Joules. )


249

2.190 Some of the heat used in heating water to the boiling temperature
T= 100°C = 373 K The remaining heat

Q-mcAT

(c = specific heat of water, AT= 100 K) is used to create vapour. If the piston rises to a
height h then the volume of vapour will be • sh(neglecting water). Its mass will be
po sh posh Mq
M and heat of vapourization will be . To this must be, added the work
RT x RT
done in creating the saturated vapour •• posh. Thus
Q-mc AT
Q-mcAT.poSh(1+ or h = = 20 cm
l+gLif\
RT

2.191
pos(
A quantity mc (T- 1.°) of saturated vapour must condense to heat the water to boiling
q
point T= 373°K
(Here c= specific heat of water, To = 295 K = initial water temperature). The
work done in lowering the piston will then be
me (T - To) RT
x = 25 J,
q
RT
since work done per unit mass of the condensed vapour is pV=

m
RT
2.192 M RT
Given AP —Pv 2a Pv 4a Pv
Pt r sir —pi PvaP si 11 V
x yap
4 aM
Or d=
piRTT1
For water a = 73 dynes/cm, M.. I.figm, Pi = gm/cc, T= 300 K, and with Ti ••• 0.01, we get
dm 0.2 µ m

2.193 In equilibrium the number of "liquid" molecules evaporoting must equals the number of
"vapour" molecules condensing. By kinetic theory, this number is

8 kT
rixin<v>.
Tixinx11x4 <v>= nm

Its mass is
_ r171'—
mxixnx v = it:kr Fr1 -
2nm 2nkt

V g/cm2 •
= 11P° 2 nRT = 0.35 s.

where po is atmospheric pressure and T= 373 K and M= molecular weight of water.


250

2.194 Here we must assume thatµ is also the rate at which the tungsten filament loses mass
when in an atmosphere of its own vapour at this temperature and that rl (of the previous
problem) • 1. Then

2 RT
P — 0.9 n Pa

from the previous problem where p = pressure of the saturated vapour.

2.195 From the Vander Waals equation


RT a
P = V — b v2
where V= Volume of one gm mole of the substances.
For water V 18 c.c. per mole = 1.8 x 10 -Mitre per mole
litre2
a = 5.47 atmos 2
mole
If molecular attraction vanished the equation will be
, RT
=V—b
for the same specific volume. Thus
a 5.47
2— x 104 atmos
4
1.7 x 10 atmos
AP v 1.8 x 1.8

2.196 The internal pressure being --I, the work done in condensation is
V

dv a a a
l7a2 Vg Vi

This by assumption is Mq , M being the molecular weight and VI , Vg being the molar
volumes of the liquid and gas.
a Mq
Thus PI' 2 pq
TVi1

where p is the density of the liquid. For water pi si 3.3 x 1013 atm

2.197 The Vandar Waal's equation can be written as (for one mole)
RT a
P(1)
V — b v2
2
At the critical point (1 and --2i vanish. Thus
av T av T
RT 2a RT 2a
0- , o r _2.. ,
-(V.-W + V'' (V— b) V'
2 RT 6a RT . 3a
0 — or
(V— b)3 V4 (V— b)3 V4
251

Solving these simultaneously we get on division


2
V - b = V, V = 3 b VMCr

This is the critical molar volume. Putting this back

R Tcr 2a 8a
- or To, =
4b2 27b3 27 bR

=
Finally PCr

PCr VMCr a/9b 3


From these we see that
RTcr 8a/27 b 8

PCr a/27 b 2 1
2.198
RTcr 8a/27 b 8b
To.
Thus b=R -
8 pc,
(RTcr)2 64a
and or
PCr 27

2.199 Specific volume is molar volume divided by molecular weight. Thus

VMCr 3 RTcr 3 x •082 x 562 litre cc


Vcr g 4.71 —g
M 8 MPcr 8 x 78 x47
a
2.200 (12 + b)= RT
VA(
a
P x V. - b 8 T
or = p
PCr VMCr 3 1Cr

or + a 2) X (V - b =
PCr VMCrMCr 3

Von
where = , V —, t=
PCr VMCrv Cr

it4. 27 b 2) (v 1 8 8

When
( V2 A2f

n= 12 and v =
3
—T
3 '
or (7 r +

T x 24 x 1 =
v -3 3

2' 8 6 2
is the maximum volume in the liquid phase and the minimum
2.201 (a) The ciritical Volume VMCr
volume in the gaseous. Thus
1000
Vm. —18 x 3 x -030 litre • 5 litre
252

(b) The critical pressure is the maximum possible pressure in the vapour phase in equilibrium
with liquid phase. Thus
a 5.47
=225atmosphere
Pn l a X =
27 b 2 27 x •03 x •03

8 a 8 3.62
2.202 . 304 K
TCr = 27 bR =
27 x -043 x •082
M 44
CrP — 3b
-
43 gm/c.c. — 0.34 gm/c.c.
3x

2.203 The vessel is such that either vapour or liquid of mass m occupies it at critical point. Then
its volume will be
3 RTc m
V Cr = amL VMCr — 2—
8 Pcr M
The corresponding volume in liquid phase at room temperature is
m
V=
I)
where p = density of liquid ether at room tmeperature. Thus

V 8 MPcr
11 — — 3 RTc, p . 0.254
Vo.
using the given data (and p = 720gm per litre)

2.204 We apply the relation (T= constant)

Tf dS 4 dU+f pdV

to the cycle 1234531. 12


Here f dS 4 titl= 0

So # pdV= 0

This implies that the areas I and II are equal.


This reasoning is inapplicable to the cycle 1231,
for example. This cycle is irreversible because
it involves the irreversible transition from a
single phase to a two-phase state at the point 3. V
2.205 When a portion of supercool water turns into ice some heat is liberated, which should
heat it upto ice point. Neglecting the variation of specific heat of water, the fraction of
water turning inot ice is clearly

f . cid . 0.25
q
where c= specific heat of water and q .. latent heat of fusion of ice, Clearly f = 1
at t = — 80°C
253

2.206 From the Claussius-Clapeyron (C-C)equations

dT T (V2- ifi 1)

dp 412
gals the specific latent heat absorbed in 1 --1. 2 (1 = solid, 2 = liquid)

T(Vw- Vice) 273 x .091 atm x cm3 x K


AT= _ x1
AP — 333 joule
q12
5 N
10 —m2 x 10 - 6 m3
3
atm x cm = 10 -1, AT = - -0075 K
1 pi
Joule Joule

2.207 Here 1 = liquid, 2 = Steam

AT - T (V s - V lig) AP
q12

412 AT 2250 09
or rs. 0, - x 5-- x 10 - 3 ii3/g = 1.7 m3/kg
T Ap 373 -2

2.208 From C-C equations

dp _ 412 q12
a T (V2- V'1) T 1r2

Assuming the saturated vapour to be ideal gas

nik Mq
Thus Ap - 2p AT
V2 — RT , RT

M
and P'" Po (1 + q 2 A T)ag 1.04 atmosphere
RT

2.209 From C-C equation, neglecting the voolume of the liquid

dp_ qu Mq

dT a TV' a RT-212' (q = q12)


CiE . Mg dT
or
p RT T

MpV
Now pV = -1?;i RT or m = for a perfect gas
RT

So L. clp. _ dT (V is cons. = t speci fi c volume)


m p T `

(Mq 1)dT . (18 x 2250 _


1) x gs 4-85 %
Sa RT - T 8.3 x 373
254

2.210 From C-C equation


_2. q . Mq
dT Tr 2 R T2P
Mq
Integrating lnp - constant -
RT
Mg ( 1 1 I
So p = poexp [ R 7,0- - i)

This is reasonable for IT - T0I « T0, and far below critical temperature.

2.211 As before (2.206) the lowering of melting point is given by


T AV'
AT= -p
q
The superheated ice will then melt in part. The fraction that will melt is
C T AV'
T1 = q P ' .°3

2.212 (a) The equations of the transition lines are


1 800
logp = 9.05 - . Solid gas
T

1 31 0
= 6.78 - T . Liquid gas
At the triple point they intersect. Thus
490 490
2.27= — or Tv = - 216K
To 2.27
is 5.14 atmosphere.
corresponding Pb.
b
In the formula log p = a - - we compare b with the corresponding term in the equation
r
in 2.210. Then

In p = a x 2.303 - 2.307,3 b So, 2.303 = Mg


R
2.303 x 1800 x 8.31
or, qsublimation=
= 783 J/gm
44
2-303 x 1310 x 8.31 -570 j/gm
illiquid-gas =
44
Finally qs.lid_iil„id= 213 J/gm on subtraction

T2

2.213 dT m_ T2
AS - f me 7,- + 2 11111 m (C ln — + --q--)
Ti. T2
T1

+ 225
103 4.181n 373 0283 3,T3-) . 7-2 kj/K
255

T2 q),
2.214 L S= + c ln
Ti T T2
333 373
+ 4.18 In — 8.56 J/°K
273 283

2.215 c = specific heat of copper = 0.39 i--• •Suppose all ice does not melt, then
•K
heat rejected = 90 x .0-39 (90 - 0) = 3159 J
heat gained by ice = 50 x 2.09 x 3 + x x 333
Thus x= 8.5 gm
The hypothesis is correct and final temperature will be T = 273K.
Hence change in entropy of copper piece

me In 22-3- - 10 J/K.
363
2.216 (a) Here t2 = 60°C. Suppose the final temperature is t °C. Then
heat lost by water = m2 c (t2 - t)

heat gained by ice = ml gm+ m1 c (t - t1) , if all ice melts In this


case mi gm = m2 x 4.18 (60 - t), for m1= m2
So the final temperature will be 0°C and only some ice will melt.
Then 100 x 4.18 (60) = mil x 333

mil = 75.3 gm = amount of ice that will melt

333 273
Finally AS= 75.3 x — + 100 x 4-18 ln
273 333
mil qm
6S= + c lnT
1 -2
(T2 - T1) T
MI n ?
M2 C i Tt

(b) If m2 c t2 > mi q„, then all ice will melt as one can check and the final temperature can
be obtained like this
m2 c (T2 - T) = mi q„, + mic (T - T1)
(M2 T2 + Ti) C - Mi q„,= (mi+ m2) cT
migne
M2 T2 + -
or T si 280 K
mt. + m2
mi T T2
and AS= Ti + c 1 ln Ti - m 2 — = 19 J/K
T
256

l M qke
T2
2.217 AS —
1
T +
m
c
ln

where
47.2 M thee al M (

q2

C ( T2

T1 ) )

m„_ ( 1 1) ( T2 T2
+mc 1-
.12 Ti T2

= 0.2245 + 0.2564 - 0.48 J/K

2.218 Wh e n h e a t dQ is given to the vapour its temperature will change by dT, pressure by dp
and volume by dV, it being assumed that the vapour remains saturated.

Then by C-C equation

» VLq), or dp dT
(Vvapour
dT TV'

on the other hand, pi,' = RitiT

pdv, v, dp RmdT
So

Hence pdV' = (ly -I)dT

fmally dQ = CdT = dU + pdV'

= Cy dr + C p dT - dT

(Cp, Cv refer to unit mass here). Thus

C= C
P T
1 wi y
For water C tn F32 a n d M = 18
Y 1 M
So C 1.90 J/gm K

and C = - 4.13 J/gm°K = - 74 J/mole K

2.219 The required entropy change can be calculated along a process in which the water is

heated from T1 to T2 and then allowed to evaporate. The entropy change for this is

T2 nAl
/14.5 = C In — +
P T T2
w her e q = specif i c latent heat of vapo ri z at i o n .
257

2.7 TRANSPORT PHENOMENA


2.220 (a) The fraction of gas molecules which traverses distances exceeding the mean free path
without collision is just the probability to traverse the distance s . A. without collision.
1 1
Thus P . e- . — . 0.37
e
(b) This probability is
-1
P- e - e-2 = 0.23
2.221 From the formula
1 ...t,v), A/
-- e or A, .
T1 In Ti
2.222 (a) Let P (t) = probability of no collision in the interval (0, t). Then
P (t + dt). P (t) (1 - a dt)

dP - 01 t
or dt - - a P (0 or P (t) - e
where we have used-P (0) = 1
(b) The mean interval between collision is also the mean interval of no collision. Then
00

f te " dt
o 1 r (2) . 1
‹./ >2.
cc a r (1) a
5 Cat dt
o
2.223 1 kT
(a) I. . , ,
V2 nen Vind2p
1.38 x 10-23 X 273
, .. .. 6.2 x 10-8m
V2 n (0.37 x 10-9)2 x 105
62 x
T -8 s- .136 n s
<V> 454
k - 6.2 x 106 m
04
(b) 1= F36 x 1 s = 3.8 hours
2.224 The mean distance between molecules is of the order
IA
(22.4 x 10-3 )115 . 224 x 10 - 9 meters a3.34 x 10 -9 meters
6.0 x 1023 6

This is about 18.5 times smaller than the mean free path calculated in 2.223 (a) above.
2.225 We know that the Vander Waal's constant b is four times the molecular volume. Thus
1/3

b= 4N 3/ d3 or d- (A 6 312 )
2 n NA
2/3

Hence
258

2.226 The volocity of sound in N2 is


Virf
P = M
1 1/7/50. RT0
SO,
M d2 po NA
or, V = 7( d2po NAA P Y 7
MRT0
kT
2.227 (a)
k>1 if P< Vind21
kT
Now for 02 of 0 is 0.7 Pa.
VI d2 /
(b) The corresponding n is obtained by dividing by kT and is 1.84 x 1020 per m3 =
1.8414per c.c. and the corresponding mean distance is vs.

10-2
1-8 x 10- m = 0.18ttm.
(0.184)1/3 x 105
1 1 <v >
2.228 (a) v -
k/<v >
d2n <v > •74 x 1010 s-1(see 2.223)
(b) Total number of collisions is

v a 1.0 x 1029 s cm-3


2n
1
Note, the factor • When two molecules collide we must not count it twice.

1
2.229 (a) X -
V2- d2 n
d is a constant and n is a constant for an isochoric process so k is constant for an isochoric
process.

<V >
v= ,
A,
1 kT
(b) = - a T for an isobaric process.
v2 d2 P
<v >
v afor an isobaric process.
T VT
2.230 (a) In an isochoric process X, is constant and
v a 1ff a pV a Vp- a
kT
(b) = _ r- must decrease n times in an isothermal process and v must increase
v2 nd2p
n times because <v > is constant in an isothermal process.
259

1 1 V
2.231 (a)
ka;"N/V"il
T1/2
Thus ka V and v a V

But in an adiabatic process (y - i here)

TVY-1.. constant so 7V2/5411 constant

or T1/2 a V- 1/5 Thus v a V- 6/5


(b) X, a 1:
P
Y
_ vi
But p(-;7) ... constant or Taapp or Tap1-1/Y
P

Thus A, a p-1ly ..1 p-5/7


1 L+.1
<v> +— 2 y
v- a1'. ap 1/2 ai p 2Y = p 6/7
X VT
(c) A. a V
But 7'V" - constant or V a T-572

Thus A. a T-5/2

T1/2 3
v a - ,i a T

2.232 In the polytropic process of index n


pVn .. constant, TV"' .. constant and pl-n rig constant
(a) )s. a V
1-a -n+1
T1/2
va l- V 2 V-1- 'V 2

, 1
T"ap"-
(b) A. a —T, 1 or Tap ' n

SO kap-1/11
i n+1
<v> p 1-- +—1
v= a vi,r ap 2 2n la p 2/1
x
n

(c) kaT,paTn-1

4 n 1 1
1 " • ...........
A m T1-n
kar n 1
T K"
260

2.233 (a) The number of collisions between the molecules in a unit volume is
1 1 2 VT'
- nv —7--7td2 n <v>a 2
2 112 V
This remains constant in the poly process pV-3 I. constant
Using (2.122) the molar specific heat for the polytropic process
pVa = constant,
1 )
is C= R
Y-1 a - 1
1 11 5+1 11
Thus C = R —y 1 + 74) 1( as R
1
It can also be written as — R (1 + 2 i) where 5
4
_
(b) In this case constant and so pV = constant
V

so C = R(-y1"f+ R (13+ = R

It can also be written as — (i + 1)


2
2.234 We can assume that all molecules, incident on the hole, leak out. Then,

- dN - d (nV). <v > S dt


4n
dt dt
Or dn- - n
4v/S <v > =
-n —

y8RT
Integrating n. no e . Hence <v > =
M
2.235 If the temperature of the compartment 2 is times more than that of compartment 1, it
1
must contain — times less number of molecules since pressure must be the same when
1
the big hole is open. If M = mass of the gas in 1 than the mass of the gas in 2 must be
So immediately after the big hole is closed.

o M o M
n mV n MI711
2

where m — mass of each molecule and 7,°, , n2 are concentrations in 1 and 2. After the
big hole is closed the pressures will differ and concentration will become n1 and n2 where

n +n (1
1 2 m 171 +11)

On the other hand


n1 <v1 > = n2 <v2 > i.e. ni = WI n2
261

Thus n2(1 + rri) ÷n:li (1+ f(1+i1)

n2. n)
So
1 +Nrri
2.236 We know

T 3 <v > X, p <v > d2 m a if

Thus 1 changing a times implies T changing a2times. On


the other hand
1 1 Virf i kT
—<v>k=
3 3 It"' iiitd2p
73/2
Thus D changing 13 times means changing 13 times
p
a3
So p must change — times
Pn

2.237 Da NriccVNIT , nocif


(a) D will increase n times
n will remain constant if T is constant
T3/2 (P ,13i2
(b) D a a = p1/2 T7312
p p
a VT/.
Thus D will increase n 312 times, ri will increase n1/2 times, if p is constant
2.238 D a VIrf , rl a if
In an adiabatic process
TVY-1= constant, or T cc
1
Now V is decreased — times. Thus
282

So 1 remains constant in the isothermal process


pV = constant, n = 1, here
(c) Heat conductivity K 311 1 Cv
and Cv is a constant for the ideal gas
Thus n = 1 here also,

1 VcrcT m 2 .,{7ncT 1
2.240 71 s m
1/2 c/2 3 7t3 d2
1 1/4 1/2 1/4
( 2 ) /2 (m k ( 2 6) (4 x 8.31 x 273 x 10')
or d = x 10 3
3r1 3.( 37) 3 x 18.9 7E X 36 x 10"
1/2 4
4 x 834 x 273)1/ .,
= 10-1° ( 2 0.178 m
3 x 18.9) ( x .36
1
2.241 K= — <v > X, p cv
3
1 8 kT 1 Cv
mn
3 mn Vind2n M
Cv)
( - . Now Cv is the same for all monoatomicCv is the specific
heat capacity which is ---"
M
gases such as He and A. Thus
1
Ka
1,41 a'

K He= dA2 Viii"


or 8.7 =
KA 547/dH CIH2 .

dA rcr

1.658 Pi F7
c/H. VT.6

2.242 In this case

N1 = 4nriw

2 R AR 2 7t 1 CO R3
or N, RA I" 4 7c w or
AR
To decrease N1, n times 1 must be decreased n times. NOW 1 does not depend on pressure until
the pressure is so low that the mean free path equals, say, 1 AR. Then the mean free
2
path is fixed and ri decreases with pressure. The mean free path cquals I AR when
2
1
= AR (no = concentration)
d2 no
263

Ili k T
Corresponding pressure is po a. 2
a d AR
The sought pressure is n times less
Nr2- k T 10- 23
P- 2 - 70.7 x ..• 0.71 Pa
nd nAR 10 - 1-8 X 10 - 3
The answer is qualitative and depends on the choice 1 AR for the mean free path.
2
2.243 We neglect the moment of inertia of the gas in a shell. Then the moment of friction forces
on a unit length of the cylinder must be a constant as a fUnction of r.

3 N1 (1 _ 1)
So, 2 n r Ti —dw a. N1 or (o (r) =
dr 4 71 Ti r2 r2
' 1 '
N1 1 _1 N1 (1 1
and oi . or ri .
4 it ri 1.21 r22 — 2- r 2)
4 n 0) r1 2
2.244 We consider two adjoining layers. The angular velocity gradient . is cos Ti. So the moment of

the frictional force is


a

71 a4 co
N= f r•27trdr-nr—°). —1--
h 2h
0
1
2.245 In the ultrararefied gas we must determine rl by taking X, .2 - h. Then
2
1 8kT 1 , . _ liiE . 1 V72M --h
11 22 -3- —WE XinX kT 3 7t RT "P
1 1/7171
SO, N = po a4 p 2T7-,
2.246 Take an infinitesimal section of length dr and apply Poiseuilles equation to this. Then
dV - 7t a4 pi
dt 8T1 az

From the formula pV = RT • iitii

RT _,
pdV al — UM
M
dm It a4 M pit
Or . - . L - Ti dx
dt 8 RT
42.
This equation impliei that if the flow is isothermal then p dr must be a constant and so
1 p22 _ p21 1
equals I in magnitude.
21
2
7C a4 M IP22
Thus, 14 - -Pi
16-1-0 T 1
264

2.247 Let T= temperature of the interface.


Then heat flowing from left = heat flowing into right in equilibrium.
(xi Ti K2 T2+
1 12
T1- T T- T2 1
Thus, K1 1 = K2 or T=
1 122 J. K2)
, + 12
2.248 We have
Ti-T T- T2 Ti. - T2
K1 =K2 IC K , ,
11 12 • 41 + 42
or using the previous result
2
K1 T1 K T2
, +
K1 41 /2 T1 - T2
T1- =K
li 1
K
11 + 12
1 K1

1 1+ 1, 1
K2
—, (T1 - T2)
Ki 12 T1 - T2 11 + 12
Or K2 =K + or K do 1 1
li K1 4. __ 11 12 ti gl
- 11 -. + —
12 Ki -K2

2.249 By definition the heat flux (per unit area) is


dT d In T1/T2
Q= -K—dx= -a —dr ln T= constant = + a
/

x T2
Integrating ln .T= 7 ln — 1nT
T+1 1
where T1= temperature at the end x. 0
za /
a In T1 T2
So T= Ti(-77-2) and Q=

time
2.250 Suppose the chunks have temperatures T1, T2 at time t and T1- dri , T2 + dT2 at
dt+t.

Then K S
Ci c/Ti = C2 dT2 - 7 (T1 - .7.2) dt

K SII . 1
Thus dAT= - 1 [&- + -a- AT dt where AT = Ti - T2
1 2

KS 1 1
Hence AT = (AT)0e-th where 1 =
t / Ci + C2)
266


2.251 Q = K ar al -A if ar
ax ax
2 aT 3
=- A /2 ( = constant )
3 ax ' A
2 3/2 3/2
( T1 - T2 )
= 3A 1

Thus T34 = constant - =xi( Ti3/2- T23/2)

or using T= T 1 at x= 0
3/2
3
( T 1

)
T3/2= /2 + X ( ( T2
Tr + '"I ( T3/ 2 /2 - T3/2 =
)1
Or
Ti 1 T )
-1

-L
1 \FITT 1 mn 2 . R3/2 i T3/2
2.252 K En -3- am vi 7td2n Al 3n 3/2 d 2 1.617 NA

Then from the previous problem

(T1 + T2)
2.253 At this pressure and average temparature = 27°C = 300K. T =
2
2330 x 10 5III = 2.3.3mm » 5.0mm = 1
-
A= 1
V2 nd 2
KT-
P
1
The gas is ultrathin and we write X.= - here
2/
dT T2 - Ti.
Then q= K — mi K

dx 1
1 1 MP R 1 p<v> I
where K= i<V>X
21x RTxy-lxM= 6T(y-1)
p<v>
and q - 6T (y -1)(T2- TO
where <v>- —
T2 +T1 here.
3t e have used T2 - T1 «
Mn
M W.
266

A
2.254 In equilibrium 2rr K dT = —A = constant. So T = B — In r
dr LII
K
But T= T1 when r= R1 and T= T2. when r=R2.
T2 — r
From this we find T = T1+ In —
In
Ri
2 dT
2.255 In equilibrium 47t r K— — A = constant
dr
A l1
B+ —
ztioc r
Using T T1 when r R1 and T T2 when r R2,
T2 — (1 1
T T+
1-1 r R1
R2 Ri
2.256 The heat flux vector is — K grad T and its divergence equals w. Thus
T.
v2
K

_a _ aTr in cylindrical coordinates.


or K
r ar ar

0)
or T — B+Alnr-- r
2x
Since T is finite at r = 0, A = 0. Also T = To at r = R
2
SO B= T +—R
° 4K
Thus T= To+ --AW- (R2—r2)
44K
r here is the distance from the axis of wire (axial radius).
2.257 Here again
v 2 7. . w
K

So in spherical polar coordinates,

1 a ( 2 aT 2 aT w 3
— or r =— r +A
ar r ar K a? 3K
'

A w 2
or T= B — r
r OK
so finally
Again
and B = To+ —
w 2 6K R
A
T. T +w (R2_r2)
=
0

° 6K
PART THREE

ELECTRODYNAMICS

3.1 CONSTANT ELECTRIC FIELD IN VACUUM

2 2
3.1 Fd (for electorns) = :---q--2-and F -"I
r=
4 ncor r
Fei
e
Thus (for electrons) =
Fir 4neoym2

(1.602 x 10 -19 C)2


=4 X 1042

x 10 -11 m3 / (kg • s2) x (911 x 10 -31 kg)2

( 1
9 x 10'

Fd
n) X 6.67

q2

Similarly Fr (for proton) = - 4ncoym


-1

(1.602 x 10 -19 C)2


= = 1 x 1036
( 1 - 11
9) X 6.67 x 10 m3 /(kg • s2) x (1.672 x 10 -27 kg)2
9 x 10
For Fd = Fr

42 v m2 n
'----- or Zi = r4 7JT07
4neor2= r2

V 6 - 6 7 x 10 -11 m3 (kg — s2)


= 0.86 x 10 -1° C/kg
9 x 109
1
3.2 Total number of atoms in the sphere of mass 1 gm = x 6.023 x 1023
63.54
6.023 x 1023-1
So the total nuclear charge A. = x 1.6 x 10 9 x 29
63.54
Now the charge on the sphere-,z. Total nuclear charge — Total electronic charge
288

6.023 x 10 -19 29 x 1
- 23 x 1-6 x 10 x ... 4.298 x 102 C
63-54 100
Hence force of interaction between these two spheres,

1 [4.398 x 10212
F. N= 9 x 109 x 104 x 19.348 N= 1-14 x 1015N
4 n eo 12

3.3 Let the balls be deviated by an angle 0, from the vertical when separtion between them
equals x.

Applying Newton's second law of motion for any one of the sphere, we get,

T cos 0 . mg (1)

and T sin 0 - Fe (2)

From the Eqs. (1) and (2)

F
tan 0= e (3)
mg

as x « / (4)

> Fe

Fe. Ty_ or
47tE0X
q
2

2=
mg z

21
ing

2 ,t eo mg x3
2
Thus
4 - (5)
I
Differentiating Eqn. (5) with respect to time

da 2 it eo mg 2 eh
4 s . 3 x-
1 dt
dx dr
According to the problem —dt . v = aAri (approach velocity is )
dt '
1/2
(2 it E0 mg 3) dq 3 x Eo mg x2 a
SO, x
1 dt g. 1 Nrx-

3 2 Jt eo mg
Hence -—a
' dt 2 1
3.4 Let us choose coordinate axes as shown in the
figure and fix three charges, qi. , q2 and q3
--, --a. --,.
having position vectors ri, r2 and r3
respectively.

Now, for the equilibrium of q3


-30 --0 •
+ q2 q3 ( r2 - r3 ) qi q3 ( ri. - r3 )
+ --0 --a. 3 - 0
I r2 - r3 I 3 I ri - r31
269

q2 qi.
or,
I 2° - r31 2 I. I 737 - 73'] 2
--lb —10 —is —II.
r2 - r3 ri - r3
because
1 r7- r3'I = - I ?.-- 731

or, Nq; (IT- 3)=Nriii ( 3- 2)


= — —0
—P. Vq2 r--■+ V Ii tii. .r2
or, r3 =
Nrii7+ V-4;
Also for the equilibrium of q1,
• •-ilP ..41, .10.
r
q3(
3 - ri ) + q2( r2 - ri ) =
117:- t7n3 1772- 77-113

-q2 --l• -1.2


or, q3 - , 21 r1 - r3 I
r1
1 2- I
Substituting the value of r3 we get,

- ql q2
q3 -
( Nrq-1- + Nri; )2

3.5 When the charge q0 is placed at the centre of


the ring, the wire get stretched and the extra
dFe
tension, produced in the wire, will balance the
electric force due to the charge q0. Let the
tension produced in the wire, after placing the
charge q0, be T. From Newton's second law in
projection form F, = mw,.

Td 0 --j. - 1° (--2---r d 0) = (dm) 0,


4 n E0 r2 2 nr

q qo
or, T=
82t2E0r2

3.6 Sought field strength


-. 1 q
E-
4 n E0 I r-. 712

= 4.5 kV/m on putting the values.


3.7 Let us fix the coordinate system by taking the point of intersection of the diagonals as
the origin and let k be directed normally, emerging from the plane of figure.
Hence the sought field strength :
270

q + xk —q l(—i)+xk
E— 4 n Eo (12 + x2)312 +
4 1E Eo (12 + x2)3/2
--9,
—q 1j +xk q l(-1)+xic*
4nEo + x2)3/2
(12 4 rc Eli) (12 + x2)3/2

ql
Thus E —
n go (12 + x2 )3/2

3.8 From the symmetry of the problem the sought field.


4-
E = f d Ex di
±
+ do
where the projection of field strength along x 1-
— axis due to an elemental charge is
? -->
1f 9 0 X"
dq cos 0 _ q R cos 0 d 0 +
Ex >4.
x 4 n £0 R 2 4 7t2 E0 R3 t E
+
It/2
Hence
q q ++
E.' 2 2 cos 0 d 0
4 Tt £0 R 2n2£0R2
N.,/2

3.9 From the symmetry of the condition, it is clear that, the field along the normal will be
zero
i.e. E.= 0 and E = Ei

dq
Now
dEi — 2 cos 0
47t Eo (R 2 1- 1 ' )
1
But a q dx and cos 0= 2 2 1/2
d =
2nR (R +l )
Hence
2zR

dx
E= f dEi= f 4 n E0 (R 2 + / 2 )3/2
2 TER
0
1 ql
or E — 47c e 0 (/ 2 +R 2 )3/2

and for 1» R, the ring behaves like a point


charge, reducing the field to the value,
1 q
E
4neo 12
271

dE
For Emax, we should have 7ll- = 0

)3/2 _ 1 (12 4. R 2 )1/2 2/ .


So,(12 + R2 0 or 12+R2-312= 0
2
R q
Thus /-= --,—_- and E max
V2 6 13 - ne oR 2

3.10 The electric potential at a distance x from the given ring is given by,

cp (x) =
q q
4 n Eo x 4 7t Co (R2 + X2)1/2

Hence, the field strength along x-axis (which is the net field strength in 'our case),

dcp q 1 qx
= - —=
dx 4 n go x2 4 n Eo (R 2 + X2 )3/2

R2)3/2
q 3 { (1 + -T —1
x X
4 it co
x2 (R2 + x2 )3/2

3 R2 3 R4
q X3 1 —— —— + I
4 7C Eo 2x2 8 x •••

2 2 3/2
x (R + x2)

Neglecting the higher power of R/x, as x » R.


2
3qR
E—
8 n eo x4.
Note : Instead of cp (x), we may write E (x) directly using 3.9

3.11 From the solution of 3.9, the electric field strength due to ring at a point on its axis (say
x-axis) at distance x from the centre of the ring is given by :

And from symmetry E at every point on the axis is directed along the x -axis (Fig.). Let

us consider an element (dx) on thread which


carries the charge (X. dx). The electric force
experienced by the element in the field of ring.
kqx dx
dF = cbc) E (x)
4 It Eo (R2 + X2)3/2

Thus the sought interaction


00

kqxdx
2 2.3/22)3/
F = 6 / 4 n eo (R + x2
0

On integrating we get, F—
4 TE eo R
272

3.12 (a) The given charge distribution is shown in Fig. The symmetry of this distribution
implies that vector Eat the point 0 is directed to the right, and its magnitude is equal to the
sum of the projection onto the direction of E of vectors difrom elementary charges dq. The
projection of vector dZonto vector E is

dE cos cp — 1 1(1 cos fp,


4 n E0 R 2
where dq= kRdq)= R cos cp d cp.
Integrating (1) over qi between 0 and 2 7c we
find the magnitude of the vector E:
2,

, f 2 ,/
E— cos yu =
4 7c Eo R 0 4 E0

It should be noted that this integral is evaluated in


the most simple way if we take into account
that <cos2 > = 1/2. Then
2x

0
cos2q dq <COS
2
cp 2 n = 7E. f
(b) Take an element S at an azimuthal angle cp from the x-axis, the element subtending an
angle d y at the centre.
The elementary field at P due to the element is
X0 coscpdcpR
along SP with components
4 3.c Eo (x2 + R2 )
ko cos cp d p R
2 2 x { cos 0 along OP , sin 0 along OS }
4 n eo +R
x
where cos 0 •B 2 2 )1/2
(X + R
2x

The component along OP vanishes on integration as 5 cos cp d cp = 0


o
The component alon OS can be broken into the parts along OX and OY with
Xo R 2 COS cp d q9
x { cos ip along OX, sin cp along OY }
4 a Eo (X2 +R 2 )3/2
On integration, the part along OY vanishes.
Finally
x0R2
E = Ex=
4 Eo (x2+ R 2 )312
For x » R
273

3.13 (a) It is clear from symmetry considerations that vector E must be directed as shown in
the figure. This shows the way of solving this problem : we must find the component
dEr of the field created by the element dl of the rod, having the charge dq and then

integrate the result over all the elements of the rod. In this case
1 dl
dE r= d E cos a = , cos a,
4 eo 1,6

whereX = is the linear charge density. Let us reduce this equation of the form convenient
2a

for integration. Figure shows that dl cos a = ro d a and ro


COS II
Consequently,
rod k
dE,= r cos a d a
=
4 n 60 r20 4 n eo
This expression can be easily integrated :
Q0

E= 2icosada- 2sinao
4 n E0 r 4neor
0 dF
where ao is the maximum value of the angle a,

sinao= aira2.f.r2
q/2a a
Thus, E- 2
4 7t Eo r 2 2 4
a + r Eo r a +r

Note that in this case also E 2 for r » a as of the field of a point charge.
4 n Eo r
(b) Let, us consider the element of length dl at a distance 1 from the centre of the rod,
as shown in the figure. dl
Then field at .13 due to this element. l vA

if the element lies on the side, shown in the


dl
diagram, and dE =
2'
if it lies on
4 n eo (r +1)

other side.
a a

f A dl dl
Hence E = f dE =
2+
4 7t Eo (r - 1) 4 7t Eo (r + 1)2
0
q 1
On integrating and putting X. = we get, E= 0.2 2)
2a' 4 n eo a

For r » a, E q
4 7t Eo
274

3.14 The problem is reduced to finding Ex and ; viz. the projections of E in Fig, where it is
assumed that k > 0.
Let us start with Ex. The contribution to Ex from the charge element of the segment dx is
1 X dr in
dEx - s a (1)
4 eo r2
Let us reduce this expression to the form convenient for integration. In our case, dx =
r da/cos a, r = y/cosa. Then
k
dEx =.r 4 n soy sin a d a.

Integrating this expression over a between


0 and n/2, we find
Ex= X/4 7r eo y.
In order to find the projection ; it is
sufficient to recall that dE differs from dEx
in that sin a in (1) is simply replaced
by cos a.
This gives
dE = (X. cos a d a )/4 y and E = X./4 it e0 y.
We have obtained an interesting result :
Ex = ; independently of y,
--3.

i.e. E is oriented at the angle of 45° to the rod. The modulus of E is


E. VEx2 +Ey2 kli/4neoy.

3.15 (a) Using the solution of 3.14, the net electric field strength at the point 0 due to straight
parts of the thread equals zero. For the curved part (arc) let us derive a general expression i.e.
let us calculate the field strength at the centre of arc of radius R and linear charge density
k and which subtends angle 00 at the centre.

From the symmetry the sought field strength dl


will be directed along the bisector of the angle
00 and is given by
+ 00/2

f (R dO) 00
E_ cos 0 - sin —
4 IT e0 R 2 2 it E0 R 2
- 0V2

In our problem 00 = 7t../2, thus the field strength


due to the turned part at the point
12-
which is also the sought result.
- 4 3r e R
(b) Using the solution of 3.14 (a), net field strength at 0 due to stright parts equals

and is directed vertically down. Now using the solution of 3.15


4 IT EoR 2 7t E0 R
275

(a), field strength due to the given curved part (semi-circle) at the point 0 becomes

and is directed vertically upward. Hence the sought net field strengh becomes
2 7t eo R
zero.

3.16 Given charge distribution on the surface a = r iiss shown in the figure. Symmetry of
this distribution implies that the sought E at the centre 0 of the sphere is opposite to
dq= cr(2nrsin0)rd0= cs•t12nr2sined0= 2nar3sin0cos0d0
Again from symmetry, field strength due to any ring element dE is also opposite to a i.e.
dE f a-,
Hence
dq r cos 0 -a
de- 0. 2 0 r2 cos2 0)3/2 a---arsing the result of 3.9)k`)
4 n Eo sin A+

(2 n a r3 sin 0 cos 0 d 0) r cos 0 (-


4 n eo r3 a

= r 0 cos2 d
a sin
2 0eo

Thus
de= (- illrfsinOcos20d0
2 E0
0
Wr2 ar
Integrating, we get z = - — =-
2E03 3Eo

3.17 We start from two charged spherical balls cad} of radius R with equal and opposite charge

densities + p and - p. The centre of the balls are at + -


and- -a respectively so th
–0 2 e2
a a a
equation of their surfaces are r - - R or r - -cosON R and r + -cosOeg R, considering
2 2 2
a to be small. The distance between the two surfaces in the radial direction at angle 0 is
acosO f and does not depend on the azimuthal angle. It is seen from the diagram that the
surface of the sphere has in effect a surface density a = ao cos() when
cr0 = pa.
Inside any uniformly charged spherical ball, the field is radial and has the magnitude given by
Gauss's theorm
4n
47cr = ar p/e0

Prr
Or
3 e.
In vector notation, using the fact the V must be
measured from the centre of the ball, we get,
for the present case

13 77*()--L1-(77.+ i)
3E0 2 3E0 2)
2 76

rw
= -pa/3E0 -

When kis the unit vector along the polar axis from which 0 is measured.

3.18 Let us consider an elemental spherical shell of thickness dr. Thus surface charge density
of the shell a = p dr.= (54:r) dr.
Thus using the solution of 3.16, field strength due to this sperical shell
ar
dE = - —
3 eo dr

Hence the sought field strength


R

a it
E= - a fr dr = -
3 Eo 6 eo

3.19 From the solution of 3.14 field strength at a perpendicular distance r < R from its left end
k A

t(r) - (-i )+ (er)


4nE 4ne r
r0

Here er is a unit vector along radial direction.

Let us consider an elemental surface, dS = dy dz = dz (r d 0) a figure. Thus


flux of E (r) over the element dS is given by
A

ci E • dS = [ (-i )+ ( d•dr(rd0)i
43tEr o 47te or er

= 4 n codr d0 (as e-7. I I- 4 °)

R 2x

R
The sought flux, (13 = - k f dr f d 0=
4 7r eo 2 Eo
0 0

If we have taken ciS t t ( - A then 41) were


X,R

2 E0

Hence
Eo

3.20 Let us consider an elemental surface area as shown in the figure. Then flux of the vector
E through the elemental area,

d (I) = di*= E dS = 2 Eo cos cp c/.5 (as Etf dS)

2q I 2q1 r dr de
( d 0) dr =
4 E0 (12 + r2 ) (/ 2 + r2 )- 2 4 n eo (r2 + / 2 )312
277

where E0 - magnitude of
q2 2
2 +r )
4 Jt go (/

field strength due to any point charge at the


point of location of considered elemental area.
2:t

rdr f
Thus (13 -11 1 - f d0
4 it go 0.2 + 12 )3/2
0 0
R

2 q/x 2n f r dr i /
[1 -
4 n go (r2 + 12)3/2 - EO V/2- -;—.R 2
+
o
It can also be solved by considering a ring element or by using solid angle.

3.21 Let us consider a ring element of radius x and thickness dx, as shown in the figure. Now,
fl ux over the considered element,

dam= E • dS = E r dS cos 0
pr E
But EE r= — from Gauss's theorem,
3E 0

ro
and dS= 2 x , cos 0=
r

Thus dcl) = 2 x dr - _r° 2 nx dr


3 E0 to

Hence sought flux

R217r1

2 ap ro 2np ro (R2 - ro2) P ro


c1) - fx dr 2- ro)
3 go 3 Eo 2 3 Eo
0
3.22 The field at P due to the threads at A and B are both of magnitude
2 1/2
2 a go (x + 12/4)
and directed along AP and BP. The resultant is along OP with

2 A, cos 0 x
E- 1/2.1/2 =
2 ned 2+ n Bo (.X2 + 12/4)

2
En X —- 2
" 4x 2 Vx

x
2 7, 1 2/1
Eo 7c - 2 v` r- +1 71 I
.4_
A
This is maximum when x = 1/2 and then E=E E0
—max =
278

3.23 Take a section of the cylinder perpendicular to its axis through the point where the electric
field is to be calculated. (All points on the axis are equivalent.) Consider an element S
with azimuthal angle q. The length of the element is R‘dq) , R being the radius of cross
section of the cylinder. The element itself is a section of an infinite strip. The electric field at 0
due to this strip is
R d5,
Go cos cp (R dcp)
along SO
2 n EoR
This can be resolved into
cro cos p d cp cos cp along OX towards 0
2 7c Eo sin p along YO
On integration the component along YO
vanishes. What remains is

23,
ao cos' cp dy_ cro
0 f 2 n eo
2 co along XO i.e. along the direction cp

3.24 Since the field is axisymmetric (as the field


Oc \\%\c'M%\l AcSttttilAmiten1), we conclude
= 7c.

that the flux through the sphere of radius R is


equal to the flux through the lateral surface of
a cylinder having the same radius and the height
2R, as arranged in the figure.

Now, (I) E • dr- E. rS

But Er =R

Thus (I) = 11-S = -11 2nR•2R= 47caR

3.25 (a) Let us consider a sphere of radius r < R then charge, inclosed by the considered sphere,

qindosed= f 4nr2drp= f 4 7( r2 po (1 - ti)dr (1)


0 0
Now, applying Gauss' theorem,

2 qinclosed
Er 4 7t r = , (where Er is the projection of electric field along the radial line.)
go

or,
279

And for a point. outside the sphere r > R.


R

r
=f 4nr2dr pc, 1 — — (as there is no charge outside the ball)
q
o R
lm'cosed
Again from Gauss' theorem,
R

_R
4 It r2 dr po 1 r
Er 4 n r2 =
go
0
Po [R3 R41 poR3
Or,
Er= r2 go 3 — 4R — 12 r2 e o
(b) As•magnitude of electric field decreases with increasing r for r > R, field will be
maximum for r <R. Now, for Er to be maximum,

3 r2) n , 3r n = 2R
or .1. — -- = 0 or r = rM —3
r — ,', 1? )— u

Hence
d( PoR
En.— 9 co
3.26 Let the charge carried by the sphere be q, then using Gauss' theorem for a spherical surface
having radius r > R, we can write.
r
q 1 f a
E 4Ttr2 — q inclosed— +— — 4 7t r2 dr
Eo go E0 r
R

On integrating we get,

q — 2 n a R2) 4 n a r2
E 4 n r2 — ( +
Ea 2 e0
The intensity E does not depend on r when the
experession in the parentheses is equal to zero.
Hence
a
q= 2 n a R2 and E—
2 eo
3.27 Let us consider a spherical layer of radius r and thickness dr, having its centre coinciding
with the centre of the system. Then using Gauss' theorem, for this surface,
r

qi nclaved f pdV
Er 4 n r2 = —
Eo eo
0
280

After integration
3
e-ar
Er 4 n r2 =
3 Eo a
3
or, Er - -C"
3Ea

3 r
Now when a r « 1, Err X
3 "0

3 PO
And when a r » 1, Er. 2
3 e0 a r

3.28 Using Gauss theorem we can easily show that the electric field strength within a uniformly.

charged sphere is E =
3 Eo
The cavity, in our problem, may be considered as
the superposition of two balls, one with the
charge density p and the other with - p.

Let P be a point inside the cavity such that its


position vector with respect to the centre of
cavity be 77:" and with respect to the centre of
the ball r Then from the principle of
superposition, field inside the cavity, at an
arbitrary point P,
I. I+ +I
--P— ( - -P--3EOa
3 Eo -
Note : Obtained expression for E shows that it is valid regardless of the ratio between the
radii of the sphere and the distance between their centres.

3.29 Let us consider a cylinderical Gaussian surface of radius r and height It inside an infinitely
long charged cylinder with charge density p. Now from Gauss theorem :

Er 2 it r h - qinclosed

Eo

(where Er is the field inside the cylinder at a


distance r from its axis.)

r
or, Er 2 71 rh = P r2 h or Er = 2p
0 Eo
Now, using the method of 3.28 field at a point P,
inside the cavity, is

E_ + = - 7°) =
2E0 ' 2E-0
281
3.30 The arrangement of the rings are as shown in the figure. Now, potential at the point 1,
= potential at 1 due to the ring 1 + potential at 1 due to the ring 2.
-q
+ 4 3T Eo (R2 + C12)112
4 n Eo R
Similarly, the potential at point 2,
q
CP2 4 n- etloR 4 1t Eo (R2 + a2)1/2
Hence, the sought potential difference,
A
-q
cP2 = = 2(—A— +
4 n eo R 4 n Eo (R2 + C12)1/2

3.31 We know from Gauss theorem that the electric field due to an infmietly long straight wire,
at a perpendicular distance r from it equals, Er = 2 n . So, the work done is
2 x 0r

f Er dr = dr
2 n Eo r 1
(where x is perpendicular distance from the thread by which point 1 is removed from it.)
Hence A X12 - 2 n eo in TI
3.32 Let us consider a ring element as shown in the figure. Then the charge, carried by the
element, dq = (2 nfl sin 0)R d 0 a,
Hence, the potential due to the considered element at the centre of the hemisphere,
1 41_ 2 naRsin 0 d0 a R sin d 0
d" 4 It Co R 4 1t E0 2E0
So potential due to the whole hemisphere
x./2

oR
cp = R ° sin 0dd 00 ==
Eo 0 2 Eo
Now from the symmetry of the problem, net
electric field of the hemisphere is directed
towards the negative y-axis. We have
dE - 4 n1eod Rq cos
2
0 =
2 co sin 0 cos 0 dO
x./2 x./2
Thus E=E ' . a f sin 0 cos 0 d 0 = , along YO
Y 2E0 0 4 eo f sin 2 0 d 0 - 4 E0
282

3.33 Let us consider an elementary ring of thickness


dy and radius y as shown in the figure. Then
potential at a point P, at distance 1 from the
centre of the disc, is

a2nydy
d = 2 + 12 )1/2
4 eo(y
Hence potential due to the whole disc,

f a 2 xydy al
(P3E 431 so (y2 + 12 )1/2 + ( R/1) 2
2e0

From symmetry

E Ei=
dl
cr 21 cr 1
2 eo 2
2V/72-47/2 E0
+ (R//)2
aR a
when / 0, ('14 E= and when / »R ,
2e 2 eo
o

3.34 By definition, the potential in the case of a surface charge distribution is defined by integral

1
= 2c--d3. In order to simplify integration, we shall choose the area element dS
4 7r e0 r
in the form of a part of the ring of radius r and width dr in (Fig.). Then dS = 20 r dr,
r = 21? cos 0 and dr = — 21? sin 0 d O. After substituting these expressions into integral

1 a dS,
cP= we obtain the expression for p at the point 0:
4ne0 r
0
R
= —° f 0 sin 0 d 0.
n Ea

We integrate by parts,
denoting 0 = u and sin 0 d 0 = dv :
fOsin Ode —Ocos 0

+fcos 0d0= —Ocos 0 +sin 0

which gives -1 after substituting the limits of


integration. As a result, we obtain
= aRht e0.
283

=a r
3.35 In accordance with the problem p --0. - -II,

Thus from the equation : i= - g(49

E_ - -aix (ar x) i + —
(ay y) j + -7,- (a, z) k l
[ax i + ay j + z k ] = - a+
a, a
°.z
3.36 (a) Given, (4) = a (x2 - y2)
-. --. ---. --.
So, E= -Vcp..-2a(xi-yj )
The sought shape of field lines is as shown in the figure (a) of answersheet assuming a > 0:
(b) Since (43) = axy
So, E= -VT.. -ayi-axj
Plot as shown in the figure (b) of answersheet.
= a (x2 + y2) + bz2
3.37 Given, p

So, :E= - 1.4 = - [2 axr+ 2 ayr+ 2 bz ic]

Hence I Z1 = 2 Va2 (x2 + y2) + b2 2

Shape of the equipotential surface :


--9. 7 . .-1. 2
Put p= xt+yi or p 2= X2 +y
Then the equipotential surface has the equation
a p2 + b z2 - constant = p
If a > 0, b > 0 then q) > 0 and the equation of the equipotential surface is

-P-- + -1-2 sim 1


cp/ a p/b
which is an ellipse in p , z coordinates. In three dimensions the surface is an ellipsoid of
revolution with semi: axislcp 7 a , 7a , iq7/72.
If a > 0, b < 0 then p can be 2 0. If IT > 0 then the equation is
p2 z2
1
cp/ a cp/Ibl
This is a single cavity hyperboloid of revolution about z axis. If (4) = 0 then
ape - 1bl z2 = 0

or z = ± 11;67 p

is the equation of a right circular cone.


If cp < 0 then the equation can be written as
lbj z2 - a p2 = 1(491
z2 _ p2 1
Or
Id /1b1 14)1 /a
This is a two cavity hyperboloid of revolution about z-axis.
284

3.38 From Gauss' theorem intensity at a point, inside the sphere at a distance r from the centre
1 q .2
is given by, Er= p and outside it, is given by Er =
3 EE o • 71 Eo o r
(a) Potential at the centre of the sphere,
R cc
00

R2
E f dr + f dr = + q
3 Eo 4 Tr e ro 3e 2 4 It eo
0

q q 3q P
as + ( as
8 n eo R 4 n eo R
8 21 eoR 4 7t R
(b) Now, potential at any point, inside the sphere, at a distance r from it s centre.
R CO

49 (r) f —P— r dr f —q-- dr


3 Eo 4 it Eo
r r
2 2
On integration (r) = [1 — 1 = cp [1 — LH
8 neoR 3 R2 3R2
3.39 Let two charges +q and —q be separated by a distance 1. Then electric potential at a point
at distance r » 1 from this dipole,
+q —q q (r_ — r+
cp (r) = +
4 n eo 4 n eo r_ 4 It Eo r+ r_

But r-—r+=
1 cos 0 and r r2
+ r-
From Eqs. (1) and (2),
—Jo —310

q 1 cos 0 p cos 0 p • r
cp (r)
4 n eo r2 4 n eo r2 q) 4 It e r0 ±
where p is magnitude of electric moment vector.
acp 2p cos 0
Now, Er.
ar 4 it eo r3

p sin 0
and E0 —
ra0 4 n eo

Er 2 02
3 V4 cost 0 + sine 0
So E= E
4neor
3.40 From the results, obtained in the previous problem,
2p cos 0 p sin 0
Er = 3 and E0
4 n eo 4 n eo
From the given figure, it is clear that,
ti
E, = Er cos 0 — Eosin 0 (3 cos2 0 — 1)
4ne0r
285

3p sin 0 cos 0
and El= Er sin 0 +Eo cos 0 -
4iteor3
When E l p, lEl. El and Ez = 0
1
So 3 cost 0 = 1 and cos 0 =
V3
Thus EZp-I.at the points located on the lateral surface of the cone, having its axis, coinciding with
the direction of z-axis and semi vertex angle 0 = cos -11 /
3.41 Let us assume that the dipole is at the centre of the one equipotential surface which is
spherical (Fig.). On an equipotential surface the net electric field strength along the tangent
of it becomes zero. Thus

p sin 0
- E0 sin 0 + = 0 or - E0 sin 0 + -0
4 n eo

1/3

Hence r P
4 it co Eo
Alternate : Potential at the point, near the dipole is given by,
p • r
(13 - - z • r + constant,
4n cor

( P
- E0 cos @ + Const
4 it co r-

For q' to be constant,

p 3 E0 = 0 or, 3- E 0
4 co r 4 it co r
1/3

Thus r- ( P
4 it E0E0

3.42 Let P be a point, at distace r »1 and at an angle to 0 the vector /

Thus E at 13
2 it co 2 It co .

+l/2 r-I12

2 it e0 { 2 l2
r + + r 1 cos 0 r2 + —1 2 - r cos 01
4 4

X (1 2 I rcos 0)
2 n E0 r2 r3

/
Hence E IE I
2
r» 1
2 it co r
286 ▪ ▪

Also, (I) Ink7+1721— /72]


2ngo 2 go

r2 + r 1 cos 0 + /4 / cos 0
In ,r»1
4n o g r2—r/cos0+ /`/4 2 go r

3.43 The potential can be calculated by superposition. Choose the plane of the upper ring as
x = 1/2 and that of the lower ring as x = —1/2.

Then (1 9 l"
q q
/2
4ngo[R2+x -1x] 2 112 4It go[R + x2 + /x)1
q lx ,,,, 1— rZ

2 ,
4 n go (R2 + X2)112 (1 + 2 (R +f) 4 7t E0 2 +
(B 2 (R"+

For

The electric field is E =

ql 3 q1 ql (2x2 — R2)
(R2 4. x2)3Z2 4- 2 (R2 x 2x =
4 n go 2
+ x )5/2 4 n go 4 t go (R2 + X2)5/2

For ql 3. » R, E
The plot is as given in the book.
2 n go x
3.44 The field of a pair of oppositely charged sheets with holes can by superposition be reduced
to that of a pair of uniform opposite charged sheets and discs with opposite charges. Now
the charged sheets do not contribute any field outside them. Thus using the result of the
previous problem

R I
(—o)12nrdrx
=
(I) 4 n ea (r2 X2)3/2

0
R 2
+X
2

a x/ f dy o xl
= _
4 co y3/2
2 2 eo \fR"7272
x
_
of 1 x 2 a /R2
Ex. — -E..
ax 2 El) VR2 2 - ( +x R2 + 4 3/2 2 €0 (R2 .1. x2)3/2•
The plot is as shown in the answersheet.

287

3.45 For x > 0 we can use the result as given above and write
/ xj
± 1- x 1j
2 eo ( (R2 + 2) 2
for the solution that vanishes at a. There is a discontinuity in potential for I xi = 0. The
solution for negative x is obtained by a - a. Thus
alx
(I) - 2)1/2 + constant
2 eo (R + x
Hence ignoring the jump
al R2
E
€0 (R2 + x2)3/2
ax = 2
for large Irl (pm and E 3 (where p = nR2 a 1)
4neox2 22reolxl

-) r
3.46 Here E Ee. 0 and F
r= 2 eo r ' = p aai
(a) Fialong the thread.
E does not change as the point of observation is moved along the thread.
F=
(b) p along

F W*.= kP 2 j:ze -
a
2 it eo r
, On using — e= o)
nreor- ar
(c) iralong
— a X,
F=p 7( r er
ra0 2 e o
--0
P a er pX
2 rt eo r2 a0 = 2 J't eo r2 am 2 n eo r2.

3.47 Force on a dipole of moment p is given by,

F= (I) 29

In our problem, field, due to a dipole at a distance 1, where a dipole is placed,

ri up ........... ■E■ ■

2 II eo /3
Hence, the force of interaction,
2
F- 3 2.1x10-16N
2 eo r

3.48 -dq= E dr= a (y dx + x d y) = a d (xy)


=
On integrating, p - a xy + C

3.49 - d = E dr.= [2 a xy r+ 2 (x2 - y2):1] • [dx +dy1 ]


or, d (19 xi: 2 a x y + a (x2 - y2) dy = ad (x2 y) - ay2 dy
288

On integrating, we get,
2
= ay(Y-- x2) +c
3
3.50 Given, again
--0 7110
– d cp.= E•(Ft (ayi+(ax+bz)j+byk)•(cfri+dyj+cbck)
= a (y + ax dy) + b (z dy + y dz) . ad (x y) + bd (yz)
On integrating,
= - (a xy + b yz) + C
3.51 Field intensity along x-axis.

E
- --L = 3 a x2
ar
Then using Gauss's theorem in differential from
aEx p (x)
so, p (x) = 6a Eo x.
ax go

3.52 In the space between the plates we have the Poisson equation
32
PO
ti)
a x2 E0
or, cp= - P° X2 +Ax+B
2 co
where Po is the constant space charge density between the plates.
We can choose cp (0) = 0 so B = 0

Po d2 cp Po d
Then (d) = A
= Ad 2 E0 or, A -
d + 2 E0
Now E - x - A = 0 for x 0
ax 0
A cp Po d
if d + —
2 co
=
2E0A•zp
then Po = d2

Po d
Also Ef (d)
go
3.53 Field intensity is along radial line and is

Er= -2ar
ar
From the Gauss' theorem,

4 3t T2 Er 'II f

Eo,
where dq is the charge contained between the sphere of radii r and r + dr.

Hence

Differentiating (2) p = - 6 co a
289

3.2 CONDUCTORS AND DIELECTRICS IN AN ELECTRIC FIELD


3.54 When the ball is charged, for the equilibrium
of ball, electric force on it must counter balance the
excess spring force, exerted, on the ball due to
the extension in the spring.
Thus F d = Fs pr
2
Or, The force on the charge
2 = K X, (
4 it co (21)
q might be considered as arised from attraction by
the electrical image)
or,q = 41 Vit eo K X,
sought charge on the sphere.

3.55 By definition, the work of this force done upon an elementry displacement dx (Fig.) is
given by

q2
dA = Fx dx = dx,
4 it eo (202
where the expression for the force is obtained
with the help of the image method. Integrating
this equation over x between 1 and 00, we find
CO

dx
A- q2 f q2
16 it eo x2 - 16 it eo /

3.56 (a) Using the concept of electrical image, it is clear that the magnitude of the force acting
on each charge,
2
q2
111= Nri 4 it eo / 2 4 it eo (Vi 02

q2
Nri - 1)
8 It E0

(b) Also, from the figure, magnitude of


electrical field strength at P
1
E= 2 1- 1,— )
5 V5 TC co /-
3.57 Using the concept of electrical image, it is easily
seen that the force on the charge q is,

(_ 02
F-
4 it eo (202 + 4 it eo (2 V-2- 02

(2 V - 1) q2 „
2 (It is attractive)
32 it eo /
290

3.58 Using the concept of electrical image, force on the dipole g


--a a E , where E is field at the location of
F p
.rdue to (-I")

or, IF 1= al
IP al 327tE0/4

as, 4= p

3
3
4 It to (21)
3.59 To find the surface charge density, we must know the electric field at the point P (Fig.)
which is at a distance r from the point 0 .
Using the image mirror method, the field at P,
q 1 q1
E= 2 E cos a = 2 2
4 it eo x X 2 It go (1 2 + r2 )3/2
Now from Gauss' theorem the surface charge
density on conductor is connected with the
electric field near its surface (in vaccum)
through the relation a - e0 En, where En is the
projection of E onto the outward normal titwith
respect to the conductor).
As our field strength it 4. I17 so
a 216 E 2 7t(1
ql
+ r2)3/2
3.60 (a) The force F1 on unit length of the thread is given by

=E i

E
where 1 is the field at the thread due to image
charge :
-x
E 1= 27(E0 (20
1 k2
Thus
47teo /
minus singn means that the force is one of
attraction.
(b) There is an image thread with charge
density- X behind the conducting plane. We
calculate the electric field on the conductor. It is

E(x)=En (4
it €0 (X2 + 12)

ou considering the thread and its image.


Thus
291

3.61 (a) At 0 ,
00

En (0) 2
=
43TE0x2s. 231E01

So a (0) A. to
En= 2711

00

dy 2
on putting y = x + r2,
4 ?ACE() 3/2
12+r2

A
l 2 + r2
2 n eo

A
Hence a (r) = E0 En=
231 V12 r2

3.62 It can be easily seen that in accordance with the image method, a charge —q must be
located on a similar ring but on the other side of the conducting plane. (Fig.) at the same
perpendicular distance. From the solution of 3.9 net electric field at 0,
ql
E=2 ( -4 where is
4 it eo (R2 4. 12)"
outward normal with respect to the conducting
plane.

Now En.
Eo
— ql
Hence a= /2
2 it + 12)3

where minus sign indicates that the induced


carge is opposite in sign to that of charge q>
O.
3.63 Potential q is the same for all the points of the sphere. Thus we calculate its value at the
centre 0 of the sphere. Thus we can calculate its value at the centre 0 of the sphere,
because only for this point, it can be calculated in the most simple way.

1 /
EP 2° + szp' (1)
4 7t eo /
292

where the first term is the potential of the charge q,


while the second is the potential due to the
charges induced on the surface of the sphere.
But since all induced charges are at the same
distance equal to the radius of the circle from
the point C and the total induced charge is
equal to zero, p' = 0, as well. Thus equation
(1) is reduced to the form,
1 q
cP 4 it co 1
2=

3.64 As the sphere has conducting layers, charge


—q is induced on the inner surface of the sphere
q and consequently charge + q is induced on
the outer layer as the sphere as a whole is
uncharged.

Hence, the potential at 0 is given by,


q (— 4) q
+
cP0 — 4 7t E0 r 4 it E 4 it e R
0 R1 0 2
It should be noticed that the potential can be
found in such a simple way only at 0, since all
the induced charges are at the same distance from
this point, and their distribution, (which is
unknown to us), does not play any role.
3.65 Potential at the inside sphere,

ql q2
(Pa— 4ne0a+4itE0b

b
Obviously = 0 for q2 = — —a ql (1)

When r a b,

ql q2
(1- r, using Eq. (1).
+
(Pr — 4 n eo r 4 n eo r 4 n Eo a
And when r s b
ql q2 ql (1 1
+
CPr - 4 n eo r 4 Tt eo b 4 n eo r a

3.66 (a) As the metallic plates 1 and 4 are isolated and conncted by means of a conductor,
= cp4. Plates 2 and 3 have the same amount of positive and negative charges and due to

induction, plates 1 and 4 are respectively negatively and positively charged and in addition

to it all the four plates are located a small but at equal distance d relative to each
293

other, the magnitude of electric field strength between 1 - 2 and 3 - 4 are both equal in
magnitude and direction (say E ). Let E' be the field strength between the plates 2 and 3,
-4. -4.
which is directed form 2 to 3. Hence E' t 1 E (Fig.).
According to the problem
E' d = Aq) 31 I:1)2 - (1)3 (1)
In addition to
0-
931 - 9)4 = (9)1 - 9)2) + (c1)2 - 9)3) + (9)3 - 9314
or, 0 - - Ed + Ay-Ed

or, .. Ai
AY - 2Ed or E
2d

Hence E . —' . (2)


2 2d
(b) Since E a a, we can state that according to equation (2) for part (a) the charge on
the plqte 2 is divided into two parts; such that 1 / 3 rd of it lies on the upper side and
2 / 3 rd on its lower face.
Thus charge density of upper face of plate 2 or of plate 3a 1 or plate 4 and lower face of
9)
= eo E . AEO

and charge density of lower face of 2 or upper face of 3


2d
' E'
(3 ' Eo ' Eo d4T-

3 Eo A 9)
Hence the net charge density of plate 2 or 3 becomes a + a = which is obvious
2a'
from the argument.
3.67 The problem of point charge between two conducting planes is more easily tackled (if we
want only the total charge induced on the planes) if we replace the point charge by a
unifonnly charged plane sheet.
Let a be the charge density on this sheet and E1, outward electric field on the two E 2

sides of this sheet.


Q
Then Ei + E 2 -=

CO

The conducting planes will be assumed to be


grounded. Then E1 x = (1- x). E 2

a ,, , , r, a
Hence E1- - - ki - X) , E.2 2 -. X

1 E O 1 0

This means that the induced charge density on


the plane conductors are
a a
a1= - 7 (1 - -0, 02 m - T x
294

3.68 Near the conductor E = E,,..


E0
This field can be written as the sum of two parts E1 and E2. E1 is the electric field due to an
infinitesimal area dS.
a
Very near it Ei . ±
2 Eo
43
The remaining part contributes E2 = — on
2 eo
both sides. In calculating the force •on the S
element dS we drop E1 (because it is a
T
self-force.) Thus
02
dF a
dS 2e0= 2E0
3.69 The total force on the hemisphere is
/4/2
2
F= i a
— cos 0 • 2nRsin0Rd0
0 2 E0
x/2
2 R2 02 f
= cos 0 Sin 0 d 0
2 eo
0
2 2
2 7C 1t2 1
2 £0 x 2 x 4nR2)
LA____) q
32ne0R

3.70 We know that the force acting on the area element dS of a conductor is,
--0 1 --*
d F= — a E dS (1)
2
It follows from smmetry considerations that the
resultant force F is directed along the z-axis, and
hence it can be represented as the sum
(integral) of the projection of elementary forces
(1) onto the z-axis :
dFz= dF cos 0 (2)
For simplicity let us consider an element area dS
= 2 n R sin 0 R d 0(Fig.). Now considering that E
= a/eo. Equation (2) takes the from
2
Tt cr2 R
dFz = sm 0 cos 0 d 0
Eo
2 2
n clo R 3
MI
- COS 0 d cos 0
go
295

Integrating this expression over the half sphere (i.e. with respect to cos 0 between 1 and 0),
2 2
TC On
we obtain F = Fz = R
4 eo
noP
3.71 The total polarization is P = (e - 1) eo E. This must equals where nois the concerntation
of water molecules. Thus
nop
N= = 2.93 x 103 on putting the values
(e 1) e0E
3.72 From the general formula
-10 -11.11, -IP 2
1 3 — r p p
E
4 rc eo r5
1 2ir
E 4 Eco 13 where r= 1 and
This will cause the induction of a dipole moment.
a 1 2p
P P 4 n Eo 13 x e°

Thus the force,


13 2p a 1 2E. 3 13p2
413 al 4 7r Eo 13 4 7c2
7 t

3.73 The electric field E at distance x from the centre of the ring is,
qx
E (x) - 4 7c eo (R2 + x2)3/2
q x
The induced dipole moment is p = 13 Eo E = 4 7c(R2+ X2)3/2
The force on this molecule is
2
8213 x (R — 2x2)
2
16 7E2 eo (R + x2)4
This vanishes for x = (apart from x = 0, x = co)
V2
It is maximum when
a x (R2 - A2 x 2)
4
ax (R2 + x2) -
(R2 2 x2) (R2 + x2 (R24. x2) 8 x2_ (R2_ _ 2 2 = 0
or, ) 4 x2
R2
or, R4 —13 x2 R2 + 10x4 = 0 or, x2 = —20 (13 ± VI191
R ✓
orrx = 13 ± Nifg (on either side), Plot of Fx (x) is as shown in the answersheet.
20-
296

3.74 Inside the ball


--1,. —• --*
a
Dr; ) = —A-- — = EEO E.
4 7t r3
E-1 -41' E— 1 q T*
Also E0E+P. D or P— D=
E E 4 7t r3

f i cir _E-1_q_ r d c 2 ......... E 1


Also, q'= • q
E 4,7( J E

a or, Edid =
175 Ddiel =t EOE did m D conductor 1. a
E E0

E—1
Pis= (E - 1) E0Edid = a
E

— Pal. E-1 a
E

This is the surface density of bound charges.

3.76 From the solution of the previous problem q' in - charge on the interior surface of the
conductor

= — (E — 1)/E fo dS = —e;1q

Since the dielectric as a whole is neutral there must be a total charge equal to
E—1
q' outer = + q on the outer surface of the dielectric.
E
3.77 (a) Positive extraneous charge is distributed uniformly over the internal surface layer. Let
o0 be the surface density of the charge.
Clearly, E = 0, for r < a
For a < r
E0E x 4 7t r2 = 4 it a2 oo by Gauss theorem.
2
GO (a
or, E— a<r<b
eoE r
For r > b, similarly

Go (a)2
E_ r>b
Eo r
19_ E
Now, E_ -

So by integration from infinity where cp (cc) = 0,


00 a2
PC r>b
I` E0 r
297

ajo a2
a <r<b q_ + B, B is a constant
Er

2 2
Go a (1 1) Go a
or by continuity, cp = a<r<b
E0 E r b eob '

For r < a. cp = A = Constant


2 2
Go a (1 1) Go a
By continuity, q=
Eo E a b Eo b
(b) Positive extraneous charge is distributed uniformly over the internal volume of the
dielectric
Let Po = Volume density of the charge in the dielectric, for a < r < b.

E= 0, r< a

4n 3 3
Eo E 4 n r2 E = — (r - a ) po ,(a< r<b)
3

Po a3
Or, E= r- --1-)
3 Eo E r

4 3
E= 1 (b - a3) po / eo 4 n r2, r > b
3

(b3 - a3) Po
Or, E= for r > b
3 go r2
By integration,

Or,

By continuity
b3 — a3 Po b2 a3
o= - +
3 Eo bP B 3 Eo E 2 b

PO f E (b3 - a3) (b2 a3


or, B -
b + + —.)
3 Eo e 1 2 b
2
Po (a2 Po a
Finally B- +a2 = B - 2 e Eo , r < a
3 eo e
(13 2
On the basis of obtained expressions E (r) and (fp) (r) can be plotted as shown in the
answer-sheet.
298

3.78 Let the field in the dielectric be E making an angle a with iii..Then we have the boundary
conditions,
Eo cos czo = EE cos a and Eo sin co = E sin a

So E = EoVsin2 ao + cost ao and tan a = E tan ao

In the dielectric the normal component of the


induction vector is
D„= Et) E E„= E0 EE cos a = co Eo cos ao

a' = Pn= D,,— e0E,, = 1-1eo Eo cos ao

E—1
Eo E os ao
or, = 0c

e —6 1
3.79 From the previous problem,a' = eo E0 cos 0

1> ..,,Lefigilz1,

MUD

(a) Theni = 1Q = n R2 Eo cos 0 1


60

(b) (D1t- D2j) 1= (e0E0 sin 0 — E eo E0 sin 0) = — (e — 1) eo E0 / sin 0

aD
3.80 (a) divD = x — p and D = p 1
ax
Ex= - , / < d and Ex= pd constant for />d
EEO
60
2
p Id (d _ ,
p(x)= — — , 1 < d and cp (x) = A — , I> d then p (x) = -d
2Pe eo o o a —1 )
by continuity.

On the basis of obtained expressions Ex (x) and cp (x) can be plotted as shown in the figure of
answersheet.
299

(b) p' = — div P= — div (e — 1) E01= —p .e -.11


E
a' ' Pin — P2n, where n is the normal from 1 to 2.
= Pm, (P2= v
0 as 2 is vacuum.)

E —1
= (pd — p d/e) = .p d E

--a. la _ p
3.81 divD = ;2- —ar r2 Dr =
1 A
2
D= Dr= - pr+-2
r r<R
r r p -?:3-3-+A

3 pr
A = 0 as Dr* co at r = 0 ,Thus, Er.. 3 E eo

B
For
2

By continuity of Dr r= R ; r>R, Dr= rr


at B — 213-
3
so, EE - --PL?3-- r>R r
*" tO r
, _ _.2 '

nr2
pR3
14) =
3Eol.'
r>R and q) = — —r=--
6 E Eo
++ CC r<R r<R
,

R2 n R2
C = + 2.:"-. '' + -r-1=-- , by continuity of q.
3Eo 6EEo
See answer sheet for graphs of E (r) and cp (r)

(b) p' = div r) . _ 1


re aar
{3
r3
p (1 — -14 1 = — p(e-
8
1)

Cif = Plr — P2r ' Plr I" 3pR 1—

3.82 Because there is a discontinuity in polarization at the boundary of the dielectric disc, a
bound surface charge appears, which is the source of the electric field inside and outside
the disc.
We have for the electric field at the origin.

E= -f a'
i dS F.3 '

4 It E0 r
where F.= radius vector to the origin from the element dS.
300

cr' = Pn = P cos 0 on the curved surface


(P„= 0 on the flat surface.)

Here 0 - angle between rand P

By symmetry, E will be parallel to P. Thus


Dt

f P cos 0 R dO-cos 0
E It - d
4 n e0 R2
0
where, 7... R if d «R.

So, E. -
Pd
4 e0 R
and r- _ 4.-1:7de0R
3.83. Since there are no free extraneous charges anywhere
--O. 19D

divD .. —21= 0 or, Dx = Constant


ax

But Dx - 0 at 00, so, Dx = 0, every where.

:70 ( _Po (1 - L2)


Thus,
E_ - —1 1 - or, Ex.. d2
Eo d €o
PO X PO X3
So, cP ma - 2 + constant
Eo 3 e0d
Hence,
2 Pod 2 Pod3 4 Pod

3.84 (a) We have D1: D2, or, e E2 .. E1


d
Also, E12 + E2-f = E0d or, Ei + Ef. 2 E0

2 E0 2 e E0 2 e e0E0
Hence, E - — and El - and DI . D2 as
2 e+1 e+1 e+ 1

(b) D1= D2, or, e E2 as E1= —aam Eo


CO
E0
Thus, E1 am E0 , E2 = — and D1= D2 ma e0
E0 e
301

3.85 (a) Constant voltage acros the plates;


El= E2= E0, D1= E0 E0, D2= E0 E Eo.
(b) Constant charge across the plates;
El= E2 , D1 m• Eo Et, D2 i• e eo E2 = e Di

2 E0
E1(1 + E) .= 2E0 or E1= E2 'NI
E+1
3.86 At the interface of the dielectric and vacuum,
E1, ' E2t
The electric field must be radial and
A
El= E2= 2, a<r<b
Er
E0

A 2 A
Now, q = --1- (2 t R ) + — 2 7t R2)
2(
R ER
se A(1+1)2TE
E

q
or, El= E2 =
2 it e0 r2 (1 +E)

3.87 In air the forces are as shown. In K—oil,

F —' F' = F/E and mg —1. mg (1 — P ).


P
Since the inclinations do not change
1 Po
-.". al 1 —
E p

Po , 1 E—1
Oor,r, — 1 — 3r

P E £

E
my:
or, 1) = Po E _ 1

where p0 is the density of K—oil and p that of the material of which the balls are made.

3.88 Within the ball the electric field can be resolved into normal and tangential components.
En= E cos 0,Et= E sin 0

Then, D.= E E0 E cos 0

and P.= (E — 1) eoE cos 0

or, a' = (E — 1) E0E cos 0

so, cr. = (E — 1) E0 E,
and total charge of one sign,

302

1
q' = f (E— 1) EoE cos 0 2 jr R2 d (cos 0) = It R 2 eo (e — 1) E
0
(Since we are interested in the total charge of one sign we must intergrate co§ 0 from 0
to 1 only).
3.89 The charge is at A in the medium 1 and has an image point at A' in the medium 2. The
electric field in the medium 1 is due to the actual charge q at A and the image charge
q' at A'. The electric field in 2 is due to a corrected charge q' at A. Thus on the boundary
between 1 and 2,

El = cos 0 — 2 cos 0
-n 4nE0r2 4 7C E0 r
II

E cos 0
4 7t eo r2
q, •
E11 — sin 0 + sin 0
4 n e0 r2 4 7E E0

E2t q 111 0
4nE0r2
The boundary conditions are
= D2,, and El, = E2!

( 1 7

q ql

So, qI is 2q a E — 1

q
e+1 E + 1

(a) The surface density of the bound charge on the surface of the dielectric
a'= P2, = D2,, — eo E2,, (e — 1). eo E2,,

e—1 q e — 1 ql
cs0=
4. 1 2 r` 6 + 1 2 it r3
CO

1
(b) Total bound charge is,— q 2 nxdx— — q
E+ 1 2 n (1 2 + x2)3l2 e+1
0

3.90 The force on the point charge q is due to the bound charges. This can be calculated from
the field at this charge after extracting out the self field. This image field is
e—1
iMt
image
E + 1 4 n eo (242
2

Thus, F E
— 1

E + 1 16 IC E0 / 2
303

Thus,
2nEo(1+e)r2

=
27tE0(1+E)r

q { 1 in vacuum
D= x
2 7r Eo (1 + E) r2 e in dielectric

Using the boundary conditions,

E2t
Ein
This implies
1
q— E q' q" and q + E q' eq"

n 24 E —1
So, E+1 q'
E+1 E

Then, as earlier;

Q,- 1) 1
2 nr3 E+1 E

3.93 To calculate the electric field, first we note that an image charge will be needed to ensure
that the electric field on the metal boundary is normal to the surface.
304 -▪

The image charge must have magnitude -

so that the tangential component of the electric


field may vanish. Now,
1 (_q__) ql
E n= 2 cos 0 =
4 E0 E r2 2 n Eo r3

(E - 1) ql
Then P, = Dn- eo En = -
2 E r3
This is the density of bound charge on the
surface.

3.94 Since the condenser plates are connected,


Ei h + E2 (d - h) = 0
E1 =
and P Eo EO E2
2
r,
or, E, +P 1/ 2

E0

, n Ph
at U, Or, E2
PhPh
Thus, E2 a -— Eo d
P (,
- I

Ei 0

-
3.95 Given P = a r, • where r = distance from the axis. The space density of charges is given
by, p' = - div P= - 2a
a
On using. div 7*. =2
r ar
3.96 In a uniformly charged sphere,

Po r -,4 Po
E = 3 E0 or, = r
3E0
The total electric field is

1
E= 1 p r -— (r 6 Po
3E ° 3Eo
o 1 P
P
3Eo
-IP
= 3Eo
where p 6 r - = P (dipole moment is defined
with its direction being from the -ye charge
to +ve charge.)
The potential outside is
--i■

1 Po • r
- = 3 r>R
CI) 4 n Eo r I 4 it eo r
-• zin 3
where pc, - —3 R po ar is the total dipole moment.
305
3.97 The electric field E0 in a spherical cavity in a uniform dielectric of permittivity e is related
to the far away field E, in the following manner. Imagine the cavity to be filled Jr, with
the dielectric. Then there will be a uniform field E everywhere and a polarization P, given

by,
--IP

P (E - 1) E0E
Now take out the sphere making the cavity,
the_Vectric field inside the sphere will be

3E0
p
By superposition. E0 - E

or, E0 = E + (e-1)1= (e + 2)

3.98 By superpositionthe field E inside the baj is given by


P
E = En-
3eo
On the other hand, if the sphere is not too small, the macroscopic equation P =
(E - 1) Eo imust hold. Thus,

E-1
Also P. 3 Eo e 2 Lso

3.99 This is to be handled by the same trick as in 3.96. We have effectively a two dimensional
situation. For a uniform cylinder full of charge with charge density p0 (charge per unit
volume), the electric field E at an inside point is along the (cylindrical) radius vector F*
and equal to,
1 -4.
E= — p r
=
2E0

(div E= 1 (rEr) 2- hence, Er= --P-r)


r eo 2E0
Therefore the polarized cylinder can be thought of as two equal and opposite charge dis-
tributions displaced with respect to each other
1 -I. 1 ---k 1 P
E= — pk r - or).
‘-() 2e0 2Eo 2€0
Since P = - p 6 i..(direction of electric dipole moment vector being from the negative
charge to positive charge .)
p
3.100 As in 3.98, we write E = En-

using here the result of the for oing problem.


Also P. (e - 1) E0E

2En e-1
So, 10 , or, E- and 2e0 E0
+1 e+1
306

3.3 ELECTRIC CAPACITANCE ENERGY OF AN ELECTRIC FIELD

3.101 Let us mentally impart a charge q on the conductor, then


R2 00

+-
(19_ = f
R1
q
431E er2dr +
0
R2
4 eo r2 dr

q
r 1 1 1 q 1
4neoe R1 R2 4neoR2

(e — 1) 1
4 n eo E R2 R1

Hence the sought capacitance,


q4neoe 4iteoeRi
C— q
q). — (e — 1) 4. 1
(e-1)-. 7+1
R2 RI

3.102 From the symmetry of the problem, the voltage across each capacitor, eq) = /2 and
charge on each capacitor q = C 2 in the absence of dielectric.
Now when the dielectric is filled up in one of the capacitors, the equivalent capacitance of the
system,
Ce
Ci 0 =
1+e
and the potential difference across the capacitor, which is filled with dielectric,
Cc
Acp' =
eC (1 + e) CE (1 + e)
But cp a E

So, as p decreases (1 + e) times, the field strength also decreases by the same factor

and flow of charge,Aq = q' — q

Ce C 1 (e — 1)
= — =
(1 + e) 2 2C (e + 1)

3.103 (a) As it is series combination of two capacitors,


1 d1 d2 Eo S
S+ or
/e + C=
C e0 el E0 E2 S (dl l) ( d2/e2)
(b) Let, a be the initial surface charge density,
then density of bound charge on the boundary
plane.
a _ 1 ) a 1 ) a (1 —1 )
a' = 1—
el e2 E2 E1
307

1 CV go S el e2 V
But, a as as
S S £2 di + el d2 S

eo V (ei — e2)
So, a'
e2 dl + el d2

3.104 (a) We point the x-axis lowards right and place the origin on the left hand side plate.
The left plate is assumed to be positively charged.
Since e varies linearly, we can write,
e(x) = a + bx
where a and b can be determined from the boundary condition. We have
e=el at x = 0 and e = E2 at x = d,

(E2— ei
Thus, e(x)— ei + x
d
Now potential difference between the plates

-f 0 E0 (El +
a
e2-ei
d x
dr -
ad
(E2 — el) eo
In
£2

ei

aS (E2 — El) eo S
Hence, the sought capacitance,C =
cp+ — cp_ — (In e2/ei ) d

(b) D = and P = -q---


S S Se (X)
and the space density of bound charges is

p' = — divP = -q( E2 El


sd£2 ( X )

3.105 Let, us mentally impart a charge q to the conductor. Now potential difference between the
plates,
R
.d;-*
2

R fr
1
q 1
4 n eo air r2 dr - --9--- 1nR2/R 1

=f Rt
- 4 it eo a

Hence, the sought capacitance,


— q 4 n so a 4 at eo a
C= q —
cp+ - T.. q In R2/R1 In R2/Ri
3.106 Let k be the linear charge density then,

Ei,„= (1)
2 It €0 R1 el

and, (2)
E2'" am 2 It e
0 R2 E2
The breakdown in either case will occur at the smaller value of r for a simultaneous
breakdown of both dielectrics.

From (1) and (2)

ei = E2,„ R2 e2, which is the sought relationship.

3.107 Let, k be the linear charge density then, the sought potential difference,

R2 R3

(14 - (P- f dr+ f dr


eo et r 2n e 0 e2 r
1 R2

1
— In R —1
2 7t e0 El 2/R1 + In R3/R2
€2
Now, as, Ei Ri e < E2 R2 E2, so

"' Et RI Et
2t €0

is the maximum acceptable value, and for values greater than E1 R1 el, dielectric breakdown will

take place,

Hence, the maximum pl,ttential difference between the p ates,

1 1 et
cp, - cp_ = El Ri ei { — In + — In R3/R2 1 = Ei Ri In R2/R1 + — In R3/R2
El E2 62

31 0 Let us suppose that linear charge density of

the wires be k then, the potential difference,

fp+ - fp_ = p - (- fp) = 2 cp. lite intensity of the


electric field created by one of the wires at a

distance x from its axis can be easily found

with the help of the Gauss's theorem,

27tEex
b-a
k b - a
Then, cp f Edr i. In
2nce, a
a
`Hence, capacitance, per unit length,

2.7t €0

In b/a
309

3.109 The field in the region between the conducting plane and the wire can I* obtained by
using an oppositely charged wire as an image on the other side.
Then the potential difference between the wire
and the plane,

f E

+
2 7t co r 2 7t co (2b - r)dr
a

,
2 7t co In - m
a 2 it co 2b - a
•:'
- I 2b - a
2 Jt co n a
2b
— as b » a
In
• 2 eo a '
Hence, the sought mutual capacitance of the system per unit length of the wire
k 2 n eo
Ay In 2b/ a
3.110 When b » a, the charge distribution on each
spherical conductor is practically unaffected by
the presence of the other conductor. Then, the
potential ( ) on the positive (respectively
negative) charged conductor is

4neoea zliteoca

Thus cp. -
27cccoa

and C - 211c ca.


b
a
Note : if we require terms which depend on —
b' we have to take account of distribution
of charge on the conductors.
3.111 As in 3.109 we apply the method of image.
Then the potentical difference between the
+vely charged sphere and the conducting plane is
one half the nominal potential difference
between the sphere and its image and is

A (P (1)- )21 —47tgc-o-a


Thus

C= - 4ncoa. for /» a.
A cp
310

3.112

Cr
11
C2
Ci C2 C317
1
C3
(a) Since cpi = cps and cp2 = cpA
The arrangement of capacitors shown in the problem is equivalent to the arrangement
shown in the Fig.

and hence the capacitance between A and B is,


C= C1+ C2 + C3
(B) From the symmetry of the problem, there is no P.d. between D and E.. So, the
combination reduces to a simple arrangement shown in the Fig and hence the net capacitance,
C C
C = —+ -- C
° 2 2
3.113 (a) In the given arrangement, we have three

605
capacitors of equal capacitance C— and
d
the first and third plates are at the same C
potential.

Hence, we can resolve the network into a simple


form using series and parallel grouping of
capacitors, as shown in the figure.
Thus the equivalent capacitance
(C + C) C 2
— — C
C° (C + C)+ C 3
311

(b) Let us mentally impart the charges +q and -q to the plates 1 and 2 and then distribute
them to other plates using charge conservation and electric induction. (Fig.).
As the potential difference between the plates 1 and 2 is zero,

EoS
d

or, q2= 2417


)
The potential difference between A and B,
'49= TA - TB = q2/C
Hence the sought capacitane,
g_ . qi + q2 3q1 3 3 eo S
L' 0 - IS = C..
cp q2/C 2q1/C 2 2d
3.114 Amount of charge, that the capacitor of capacitance C1 can withstand, q1 = C1 V1 and
similarly the charge, that the capacitor of capacitance C2 can withstand, q2 = C2 V2. But
in series combination, charge on both the capacitors will be same, so, qmax, that the combination
can withstand = C1 V1,
as C1 V1 < C2 V2, from the numerical data, given.
Now, net capacitance of the system,
CI C2
- +
C0 C1 C2

qma C
and hence, V = x - - V1 (1+-C 1)= 9 kV
' Ci C2/Ci + C2 C2
Co
3.115 Let us distribute the charges, as shown in the figure.
Now, we know that in a closed circuit, - Ay = 0
-So, in the loop, DCFED,
41 42 42 ( 1 1
— -C, -C = s or, q1 = Ci g + q2 + —) (1)
Cis Ci C2 Ci C2
Again in the loop DGHED,

q1 ql 4. q2
„.., r.„ g (2)
....1 4/1-2
312

— 172 1
Now, — TB "I CZ c22/ci 1 3
C1
2
CI C2 + C2

Or, CPA — CPB a = 10


C22 3 C2 y12 + 3 yi + 1
+ +i
C2 C
1 1

3.116 The infinite circuit, may be reduced to the circuit, shown in the Fig. where, Co is the net
capacitance of the combination.
1 1 1 C
So, +_a
C + Co C Co
Solving the quadratic,
CC o+ co2 - c 2in 0, . - - - - -I I cT TI o
I
C
we get,
( 4 — I.)
Co = C, taking only +ve value as Co can not be negative.
2

3.117 Let, us make the charge distribution, as shown in the figure.

Now, (PA cPa q


-c"7" 2
e2
g r., r.., +/ ILA_
(TA — cPB)
or, q= A *—±17=t—i1&
C +
1 C2
Hence, voltage across the capacitor C1

= (TA (1)/3) c
C1 C1 + C2 2 = 10V

and voltage across the capacitor, C2

3.118 Let In > 11, then using — xp = 0 in the closed


circuit, (Fig.)
rs. + g - s- 0
2 C2 1

(g2 gl) Cl C2
or,
qua (Ci + C2)
D -sH q
C2
313
Hence the P.D. accross the left and right plates of
capacitors,
_ q _ & U C2
(P1 Cl el + e2
and similarly
-q (g1-g2)C1
(P2- C2 Ci + C2

3.119 Taking benefit of tire foregoing problem, the amount of charge on each capacitor
I - il Ci C2
I ql ' Ci + C2
3.120 Make the charge distribution, as shown in the figure. In the circuit, 12561.
-A9= 0 yields
41 + 41 - = 0 or, C3 C4 - 'VI+ 612 A -4 C2-42 3
q1- e3 + e4 4 • 21}
C4 C3
and in the circuit 13461, B
q2 q2 Cl C2 5 -cisI itqi
+ e2 -11+41 •
/.., + (_, - g = 0 or, q2 -
4,..2

Now

=
%,-,1
q2 q1
(PA - q)B 1" Zli: - c13

[ C2 c+4 I -c4
Ci + C2 C3
Cl
6

[ CC -CC2 32 1 4
(Cl + C ) (C3 + C4)
C3 a

16
It becomes zero, when
(C2 C3 - C1 C4) = 0. or Cl_
C2 = C4
3.121 Let, the charge q flows through the connecting wires, then at the state of equilibrium,
charge distribution will be as shown in the Fig. In the closed circuit 12341, using
- Aq9= 0
(Ci V- q) a a
- Cl + -A-- + =I-- - 0 14-s
C2 C3
+(C v-4) c2
V - 0 06 mC CI7(cv,i)
or q - (1/C1 + 1/C2 + 1/C3)
3.122 Initially, charge on the capacitor C1 or C2,
Ci C2
q - C1+ C2 , as they are in series combination (Fig.-a)

I
314

when the switch is closed, in the circuit CDEFC from - Ay = 0, /(Fig. b )


q2
s - = u0 or q2 = C2
c2

And in the closed loop BCFAB from - Ay = 0


-q1 q2
0
C1 F2

Swi
C

IIIC2 qi

1
T' F 2
-14
A
cad (b)

From (1) and (2) q1= 0


Now, charge flown through section 1 = (q1 + q2) - 0 = C2
CI C2
and charge flown through section 2 = - -q c +c,2
1

3.123 When the switch is open, (Fig-a)


2 C'i.C2
go - el e2
7 1

Cl

3 -go-qo 2 3
Sv4

TC
2 +g 2

and when the switch


q1=
C and is
1
o
q2closed,
=
C2
Hence, the flow of charge, due to the shortening of switch,
ugh the section 2 = - q2 - (q0) =
t
through section 1 = q1- qo = C1 h
r and through the section 3 = q2 -
o
C - C_I 2
2
CI + C2 4

p
C2 [ ,

CI - C2
=
-
3
6
I
X
C
I
+
C
2

0= C2 - C1) = - 60 [tC
(q2 - q1) - (
315

3.124 First of all, make the charge distribution, as shown in the figure.
In the loop 12341, using — Aq = 0

q1
-, + i q2 - 0 (1) Z
1441 I / -(1211+42.
6
z--1 1 q c 3
Similarly, in the loop 61456, using — Acp = 0
q2 q2 - q1
+ (2)
C2 C3 - = 0
From Eqs. (1) and (2) we have
5
2 C2- C1

q2 - q1 = C2 C1
+ —+1
C3 C3

q2 - q1 42 C2 - 1C 1
Hence, CPA - CPB = c3
NB
Ci + C2 + C3

3.125 In the loop ABDEA, using — AT = 0 -0,1+q2)


_ + ql+ql+q2+ . 0
1 ilqi-H2 D
3 C3 C1 1
(1) E.—if
&I CI
Similarly in the loop ODEF, 0
ql q2

-1. 1-i2+-er = 0
112
(2) F II C
C1 2 2 q21 qz
Solving Eqs. (1) and (2), we get, bz 1-
2
C2 - C2 - 1 C3 + 3C3
q1 4. q2 -
C3 C2
— +— +1 A il l
h:iT
h 5
Ci Ci
3
C3
(q1 + q2)
Now, (Pi - TO 2: (1)1 = - as ((o = 0)
Cl

i (c2 + c3) - c2 - c3
CI. + C2 + C3

2 (ci + c3) -1c11- c3


And using the symmetry, CP2 - Ci+C2+C3

(Ci + C2) - Ci - C2
and 1:1D3 -
Ci + C2 + C3

The answers have wrong sign in the book.

3.126 Taking the advantage of symmetry of the problem charge distribution may be made, as
shown in the figure.
In .the loop, 12561, -: Acp = 0
316

q2 q2 ql q1
or 0
C2 C3 C1
6 5 42 4
q1 CI (C3 + C2)
or (1) Cr Cg
q2 C2 (C1 + C3)

Now, capacitance of the network, C3

T2-1,
R1+ R2 q1+ q2
C0
(PA- 'PB q2/C2 + q1/C1

1 qg I -qg 2 qi 111,
(1 + q1/R2)
(2) C2 C
( 1
—+
C2 q2 CI

From Eqs. (1) and (2)


2 C1 C2 + C3 (C1 + C2) Co
C1 + C2 + 2 C3
qi
3.127
(a) Interaction energy of any two point charges q1 and q2 is given by where r
4 n eo r
is the separation between the charges.

+4. + ■• • • • ■•■•
+4.
1
I
I I

Hence, interaction energy of the system,


2 2
—4 q 2
Ua zineoa 4ireo(v2q./—

U —4 +2 q2 r—
b 47c eo a 4 eo (V2 a)

2
2 q2 2 q2 /27 q
2
and U, = 2 — —
4n eo a 4n E0 a 4 jt Eo WY a) 4 TE eo a
317

3.128 As the chain is of infinite length any two charge of same sign will occur symmetrically
to any other charge of opposite sign.
So, interaction energy of each charge with all the others,

U= — -1+ - + up to 00 1 (1)
4nEoa 2 3 4

1
But In (1 + x) = x - — x2 +1 x3 up to 00
2

1 1
and putting x 1 we get In 2 = 1——+— + up to 00 (2)
2 3
From Eqs. (1) and (2),

- 2 q2ln 2
U=
4 n eo a
3.129 Using electrical image method, interaction energy of the charge q with those induced on
the plane.
2
q q2
U— — - 8nEO 1
4 n go (20
3.130 Consider the interaction energy of one of the balls (say 1) and thin spherical shell of the
other. This interaction energy can be written as f dcpq

2
q1 2 p2 r sin 0 d 0 dr
p2 (r) 2 n r sin 0 d 0 dr —

=1 4 n co R

l+r
0
2 co (/ 2 + r2 + 21r COS 0)1/2

ql r
= dr • 2r p2 (r) 2
2 E0 l

-. 4 7t r2 dr p2 (r)
4 ic co /
Hence finally integrating
CO

ql q2
U. = where, q2 = f 4 r2 p2 (r) dr
int 4 It co 1
0
3.131 Charge contained in the capacitor of capacitance C1 is q = C1 q and the energy, stored
in it :

U 9.2 = 1 C QD2
1 1

Now, when the capacitors are connected in parallel, equivalent capacitance of the system, C =
C1 + C2 and hence, energy stored in the system :
318

2 2
C1 (40
U , as charge remains conserved during the process.
-
1 2 (C1+ C2)
So, increment in the energy,
2 r, 2
C1 cp2 ( 1 1 ) - C2 `-' 1 IP
AU= = — 0-03 mJ
2 ( 1+ 2 —Ci ) 2 (Ci + C2)

3.132 The charge on the condensers in position 1 are as shown. Here


q qo q + qo
ru Co — C + Co

and

C Co
Hence,
qo- Co + 2 C

+qo 11--qo ii-KoG.10 —20-*71 ) 1:1,


Co C C C C
—qo +q+clo —(1.-clo +40 +q

2 1 2
C IO
(b)

After the switch is thrown to position 2, the charges change as shown in (Fig-b).
A charge qo has flown in the right loop through the two condensers and a charge q0
through the cell, Because of the symmetry of the problem there is no change in the energy
stored in the condensers. Thus
H (Heat produced) = Energy delivered by the cell
2
CCo
= Aq = q() — Co+ 2C

3.133 Initially, the charge on the right plate of the capacitor, q = C (l;i - 2) and finally, when
switched to the position, 2. charge on the same plate of capacitor ;
- C 1
q

So, Aq= q' = C

Now, from energy conservation,


AU + Heat liberated = A ceo, where AU is the electrical energy.
319

2
C l i-
2
C - ,2)2+ Heat liberated = A q

as only the cell with e.m.f. is responsible for redistribution of the charge. So,

2C Heat liberated
C 1 1 2— 2+ C
1
Hence heat liberated — C2
2
3.134 Self energy of each shell is given by 1-1, where q is the potential of the shell, created
only by the charge q, on it.
Hence, self energy of the shells 1 and 2 are :
2 2
W1. q1 a nd W q2
87ts0 R1 2 = 8 7t eo R2
The interaction energy between the charged shells equals charge q of one shell, multiplied
by the potential q, created by other shell, at the point of location of charge q.
q2 q1 q2
So, 12 W
12 - '1 4ne R0 2 4n E R O 2
Hence, total enegy of the system,
Wi+ W2 + Wi2

1 qiq21
[ qi qz
4 n eo { 2 R1 2 R2 R2
3.135 Electric fields inside and outside the sphere with the help of Gauss theorem :
E, 2 (r s R), E2 A 1 r> R)
( >
4 e oR
q r CO

Sought self energy of the ball


U- W1+ W2

R co

ICO Elc'l2 E0 E 22
4 a r2 dr - —2--(1 +
n2
1)
= 4nr2 dr + f
2 2 , Wi 8 neoR 5
0 R
3q2 ana 2- 1
U=
Hence, 4
iii

1
n E 0 5 R

3.136 (a) By the expression — -


1 4 n r2 dr, for a spherical layer.
E e E2
2 ° E 2 dV = 2e E°
To find the electrostatic energy inside the dielectric layer, we have to lategrate the upper
expression in the limit [a, b]
320

3.137 As the field is conservative total work done by the field force,
1
A fd = tit- Uf= 2 q ((pi - Cp2)

- 1 q2 1 - 1 , q2 1 - 1
2 4 n E0 R1 R2 8 n Eo R1 R2

3.138 Initially, energy of the system,


Cii = W1 + W12 where, W1 is the self energy
and W12 is the mutual energy.
, 1 q2
qq0
So, ui = +
2 4 a eoRi 4 a coRi

and on expansion, energy of the system,

2 4 a eo R2 4 a eo R2

Now, work done by the field force, A equals the decrement in the electrical energy,

q (qo + q/2) ( 1 1
i.e. A = (U, - Uf) -
4 a 60 Ri - R2

Alternate : The work of electric forces is equal to the decrease in electric energy of the
system,
A= Lli - Uf

In order to find the difference UL - Up we note that upon expansion of the shell, the electric field

and hence the energy localized in it, changed only in the hatched spherical layer

consequently (Fig.).
R2

R
1

where E1 and E2 are the field intensities (in the hatched region at a distance r from the

centre of the system) before and after the expansion of the shell. By using Gauss' theorem, we

find

1 g + go 1 go
E - and E2 - 4 a go r2
I- 4 n E0 r2
As a result of integration, we obtain

q (qo + q/2) ( 1 1
A- -
471E0 R1 R2
321

3.139 Energy of the1 charged sphere of radius


r, from the equation
4 2
--A____

1q
q
2 cPq42 n r

8e
U= —

eo r n o
If the radius of the then work done is
dr

shell changes bya r2 Fu dr = - dU =

q2/8n eor2
Thus unit ( r2a)2 a2
,- 7C

sought area 4
force per ,
Fu -

e
4 n r2 (8
7C E 0 rl

r2
x8n
TE

eo
r2
4
2 so

Initially, there will be induced charges of


3.140

magnitude -qand on the inner and outer +q

surface of the spherical layer respectively Hence,


the total electrical energy of the system
is the sum of spherical a and their
a b,

ofself shells,having n mutual


energies radii d
energies including the point charge
U 1 q2 1 (-02 _q q
q.

24 neo 24n 4n eo 4 neo 4neo


+ qq + - qq

+ +
a a b b
b 2

eo
Or ui = ____q___

r__ _I.1
8 eo b a

n
Finally, is at infinityhence,Uf = 0
q

charge
Now, y the agent= incrementin the energy
work = U1- U = —q—2 [1 11

3.141
done b 1 8 n eo a b

(a)Sought alent e work performed against the


work is to th electric field created by
equiv
one plate, and to er plate away. Therefore the
holding at bringthe required work,
rest oth
where
2a create
E- age
nt
=q
E
(x
2-
x1
),
d by
ti one
ys plate
t o at the
fh locatio
e t n of
h other.
ei 2

A
nf
it
ee
ln
ds
0
i
So, A (x2 -
agent = q ,1

x1) L'

eo 2 eos (x2 - x1)


A ext =AU(as field is potential)
Alternate :

2 2 eoS 2 eoS

2
q— X

E0S

2
2
— --q—
2
X — —q-- (x2 —xi)X1)

(b)When voltage is kept const., the forceacingon


each plate of capacitor will depend
on the distance betweentheplates
So, elementary work
a done byagent, in
its displacement over a distance relative to the dx,

other,
But, Fx = -
0(x)

2eo
x2
dA = - Fx cbc

S a(x) and a(x) =


EoV

Hence, A= fdA 1 e° e 2 x1 x2
dx

=5 —s o
v2 S
2x V
2
[
1
1
1
x
-
322

Alternate : From energy Conservation,


Ur— Ui = A eell + A agent
1 coS 2 1 eoS 2
or V — -- V Mg
eoS
— e0S g
V2 + A a ent
"2 x2 2 x1 x
x2 1
( as Aceo = (q1— qi) V = (Cr Ci) V2 )
EoS V2 [ 1 1
So A agent '
2 xi x2

3.142
I
(a) When metal plate of thickness rid is inserted inside the capacitor, capacitance of the
EOS
system becomes Co - d (1 _ 1)
eo S V'
Now, initially, charge on the capacitor, q0 = Co V= d (1 — TO
805
Finally, capacitance of the capacitor, C = d
As the source is disconnected, charge on the plates will remain same during the process.
Now, from energy conservation,
Uf — Ui = A agent (as cell does no work)

q02
1 1 q°2
or,
I c — i Co - A agent
2
1[ 60 S V { 1 (1 — i) 1 c v21-1
Hence 2— 1.5 mJ
A agent — 2 d (1 — TO a— C wi 2 (1 — 1)
(b) Initially, capacitance of the system is given by,
Ce
Cn '1' (this is the capacitance of two capacitors in series)
ri (1 — E) + E

So, charge on the plate, q0 = Co V


Capacitance of the capacitor, after the glass plate has been removed equals C From
energy conservation,
A agent = Uf — Ui

1q2 { 1 1. 1 C V2 E Ti (e — 1) - 0 .8 mj
- i q° C' . — Co 2 [E _ 1 (E — 1) J2

3.143 When the capactior which is immersed in water is connected to a constant voltage source,
it gets charged. Suppose ao is the free charge density on the condenser plates. Because
water is a dielectric, bound charges also appear in it. Let a' be the surface density of
bound charges. (Because of homogeneity of the medium and uniformity of the field when
we ignore edge effects no volume density of bound charges exists.) The electric field due
ao a'
to free charges only —; that due to bound charges is s — and the total electric field is
eo 80

ao
— . Recalling that the sign of bound charges is opposite of the free charges, we have Eeo
323

a ao a'
o = -— or, a' - ( e - 1 ) a0
EEEE )( E0 E0 E
Because of the field that exists due to the free charges (not the total field; the field due to
the bound charges must be excluded for this purpose as they only give rise to self energy
effects), there is a force attracting the bound charges to the near by plates. This force is
2
1 , 0, ( e 1) 00
-C - per unit area.
r e0 2eE0
2
1
The factor - needs an explanation. Normally the force on a test charge is qE in an
2
electric field E. But if the charge itself is produced by the electric filed then the force must
be constructed bit by bit and is

F =j. q(E' )dE'

if

1
This factor of -, is well known. For example the energy of a dipole of moment irin an
-4
electric field E0 is -p • E0 while the energy per unit volume of a linear dielectric in an
1
electric field is - - • E0 where P is the polarization vector (i.e. dipole moment per unit
2
volume). Now the force per unit area manifests ifself as excess pressure of the liquid.
V ao
Noting that -=—
d EE0

E0E(6 - 1)V2
We get Ap -
2d2
Substitution, using E = 81 for water, gives Ap = 7.17 k Pa = 0.07 atm.
3.144 One way of doing this problem will be exactly as in the previous case so let us try an
alternative method based on energy. Suppose the liquid rises by a distance h. Then let us
calculate the extra energy of the liquid as a sum of polarization energy and the ordinary
gravitiational energy. The latter is

1
2 h • pg • Sh = - pgSh2
2
If a is the free charge surface density on the plate, the bound charge density is, from the
previous problem,
E-1
a' - a
E
This is also the volume density of induced dipole moment i.e. Polarization. Then the
energy is, as before
1 -1 ,a
- a,E0 = 2 ac " (e - 1) (32
2 o 2cog
324

and the total polarization energy is

(e — 1) 02
-S(a+h)
2e0e
4 1 ". " •• ■•, •

The actual height to which the liquid rises is determined from the formula
dU
.U' (h).0
dh
E — 1 )Cr2
This gives
h •
2e0 e pg
2
3.145 We know that energy of a capacitor,U =
2C

aU
Hence, from F = — we have, Fx C2 (1)
x ax q Const.

Now, since d «R, the capacitance of the given capacitor can be calculated by the formula
of a parallel plate capacitor. Therefore, if the dielectric is introduced upto a depth x and
the length of the capacitor is 1, we have,
2 7t eo eRx R eo (/ — x)
C— (2)

From (1) and (2), we get,


2
Fz = eo(e — 1) R dV
3.146 When the capacitor is kept at a constant potential difference V, the work performed by
the moment of electrostatic forces between the plates when the inner moveable plate is
rotated by an angle dp equals the increase in the potential energy of the system. This
comes about because when charges are made, charges flow from the battery to keep the
potential constant and the amount of the work done by these charges is twice in magnitude
and opposite in sign to the change in the energy of the capacitor Thus

au ir,2 ac
NZ
V

ay 2 atp
Now the capacitor can be thought of as made up two parts (with and without the dielectric)
in parallel.
eoR2cp eoe (It — )R2
Thus C +
2d 2d
1cp.
2 Differentiation then gives
as the area of a sector of angle pis —R
2
E0R2v2
Nz — —
4d
The negative sign of Nz indicates that the moment of the
force is acting clockwise (i.e. trying to suck in the. dielectric).
325

3.4 ELECTRIC CURRENT

3.147 The convection current is

I (1)
dt
here, dq = k dr, where A, is the linear charge density.
But, from the Gauss' theorem, electric field at the surface of the cylinder,

E—
2 n co a
Hence, substituting the value of A. and subsequently of dq in Eqs. (1), we get
2E n co adx
I=
dt
dx
= 2 n coE av , as , v
dt
3.148 Since d << r, the capacitance of the given capacitor can be calculated using the formula
for a parallel plate capacitor. Therefore if the water (permittivity e) is introduced up to a
height x and the capacitor is of length 1, we have,

Ec .2 n rx eo (1— x) 2 nr E02 nr
C— ° (E x + I — x)
d d = d
Hence charge on the plate at that instant, q = CV
Again we know that the electric current intensity,
I d (CV)
. a—
dt dt
VE0 2nr d(Ex+i_ V2nrc0 4.1
' (E — 1)
dt dt

But, =V
dt

2nrE0(E-1)V
So, /— d v — 0.11 [t A

3.149 We have, Rt = R0 (1 + at),(1)


where R, and R0 are resistances at t° C and 0° C respectively and a is the mean temperature
coefficient of resistance.
So, R1= R0 (1 + al t) and R2= R0 (1 + a2 0
(a) In case of series combination, R = LR.
SO R = Ri+ R2 = R0 [(1 + 11) + (a1 + 11 a2) t] (1)

=R0(1+T)) l+2a1+71a2t1
1+
326

Comparing Eqs. (1) and (2), we conclude that temperature co-efficient of resistance of the
circuit, a
l +1 a2
a -
1 + rl
(b) In parallel combination
R0 (1 + al t) R0 (1
R- + a2 t) - R' (1 + t), where R' 1Ro
R0(1 + ai t) + Ro (1 + a2 1+

Now, neglecting the terms, proportional to the product of temperature coefficients, as being
very small, we get,
, 1 ai a2
Ct
1+
3.150 (a) The currents are as shown. From Ohm's law applied between 1 and 7 via 1487 (say)

173 6 1i-13

(b)
I 5
IRS== —R + + = — RI
6 3 6
5R
Thus, Rte=
6
(b) Between 1 and 2 from the loop 14321,
11 R = 2 R + /3 R or, /1 = /3 + 2/2
From the loop 48734,
(/2 - /3) R + 2 (12 - /3) R + (12- 13) R 13 R.

4 I
or, 4 (12 /3) = /3 or 3= 12

SO
327

7
Or R = — R, ii/2
eq 12
(c) Between 1 and 3
10'21'2
From the loop 15621
/1 /1 2
/2 R = IiR + -y or, I2= 32

Then, Vi + 2/2) Reg= 4 /1 Req

= 12R + 12R = 311R

3 12+212 (C)
Hence, Req = —
4
3.151 Total resistance of the circuit will be independent of the number of cells,

2R A

R < > Rx

(R., + 2R) R
if
Rx = Rx + 2R + R

or, Rx2 + 2R Rx - 2 R 2 = 0
On solving and rejecting the negative root of the quadratic equation, we have,
Rx = R (VT - 1)
3.152 Let Ro be the resistance of the network,

A
A•

13 • RI
R2

1 R0 <—> /?0
B

Ro R2
then, Ro = or R2-R0RI-RIR2 = 0
R
0 + R2
On solving we get,
328

3.153 Suppose that the voltage V is applied between the points A and B then
V= IR = I0R0,
where R is resistance of whole the grid, I, the current through the grid and 4, the current
through the segment AB. Now from symmetry, //4 is the part of the current, flowing
through all the four wire segments, meeting at the point A and similarly the amount of
current flowing through the wires, meeting at B is also //4. Thus a current //2 flows
through the conductor AB, i.e.

R0
Hence, R= T
3.154 Let us mentally isolate a thin cylindrical layer
of inner and outer radii r and r + dr
respectively. As lines of current at all the points

of this layer are perpendicular to it, such a if-


7 le - ............ - -4— ,
, .-a•o•
layer can be treated as a cylindrical conductor / . •C'N — --1-- 11.
0
of thickness dr and cross-sectional area ;I %\
; li
2 It rl. So, we have,

dr dr I1
I,
III 0
I I
I
i
1
I

t + I
---=---1---- ==--- - ----'\-Jz)
‘I i
dR = P .S -) P It rl
■12--
and integrating between the limits, we get,

R . --P-- in 2/-
Id a
3.155 Let us mentally isolate a thin spherical layer of inner and outer radii r and r + dr. Lines
of current at all the points of the this layer are perpendicular to it and therefore such a
layer can be treated as a spherical conductor of thickness dr and cross sectional area

4 n r2. So
dr
dR = p 2 (1)
4nr
And integrating (1) between the limits [a, b], we get,
P 1 1
R= 4 n[a b

Now, for b —* 00, we have

R --
— P4 n a
p [1 1
3.156 In our system, resistance of the medium R = (1-— —
4n 61'
where p is the resistivity of the medium

The current

4 n la b
329

or,
f dam_ At
Cp[1
4n a bi

in At4nab
or,
Cp(b-a)
Hence, resistivity of the medium,
47t At ab
C (b - a) ln

3.157 Let us mentally impart the charge +q and -q to the balls respectively. The electric field
strength at the surface of a ball will be determined only by its own charge and the charge can
be considered to be uniformly distributed over the surface, because the other ball is at
infinite distance. Magnitude of the field strength is given by,

E=
4 7t eo a2

So, current density j = 1 q and electric current


P 4 n eo a2

I= f iS - q , 4 a2 -
43t eo a- eo
But, potential difference between the balls,

cp+ - Cp_ 2
4 It eo a
Hence, the sought resistance,

- ozp_ 2q/4 n Eo a p

R=
I f i lP E0 2 na
3.158 (a) The potential in the unshaded region beyond the conductor as the potential of the given
charge and its image and has the form

(19 =A 1-1
r1 r2
where r1, r2 are to distances of the point from
the charge and its image. The potential has
been taken to be zero on the conducting plane
and on the ball

cp a. A ( 1 =V
a-2
330

So A as Va. In this calculation the conditions a « 1 is used to ignore the variation of (pi
over the ball.
The electric field at P can be calculated similarly. The charge on the ball is
Q 42tEoVa

Va 2a1V
and Ep „
coo =
1
Then j = —E = 2a1V § normal to the plane.
pr
(b) The total current flowing into the conducting plane is

2a1V
I = f 27txdxj = f 21txdx
2)3/2
0 o P (n2 + 1

(On putting y = x2 + 12 )
00

2 7C alV f dy 47taV
I= p J y3/2 p

V p
Hence
2na

3.159 (a) The wires themselves will be assumed to


be perfect conductors so the resistance is
entirely due to the medium. If the wire is of
length L, the resistance R of the medium is
1
a — because different sections of the wire are
connected in parallel (by the medium) rather
than in series. Thus if R1 is the resistance per
unit length of the wire then R= R1/L. Unit of
R1 is ohm-meter.
The potential at a point P is by symmetry and
superposition
(for / » a)
A T1 A r2
cp si — ln — - In
2 a —2 —a
A T1
am 2 In
r2
a
V A
Then qh = = In 7 (for the potential of 1)
331

I
Or, A — V/In —
a
V
and s`
2 In 1/a In ri/r2
We then calculate the field at a point P which is equidistant from 1 & 2 and at a distance r
from both :

Then E x 2 sin°
21n117 /a (r1 )
VI 1
21n 1/a r2
1 V 1
and J aE p 21n1/a r2
V 1
(b) Near either wire E
21n1/a a
1 V
and J E
p 21n1/a
V 7r
Then I= V= L — J 2 aL
R
Which gives R1 = -e In I/a

3.160 Let us mentally impart the charges +q and —q to the plates of the capacitor.
Then capacitance of the network,
ceof En&
C (1)
9)
Now, electric current,
--3.
i= f j•dS= faEndS as j T T E. (2)
Hence, using (1) in (2), we get,
1.5 ti A
iV a= --
EEO
P Ego
3.161 Let us mentally impart charges +q and —q to the conductors. As the medium is poorly
conducting, the surfaces of the conductors are equipotential and the field configuration is
same as in the absence of the medium.
Let us surround, for example, the positively charged conductor, by a closed surface S, just
containing the conductor,
— (1) asitti
then, R
.rd f a En clS.
f geof ER clS
and C = g--

Ego
So, RC = — p EE0
a
332
3.162 The dielectric ends in a conductor. It is given that on one side (the dielectric side) the
electric displacement D is as shown. Within the conductor, at any point A, there can be
no normal component of electric field. For if there were such a field, a current will flow
towards depositing charge there which in turn will set up countering electric field causing
the normal component to vanish. Then by Gauss theorem, we easily derive

a = Dn= D cos a where a is the surface charge density at A. The


tangential component is determined from
the circulation theorem

fI. ch7 0
It must be continuous across the surface of the
conductor. Thus, inside the conductor there is a
tangential electric field of magnitude, (Conductor)
D sin a
at A .
eoe
This implies a current, by Ohm's law, of
D sin a
.1
eoe P
3.163 The resistance of a layer of the medium, of thickness dx and at a distance x from the first
plate of the capacitor is given by,
1 dx
dR = — (1)
a (x) S
Now, since a varies linearly with the distance from the plate. It may be represented as,
(02 — al)
a = al + x , at a distance x from any one of the plate.
d
From Eq. (1)
1 dx
dR =
(02 — al) S
CI1 + x
d
d
1 dx d 02

01
+
or,

(
R=
I
3.- 02 — al)
d
x S
(a2
— al)
In
al

v S V (a2 — al)
Hence, — 1=— 511A
R a2
d In —
al
3.164 By charge conservation, current j, leaving the medium (1) must enter the medium (2).
Thus
j1 cos al = j2 cos a2
Another relation follows from
Elt = E 21
333

which is a consequence off E d r 0

1 . 1
Thus — ji sin al = —)2 sin a2
ai 02 (2)
tan al tan oi2
Or,
al 02
(1)
tan al al
or,
tan a2 _ J
02
3.165 The electric field in conductor 1 is

x. P11
LI 1 1.

R
P2
and that in 2 is E2 on 2
R
Applying Gauss' theorem to a small cylindrical pill-box at the boundary.

Pi/ P21
c/..5 + --- dS cl*5
R nR Eo
1
Thus, a = E0 (P2- Pi)

and charge at the boundary= e0 (p2 - pi)/

3.166 We have,E1 dl + E2 d2 = V
and by current conservation
1 E 1-
1 E2
151 P2
+V 0
Pi V
Thus, Ei =
PI dl + P2 d2 di d2
P2 V
E2
P1 dl + P2 d2
At the boundary between the two dielectrics,
a= D2 - Di = E0 e2 E2 - eo el Ei

Eo V
2 P2 - el Pl)
P1 di+ P2d2 (E
3.167 By current conservation dx
E (x) = E (x) + d E (x) = dE (x) p
(x) p (x) + d p (x) d p (x) 4
0
This has the solution,
E Etclk I
/ p (x)
E (x) C p (x) = A
334

Hence charge induced in the slice per unit area


/ I
da= E — [{E(x)+dE(x)Hp(x)+dp(x)}-E(x)p(x)]= E — d[E(x)p(x)]
°A °
Thus, d Q = Eo Id [ E (x) p (x) I
Hence total charge induced, is by integration,
E
Q = o /(E2 P2 - El Pl)
3.168 As in the previous problem
E (x) = C p (x) = C (po + pi x)
(Ti - 1) Po
where po + p1 d = 11 po or, Pi - d
d

By integration V= 5 Cp(x)dx = C po d (1 + -- - 1 C 0 d (i + 1)
2 2 •°
o
2V
Thus C-
po d (ri + 1)
Thus volume density of charge present in the medium
dQ
= - E dE (x)/dx
Sthc °
2E0 V (i - 1) po 2E0 V (i) - 1)

3.169 (a) Consider a cylinder of unit length and divide it into shells of radius r and thickness
dr Different sections are in parallel. For a typical section,
1 )._ 2n r dr _ 2 n r3 dr

Integrating,
d
(
R1
1
(a/r2)
nR4
a
S2
R1 2a 27ta
or, R1 = 2 a 2
7c2 , where S= it R
S
(b) Suppose the electric filed inside is Ez = E o ( Z axis is along the axiz of the conductor).
This electric field cannot depend on r in steady conditions when other components of E are
absent, otherwise one violates the circulation theorem
--> --).

f E • dr = 0
The current through a section between radii ( r + dr, r) is
1 3
2nrdr 2 E = 2nr Edr —
cc/r oc

Thus I= E - ItR4E
cc 2cc
0
335

3.170 The formula is,


q= C V0 (1 _ e-t/RC )
_V 1_ e-t/RC
or, V= -(= V or,
(1_ e-t/R
C ° V0
t/RC Vo— V
or,
e- —
Vo V0
Vo
Hence, t = RC In — R C In 10, if V = 0.9 Vo.
Vo — V
Thus t = 0.6 tiS.

3.171 The charge decays according to the foumula


- t/R C
=q0 e q
Here, RC = mean life = Half-life/in 2
So, half life= T = R C In 2
Eeo A pd
But, C= R=
d A
1
Hulce, — 1.4 x 10 3E2•m
p= EEO ln 2
3.172 Suppose q is the charge at time t. Initially q= at t = 0.
Then at time t,

—iR— =0 F

dq
But,i = — dt (- sign because charge decreases)
- iR
So rig +
C dt

dt RC R
t i/R C = t n/R C
or, e
e '

or, q— +A e- "VR
rl

A= Cl; ( i -- - 1, from q= Cat t= 0


t/R
Hence, q= C
11
dq (1— 1) e- t TIM C
Finally,
dt

3.173 Let r = internal resistance of the battery. We shall take the resistance of the ammeter to
be = 0 and that of voltmeter to be G.

Initially,V= ,I—
r G
336

G
So,
V 2° r + G
After the voltmeter is shunted
V r
=
— (Voltmeter)
11 RG
r+
R G

and (Ammeter)
RG r G+
r+
R+G
From (2) and (3) we have
V
(4)
r +G R
From (1) and (4)
G
— = r + G — rir or G
11
Then (1) gives the required reading
V
+1

3.174 Assume the current flow, as shown. Then potentials are as shown. Thus,
-IR1+
p1=cp 1 1 -IR2- 2 eigrIRM

Or, I.
R1 +R2
p1— / Rr 2
And P2= R1+

So, —1 R2
(Pi — (P2 = +
+ R2

= — ( 1 R2 R1)/(R1 + R2) = — 4 V
+ 9-I 7 -61-1R R1 2
2
3.175 Let, us consider the current i, flowing through the circuit, as shown in the figure.
Applying loop rule for the circuit, — A p = 0
—2 iRi + iR2 + iR = 0

or, i (Ri + R211- R)=


g- R1 2 6 R2 3

or, i=
R + Ri + R2
Now, if itP2 =

— + iRi = 0

2 R1
R
or, = and 2R1 = R2 + R + R
R + R + R2i
or, R R1 — R2, which is not possible as R2 > Ri
Thus, CP2 — CP3 -1R2
337
2

Or, g
R + Rl+ R2
mi

So, R.. R2 - R1, which isRthe required resistance.


2
l‘r Na R
given)
3.176 (a) Current, i=
N R N R
= a, as g= aR (

(b) cpA — cps = pz - n i R . - n a R - n a R = 0

3.177 As the capacitor is fully charged, no current


fl ows through it. So, current
6.14
i= (as 2> U
R1+R2

— cps = c
And hence, PA 1- 2 + i R2
1 b-, ,
= 1
t
- 452 + R 1
-
+R2
1 D

R2
11 701 B

3.178 Let us make the current distribution, as shown in the figure.

Current i = g (using loop rule)


RI R2
R +
R1+R2

So, current through the resistor R1,

R2

it - Ri R2 RI + R2
R +
R i + R 2

2
— 1.2 A
RRI+RR2+RiR2

and similary, current through the resistor R2,

g R1 = 1
if = .8 A
0
RiR2 R1+R2 RRI+RiR2+RR2m
R +
R i + R 2

x Ro
R
i x I
3.179 Total resistance= I- RO + xR0
R +
I

x R R 0
I - x

1 R°+ IR + xR0

D
1 - x
xR 1
zg '1 0 [ 1

+ IR + x Ro
338

xR 1( _x+ xR
Then V= V VoRx I {1R +Rox(1-7)}
°1R+xR0 I 1 1 xR0 + 1R )

For R»RolVsi Vo

3.180 Let us connect a load of resistancc K between the points A and B (Fig.)
From the loop rule, A q' = 0, we obtain
iR = 1 — 11 R1 (1)

and iR = — (i ii) R2

Or i (R + R2) = + i1 R2 (2)
Solving Eqs. (1) and (2), we get

+ R 1R2
1R1 2R2
1 == R+ (3)
+R2 Ri+ R2 R + Ro

1 RI g2 R2 R1 R2 R
where R1 + R2 and Ro R1 + R2
Thus one can replace the given arrangement of the cells by a single cell having the
emf and internal resistance Ro.

3.181 Make the current distribution, as shown in the


R2
diagram. 5
Now, in the loop 12341, applying — A fp = 0
iR+iiRi+ 1= 0 (1)
and in the loop 23562, 4
iR— + (i— R2 = 0 (2)
Solving (1) and (2), we obtain current through
the resistance R,
2
i
3
R1- 1 R2)
= +RR2++ RR 1R2 — 0.02 A

and it is directed from left to the right

3.182 At first indicate the currents in the branches using charge conservation (which also includes
the point rule).
In the loops 1 BA 61 and B34AB from the
6t
loop rule, — A cp = 0, we get, respectively

2"4" i1)R2 Ri = 0 (1)


i R3 + 3 — (i — ii) + =0 (2)
On solving Eqs (1) and (2), we obtain
( 31 — 2) R3 + R2 (.1 +
— sto 0.06 A 5
R1 R2+ R2 R3+ R3 R1
— cpB = R a 0.9 V
Thus 1A — it 1
339
3.183 Indicate the currents in all the branches using charge conservation as shown in the figure.
Applying loop rule, - A cp = 0 in the loops 1A781, 1B681 and B456B, respectively, we
get
R2 21 6
k) = (io - i1) R1 • (1) 8 5-
i3R3 + ii R2 - =0 (2) and
(i1 — i3) R — 0
— i3 R3 =
(3) go
Solving Eqs. (1), (2) and (3),
we get the sought current
_ (R2 + R3) + ,3R3
-13) R
(11 (R2+ R3) + R 2R3
3.184 Indicate the currents in all the branches using charge conservation as shown in the figure.
Applying the loop rule (- 6, cp = 0) in the loops
12341 and 15781, we get
- i + i3 Ri - (4 — i3) R2 = 0 (1) 7
R3
and (i1- i3) R2 — 2 + it R3 = 0 (2) 8 --.\./../ ..... "--- A118
I,
Solving Eqs. (1) and (2), we get ii v s,
1
i (R2 + R3) + '2 R2
13
3 R1R2 + R2R3 + R
3R1
Hence, the sought p.d. 67 3
(PA — (PB = — i3 Ri

2 R3 (RI + R2) — i R1 (R2 + R3)


— — -1V
R1 R2 + R2R3 + R3R1

3.185 Let us distribute the currents in the paths as


shown in the figure.
Now, cpi - 4)2 = iRi + it R2
(1) '

and (p1 - cp3 = iRi + (i- i1) R3 (2)


Simplifying Eqs. (1) and (2) we get
R3 ((Pi — cP2) + R2 (T1 — (p3)
i =
R1 R2+ R2R3 + R3 R1 = 0.2 A
R,
3.186 Current is as shown. From Kirchhoff's Second law C 2-i3
_A/WV`
iiki = i2R3 ,
.- R2 --o
iiRi + ( ii - i3 )R2 = V , A ?3 R4 B
i2R3 + ( i3 + i2 ) R4 m V 22 R3) D 12+13
340

Eliminating i2

2
Hence i3 { R
4
( R1
+
R2 )
+ - - - - -
D --( R 3 + R 4 )
/1..3

-11[(Ri+ R2 ) - 1 7: 1 ( R3 + R 4 )
/13

R3 ( RI + R2 ) - R i ( R3 + R 4 )

or,
i3 - R3R4 ( RI + R 2 ) + R iR 2 ( R 3 + R 4 )

On substitution we get i3.. 1.0 A from C to D

3.187 From the symmetry of the problem, current


flow is indicated, as shown in the figure.
Now, TA — q),3 .. ii r+(i—ii)R In the (1)
loop 12561, from — A q) . 0
(i — OR + (i —24) r — iir = 0

(R + r) i
Or, (2)
I11 3r + R
Equivalent resistance between the terminals r r
A and B using (1) and (2),

[R+r (r—R)+Rl
3r+R i r(3R+r)
Ro. (PA—cPB 2g 2g

i i 3r+R

3.1118 Let, at any moment of time, charge on the plates be +q and —q respectively, then voltage

3 6
341

From (1) and (2),


41 2q dq . dt
or, (5)
dt R = 1-- C t _ 21 R

On integrating the expression (5) between suitable limits,


q
t
dq 1 C -C
- f dt or, - —In = t—
t _ 21 R0 g R
`' C

Thus CV= i 1 g (1 _e-2t/Rc)

3.189 (a) As current i is linear function of time, and at t = 0 and At, it equals io and zero
respectively, it may be represented as,

i= io 1-

At At
io At
Thus q= f i dt = 5 io(1 - -
Aoft 2
o o
; .__ 2q
So, io At

i. 2/(1 _ t )
Hence,
At At
The heat generated.
At

3iti t ) 12 R dt = 4q2 R
At

H= f i2 R dt =
o
o
[ At - rt 3

(b) Obviously the current through the coil is given by


t/At

1= to (12)
io At
Then charge q=
f idt = f io 2 -"I dt - In 2
0 0
; _ q In 2
So, ‘ At
And hence, heat generated in the circuit in the time interval t [0, co],
co
142

1.190 The equivalent circuit may be drawn as in the figure.

Resistance of the network = Ro + (R/3)


Let, us assume that e.m.f. of the cell is then
current

i
Ro + (R/3)
Now, thermal power, generated in the circuit

P = i2 R/3 = (R/3)
(R0 + (R/3) )2
P
For P to be maximum, d—dR . 0, which yields
R. 3 Ro
&,R0
3.191 We assume current conservation but not Kirchhoff's second law. Then thermal power
dissipated is R1
2
P(ii) il2R1 ( R2
)
= i12 Ri + R2) - 2i4R2 + i2R2
2
R2 2 R1R2
= [R1 + R21 1- + ] -11
[i R1 R2 4-1 R 1+ R2
The resistances being positive we see that the power dissipated is minimum when
R2
i -i
1 R1+ R2
This corresponds to usual distribution of currents over resistance joined is parallel.

3.192 Let, internal resistance of the cell be r, then


V = - ir (1)
where i is the current in the circuit. We know
that thermal power generated in the battery.
Q = i2r (2)
Putting r from (1) in (2), we obtain,
Q= - = 0.6W
In a battery work is done by electric forces
(whose origin lies in the chemical processes
going on inside the cell). The work so done is
stored and used in the electric circuit outside. Its
magnitude just equals the power used in the
electric circuit. We can say that net power
developed by the electric forces is
P = - IV = - 2 • OW
Minus sign means that this is generated not consumed.
343

3.193 As far as motor is concerned the power delivered is dissipated and can.be represented by
a load, Ro . Thus
V
I.
R + Ro
V2 Ro
and P . 12 Ro. i/
_ (R0 +R)2
This is maximum when Ro = R and the current I
is then
V
I= —2R
The maximum power delivered is
V2
4R
-P
2 v2
The power input is R + Ro and its value when P is maximum is 2R

The efficiency then is 12- = 50%

3.194 If the wire diameter decreases by 6 then by the information given


__
v2
P . Power input = — = heat lost through the surface, H.
R
Now, H c< (1 - 6) like the surface area and
R cc (1 - 8)— 2
V2
So, (1 - 6)2 . A (1 - 6) or, V2 (1 - 6) = constant
Ro
But V «1 +1 so (1 + 1)2 (1 - 6) = Const = 1
Thus 6 = 2 i = 2%
3.195 The equation of heat balance is
V2 T
- k (T - To). C

Put
R R d dt
T - To=
V2 k V2
So, C +k . R or, -,--
2 C CR
d ka V /u/c
Of,

-di (e c) - crif e
y V2
ac
Of, ek" = kR e +A
where A is a constant. Clearly
v2
= 0 at t= 0, so A = - and hence,
kR
7 (i - e - c
, 0 + —kR
V2 l
T- i ")
344

— cpB = cp
3.196 Let, PA
Now, thermal power generated in the resistance Rx,

= if2 R.. _ [ R 2
P Rx,
R + CPR2 Rx R2 +Rx
+
1 R 2 Rx

For P to be independent of Rx,
dP— 0, which yeilds R
dRx

R Ri. R2 .
12 Q
x = R1+R2 =
3.197 Indicate the currents in the circuit as shown in the figure.
Appying loop rule in the closed loop 12561, — Acp = 0 we get
iiR — i'f iRi= 0 (1)
and in the loop 23452,
(i — i1) R2 + 2 — it R. 0 (2)
Solving (1) and (2), we get,
M24-g2Ri 62
i =
1 RR1+R1R2+RR2

So, thermal power, generated in the resistance R,

{ I. 2 12R
P= ill R = R2 + R1 fR2
RR1+R1R2+RR2
dP
For P to be maximum, ---fi — 0, which fields
3
RI R2 1 1-1,
R..
R1+ R2

( 1 R2 + g2 R1)2
Hence,
PMaX — 4 Ri R2 (Ri + R2)

3.198 Let, there are x number of cells, connected in series in each of the n parallel groups

R
Now, for any one of the loop, consisting of x cells
and the resistor R, from loop rule
i
iR + —xr — .x. .. 0
n
N_

So,i — x — using (1)


xr Nr'
R+— R+
n n2
345

Heat generated in the resistor R,


2
2 ( 2 ■ / " 7: )
Q= i R - / R (2)
n R +NR
dQ
and for Q to be maximum, tht = 0, which yields

TF
—=3 n-
R
3.199 When switch 1 is closed, maximum charge accumulated on the capacitor,
qmax — C , (1)
and when switch 2 is closed, at any arbitrary
instant of time,

(Ri +R2) (— = q/C,


dt
because capacitor is discharging.
q t
1 1
or, f - .
qdq -(R1+ R2) C fodt
q„.
Integrating, we get
—t
-t
4, . qmax e (R, +Rd c
In 2— = or, (2)
gmax (Ri + R2) C
Differentiating with respect to time,
-t
1 )
i (t) = e (R1 "2)C (
a
dt = i (R1 + R2) C

C t
-t
or i (t) - e (R1 +R2)C
(Ri + RO C
Negative sign is ignored, as we are not interested in the direction of the current.

(R +R )C
thus, i (t) — e 1 2 (3)
(Ri+ R2)

When the switch (Sw) is at the position 1, the charge (maximum) accumalated on the
capacitor is,
q-C
When the Sw is thrown to position 2, the capacitor starts discharging and as a result the
electric energy stored in the capacitor totally turns into heat energy tho' the resistors R1
and R2 (during a very long interval of time). Thus from the energy conservation, the total heat
liberated tho' the resistors.

H= Ili = fc-,- =121 Ce


346

During the process of discharging of the capacitor, the current tho' the resistors R1 and R2

is the same at all the moments of time, thus


H1 oc R1 and H2 cc R2

H R1
So, H1- R) ( as H + H2)

21 RiC+RiR2 e
Hence =

3.200 When the plate is absent the capacity of the condenser is


Eo S
C=
d
When it is present, the capacity is

S
C' = d(1 —r)) 1-Ti
(a) The energy increment is clearly.

AU— 1 CV2 — —1 C'V2 C 11 V2


2 2 2 (1 —
(b) The charge on the plate is
CV
qi. initially and q1. CV finally.
1 —1-1
CVTI CV2Ti
A charge 1 — has flown through the battery charging it and withdrawing 1— units
of energy from the system into the battery. The energy of2 the capacitor has decreased by
1 CV
just half of this. The remaining half i.e.2 1— must be the work done by the external
agent in withdrawing the plate. This ensures conservation of energy.

3.201 Initially, capacitance of the system = C e.


1 2
So, initial energy of the system : Ul = — (C E) V
2
1 2
and finally, energy of the capacitor : Uf= CV

Hence capacitance energy increment,


it
AU= C V2 (C e) V2 = — CV2 (e — 1) = — 0-5 mJ
2 —2 2
From energy conservation
A U— Amu + Aagen t

(as there is no heat liberation)


But Acell = (C1— C,)1/2 = (C — Ce)V2'
347

= A U— Awn
Hence Anent
1
= —C (1 — e) V2 = mJ
2
3.202 If Co is the initial capacitance of the condenser before water rises in it then
1 eo 21n R
Ul = —2 C0 V2 , where Co

(R is the mean radius and 1 is the length of the capacitor plates.)


Suppose the liquid rises to a height h in it. Then the capacitance of the condenser is

EE0h27GR e (1— h) 2TER E023tR


C— (1 + (e — 1) h)
d d d
and energy of the capacitor and the liquid (including both gravitational and electrosatic
contributions) is
E o 27ER
d (1 + (E — 1)h)V2
2 pg (2 R hd) 1
2
If the capacitor were not connected to a battery this energy would have to be minimized. But
the capacitor is connected to the battery and, in effect, the potential energy of the whole
system has to be minimized. Suppose we increase h by oh. Then the energy of the capacitor
and the liquid increases by
E° 2nR
Oh (E —1) V2 p g (2.7tRd)h)
2d

and that of the cell diminishes by the quantity Aceo which is the product of charge
flown and V
E0 (2itR)
Oh (e — 1 ) V2
d
In equilibrium, the two must balance; so

E0 (E - 1) V2
pgdh —
2d

E0 (E 1)V2
Hence h=
2pgd2

3.203 (a) Let us mentally islolate a thin spherical layer with inner and outer radii
r and r + dr respectively. Lines of current at all the points of this layer are perpendicular to
it and therefore such a layer can be treated as a spherical conductor of thickness dr
and cross sectional area 4 it r2. Now, we know that resistance,
dr - dr
(1)
dR = S (r) 4 n r
Integrating expression (1) between the limits,
348

R
fdR=fP drr2 a
Or,
4n, 4n a b

4 Tt eo e
Capacitance of the network,C =
[1 1
a b

where q is the charge


and q= C
q) at any arbitrary moment]

-
also, — R as capacitor is discharging.
, dt
From Eqs. (2), (3), (4) and (5) we get,
411 [1 1

q-
4 it eo E [ dt p a bi
or
dt
,
41t q PEE°

r
f —amd 1 r dt dt
Integrating q PEe P 8Eo
go
-t
Hence q
e P E°6

(b) From energy conservation heat generated, during the spreading of the charge,
— Uf [because ken
H=
2 2 0]

1 q0 1 1 0= go b—a
= I 4 a Ee a b 8 1t E08 ab

3.204 (a) Let, at any moment of time, charge on the plates be (qo — q) then current through

d (go —
the resistor, i = —
dt ' because the capacitor is discharging.

Or, i= dt
-00-4) --q)
Now, applying loop rule in the circuit,
go —
— —o
C

or,
go R
dt

dq 1
or,
— dt
qo — q RC dt
-▪ 349

At t= 0, q= 0 and at t= i, q= q

q --r
So, Ingo-
qo RC
/
Thus q = qo (1- e -T R C = 0.18 mC
(b) Amount of heat generated = decrement in capacitance energy
2
q0 q0 (1 — e - s/R C)]
1 4f 1 [

2C 2

dq
3.205 Let, at any moment of time, charge flown be q then current i =
dt
Applying loop rule in the circuit, - 0, we get :
( CVo - q) a
SIR- •=1-0
dt C C
or,
dq 1 R
dt
CV0-2q RC

C170 - 2q t
So, In CV0 - 2 R C for Ostst

C Vo i=o8
or, q= 2 1-eR2cj) dt
dq C Vc, 2 - 21/R C Vo 2t/R C
Hence, = = = -
dt 2 RC
Now, heat liberaled, •
00
co
-41
Q= V 2
f i2Rdt= o 1
2R f e RC dt= — CV2
4 °

3.206 In a rotating frame, to first order in co, the main effect is a coriolis force 2 m v x

This unbalanced force will cause electrons to react by setting up a magnetic field B so that
the magnetic force e Tic B balances the coriolis force.
e 2m --•
Thus -
2m
B = co or, B = - —
e
The flux associated with this is

J= Nnr2B= Nnr2 —2m w


350

1
where N= is the number of turns of the ring. If co changes (and there is time fo
2r rn
the electron to rearrange) then B also changes and so does O. An emf will be induced and
a current will flow. This is

I= N r2 -2-n1 co/R

The total charge flowing through the ballastic galvanometer, as the ring is stopped, is

q= N n r2 I -2Ln co/R

e 2Nitr2co lwr
So,
m qR qR

3.207 Let, no be the total number of electorns then, total momentum of electorns,
p= nomevd (1)

no e ne
Now, I= p Sx Vd - v-S xV d= T Vd (2)

Here Sx = Cross sectional area, p = electron charge density, V = volume of sample From
(1) and (2)

p= 11= 0.40 tiNs

3.208 By definition
nevd= j (where vd is the drift velocity, n is number density of electrons.)
1 nel
Then
Vd J

So distance actually travelled


nel < v >
< V>t

(<v> = mean velocity of thermal motion of an electron)

3.209 Let, n be the volume density of electrons, then from I- p ;vd,

I= neSx1 <17*> I =

n e Sx1
So, t= I = 3 vs.

(b) Sum of electric forces

= Knv)er1=Inslepli, where p is resistivity of the material.

= nSlep—= nelpI. IAN


351

3.210 From Gauss theorem field strength at a surface of a cylindrical shape equals,

where k is the linear charge density.


2 it eo r'

(1)

I I
or, 1= k v or, A. - = , using (1)
v
1277

me

I Nrne
Hence E = - 32 V/m
2iteor 2eV

(b) For the point, inside the solid charged cylinder, applying Gauss' theorem,

2nrhEe- nr2h q
2
eo n R 1

q/1 Ar
or, E= , r=
2iteoR` 2iteoR 2

So,from E - - -d-R
th .
'

R
14)2

Or,

Hence,

3.211 Between the plates


49 -a x4/3
or, acp 1/3
= a x -Ix
ax 3

d2cp 4 2/3
—2- in v ax . - p/ eo
ax

4 eo a
- 2/3
or, P- x
9
Let the charge on the electron be - e,
352

1
then - mv2 - e = Const. = 0,
2
as the electron is initially emitted with neligible energy.

2_ts
V2= , v=

4e a
So, j= -pv= ° V
9
(j is measued from the anode to cathode, so the - ve sign.)

3.212 E_ =

So by the definition of the mobility


+ + V V
v u0d,v u0 - d

eV
and j= (n+ uo + n_ ) 7

(The negative ions move towards the anode and the positive ion towards the cathode and
the total current is the sum of the currents due to them.)
On the other hand, in equilibrium = n_

eV
So, n + = n_ = I (uo+ + uo" )d

Id
- 2-3 x 108 cm-3
e V S (u+0 + uo )
3.213 Velocity = mobility x field
Vo
or, v= u 7- sin co t, which is positive for 0 s w t So,

maximum displacement in one direction is


Vo 2 u Vo
x =f u— smoatdt -
I Iw
0
2u Vo
At co = coo x = 1, so, =1
1 co
2
CO
Thus U
2 170

3.214 When the current is saturated, all the ions, produced, reach the plate.

Then, = .!1'111- = 6 x le cm3


n.eV
(Both positive ions and negative ions are counted here)
dn
The equation of balance is, -tii= ni - rn2
353

The first term on the right is the production rate and the second term is the recombination
rate which by the usual statistical arguments is proportional to n2 (= no of positive ions x no.
of -ye ion). In equilibrium,
dn n
— = ll
dt

so, n as Nit 71:.-= 6 x 10


7
CM- 3

3.215 Initially n = no = ViT77


Since we can assume that the long exposure to the ionizer has caused equilibrium to be
set up. Afer the ionizer is switched off,

dt d
21 '' rn 2

1
t —

dn
Or r dt = - — o rt = + constant
2 ' r, n
n
1- 1
But n = no at t = 0, so, rt =
n no
The concentration will decrease by a factor Ti when
1 1 Ti - 1
r t„ =
u no/Ti no no

or, to - T1 -1 = 13 ms
1/7771.-,
3.216 Ions produced will cause charge to decay. Clearly,
evi
V = decrease of charge = ni e A dt = — V 71
11 C d
eo 11
V
or t- . 2 = 4.6 days
nie d
Note, that n1, here, is the number of ion pairs produced.
3.217 If v = number of electrons moving to the anode at distance x, then
7dv e
x o 11 X
- a v or v= v
Assuming saturation, I= e vo e a d
3.218 Since the electrons are produced uniformly through the volume, the total current attaining
saturation is clearly,
d

I= f
0 1)
Thus,
354

3.5 CONSTANT MAGNETIC FIELD. MAGNETICS


3.219 (a) From the Biot - Sayan law,
S 11/4 dl x F7so
4n r3

(R d 0) R
dB— I (as dl
4 R3
From the symmetry
2

110 i
0 110 6.3 T
f dB"if 47c 2R
0

and f dl xR* R dl 714

1144 dl 23tR2ir

Here n is a unit vector perpendicular to the plane containing the current loop (Fig.) and
in the direction of x
2 nR2i
Thus we get B= 4 31 (x2 +R .2)3/2 n

2
3.220 As L AOB = 3 t— , OC or perpendicular distance of any segment from centre equals

R cos —It . Now magnetic induction at 0, due to the right current carrying element AB

µo i
2 sm —
4n 3t n
R cos —

(From Biot— Savart's law, the magnetic field at 0 due to any section such asAB is perpendicular to
the plane of the figure and has the magnitude.)
355

x
x
a 2
f R c o s - sec OdO
1-1. dx Po i n n ROi 1
B - f —° i — cos() = COS u al 2sin;
43t r2 43t 2 l it 2 47t a
R cos— sec 0 Rcos;
x n
M

As there are n number of sides and magnetic induction vectors, due to each side at 0, are
equal in magnitude and direction. So,

go ni n
Bo = 2 sin - • n
4n 7t n
R cos -
n

ni
= — tan - and for n --* 00
2 ir R n

Ro i jtall7t Ro i
Bo- -i- R- .Lt ItIn
= i ii
B
3.221 We know that magnetic induction due to a straight current carrying wire at any point, at
a perpendicular distance from it is given by :
tto
B- — - (sin 01+ sin 02 ),
4 3t r
where r is the perpendicular distance of the wire from the point, considered, and 01 is the

angle between the line, joining the upper point of straight wire to the considered point and

the perpendicular drawn to the wire and 02 that from the lower point of the str aight

wire.

Here,

go i
and B2 = B4 = ( sin 1 + sin 1
4 n 2 2
(d/2) cos 1
2

Hence, the magnitude of total magnetic


induction at 0,

= B
4
B0 1+ B 2 +B3 +B4

cos I sin
4i 2 2
3
as 43t d/2 in1 +cos/
1
s 2

- 4 Ro i
stp - 0.10 mT
d sin
356

3.222 Magnetic induction due to the arc segment at 0,


go i
B. — (2 It — 2 cp)
4 It R
and magnetic induction due to the line segment at
0,
go i o
Blin = [2 sin cp]
e 4 it R cos cp „ 2 -.SS!!
So, total magnetic induction at 0, • s/
B arc + ne=
Bo= Bli 231 Ri [it — + tan p]= 28 it T

3.223 (a) From the Biot-Savart law,


(d1 x
PO idB =
4n r3
So, magnetic field induction due to the segment 1 at 0,

B1= (2 it — q)
4 7C a
also B2 = B4 = 0, as dl T t

and B = PO cp
3 4 it b
Hence, Bo = Bi + B2 + B3 + B4

i{ 2it—cp+1
4tt a b

Poi 3 it
(b) Here, B1= , B = 0,
4 n—aa 2

sm 45°,
B3 -47Cb

go
B4 = 4 nb sm 45°,

and B5 = 0

So, Bo. B1+B2+B3+B4+B5

i 3n ila i Poi
= tto— — — + 0 + -- — — sin 45° + 0
4 it a 2 sin 45°
4xb +47Cb

= 110t.{ 3n
— +—
4 it 2a b
357

3.224 The thin walled tube with a longitudinal slit can be considered equivalent to a full tube
and a strip carrying the same current density in the opposite direction. Inside the tube, the
former does not contribute so the total magnetic field is simply that due to the strip. It is

o (//2 n R) h 1101 h
B P 2
2n r 4n Rr
where r is the distance of the field point from the strip.
3.225 First of all let us find out the direction of vector Bat point 0. For this purpose, we divide
the entire conductor into elementary fragments with current di. It is obvious that the sum
of any two symmetric fragments gives a resultant along 'shown in the figure and con-
sequently, vector B will also be directed as shown

So, I B1= f dB sin fp (1)

R di sin q)

It

f Po . i
= , is in q) d q), as di= —dcp
2 n` R n
0

Hence B = go i/n2 R

3.226 (a) From symmetry


Bo= Bi +B2 + B3
3
-—
ft 110 i n ILO i
311 + It +
4 nR 4R
4T
(b) From symmetry
Bo = Bi +B2 + B3
3
Po i i Po i r 3n

(c) From symmetry


Bo= Bi +B2 + B3

tio i Po
11° 11/4 I- + n)
4ni4-471R " 4n/= 4nR

3.227 K. 4+;32
or, I go I = = 2.0 T, (using 3.221)
4nl�
358

3.228 (a)

4 R

tto i 2
So, 1B Ism
---iTt171 4 so 0'30 IAT
4 R
(b) K-4+4+4

110 i i

= k ) 4-3-171i71 (- tr )

Pv i
- [ k +(n+ 1)
4itR
So,

II3: = —411°it + (It + 1)2 = 0.34µT

(c) Here using the law of parallel resistances


1
+ i2 = i and - =
1:2 3'

i2 4
So,
3

3. 1.
Hence z2 - -4- z, and it -;1-

Thus ro= 'to


4 it R 4nR 4n
2 R
:4I1° (n/2) i2
R
lio i --a.
4 31 R (j+k)+0

i
Thus, µ0
=4 R "1 T
3.229 (a) We apply circulation theorem as shown. The current is vertically upwards in the plane
and the magnetic field is horizontal and parallel to the plane.

110
f
B d ri . 2 B1 = }toil or, R-
2
i
(b) Each plane contributes tinv 2- between
the planes and outside the plane that cancel.
Thus
B 1.10 i between the plane
0 outside.
359

3.230 We assume that the current flows perpendicular X=0


to the plane of the paper, by circulation theorem,
2B dl = go (2 x dl ) j

or, B- xj, I xl s d

Outside, 2 B dl go (241 dl ) j

or, vodj Ixlz d.

3.231 It is easy to convince oneself that both in the regions. 1 and 2, there can only be a circuital
magnetic field (i.e. the component B,). Any radial field in region 1 or any B, away from
the current plane will imply a violation of Gauss' law of magnetostatics, Bcf, must obviously be
symmetrical about the straight wire. Then in 1,
Bq,27tra [to/

llo I
Or, Bq,
2 7t —r
In 2, B •271r = 0, or B, = 0
2 0
4.4.0/R 2
3.232 On the axis,B 3/2 = Bx, along the axis.
2 (R 2 + X2 )
00

00

dx
Thus,
fB dr. f
Exdr. NI R2
r (R 2
+ x2)3/2

— 00
2 j
— 00

31/2
go I R R sec2 0 d 0
=
2 2 R
sec
0
f 3 3
, on putting x = R tan 0

- x./2
x/2

1
tio / — cos 0 d 0 = 11/4/
2 - x/2

The physical interpretation of this result is that f Bxdx can be thought of as the circulation
ao

of B over a closed loop by imaging that the two ends of the axis are connected, by a line at
infinity (e.g. a semicircle of infinite radius).
3.233 By circulation theorem inside the conductor
2
B 2 7c r = tio jz it r or, B,- tio iz r/2

1 —•
B — jxr
2 °
Similarly outside the conductor,
360

1 . R2
By 2 3t r = iz X R 2 or, By
14°Jz r

---0 1 R2
So, B. Li. 0)61. —
2 ° r2
2.234 We can think of the given current which will
be assumed uniform, as arising due to a negative
current, flowing in the cavity, superimposed
on the true current, everywhere including the
cavity. Then from the previous problem, by
superposition.

-0,- 1 1 -°
B - 1 ) x (A P - B P ) = go j x /
2 1° -2
If 1 vanishes so that the cavity is concentric
with the conductor, there is no magnetic field in
the cavity.
3.235 By Circulation theorem,
r

B •2irr= t.tof j(ri)x21tr' dr'

or using By.. brainside the stream,

So by differentiation,
(a + 1) bra = [to j (r) r

b ( a + 1) _
Hence, j (r) = r

3.236 On the surface of the solenoid there is a surface current density


js= nIeip
A
e x ro
Then, B= - 31nIf Rdcpdz
4 ro
where 70a is the vector from the current element to the field point, which is the centre of the
solenoid,

-v2

gon.1 f dz
Thus, B Bz x2xR-
4x (R 2 + 43/2
-U2
361

-I /
+ tan —
2R

1
cos a d a (on putting z= R tan a)

= ptoni f -1 1
tan 2B

2
1/2
= ponisina= gonI go nil V1+ ( 1
1 (1/2) + R =

3.237 We proceed exactly as in the previous problem. Then (a) the magnetic induction on the
axis at a distance x from one end is clearly,

nI dz dz
B= x2 7tR ER 2 (z 213/2 -
2
1 µo n IR
2f x) 2 (z2 + R 2)3/2
0 x
n/2
1
=— n I f cos0d0= 1Ron/ 1—
2 2 Vrx27iR2
_1 -
tan
X>0 menas that the field point is outside the solenoid. B then falls with x. x < 0 means that
the field point gets more and more inside the solenoid. B then increases with (x) and
eventually becomes constant, equal to I,to n I. The B — x graph is as given in the answer
script.
Bo — 8B 1 xo
(b) We have, = [1 = 1 —71
R + xo
xo
or, — =1—2
R2 + x02

Since Ti is small (.1%), xo must be negative. Thus xo = — I xo

xo
and 1 — 2ri
11R 2 + 1 x0 12

is,
362

By comparision with the case of a solenoid and a hollow straight conductor, we see that field
inside the coil

= to 11: 1/1 — (h/2 Tt R)2

(Cf. B = go nI ).
Outside, only the other term contributes, so
I h
Bq, x 2 7tr = [to h x 2 3tR x 23TR

[to 2/
or, B q,
= TC

7 t

r
Note - Surface current density is defined as current flowing normally across a unit length over a
surface.
3.239 Suppose a is the radius of cross section of the core. The winding has a pitch 21tR/N, so
the surface current density is
I ---3. I --1.
J id e + e
s 2aR/N 1 23ta 2
where 4 is a unit vector along the cross section of the core and il is a unit vector along its
length.
The magnetic field inside the cross section of the core is due to first term above, and is given
by
B4).2.7rR = go NI
(NI is total current due to the above surface current (first term))
Thus, B, = p.0NI/ 27r,R.
The magnetic field at the centre of the core can be obtained from the basic formula.
-
-9
di34.. tto J., x r
dS and is due to the second term.
4n r 3
o

So, B = B z a=
—• 110 f 1. Rthp x 27ta
ez 4 7t 27ta R3
1101
or, Bz= 2R
N
The ratio of the two magnetic field, is = —it
3.240 We need the flux through the shaded area.
Now by Ampere's theorem;
I 2
B 2.7tr = go -I • ;Er
q)
3tR
ttO r
Or, B = — .1 --I
4) 2-it R
The flux through the shaded region is,
363

cpi - 5 1-dr.Bv(r)
0
R

= f dr - ill r = NI
o
2it R2 4n

3.241 Using 3.237, the magnetic field is given by,

B = 1 Ito nf 1- ,x --
2 2
x +R 2
1 1
µ n I = B , here /30. 1. ni,
At the end,B = -2 o -2 0 w t0 is

the field deep inside the solenoid. Thus,


(I) = ito nIS = 00/2, where (1) = vo nIS
2
is the flux of the vector B through the cross section deep inside the solenid.

3.242 Bq, 27Er = 1.4.0N I


NI
go
or Bp 7 23tr

Ro
Then, cl) - 5 B
92
hdr,as rs b= :4 .,-,- 2N l h In Ti, where 11= b/a
a

3.243 Magnetic moment of a current loop is given by pm = n i S ( where n is the number of

o
turns and S, the cross sectional area.) In our problem, n = 1, S = aR 2 and B = g — -
2 RR
2BR BR3
So, pm= n.12 2 — 21(
110 go
N
3.244 Take an element of length rd 0 containing — • rd0 turns. Its magnetic moment is
nr

lld0--Id2/
It 4
normal to the plane of cross section. We resolve it along OA and OB. The moment along OA
integrates to
x

o f11c12/dOcos 0. 0
4

while that along OB gives


a

N d2I
pm= f sin 0 d0 = N d2/
2
0
364

3.245 ( a ) Fr o m B i o t - S av a r t ' s l a w, th e ma g ne t i c i n duc t i o n du e to a c i r c ul a r c u r r e nt c a r r y i ng wi r e

l oo p at its c e nt r e is g iv e n by ,

B,. = -111 i

2r

Th e pl a ne sp i r a l i s m ad e up of c o nc e nt r i c c i r c ul a r l oo p s , h a v i ng d i f f e r e nt ra dii, v a r y i ng

f r o m a to b. Th e r e f o r e , t he to ta l m ag n e t i c i nd u c t i o n at th e c e nt r e ,

f RO
Bo - dN (1)
2 r

Ro
wh e r e - i is the c o nt r i but i o n of one t ur n o f r a d i us r a nd dN is t h e nu mb e r of t u r n s in

2r
the i nt e r v a l (r, r + dr)
N
i . e . dN = dr
b a
Su b s t i t u t i ng in e q u a t i o n (1) a n d i nt e gr a t i ng t h e r e s u l t o ver r b e t we e n a a nd b, we ob t a i n ,

Ni N lioiN
B 2r In b

(b - a) a

f
2

= dr =
° (b - a)
a
(b ) Th e m ag n e t i c moment of a turn o f r a d i u
s r is pm = in r2 and of all turns,
b
N dr _ ni (b3 - a2)
hy j
i n r 2

P=f pm t b a 3 (b - a)
a

3.246 ( a ) Let us ta k e a r i ng e l e m ent o f r a d i u s r a nd t h i c kne s s dr, then ch a r g e o n th e r i n g e l e ment . ,

dq= a2nrdr
(a 2 n r dr) co
a n d c u r r e nt , due to th i s ele ment , tii = = a CO dr r

2 i r

ILO di
So , m ag n e t i c i nd u c t i o n at the c e nt r e , du e to th i s ele ment : dB = -

2 r

R
f Ro a co r dr tic,

an d h ence , fro m s y mm e t r y :B=.1dBi. " "'.. CF °A

r 2

(b ) Magnetic moment of the element, considered,


dpm = (di) nr2 = a co dr n r2 = on to r3 dr
H e nc e , the so u ght m a g ne t i c m o me nt ,

R
R4
pm = f dp m= fanco r3 dr = a co n 4
365

3.247 As only the outer surface of the sphere is charged, consider the element as a ring, as
shown in the figure.
The equivalent current due to the ring element,

di = -2w7t (2 nr sin 0 rd 0) a (1)

and magnetic induction due to this loop element at


the centre of the sphere, 0,

dB . di 2 n r sin 0 r sin 0 I-to sine 0


di
4n r3 4n r
[Using 3.219 (b) ]
Hence, the total magnetic induction due to the
sphere at the centre, 0,
2t/2

N./2
a
f I-to wr 3 2
Hence, B= sin 0 d 0 —3 o cr (or 29 pT
4n
0

3.248 The magnetic moment must clearly be along the axis of rotation. Consider a volume

element dV. It contains a charge q 3 dV' The rotation of the sphere causes this charge
4 n/3 R
to revolve around the axis and constitute a current.
3q 2,.,
av x
4 R 3 2n
Its magnetic moment will be

3g dV x 2n x nr2 sine 0
43TR-
So the total magnetic moment is
R
o o
2 sin2 0 3.q_x1(1 xR5 x4 1 2

f 2R 3
r2 sin ode

The mechanical moment is


x r
2
dr
2R3 2 5 3 314 —5 qR

2 m
— tR co, So,P
5n M = 2m

3.249 Because of polarization a space charge is present within the cylinder. It's density is
pp — div P. — 2a
Since the cylinder as a whole is neutral a surface charge density ap must be present on the
surface of the cylinder also. This has the magnitude (algebraically)
366

a x 2aR = 2 a nR 2 or, cr = aR
p
P
When the cylinder rotates, currents are set up which give rise to magnetic fields. The
contribution of pp and op can be calculated separately and then added.
For the surface charge the current is (for a particular element)
oi
orRx2andxx— = aR2w dx
2.7c
Its contribution to the magnetic field at the centre is
goR2(aR2codx)
2 (x2 + R 2 )3/2
and the total magnetic field is

Bs =
f
lioR
2
2
(a R 2 oidx)
2 (x + R 2)3/2
go a R
2
40)
i
r 2
dr
2.3/2 m
„4
Ro a iv
2
0) 2
x --2-= 1.10 a R2(0
(x + R ) R
00 - 00

As for the volume charge density consider a circle of radius r, radial thickness dr and
length dx.
w
The current is- 2cc x 2ardrdrx = - 2 ar dr co dr
2n
The total magnetic field due to the volume charge distribution is
R co R co
2
r
-f a t.to w r3 dr f dx (x2 + r2)3/2
g0
B„= - f dr f dr2nrco 2 3/2 21
2 (x + r2)
0 -00
0 - 00
R

2
= -f a go cor dr x 2 = - 1.10 a co R so, B= Bs + By= 0
0
3.250 Force of magnetic interaction, g M e (17'x B )

lio e (vr1
Where, B
47r r3

If 2
So, Fma = r1) e [v---,x (v--,.
x 0]
t Llit—r3

µ0 e3 t go e2 (_ v2 y1
= 171:
zln r3 1 V1.1
X 17- ( 0 X 71 = 4n r3
-4.
And Fe e - geE-= e 1 er
4 n go 1 713

I mi ag I V2
Hence, = - V2Ro E0 •It (—) = 1.00 x 10- 6
IFe lectic I c
367
\
3.251 (a) The magnetic field at 0 is only due to the
curved path, as for the line element, dl tt F.'

I-to i Po i -3"
Hence, B = it (- k ) = — (- k )
4nR 4R
-2
--* PO 1 —1'
Thus i: = iB (- .1.) - 4R (- i )

-2
not
So, F. = 4R – 0.20 N/m
-4.
(b) In this part, magnetic induction Bat 0
will be effective only due to the two semi
infinite segments of wire. Hence

B . 2• li° i sin -I! (-


(I) 2
r)
4n 2

I-to i(_ )
=
itl

Thus force per unit length,


0
:. _ I-Lo ( r)
n1

3.252 Each element of length dl experiences a force BI dl. This causes a tension T in the wire.
For equilibrium,
T d a = BI dl,
where da is the angle subtended by the element at
the centre.
dl
Then, T = BI — = BI R
da
The wire experiences a stress
BIR
ic d2/4
This must equals the breaking stress crm for
rupture. Thus,

a d2 a.
B =
4IR
3.253 The Ampere forces on the sides OP and 0' P' are directed along the same line, in opposite
directions and have equal values, hence the net force as well as the net torque of these
forces about the axis 00' is zero. The Ampere-force on the segment PP' and the cor-
responding moment of this force about the axis 00' is effective and is deflecting in nature.
368

In equilibrium (in the dotted position) the


deflecting torque must be equal to the restoring
torque, developed due to the weight of the
shape.
Let, the length of each side be 1 and p be
the density of the material then,
1
ilB (1 cos 0) = (S / p)g — sin 0 + (S / p)g —1 sin 0
2 2
+ (S 1p) gl sin 0

or, 112 B cos 0 = 2 S p g12 sin 0


pg
Hence, B - 2S tan 0
I

3.254 We know that the torque acting on a magnetic dipole.


—4. _a, —a.
N. pmxB
A A
But, /5: = i S n , where n iis the normal on the plane of the loop and is directed in the
direction of advancement of a right handed screw, if we rotate the screw in the sense of
current in the loop.

On passing a current through the coil, this torque


acting on the magnetic dipol, is counterbalanced by
the moment of additional weight, about 0.
Hence, the direction of current in the loop must be
in the direction, shown in the figure.

13: x :9.= - /.>: Ainr


Or, N iSB- Amg1

So, B .-A1171 -1 = 0.4 T on putting the values.


NiS
3.255 (a) As is clear from the condition, Ampere's forces on the sides (2) and (4) are equal in
magnitude but opposite in direction. Hence the net effective force on the frame is the
resultant of the forces, experienced by the sides (1) and (3).
Now, the Ampere force on (1),
µo iio
F . I 2
1 21( (1 714
and that on (3), 1 3
to
F3 i
1

So, the resultant force on the frame


.„„e.._._a÷.___),,
=F1- F3, (as they are opposite in nature.) 4
369

2 1.10 iio
0.40µN. a
(4 ri2 — 1)
(b) Work done in turning the frame through some angle, A= f id (I) = i (43f — cl)i), where Of is
the flux through the frame in final position, and (Di, that in the the initial position.

Here, I (Dfi - — cl) and — (Df

so, A1= 210 and A= i2(13

Hence, A= 2i f Bds

a (71 + —1)
2

go Uo a +1)
=21 f go i a— r= In 2ri — 1
2.7t r

3.256 There are excess surface charges on each wire (irrespective of whether the current is
fl owing through them or not). Hence in addition to the magnetic force Fm, we must take into
account the electric force Fe Suppose that an excess charge X, corresponds to a unit length of
the wire, then electric force exerted per unit length of the wire by other wire can be found
with the help of Gauss's theorem.
1 21X, 2A2
Fe. ),E k _ (1)
eo 4 nE0/'
where 1 is the distance between the axes of
the wires. The magnetic force acting per unit ++++++++++++-F
length of the wire can be found with the help of
the theorem on circulation of vector B
[to 1.2
F.= —
4n /
where i is the current in the wire. (2)
Now, from the relation,
X C p, where C is the capacitance of the wires per unit lengths and is given in problem
3.108 and cp = iR
aEo i
In
t. R or ,,
ne o R (3)
i
Dividing (2) by (1} and then substuting the value of — from (3), we get,
Fm 110 (1nri)2
eo 312R 2
The resultant force of interaction vanishes when this ratio equals unity. This is possible
when R. Ro, where
370

„ _ Vrio nin
/to — — 0.36 IQ
Eo it
3.257 Use 3.225
The magnetic field due to the conductor with semicircular cross section is
µ0I
B=
n2R
r2
aF go i
Then
al — = BI = n2 R
3.258 We know that Ampere's force per unit length on a wire element in a magnetic field is
given by.
A A

dFn= i (n x B) where n is the unit vector along the direction of current. (1)

Now, let us take an element of the conductor i2,


as shown in the figure. This wire element is in
the magnetic field, produced by the current i1,
which is directed normally into the sheet
of the paper and its magnitude is given by, I/

IBI= (2)
2 nr

From Eqs. (1) and (2) .(--- b ---).


/2 A --lb 12
d Fn = — dr (n x B), (because the current through the element equals — dr
b b
A ---3'

So, (IF = 11° '112 —dr ,towards left (as n .1! B ).


n a b r'
Hence the magnetic force on the conductor :
a+b

"g.. tio 1112 f dr PO 1112 a+b


F — — = — —F. in (towards left).
ks(ttowardowar
= ds left)
n 2n b r 23t a
a

Then according to the Newton's third law the magnitude of sought magnetic interaction
force
110Ill a +b
2
= In
a b a
3.259 By the circulation theorem B = µo i,
where i = current per unit length flowing along the plane perpendicular to the paper. Currents
flow in the opposite sense in the two planes and produce the given field B by superposition.
1
The field due to one of the plates is just —B. The force on the plate is,
2
1 B2
2-B x i x Length x Breadth = — per unit area.
2t.tc,
(Recall the formula F = BB on a straight wire)
371

B1 + B2
3.260 (a) The external field must be which when superposed with the internal field
2
Bl — B2
2 (of opposite sign on the two sides of the plate) must give actual field. Now
B1— B2 1
2 2 11°1
B1— B2
Or,

2—B2
F B1 2
Thus, 21.to Br 82

Bl — B2 + B2
(b) Here, the external field must be upward with an internal field, pward
2 2 'u
on the left and downward on the right. Thus,
B1+ B2 2— 2
B1 B2
i= and F —
1-to 21,to

(c) Our boundary condition following from


Gauss' law is, B1 cos 01 = B2 cos 02.

Also,(B1 sin 01 + B2 sin 02) = µo i where i


= current per unit length.
Br 82
B1 sin 01 — B2 sin 02
The external field parallel to the plate must be
2
(The perpendicular component B1 cos 01, does
not matter since the corresponding force is
tangential)
B 2 s;,,,2 _ r,e 2 ,;n2 0
n °All
Thus, F — 1 2110 2 per unit area
2 2per unit area.
B1 B2
2 µo
The direction of the current in the plane
conductor is perpendicular to the paper and
beyond the drawing.
3.261 The Current density is —aL , where L is the length
of the section. The difference in pressure
produced must be,

1 IB
Ap = —aL x B x (abL)/ab —
a
372

3.262 Let t — thickness of the wall of the cylinder. Then,


J = 1/2 n R t along z axis. The magnetic field due to this at a distance r
t t
(R —i < r <R+i , is given by,

R+2

2
Fr 1 f go12 17.2
and = 27vRL 27cRL 8 2t2 R2 t2 r 2 x 2 It rL dr


--t)
Po /2
8 3t2R3 t2

R-i

u 12
go 12
, [R t + 0 (t2)]*. °- r

8 n2 R t 8n2 R2
1
3.263 When self-forces are involved, a typical factor of — comes into play. For example, the
2
force on a current carrying straight wire in a magnetic induction B is Bll. If the magnetic
induction B is due to the current itself then the force can be written as,
I
F= f B(r)dr1

If B ) a P, then this becomes, F = B (/)//.


2
In the present case, B (I) = µo n I and this acts on nI ampere turns per unit length, so,
F 1 /xn/xlx/ 1 22
pressure p= Area = z n 1x1 = 2 n I
3.264 The magnetic induction B in the solenoid is given by B = µo nI. The force on an element
dl of the current carrying conductor is,
1 2
dF= — n dl = !go nI dl
2 ° 2
1
This is radially outwards. The factor — is explained above.
2
373

To relate dF to the tensile strength Firm we


proceed as follows. Consider the equilibrium of
the element dl. The longitudinal forces F have a
radial component equal to,
dO
dF= 2F sin — = F d 0

1 2
Thus using dl = R d 0, F = - R
2 ° nI

m when, 2 Fiml
This equals F li I= =
"to n R

Note that Flim, here, is actually a force and not a stress.

3.265 Resistance of the liquid between the plates= Pi

Voltage between the plates = Ed = v Bd,

vBd
Current through the plates =
R+

Power, generated, in the external resistance R,


2 v2B2d 2 2 2 2
v B2d 2R V B d
P= 2 2 - 2
1/2
VR-+ 5
VVR R " - (.521L4Tz- +
S

2 2
pD v B Sd
This is maximum when R= and Pm
4p

3.266 The electrons in the conductor are drifting with a speed of,

vd = —
2 '
ne R ne

where e = magnitude of the charge on the electron, n = concentration of the conduction


electorns.
The magnetic field inside the conductor due to this current is given by,

Jr
B (23tr) = TE r2 1
22
µ
o or
,
B
=
R 27t R 2

A radial electric field vB q,must come into being in equilibrium. Its P.D. is,
R
I Ro Ir2 dr I
12
(R° I)=
12°
A(I) al R2 ne 27t R itR2 ne 47t 4 R2 ne
374

3.267 Here,vd = — and j = ne vd


B

200 x 104 m2 x 1 T
SO, n= V /in
eE 1.6x10-19Cx5x10

= 2.5 x 1028 per m3 = 2.5 x 1022 per c.c.

Atomic weight of Na being 23 and its density go 1, molar volume is 23 c.c. Thus number
6 x 1023
of atoms per unit volume is
23
- 2.6 x 1022
per c.c.
Thus there is almost one conduction electron per atom.'

dirft velocity
3.268 By definition, mobility - or t=
Electric field component causing this drift EL
On other hand,

1
ET= vB = —, as given so, IA - 3.2 x 10-3 m2/(V • s)
T1 T1B

t1/4/
3.269 Due to the straight conductors - 21tr

We use the formula, F = (gi• V )134.


(a) The vector /5,*,, is parallel to the straight conductor.
a
F = pm -62- B = 0,
because neither the direction nor the magnitude of B depends on z
(b) The vector j?„, is oriented along the radius vector 7"

F=
or

The direction of B at r + dr is parallel to the direction at r. Thus only the p component of F

will survive.

a tiff po I Pm
Fcp= pm ar
coincides in direction with the magnetic field, produced by the conductor
(c) The vector pm carrying
current I

a Rol' —a. tic, Pm a k;


Pm rag) 27t 2 3-cr2 aq9
Oe
I-to I Pm
So, F - 2 er As, =-
27tr a
376

a
3.270 Fx = pm ix- Bx

But,

[to 6 it R 2 ipm X
5
43t (x2 + R 2) "2

3.271
a {Ro 3 i•;',„ • Fr— _gmr2
F= P
2m al 47t r5
a {tio Pim} — 3 tloP1mP2m - 9 nN
l3
= A 2m -51 23t = 2 nl4

3.272 From 3.270, for x » R,


1.4.0i1R 2
B
x 2x3

2B x3
or, T x 2
110 R
3.273
B' = B cos a,
1
= sin a,
n
B' = RB sin a
SO,
Vaccum
B' = B 1/1.1,2 sin2 a + cos2 a
/if
3.274 (a) j i. d :§= 7*. dS, 0
loo—j) * d 3 = -7 .s
ince f it. ciE=
Now ris nonvanishing only in the bottom half of the sphere.
13
376

1 B
Here, B'n. B cos 0, H' 1 . —B sin 0, B'1= pi B sin 0, H' n = cos 0
go ggo

B cos 0 . _ 1
Jn = 1 — and ./t = - 1-B sin 0
µo Pt go

Only .In contributes the surface integral and

- i 7. cir= -f 7.dt4 J„ds- itR2B cos o (1. -- 1


lower
tio IA lower

ii.•dr= (13,—B't)1= (1 — pt)B /sin 0

Jr
3.275 Inside the cylindrical wire there is an external current of density --2. This gives a magnetic
7r.R

field Hq, with

r2 Jr
H 2nr = I or, H .
9) 17 11) 2it R 2
two Jr x h.
Jr
From this B— ,, and .1,„— g — 1 ,, =
27TR ...
Magnetization.
tP 27tR ' , 241 R 2

Hence total volume molecular current is,


r=R

xI
f 1-*. dFt f
(P 2 dl
nR
= xI

The surface current is obtained by using the equivalence of the surface current density to
-->
.11._.
J x n, -3■
this gives rise to a surface current density in the z —direction of —
2tR
The total molecular surface current is,

xi
/' = — (27r,R) = — x/.
s 231R

The two currents have opposite signs.

3.276 We can obtain the form of the curves, required here, by qualitative arguments.

From f gal: 1,
we get H (x » 0) . H (x « 0) . nI
Then B (x » 0) . titio nI
B (x _< 0) = t.t.onI
Also,

B (x < 0) = [to H (x < 0) J


(x < 0) = 0
B is continuous at x = 0, H is not. These give the required curves as shown in the answer-
sheet.
377

3.277 The lines of the B as well as H field are circles around the wire. Thus

H17tr+H2nr= I or, Hi +H2 =

Also 1..to µ1H1= µ2H2 [to = B1= B2 = B


I
Thus II =
[ii 112 nr
111 I 111
H2 —
/41 + IA2 nr
111 1-12 I
and
B = µoo 111+ 1-12. nr
3.278 The medium I is vacuum and contains a circular current carrying coil with current I. The
medium II is a magnetic with permeability The boundary is the plane z= 0 and the
coil is in the plane z = 1. To find the magnetic induction, we note that the effect of the
magnetic medium can be written as due to an image coil in II as far as the medium I is
concerned. On the other hand, the induction in II can be written as due to the coil in I,
carrying a different current. It is sufficient to consider the far away fields and ensure that
the boundary conditions are satisfied there. Now for actual coil in medium I,

tloPm (2 cost 0, — sin2 0,) and B


SO, B2
zin 1A0Pm (— 3 sin 0' cos 0')
47t
, I (n a2), a = radius of the coil.
where pm =
Similarly due to the image coil,
1A0Pin, 2 m
Bz— 4n (2 cos 0' — sin2 0'), Bx = 7t (3 sin 0' cos 0'), p' m= I' (Jr a2 )

As far as the medium II is concerned, we write similarly


A,
Bz— Oat
tioP
(2 cost 0' — sin2 02 Bx— l-toP (— 3 sin 0' cos 02 p"m = r a2 )
378

The boundary conditions are, p,,,+ p',„ = p",,, (from Bin = B2,,)

- p,„+ p'„, = - 14p" „, (from 1111= H21)

Thus, /1 ' = --- ----


p, + 1
1 r= m-=-1-I
' t.t, + 1
In the limit, when the coil is on the boundary, the magnetic field enverywhere can be

obtained by taking the current to be -2E-I. Thus ,i.'= -----. /3., 11 + o


[I, + 1
3.279 We use the fact that within an isolated uniformly magnetized ball,
-4.
4%J
H' = - J/3, B' = , where J is the magnetization vector. Then in a uniform magnetic
3
field with induction Bo, we have by superposition,
-1.. -4. 2110./ -4. B T /I
B. = Bo+--s—, H. MT. '..
0
J 1 ..)

110

or, B„,+ 21.4.0Hi„ = 3 Bo


also, B:= ti Ito irin
3 Bo -+
Thus, H. = and B. = 3 11B°
is 110 (Ix + 2) m 1.1. + 2
3.280 The coercive forcelf, is just the magnetic field within the cylinder. This is by circulation
N I
theorem, 11,.= 1 = 6 kA/m

(from f •d r= I, total current, considering a rectangular contour.)

1-1
3.281 We use, f 11.111= 0

Neglecting the fringing of the lines of force, we


write this as

H(td-b)+-11.b. 0
1-10
-Bb
or, Hi= , - 101 A/m
[to 'ra
The sense of H is opposite to B
Bb NI lito -Bb
3.282 Here, f 4:d r= NI or, H(27(R)+---- NI, so, H=
[to 2 TER tto

= B 2nRB
Hence, 1-1. - - 3700
tioH lioNI-Bb
B
3.283 One has to draw the graph of IA = µB H from the given graph. The 4 -H graph
H
starts out horizontally, and then rises steeply at about H= 0.04 k A/m before falling agian.
It is easy to check that Ix. ft, 10,000.
379

3.284 From the theorem on circulation of vector


Bb tioN I [to d
Hnd+ — = NI or, B H = (1.51 — 0-987)H,
b b
where B is in Tesla and H in kA/m. Besides, B and H are interrelated as in the Fig. 3.76 of the
text. Thus we have to solve for B, H graphically by simultaneously drawing the two curves
(the hysterisis curve and the straight line, given above) and find the point of intersection. It is
at
H 0.26 kA/m, B = F25 T
Then, 4000.
1-to H
3.285 From the formula,
F (p: • 1V3)r__.i_ pf (J. ,g)rdv,
Thus (B-V
tt [to
or since B is predominantly along the x—axis,
x L
aB x SB2 x SB2
Fx f Bx Sdx = Sgo f dB,2,
ax 212 po 2µo
lAo 211 x 0
3.286 The force in question is,
—3. --0 BV dB
F „(p, • V ) B - ,

since B is essentiatlly in the x-direction.


x V dB2 _ X Bo2 V d x Bo2
- ax
So, Fx sa. 2µo dr 2µo cbc (e 2 ax2 ) — 4 e V
2110
This is maximum when its derivative vanishes
i.e. 16 a2 x2 — 4 a — 0, or, xn, = 1
4a
The maximum force is,
2

F 1 x B02 V.I
max . 4a -1/2 X B0 V—
{4; e 2µo Ve
So, x (R0Fnia,, T) / VB02 = 3.6 x 10-4
Bx x131 dB x V dB2
3.287 Fx 2µo dx
dx
This force is attractive and an equal force must be applied for balance. The work done by
applied forces is,
x- L
A f Fx dx _LK (_ B2 Ix. x VB2
x 0 -
go ° 2 go
380

3.6 ELECTROMAGNETIC INDUCTION. MAXWELL'S EQUATIONS

3.288 Obviously, from Lenz's law, the induced current and hence the induced e.m.f. in the loop
is anticlockwise.
From Faraday's law of electromagnetic indcution,
0—>
cV.).
dt

Here, d0= B•dS= —2Bxdy,

and from y = ax2, X =


a

Hence,
2B -V-TA a dt

1 774, X
= By — , using dt 12WY
a
3.289 Let us assume, B is directed into the plane of the loop. Then the motional e.m.f.

gin= — (v-*x •dr = vBl

and directed in the same of (i7x B) (Fig.) _L. R1 R2

ByI R2
So, i—
Ri R2 R+
R+
Ri+ R2

As R1 and R2 are in parallel connections.


3.290 (a) As the metal disc rotates, any free electron also rotates with it with same angular
velocity w, and that's why an electron must have an acceleration w2r directed towards the
disc's centre, where r is separation of the electron from the centre of the disc. We know
from Newton's second law that if a particle has some acceleration then there must be a net
effecetive force on it in the direction of acceleration. We also know that a charged particle
can be influenced by two fields electric and magnetic. In our problem magnetic field is
absent hence we reach at the conslusion that there is an electric field near any electron and
is directed opposite to the acceleration of the electron.
If E be the electric field strength at a distance r from the centre of the disc, we have from
Newton's second law.
m
= wn
m o.)2 r
eE = m r o.)2, or, E —
e
and the potential difference,
a a
--+ 2
cpcen— cprbn= f •dr = f M 03 r dr, as Et 4. di:*
e
0
381

(Pcen —(Prim ' A cp = m 2


Thus c
o2
a — 3.0 nV
e 2
(b) When field ifis present, by definition, of motional e.m.f. :
2

CP1 — (P2 = f 1-7X 13) • di.

1
Hence the sought potential difference,
a a

(Pcen (PrimMr f — V Bdr= f — cor B d r , (as v= wr)


0 0
1
Thus (Prim 419cen p= — co 2
Ba = 20 mV

eB
(In general w < — so we can neglect the effect discussed in (1) here).

3.291 By definition,
7E= ( 17X 11)

C C d

So, f • Cr it f - (TX ) •d f - y B dr

A A 0

But, v = wr, where r is the perpendicular distance of the point from A.

Hence, f —o)Brdr= —1(oBd2= — 10mV


2
A

This result can be generalized to a wire AC of arbitary planar shape. We have


C C C

A A A

- - f (B•r w-B•co r)•dr

1 A
=—— Bwd2,
2
d being AC and r being measured from A.

3.292 Flux at any moment of time,

143 IB • 1
1= R2 d S1= B
2
is the sector angle, enclosed by the ield.
where q f Now,
magnitude of induced e.m.f. is given by,
382

d'' BR2 dcp BR2


gin = co,
- dt 2 dt 2
where co is the angular velocity of the disc. But as it starts rotating from rest at t = 0 with
an angular acceleration 13 its angular velocity w (t)= 13t. So,
BR2
L-
2 i(t.
According to Lenz law the first half cycle current in the loop is in anticlockwise sense, and
in subsequent half cycle it is in clockwise sense.
nBR 2
Thus in general, = (- 1) ilt, where n in number of half revolutions.
2
The plot gu, (t), where t„ = 1/2 n n/(3 is shown in the answer sheet

3.293 Field, due to the current carrying wire in the region, right to it, is directed into the plane
of the paper and its magnitude is given by,
[to i
B = — – where r is the perpendicular distance from the wire.
2n r
As B is same along the length of the rod thus motional e.m.f.
2

= - f (nit) • dr . vB 1
1
and it is directed in the sense of (v B ) So,
current (induced) in the loop,
i,, 1 go/ vi
t. -
in R 2nRr

3.294 Field, due to the current carrying wire, at a perpendicular distance x from it is given by,
I-to i
B (x ) = — -
27c x

Motional e.m.f is given by - (173x134.) • d r

There will be no induced e.m.f. in the segments (2) and (4)


as, ill d / and magnitude of e.m.f. induced in 1 and 3, will be

gi = -"9 ) a andln=v ( 110 i )a


27t x

v( 27t (a + x) '

respectively, and their sense will be in the direction of ( ill< B ).


So, e.m.f., induced in the network = i - [as 11 > ]


a v 1.1,0 i 1 1 v a2 110 t
_ _
27c x a +x 2 7 c x (a + x)
383

3.295 As the rod rotates, an emf.


d 1 2 1 2
— a 0.B=- B w
dt2 2

(t) - - 1 2 B w
2
is induced in it. The net current in the conductor is then
R

A magnetic force will then act on the conductor of magnitude BI per unit length. Its
direction will be normal to B and the rod and its torque will be
a

of i (t) - -21 a2 B w)
dx B x
R
0
Obviously both magnetic and mechanical torque acting on the C.M. of the rod must be
equal but opposite in sense. Then
for equilibrium at constant w
1 2
(t) - a Bw 2
2 Ba in
A mga s cot
R 2 2
mg R
or, (t) = 1 a2 B w + sin w t - 1 (2 B2 (.0 + 2 mg R sin w t)
2 aB 2 aB
(The answer given in the book is incorrect dimensionally.)

3.296 From Lenz's law, the current through the connector


is directed form A to B. Here in = vBl between
A and B -
where v is the velocity of the rod at any moment.
For the rod, from Fx = mwx
or, mg sin a - i 1 B = mw

For steady state, acceleration of the rod must be


equal to zero.
Hence, mg sin a = ilB (1)

Lin . vBl
But, i=
R R \VP

in
From (1) and (2,) v -mg s aR
B212

3.297 From Lenz's law, the current through the copper


bar is directed from 1 to 2 or in other words,
the induced crrrent in the circuit is in clockwise
sense.
Potential difference across the capacitor plates,

-dI= in or, q=C L


384

Hence, the induced current in the loop,


dq
t= -C
dt dt
But the variation of magnetic flux through the loop is caused by the movement of the bar. So,
the induced e.m.f. gin = B lv
d dv
and, - B1 — = Blw
dt dt

Hence, i= C CB lw
dt
Now, the forces acting on the bars are the weight and the Ampere's force, where Fa mp

= i 1B (CB 1 w) B= C 12 B2 w.
From Newton's second law, for the rod, Fx = mwx
Or, mg sin a - C 12B2 w= mw

mg sin a g sin a
Hence w= i2B2c
C12B2+m
1+
m

1.298 Flux of B, at an arbitrary moment of time t :


2
na
CDt = B•S= B 2 cos co t,

(I)
From Faraday's law, induced e.m.f., =d
dt
2
-fL- COS 0) t)
d 2
B 7C a2 (0 .
— — Sill t.
dt 2
2co sin co t.
B a
and induced current, = —
R 2R
Now thermal power, generated in the circuit, at the moment t = t :

(13 a2 oo)2 1
P(t) = gin x — 2 R sin 0)t
2

and mean thermal power generated,


2 T
[B a a2 WI 1
fsin2 co t dt
2
0 1
P>= 2
2R (n(.0 a2

J dt 2
Note : The claculation of which can also be checked by using motional emf is correct even
though the conductor is not a closed semicircle , for the flux linked to the rectangular part
containing the resistance R is not changing. The answer given in the book is off by a factor
1/4.
385
3.299 The flux through the coil changes sign. Initially it is BS per turn.
Finally it is — BS per turn. Now if flux is cl. at an intermediate state then the current at that
moment will be
d (130
—N
i= dt
R
So charge that flows during a sudden turning of the coil is
q= f i dt = —Ls-I- polo — (— (1)]. 2 N BS /R
R
Hence, 1 qR
B=— = 0.5 T on putting the values.
2 S
3.300 According to Ohm's law and Faraday's law of induction, the current io appearing in the
frame, during its rotation, is determined by the formula,
d cl) L d io
io' —dt ' dt
Hence, the required amount of electricity (charge) is,
1
q= f iodt = -R f (d (I) + L dio) = -R (Acic,+LAio)
Since the frame has been stopped after rotation,
the current in it vanishes, and hence A io = 0.
It remains for us to find the increment of the I I
lfux A cl) through the frame (A (13= (1:02 — 49. a I
i
Let us choose the normal iito the plane of the 1
frame, for instance, so that in the final position,
n is directed behind the plane of the figure :1
(along B ). it- a ---)4 0'
• Then it can be easily seen that in the final position, 43102 > 0, while in the initial position,

ctli <0 (the normal is opposite to B ), and A cl) turns out to be simply equal to the fulx
through the surface bounded by the final and initial positions of the frame :
b+a

A (13, = 4)2 + I 4)11 = f B a dr,


b-a

where B is a function of r, whose form can be easily found with the help of the theorem of
circulation. Finally omitting the minus sign, we obtain,
AO I.to a i ln b + a
q= ---=
R 2 it R b — a
3301 As B, due to the straight current carrying wire, varies along the rod (connector) and enters
linerarly so, to make the calculations simple, B is made constant by taking its average
value in the range [a, b].
386

<B>=
b

f B dr f —il —
b

o iodr

a
f
b

a
dr
m a
2n r
b

f dr
rgm< ›,X
a
a b
g o io b
or, <B > = ln — <
2 n(b - a) a 10
(a) The flux of B changes through the loop due to the movement of the connector. According to
Lenz's law, the current in the loop will be anticlockwise. The magnitude of motional e.m.f.,

So, induced current


in in . go io v 1 b
I. R 2 n R -n —a

(b) The force required to maintain the constant velocity of the connector must be the
magnitude equal to that of Ampere's acting on the connector, but in opposite direction.

o jo b ( io b
So, F ius / <B>. g -vin— (b- a) 11/4 In
2nR
2
v go .(
a 2n (b - a) a

b
= — --z ln — and will be directed as shown in the (Fig.)
R 2n° a '

3.302 (a) The flux through the loop changes due to the movement of the rod AB. according to
Lenz's law current should be anticlockwise in sense as we have assumed B is directed into
the plane of the loop. The motion e.m.f i,,(t)= B 1 v
B1
and induced current i. =V
in R
From Newton's law in projection form Fx = mwx
mp m v
-Fa = d
dr
v B2 12
But Fa„v, = ia, 1 B =
R

v B2 12 dv
So, -
R = v —
dx

)
387

x 0
mR f Rvo
Or, — dv or,
m
f d" B2 /
vo
, x
B2 / 2

(b) From equation of energy conservation; Ef —Ei + Heat liberated = Ace/1+216a

[0-1mv 2 +Heat liberated = 0+ 0


2 °

So, heat liberated = m v2


2 °

3303 With the help of the calculation, done in the previous problem, Ampere's force on the
connector,

V B212
Fa mp - R directed towards left.

Now from Newton's second law,


dv
F Fa mp =m 80
at
R
B212 dv
So, F= VR + m — < F
dt OMp

R
B
(F B212 )
—m =
R, R
Or,
— B1 F
t B212 ) RF
Thus v= (1 — e - Ent B212

3.304 According to Lenz, the sense of induced e.m.f. is such that it opposes the cause of change
of flux. In our problem, magnetic field is directed away from the reader and is diminishing.

(a) (b) (c) (d)

So, in figure (a), in the round conductor, it is clockwise and there is no current in the
connector
In figure (b) in the outside conductor, clockwise.
388

In figure (c) in both the conductor, clockwise; and there is no current in the connector to
obey the charge conservation.
In figure (d) in the left side of the figure, clockwise.

3.305 The loops are connected in such a way that if the current is clockwise in one, it is anticlockwise
in the other. Hence the e.m.f. in loop b opposes the e.m.f. in loop a.

e.m.f. in loop a = 51- (a2 B) — a2 B (o sin cot)


dt dt
Similarly, e.m.f. in loop b . b2 Bo co cos cot.

Hence, net e.m.f. in the circuit = (a2 — b2) Bo co cos cot, as both the e.m.f's are in opposite
sense, and resistance of the circuit = 4 (a + b) p
Therefore, the amplitude of the current
(a2 b2) Bo (0

— — 0.5 A.
4 (a + b) p
3.306 The flat shape is made up of concentric loops, having different radii, varying from 0 to
a.
Let us consider an elementary loop of radius r, then e.m.f. induced due to this loop
— d (B • S )
— n r 2 Bo CO cos cot.
= dt
and the total induced e.m.f.,
a

gind m f (7c r2 Bo co cos cot) d N, (1)


o
where n r2 W cos wt is the contribution of one turn of radius r and d N is the number of
turns in the interval [r, r + dr].
So, d N = (1 dr (2)
a
a

N TE Boo) cos co tN a2
From (1) and (2), = f — (it r2B0 co cos cot) — dr -
a 3
0
1
Maximum value of e.m.f. amplitude L, = TE Bo co N a2

3.307 The flux through the loop changes due to the variation in B with time and also due to the
movement of the connector.
d (; . .75) . d (BS )
So, Lm as S and B are colliniear
dt dt
1
But, B, after t sec. of beginning of motion = Bt, and S becomes = 1 — w t2, as connector
2
starts moving from rest with a constant acceleration w.
3
So, in,,i= fB/wt2
389

3.308 We use B = Ron I


Then, from the law of electromagnetic induction
al)
d1=—
d

So, for r < a

Eq, 2 r — r2 go n / or, E = —2 nr (where/ = dlldt)


°

For r> a

;27E7.= —na2 !Joni or, Ev = — gonI a2 /2 r
The meaning of minus sign can be deduced from Lenz's law.

• d2
3.309 The e.m.f. induced in the turn is go n I n 4

The resistance is —" p.

go n/Sd
So, the current is — 2 m A, where p is the resistivity of copper.
4p
3.310 The changing magnetic field will induce an e.m.f. in the ring, which is obviously equal,
in the two parts by symmetry (the e.m.f. induced by electromagnetic induction does not
depend on resistance). The current, that will flow due to this, will be different in the two
parts. This will cause an acceleration of charge, leading to the setting up of an electric
field E which has opposite sign in the two parts. Thus,

—n aE= rI and 1+ naE= /,


2 '2
where is the total induced e.m.f. From this,
l= (fi + 1) rI,

and E= — 1) d —
2 a
2 7t a + 1
But by Faraday's law,l = n a2 b
1 —1
so, E= —
2 +1

3.311 Go to the rotating frame with an instantaneous angular velocity 53%4 In this frame, a
Coriolis force, 2 m v x (51*(0
-4"
acts which must be balanced by the magnetic force, ev --a• xB (t)

Thus, w(t)= —
2m
(It is assumed that Zis small and varies slowly, so 0)2 and to. can be neglected.)
390

3.312 The solenoid has an inductance,


L . 1.10 n2x b
2 1,
where n = number of turns of the solenoid per unit length. When the solenoid is connected to
the source an e.m.f. is set up, which, because of the inductance and resistance, rises slowly,
according to the equation,
NJ-Li= V
This has the well known solution,
V t R/L
1 = - (1 - e- ).
R
Corresponding to this current, an e.m.f. is induced in the ring. Its magnetic field
dF 2
B = 1.1,0 n I in the solenoid, produces a force per unit length, — = B i - [to n2 n a2 II/ r
dl
.11 ( 23, n
- a2 v 2 ( n2 e - tR/L (1 - e- t R/L ),
r RL
acting on each segment of the ring. This force is zero initially and zero for large t. Its
maximum value is for some finite t. The maximum value of
2
- t R/L 1_ 1_ e_ Lila 1
e (1- e - :RA ) = - is
4 2 4 •
2 2 2 2 2
d F. [to ir a V n2 [to a V
So
dl r 4RL 4rR1b2

3.313 The amount of heat generated in the loop during a small time interval dt,
d(1)
dQ- /R dt, but, l = -— = 2 at - a -r,
dt
(2 at - a -02
So, d Q - dt
R
and hence, the amount of heat, generated in the loop during the time interval 0 to T.
T

f 2 at - a T)2 1 a2 T3
Q= ( dt - 3 R
R
0
3.314 Take an elementary ring of radius r and width dr.
The e.m.f. induced in this elementary ring is x r 2 3.
Now the conductance of this ring is.

d (1). h dr h r dr ,,
so d I -
R p 2 Itr 2p P

Integrating we get the total current,


b

hr dr h 13 (b2 - a2)
I -f 13 -
2p 4p
391

110 n2 v . l_to n2 10 nR 2 ,
3.315 Given L = where R is the radius of the solenoid.

Thus, n= •FL---- 1
I-to Ion R •
So, length of the wire required is,

‘1 7aI T
1= n1021tR= = 0.10 km.
1-to

3.316 From the previous problem, we know that,

vL 14 It
/I = length of the wire needed= , where 1= length of solenoid here.
I-to
PPS r
Now, R = — (where S = area of crossection of the wire. Also m = p S I')
S'

Thus,

where po = resistivity of copper and p = its density.

Rm LI
Equating,
PPo Ro/4 n
go m R
or, L= —
4n ppo /
3.317 The current at time t is given by,
V
1 (t).. — (1 - e- tRiL )
R
V
The steady state value is, 10 = i i
1 (0 - t.R/L
and /0 = r1= 1 — e- t RA
or , e = 1 - T1
1 L 1
or, t0 — R = In or, t = In = 1.49 s

3.318 The time constant t


°L 1 is given
'n

by
° R 1- ri

L L
T= —=
R 10 '
Po s
where, p0 = resistivity, /0= length of the winding wire, S = cross section of the wire.
But p
m= 1 0 S
L mL
So eliminating S,t — , —
PO 10 PPO 10
in/ P lo
392

7111L
From problem 3.315 10 =

(note the interchange of l and 10 because of difference in notation here.)

mL m
Thus, i =
4 - [to 4 n -0.7 ms,
PP° —L 1 P Po /

3.319 Between the cables, where a < r < b, the magnetic field H satisfies

H 2 7t r = / or, H9,=
8P 2 r
11, /
So
B`P 2 7t r
b

1-to I [to/ b
r= a
The associated flux per unit length is,(13 =
f
2 it
r
x 1 x 2 7t
dr -
In —
a

(13
-t
o
Hence, the inductance per unit length L — IM
— In Th Where = —
1= I= 2n a

We get L1 = 0.26 IAH


m
NI NI
3.320 Within the solenoid,1/ -2 n r = NI or
(1)
ii-cp = r
B
CP
-
IA11° 2 r
a+b

1-ttio f a dr
and the flux, = N41)1, = N — N I
2 it r
b

2 a
Finally, L= t-to N a ln 1 +

3.321 Neglecting end effects the magnetic field B,


between the plates, which is mainly parallel

to the plates, is B = 120E.

(For a derivation so 3.229 b)


Thus, the associated flux per unit length of the
plates is,

R0,xhxi=
So,L1 = inductance per unit length = 110 = 25 nH/m.
393

/
3.322 For a single current carrying wire,B (r > a). For the double line cable, with current,
q) 2 r
flowing in opposite directions, in the two conductors,

Rol
B
c — between the cables, by superposition. The associated flux is,
3tr
d—a

f tiol dr x 1 Rol d
=
7r r In = In rl x /, per unit length
a

Hence, L1= lnrl

is the inductance per unit length.

3.323 In a superconductor there is no resistance, Hence,


dI d (13
— = +
J.,
dt dt

M n a2 B
So integrating, = —=
L L

because = - ctof = n a2 B, 43i= 0

4 2
014 ) 1 7c2 a B
Also, the work done is, A = f I dt = f I dt — - iL I2 —
dt 2 2 L

N 2S
3.324 In a solenoid, the inductance L = µµo n2 V = Ro ,
where S = area of cross section of the solenoid, 1= its length, V = SI, N = nl = total number of
turns.
When the length of the solenoid is increased, for example, by pulling it, its inductance will
decrease. If the current remains unchanged, the flux, linked to the solenoid, will also
decrease. An induced e.m.f. will then come- into play, which by Lenz's law will try to
oppose the decrease of flux, for example, by increasing the current. In the superconducting state
the flux will not change and so,
I
= constant

I _ -/c•
Hence, or, I= I0 / 10 = 10 (1 + 11)
/ - /0 '
3.325 The flux linked to the ring can not change on transition to the superconduction state, for
reasons, similar to that given above. Thus a current I must be induced in the ring, where,

'4: n a2B aaB


I= 0 - -
8a 8a
L [to a In 7;,- - 2) µ0 In b - - 2)
394

3326 We write the equation of the circuit as,


,. L di ,
nt + — — = q
11 dt '
for t a 0. The current at t = 0 just after inductance is changed, is

i = ri R4, so that the flux through the inductance is unchanged. We


look for a solution of the above equation in the form

i= A + Be- tic

Substituting C= R, B= i- 1, A= ?i-
r
i
i . II (1 + (II _ 0 e- rill t/L )
Thus,

di
3.327 Clearly, L — = R (I - 0 = g - RI
dt
So, di
2L — = g - Ri
dt
This equation has the solution (as in 3.312)

i= 1- (1 - e - tR/a)
R

3.328 The equations are,


di1 die
L1 ii. = L2 ii_ = — 1? (4 + i2)

d
Then, —(L 1 il - LL2212)== 0
1)
or, L1 it -L2 i2 = constant
But initially at t = 0, it = i2 = 0 27f12

so constant must be zero and at all times,


Li ii_ = L e h

In the final steady atate, current must obviously be = 4 Thus in steady state,
ill-if R.
gL1
Q.
ii-22 R (1, 1+ L2) and i2 - R (Li +L2)
Poi
3.329 Here, B= 5tr at a distance r from the wire. The flux through the frame is obtained as,
2
a+l
mo I pob
1.12 = f - bdr - — I in (1 +1
2 ar 2n 1
1 I
12 a
Thus, 1L —µ0bb l n__ (1 + --)
,12 = / — 2 7t /
395

Rol I f h dr xr
3.330 Here also, B = and
2 ar c1"2 " 2 it r v
a

II IAD hN b
Thus, L 12=
2n In —
a
3.331 The direct calculation of the flux 402 is a rather complicated problem, since the configuration
of the field itself is complicated. However, the application of the reciprocity theorem
simplifies the solution of the problem. Indeed, let the same current i flow through loop

2. Then the magnetic flux created by this current through loop 1 can be easily found.

[to i
Magnetic induction at the centre of the loop, : B —
2b

2 tioi
So, flux throug loop 1, 43 12 3t a 2b

and from reciprocity theorem,


2•
iko a I
14312 = (I)21 (1321 2b
4312 1 2
So, L12 — po n a /b

3.332 Let pm be the magnetic moment of the magnet M. Then the magnetic field due to this
magnet is,
3 (rm " r Pm

zin r5 r3 *
The flux associated with this, when the magnet is along the axis at a distance x from the
centre, is

4). f 5 3
( 1 —

1
where,c1,2 =
3
a

3 µo Pm x2 f 2 it pd p
and (1,1 =
4n (x2 + p2)5/2
0

2
PO Pm X ( 1 1
2 x3 (x2 + a2)3/2

2
PoPm a
So, 1, — 2
2 (x + a2)3/2
396

d N d (I)
When the flux changes, an e.m.f. - N — is induced and a current -— — lfows. The
dt R dt
total charge q, flowing, as the magnet is removed to infinity from x = 0 is,
IA° P m
q= — cl) (x = 0) -
R NR 2a
2aqR
or,
Pm_ N [to

3.333 If a current I flows in one of the coils, the magnetic field at the centre of the other coil
is,
po a2! 1.1,0 a21
B - 2 02 a2)3/2 = 213 ag 1» a.

The flux associated with the second coil is then approximately µ0n a4 1/2 13
4
1-10 71 a
Hence, L12 -
2 13
d11
3.334 When the current in one of the loop is I1= at, an e.m.f. L12 — - L12 a, is induced in
dt
the other loop. Then if the current in the other loop is /2 we must have,
d12
L
2 dt + RI2= L12
This familiar equation has the solution,
a -rR
L12 (
- 1- e 2 which is the required current
/2

3.335 Initially, after a steady current is set up, the current is flowing as shown.
1.
In steady condition i20 = R1. •, t= •
10
R0
When the switch is disconnected, the current
through R0 changes from i10 to the right, to i20
to the left. (The current in the inductance cannot
change suddenly.). We then have the equation,

• • t(R +RoYL
This equation has the solution t = /2 e—
The heat dissipated in the coil is,
cx)

Rfe-2t t
Q= f 122 R dt = i2 20 (R +120)/La
397

3.336 To find the magnetic field energy we recall that the flux varies linearly with current. Thus,
when the flux is cl) for current i, we can write 0 = A i. The total energy inclosed in the
field, when the current is I, is

d(
W= f i d t - f N — id t
dt
I
2 1
ni f Ndcloi... f NAidt- -NAI - iNOI
2
0

The characteristic factor appears in this way.


2

3.337 We apply circulation theorem,

I1-2 nb- NI, or, H= N 1/2 70.

Thus the total energy,

W — 1BH -27th • n a2 . It2a2 b BH.

Given N, I, b we know H, and can find out B from the B — H curve. Then W can be
calculated.

3338 From fi i l- d r= N I,

B
11-70+ — • b . N I , ( d » b)
P.O

NI
Also, B= µµ 0 H. Thus, H -
nd + lib'

Since B is continuous across the gap, B is given by,

N I
B = It tio both in the magnetic and the gap.
nd + ith '

B2
11 xSxb
Wgap _ 2 0 .EL)
(a) _
2
Wmagnetic B 70 .
x S x otd
2 Kt°

2
NI _ SN
(b) The flux is Nf ii'd S N giA,0 nd 0 S' Ro rd ,
+ b+—
R

140 SN2
So, L ...
ad '
b+—
R
Energy wise; total energy

tf, -2 wri ,2 =
1 µ0N25 1 2
= - :2-1.- +b S- i - L I
22 µ0 ti 2
b +Lrd
2

li
398

The L, found in the one way, agrees with that, found in the other way. Note that, in
calculating the flux, we do not consider the field in the gap, since it is not linked to the
winding. But the total energy includes that of the gap.
3.339 When the cylinder with a linear charge density
X rotates with a circular frequency co, a surface
current density (charge / length x time) of

1.co
t = — iis set up.
2it
The direction of the surface current is normal to
the plane of paper at Q and the contribution of
this current to the magnetic field at P is
S._ lio iceX 7) ds --4.
where e is the
Q
4 it r3
direction of the current. In magnitude,
--).. —3.
'x' FT= r, since e is normal to r and the
direction of dB is as shown.
It's component, d B0 cancels out by cylindrical
symmetry. The component that survives is,
--1. 1-to f id S Rai
IB_LI= —471 ---1- cos 0 = —43t f do = po i ,
r
d S cos 0
where we have used = d Q and f d Q = 4 it, the total solid angle around any
r
point.
The magnetic field vanishes outside the cylinder by similar argument. The
total energy per unit length of the cylinder is,
1 2
IO a2 x2 0)2
W1 = 1 1102 21=1-2- x it a2 =
2 pc, 2 it Ba
t

3.340 WE = - 1 - eoE 2, for the electric field,

1 2
wB - 2 go B for the magnetic field.

1 2 2
Thus, 1
B = e E
2µo 2 ° '
B 8
when E= 3 x 10 V/m
‘cFto
3.341 The electric field at P is,
ql
EP
P 4 TE 80 (a2 + /2 )3l2
399

To get the magnetic field, note that the rotating ring constitutes a current i = q w/2 it, and the
corresponding magnetic field at P is,
µoa2i
=
2 (a2 12 )3/2 •
2 2
WE EO PIO E ql x 2 )
Thus, e0 RO 2.
Wm B2 4 7C eo a
2
1 1
E
o tio a2co)
Wm 2 4 .2
or, E0 ILO (k) a /I
WE

3342 The total energy of the magnetic field is,

(T3- dV = f B
(-1113;11—j1"

f B B dV —12.1J • B dV.
2tio
The second term can be interpreted as the energy of magnetization, and has the density
1
— — • B.
2J
3343 (a) In series, the current I flows through both coils, and the total e.m.f. induced. when
the current changes is,
dI dI
— 2L — = — L'
dt dt
or, L'= 2L
/
(b) In parallel, the current flowing through either coil is, — and the e.m.f. induced is
2
—L 1 dl
2 dt

Equating this. to — L' , we find L' = 2 L .


J
2
3344 We use L1= µo n12 V, L2 = Ito n2 V

So, =
L12 µo n1 n2V =VEIT;

3345 The interaction energy is


1 r 2 r --a, 2 1 r 2
2µo f dV
2 1B 11 dV
J ri+B21 ay- 1 --a. --a.
ill21
= -f Bi • B2 dV

Here, if Bi is the magnetic field produced by the first of the current carrying loops, and B2,
that of the second one, then the magnetic field due to both the loops will be B1 + B2.
4 0 0

3.346 We can think of the sm aller coil as constituting a m agnet of dipole moment,
pm- 7t a2/1
I t s d i r e c t i o n i s n o r ma l t o t h e l o o p a n d ma k e s a n a n g l e 0 w ith the direction of the m agnetic

field, due to the bigger loop. This m agnetic field is,

Po 12
B2 =
2b

The i nt e r a c t i o n e n e r g y h a s the ma g ni t u d e ,

I W I = PI)2 Ibi 1" 2 it a2 COS 0

Its s i g n d e p e n d s o n t h e s e n s e o f t he c u r r e nt s .

3.347 (a) There is a radial outw ard conduction current. Let Q be the instantaneous charge on

the inner sphere, then,

2 dQ 1 A
j x 4 n r = — or, J— -
dt 4 n r2 dtr*
_. a5 ( Q ?) .
id - at — ddt
... i".*
On the other hand
4 it r2

A
q
(b) A
t the given moment, E=
4 n eo E r2 r

and by
Ohm 's la w,j
-:-. r
= — =
q
r
A

P 47c oepr2

7* q A
Then,
h i=
— , r
4 n E0 E pr "

q f dS cos 0 ....s___
and

The surface integral


f •I d. dS =
' '
mu s t b e
4

-ye
It 20 p r2 EOE P
71'.
b e c a u s e j d , b e i n g o p p o s i t e o f j , i s i n wa r d .

—111, 71.
3.348 He r e also w e see that neglecting edge e ffects, jd = —j. Thus Ma x we l l ' s equations reduce

to,div 73. 0, Curl H= 0, if ,,,,g.=

A
= Bo • Bo can be thought of as an
general solution of this equation is B = constant
—.$
e x t r a n e o u s m a g ne t i c f i e l d . If it is zero, B = 0.
3.349 Given I = Im sin wt. We see that

j= —
ins
sin wt = — jd.
ap
S at
I. /m
or, D= cos wt, so, Em - is the a mp l i t u d e of the electric field and is
co S
m Eo w S

7 V /c m
401

3.350 The electric field between the plates can be written as,

V. V.
E = Re --- et' ', instead of — cos wt.
d

This gives rise to a conduction current,

a i
jc= a E= Re — Vme "
d
and a displacement current,

OD V. .
.1e1 — 1" eo e --el a"
at
The total current is,

jr

Q
where, tan a - on taking the real part of the resultant.
80 8 (t)

The corresponding magnetic field is obtained by using circulation theorem,

r = nr2jr
H.2 n

V ,
or, H = H„, cos (wt + a), where, Hm = 712—da VG- + (E0E w)2

3351 Inside the solenoid, there is a magnetic field,

B= tio n ins sin wt.

Since this varies in time there is an associated electric field. This is obtained by using,

E.(177.=
dt
fB
s
dr
Br
For r< R, 2 rE - B. it r2, or, E_ -
2

BR
For r> R, E = - 2
2r

The associated displacement current density is,

aE B r/2
- eo
id= 60-67 a` •• 2
— Eo B R /2 r
The answer, given in the book, is dimensionally incorrect without the factor E O•

3.352 In the non-relativistic limit.

E*= r
3
43tEor
(a) On a straight line coinciding with the charge path,

aE q [-V 3rr i ( dr_


e° at — 4n r4
using,
402

r —I. 2 qr
But in this case, r.. —v and v — v, so, Id =
r= 4 it r3
(b) In this case,r = 0, as, 71 t7..Thus, _.

id ' --1;
4 Tt r
qx
3353 We have, Ep us
4 it co (a2 + X2)3/2

aD aE qv
then 1d = — = Eo al 4 n (a2 + x2)-5/2 (a2 — 2x2)
at at
This is maximum, when x = xn, = 0, and minimum at some other value. The maximum
displacement current density is
CI d) max 1` tiV 3
4na

To check this we calculate


a dI

ax
a id .... e._ rt._ 4x (a2 + x2) _ 5x ,a2 _ 2
ax 4 it 1 ‘ k 2x-- ) I

VT
This vanishes for x = 0 and for x = — a. The latter is easily shown to be a smaller
2
local minimum (negative maximum).

3354 We use Maxwell's equations in the form,


111 --Jr __), f
B•dr= co 1.10 E•dS,
at
when the conduction current vanishes at the site. We
know that,
--1 A:
f F. d .i. _i_f
4 3t co
dS r
r2 V

= I—f c/S2 = g 2 at (1 — cos 0),


4 n co 4 n co
where, 231(1 — cos 0) is the solid angle, formed by the disc like surface, at the charge.

Thus, f ri•c/FL 23taB=2Roq•sin0.0

On the other hand,x = a cot 0

differentiating and using —dx = — v,


dt

v 3z a cosec2 0 0
lio q yr sin 0
Thus, Ba.
43t?
403

µo4 (v
This can be written as, B
4 n r3
-4.
xr
and TH= (The sense has to be checked independently.)
n 3
3.355 (a) If B = i.(t), then,

Curl /- 0
at '
So, gcannot vanish.

(b) Here also, curl E 0, so E cannot be uniform.


(c) Suppose for instance, r= 5-7 (t)
aB
where W' is spatially and temporally fixed vector. Then - = curl E 0. Generally
dt
speaking this contradicts the other equation curl H= ap 0 for in this case the left
at
hand side is time independent but RHS. depends on time. The only exception is when f (t) is
linear function. Then a uniform field E can be time dependent.
ap 74'
3.356 From the equation Curl H --= j
at
We get on taking divergence of both sides
a

- div D= div j
at
But div D = p and hence div j +
at = 0
3.357 From V% r._
at
we get on taking divergence
a --•
0= - divB
at
This is compatible with div B = 0
3.358 A rotating magnetic field can be represented by,
Bx = Bo cos co t ; By = Bo Sill cot and Bz= Bz.
aB
Then curl, -—
E at
So, - (Curl 4 . _0) Bo sin cot = - coBy
- (Curl MM w BO cos wt MI coBx and - (Curl A . 0
Hence, Curl E = - ctrx B ,
where, (0 = e3 CO
404 -▪ - ▪

3359 Consider a particle with charge e, moving with velocity 17,.in frame K. It experiences a
force F = ev x
In the frame K , moving with velocity 17: relative to K, the particle is at rest. This means that
there must be an electric field E in lc, so that the particle experinces a force,
F'= e E' F- ev00xB

Thus, E'- 17x B

3360 Within the plate, there will appear a (ii*x B) force, which will cause charges inside the
plate to drift, until a countervailing electric field is set up. This electric field is related to B, by
E- e B, since v &B are mutually perpendicular, and E is perpendicular to both. The
charge density ± a, on the force of the plate, producing this electric field, is given by

3.361 Choose Cir.!. f B along the z-axis, and choose i;.as the cylindrical polar radius vector of
a reference point (perpendicular distance from the axis). This point has the velocity,
v= wxr,

and experiences a (T)xi B) force, which must be counterbalanced by an electric field,


- (01x 11 x - B

There must appear a space charge density,


p= eo div E - 2 E0 co • B = -8pC/m3

Since the cylinder, as a whole is electrically neutral, the surface of the cylinder must
acquire a positive charge of surface density,

2 E 07.)* • B Tt a 2
0= + eo a (.3) • B + -2 pC/m2
2na
3.362 In the reference frame K , moving with the particle,
-3.
E+
3

7/3/ a if- 17;x ://c2 = 0.

Here, vo = velocity of K , relative to the 1 frame, in which the particle has velocity v.

Clearly, i,-0).= t7.*From the second equation,

vxE q vxr µo4 (v


B
c2 E°11°x 4 n Eo r3 47c r3
405

3.363 Suppose, there is only electric field E, in K. Then in K', considering nonrelativistic velocity
i7x E
17,* E' = E , B = - 2
c
So, E' • B' = 0
In the relativistic case,

41 .w Ai'
011 11 El
- v2/c2
c2

- B'

Now, El • B' = E' + E' B = 0, since


II. B'II 1 '1

E.,. •B,I. _4,_ • (TX Te)/(1 - v2/c2) = - 4 • (7x El ) / ii _ -v,2 .0


y
A
i -xj
A
)
3.364 In •K, B = b 2 b = constant
x2 + y
A A

In K', = Tic73= by Y - , by 2
X2 + r :
A

The electric field is radial (r = x i + y j ).


A
r
3365 In K,E = a 2, r= (x 1+ y
)
rox
K, "Tir1 7.x E a
In 2 2
C r2
The magnetic lines are circular.
3366 In the non relativistic limit, we neglect v2/c2 and write,
E,11E__11 _a, II= _j
r
Elm El+vxll B'1 at - ric2

These two equations can be combined to give,


E'= E+v-10xB, B'= B-vxE/c`

3.367 Choose E M the direction of the z-axis, E = (0, 0, E). The frame K' is moving with velocity
t7.= (v sin a, 0, v cos a), in the x - z plane. Then in the frame K',

Te r r
eo l i r 1 = 0

-4° v E/c`
E' B'
- v2/c2
The vector along T'is e-t (sin a, 0, cos a) and the perpendicular vector in the x - z plane
is,

f = (- cos a, 0, sin a),


406 -
(a) Thus using E= E cos a Wa+ E sin a :f7
Esina
E cos a and E'„-
111-v2/c2

V1- (32 cos2 a tan a


So E'= E and tan a' =
1 - 02 V T : 727

TS( Z/c2
(b) B'H= 0, -
2
-v 2/C
f3 E sin a
B' -
cir=i7
-3.
3.368 Choose Bin the z direction, and the velocity v
(v sin a, 0, v cos a) in the x - z plane,

then in the K' frame,

1
= II =
0
_1. 3T'11-
1 B B
i_v2/c2 - 1/2/c2
c13B sin a

We find similarly, E' = fI 7:131

132 a
cost
B' = B
V 1-132
tan a' -
tan a

V17-(37
3.369 (a) We see that, E' •13; =
vx
B, - ,E)
vx B)-( -

v2
1- 2
2

7)•(vxE)/c
• 13.1 - (17),C
EH • Bll+

2
1- V2iC
-4.
-4. -4. ri • .81 - (179x /3.1 )2 (17.x )/c2
=E . +
l BII
l V
—2
But, AxB•CxD= A •CB •D -A •DB • C,

SO, = 4 • 4 + 4..K 2 -rs 1 c2


407

1 17x E 2
= E2 — c2 B2 + v2 (Ei vB )2 — c2
ii c2
1

1— 2

E2 B2,
= E —c + c2
ll 2 B11

2
since, (v ) v2 Al

3.370 In this case, E•B= 0, as the fields are mutually perpendicular. Also,
2

E2 — c2 B2 = — 20 x log Y- is — ye.
(
m

Thus, we can find a frame, in which E' = 0, and

/ E2 4 X 104 )2
B' = — v c B- — E- = B 1— — 0.20
2 2
— _4 = 0.15 mT
c 2 c2 B2 3 x 108x 2 x 10

3.371 Suppose the charge q moves in the positive direction of the x—axis of the frame K. Let
us go over to the moving frame K', at whose origin the charge is at rest. We take the
x and x' axes of the two frames to be coincident, and the y & y' axes, to be parallel.

1 q r—w
In the K' frame, E —
" Eo r13
and this has the following components,

1 qx' E , 1 qy'
3
Ex= 4 it E0 7/3 Y 4 it eo r, *

Now let us go back to the frame K At the moment, when the origins of the two frames
coincide, we take t= 0. Then,

v2
x= rcos 0 = x'\11 ,y= rsin 0=

E
y v2

Also,
= /c 2

Ex= Ex', Ey
p2 0)
(1 — sing
From these equations, r'2 = r2
1-132
1 p23/2 () x, .3/ j A )
— _ /32 sin2 0)3/2 (
4 7C E0 r3 67 .. 32

q 1—N2)
R2 Sint 0)3/2
4 3t eo r3 (1
408

3.7 MOTION OF CHARGED PARTICLES IN ELECTRIC AND


MAGNETIC FIELDS

3372 Let the electron leave the negative plate of the capacitor at time t = 0

A. E. 1. at
As, EX= ..... 1 1'
x dx ,
and, therefore, the acceleration of the electron,

eE eat dv eat
w=—=— Or — am
m ml ' dt ml
V t

1 ea 2
or, f dv - ell f tdt, or, v — — - - t (1)
2 m/
ml
0 0

But, from s = f v dt,

r 1
1 ea f 2 eat3 (6 m12)3
1= t dt = or, t
2 m/ 6 ml ea
0

Putting the value of t in (1),


2 1
1 ea 6m12 3 (9ale)5- .
v ... ' 16 km/s.
2 m/ ea 2 m

3.373 The electric field inside the capacitor varies with time as,

E = at.
Hence, electric force on the proton,

F .. eat
and subsequently, acceleration of the proton,

eat
w= —
m
Now, if t is the time elapsed during the motion of the proton between the plates, then

/
t = —, as no acceleration is effective in this direction. (Here v is velocity along the length
vit 11

of the plate.)

dv1
From kinematics, —w
dt
V1 r

SO, f dv 1 = f w dt,
o o
(as initially, the component of velocity in the direction, 1 to plates, was zero.)
409

ea t2 ea 12
or
'v 1-= f m 2m = 2m v2
0 II

VI eal 2
Now, tan a =
V 3
2mv
II 11
1
- eal2 2e
3 '
as v = , from energy conservation.
11
2e T 2
2m /7)
m m

al 2 {.77/-
4 2 eV3
3.374 The equation of motion is,
dv dv g_ ,,,
-- = v — =dx m kco - ax)
clt
Integrating
1 I. 1
- (E x - axe) = constant.
2 1/2 — m °- -2
But initially v = 0 when x = 0, so "constant" = 0

Thus, v2= 2m x - 1 ax2)m o 2

2E0
Thus,v = 0, again for x = x. = a
The corresponding acceleration is,
(dv) E0
dt = -q- (E0 - 2 E0) =
Xm m m

3.375 From the law of relativistic conservation of energy


2
"10
- e Ex = mo c2.
1/1 — (1,2 / C2)

as the electron is at rest (v = 0 for x = 0) initially.


Thus clearly T= e Ex.
2
MO C
On the other hand,V1 - (v2 / c2) -
m0 c2 + e Ex

v V(m 0 c2 + e Ex )2 - m20 cal


or, _
c mo c2 + eEx
2
or, Ono C + eEx) clic
ct= f cdt=
NI (mo c2 + eEx)2 — m20 c4
410

1 f dy 1 , 1,
— so — V
2 eE Vy _ mo2 c4 eE Vno c2 + eEx)2 - mo2 c4 + constant

The "constant" = 0, at t = 0, for x = 0,


1./ ., m 2o c4.
.
So, ct - — v (moc2 + eEx) 2
E
Finally, using T = eE x,

3.376 As before, T = e E x
Now in linear motion,
d mov mow mow
dt - 2 + /c2)312 c2 W
V 1 - v2 / c2 Vi - v2/C (1- V2
mo (T + m 0c2)3
(1 _ v2 / c2)3/2 w 2 6 w = e E,
MO C
_3
e E m2 c6 eE T
So, w= „ = 1+ 2
(T + moc)- m0 mo c

3.377 The equations are,


d( movx d ( movy j
0 and =eE
dt I - (v2 c2) dt

vx vo
Hence, - constant
- v2/c2 1/1 - (4/ c2)

Also, by energy conservation,


2 2
MO MO c
+eEy
I1 - (v2 / c2) I1 - (4/ c2)
2
vo E0 MO
Dividing Vx - E
E0 + eEy '
° 1 - (I? c2)
MO e0+ e E y
Also, 2
7 C
11 - (v2 / c2)
Thus, (e0+ e Ey) vy al
C2 E t + constant
"constant" = 0 as v = 0 at t = 0.
Integrating again,
1 1 2 2
y+ E y2 = - c E t + constant.
e° -2 e 2
411

2
or, ceEt = (eo + eEy)2 - Eo

or, E0 eEy = 11E02 + c2 e2E2t2

vo E0 C2 e Et
Hence, = also, v =
_2 _ e2 2 E2 r 2 Y V 2 222
tO co2 +c e E t

and tan 0 = - e E t 111 -


/ c2) .
vx mo vo

3.378 From the figure,


d dqB
sin a = -
R m '

v
As radius of the arc R - m , where v is the
qB

velocity of the particle, when it enteres into


the field. From initial condition of the problem,

1 2qV
qV = — mv2 or, v = v
2

dqB dB
Hence, sin a =
2mV
FF11-:
m
m

and a = sin- 1 dB VIZ = 30°, on putting the values.


2 mV

3.379 (a) For motion along a circle, the magnetic force acted on the particle, will provide the
centripetal force, necessary for its circular motion.
2
my
i.e. = evB or, v-
eBR
R

2n = 2 ;IR 2 sc m
and the period of revolution,T =
w v eB

(b) Generally, dt =
-0 2-•
d mo mo
mo v • 174)
But, - 3/2 2
dt dt I
11 1 - (V2/C2) - (112 / C2) (1 (V2 / C2)) C

For transverse motion, 1,: i-,= 0 so,


di* NT. mo v2
, here.
dt V i - (v2 1 c2) - (v2 / c2)
412

V2
MO V
Thus, - Bev or,
r 1I I. - (v2 / c2)

V_ Ber
or,
.,2 2 2 2

C Va,r- e r- + mo c
2 It //to
2 nr 2-7t V 2 2 2
Finally,, T= -
v - cBe B e r2 +mo c
eB111-v2/c2
3.380 (a) As before,p = B qr.

Ve2p2 + m20 c4 = Vc2 B2 q2 r2 + m c4


(b) T = 0

2 2
V C
(c) w - —
r r [1 + (mo c I B qr)2 1

using the result for v from the previous problem.

3.381 From (3.279),

2n
T= E (relativistic) ,,
c` eB

Here, moc2 /111-v2/c2 = E

23IT
Thus,
oT= , (T= K.E.)
c2 eB
OT T
Now, 1= _____T , so, T = i mo c2
To - mo c

3.382 T = eV = 1 2
—mv
2
(The given potential difference is not large enough to cause significant deviations from the
nonrelativistic formula).
VIeV
Thus, V=
m
VV 2eV
So, —
vII = cos a, vi = m sin a
m
2
m v1 MV.L.
Now,
r = Bev I or, r - Be '
,r 2nr 2nm
and i = — -
v1 Be
„, 2 7CM \Fi r ( ,, 2M V
Pitch p ... v i ■ — — cos a .. L n — cos a
II Be m \
eB2
413

3383 The charged particles will traverse a helical trajectory and will be focussed on the axis
after traversing a number of turms. Thus
1 2 7cm 2 am
n. — (n+ 1)
vo 4B1 4B2
n n +1 1
So, - B1
B B2 B2

Hence,

or,

or, m 2 (B2 _ B1)2

3.384 Let us take the point A as the origin 0 and the axis of the solenoid as z-axis. At an
arbitrary moment of time let us resolve the velocity of electron into its two rectangular
components,v along the axis and )73 to the axis of solenoid. We know the magnetic

force does no work, so the kinetic energy as well as the speed of the electron I vl I will
remain constant in the x-y plane. Thus v—ig. can change only its direction as shown in the
—3.
Fig.. vii will remain constant as it is parallel to B.
Thus at t = t
vx = vl cos wt = v sin a cos wt,
V,
= v sin cot = v sin a sin co t

and = v cos a , here w = eB


vz w
m
As at t = 0, we have x = y = z = 0, so the motion law of the electron is.
z= v cos a t
v sin a
x— sin wt
v sin a
y= (cos co t — 1)
co
(The equation of the helix)
1
On the screen, z= 1, so t —
v cos a

2 v2 sin2 a (
2 )/ )
Then, r 2 =X 2 + y -
2 1 — cos a
V cos a
(0
2 v sin a . co/ leB
r= sm =2 sm a sin
a) 2 y cos a eB 2 my cos a
414

3.385 Choose the wire along the z—axis, and the initial direction of the electron, along the x—axis.
Then the magnetic field in the x — z plane is along the y—axis and outside the wire it is,
I
B = By= —2 7rx , (Bx = Bz= 0, if y = 0)

The motion must be confined to the x — z plane. Then the equations of motion are,

mvx = eV B

d(mvz)
- + evx By dt

Multiplying the first equation by vx and the second by vz and then adding,

dvx dvz
v +v = 0
x dt x dt

or,

dvx e
Then, vx dx = —
vo- vx 2 7TX

vxdvx Role dx
or,
11--2 7 2 7T1ri X
Vo — vx
Role x
V 72 2
Integrating, vo — vx - 2 ion In —a

on using,vx = vo , if x = a (i.e. initially).

Now, vx = 0, when x = x„„

Role
v
SO, Xm = a e 0 /b , where b -2

3.386 Inside the capacitor, the electric field follows a 1law, and so the potential can be written as
r
V ln r1a -V 1
9) E -
lnbla ' ln blar

Here r is the distance from the axis of the capacitor.


my2 qV 1 2 gy_r_
Also, — or my =
r l n b l a r r ln b1a

On the other hand,


my - q B r in the magnetic field.

V V
Thus, v - and —
Brinbla m Br B2r21n(b/a)
415

3.387 The equations of motion are,


dv dv, d
vz
m — . - q Bvz , m —c . qE and m — = q vx B
dt -t dt
These equations can be solved easily.
qE qE
First, v = — t, y = — te2
y
2m
Then, vx2 + vz2 = constant = vo as before.

In fact, vx = vo cos cot and vz = vo sin wt as one can check.


Integrating again and using x = z = 0, at t = 0
vo . vo ‘.
x = — sin wt, z = — ki - cos cot)
03 CO
2x
Thus,
x = z = 0 for t = tn= n —(0
_ qE x 2n 27t _ 2 n2mEn2
2
xn x
2

vx
Also, tan an = — , (vz = 0 at this moment)
vy
mvo mvo B 1 B vo
- - x x -
qEtn qE m 2nn 2nEn•
3.388 The equation of the trajectory is,
vo vo qE
x = —cosin cot, z = T() (1 - cos cot) , y = — t 2 as before see (3.384).
2in
Now on the screen x = 1, so
col 1_,. - 1 col
Sill (Or mil — or, (Or go Sin —
vo vo
At that moment,
2
qE ( _ 1 col )
Y= --2- SM
2 m co vo

so,
ov VT„:17 11;;T,
vc, sin - sin Em
qE
V ,, wt (Ot
and Z - —co o 2 sin ' — = / tan —
2 2
1
= / tan [sin-1 1 = / tan .V7g) .32
2 vo 2 mE
For small

or,
416

3.389 In crossed field,


E
eE = evB, so v =
B
I E mIE
Then,F = force exerted on the plate = xm =
e B eB
3390 When the electric field is switched off, the path followed by the particle will be helical.
and pitch, Al = v11 T, (where v is the velocity of the particle, parallel to B, and T, the
ll
time period of revolution.)
= v c,os (90 - cp) T= v sin cp T

= v sin cp 2 nni as T= 2::,) (1)


qB DPP
Now, when both the fields were present, qE = qvB sin (90 - q)), as no net force was effective on the
system.
E
or, v- (2)
B cos cp
E 2 3tM
From (1) and (2), Al tan a, 6 cm.
Lu = B ---qB- (P

3.391 When there is no deviation,


- qE = q (i7X B)
E
or, in scalar from, E = vB (as 171B ) or, v = (1)
B
Now, when the magnetic field is switched on, let the deviation in the field be x. Then,
1 (I_ 21 2
X - t ,
2 m
where t is the time required to pass through this region.
a
also, t- -
v
2

Thus 1 (qvB )(1 . 1 _q_ a2 B2


X= 2 m v 2m E
For the region where the field is absent, velocity in upward direction
= r-)t= 5- a B
m m
Ax _ x . Tip_ t,
Now,
m
„ aB2 b b bB
= -2- when t' .: (4)
m E v E
From (2) and (4),
Ax_ a2 T,2
1 1 Li _q_a B2 b
2m E = In E

or,
g_. 2E Ax
m a B2 (a + 2b)
--▪ -▪ 417

3.392 (a) The equation of motion is,

d2
Pri — q(E+v x131
dt2
71
. k
Now, v x B= x y z By-jBx
0 0 B
So, the equation becomes,
dv, q B v„ gE qB dv
= ' = vx , and =0
dt m dt m m dt
Here, vx = x , vy = 57, i. The last equation is easy to integrate;
v = constant = 0,
since vx is zero initially. Thus integrating again,
z= constant = 0,
and motion is confined to the x - y plane. We now multiply the second equation by i and add
to the first equation.
sot V +
x y
we get the equation,
E . qB
= to) - twg, =
dt m.

This equation after being multiplied by eiwcan be rewritten as,


d
— (ge"")= iwel cot — dt
and integrated at once to give,

where C and a are two real constants. Taking real and imaginary parts.

v = + C cos (0) t a) and v = - C sin (cot +


x
Since vy = 0, when t = 0, we can take a = 0, then vx = 0 at t = 0 gives, C =

and we get,
E , E
= — (1 - cos wt) and v = — sin cot.
vx B Y B
Integrating again and using x = y .. 0, at t = 0, we get
E( „ E „
x (t) = — t - s' , y (t) = — k.i. - cos cot).
B B
This is the equation of a cycloid.
(b) The velocity is zero, when cot = 2 n n. We see that
2
v2 + v2 . E(
v2= (2 - 2 cos wt)
x Y
418

ds 2E wt
or, v= = sin
dt B 2

The quantity inside the modulus is positive for 0 < cot < 2 n. Thus we can drop the modulus
and write for the distance traversed between two successive zeroes of velocity.

4E , cot)
S = — - cos —
coB 2

Putting
cot = 2 n, we get
8E 8 mE
S =

coB /32
4
(c) The drift velocity is in the x-direction and has the magnitude,

< vx> = C E— (1 - cos cot)> = B.

Ro I
3.393 When a current I flows along the axis, a magnetic field B., - ii s set up where
2n p
2 2
p mi X + y2. In terms of components,
lx
Bx= Rb' n
, D„= —4- and Bz = 0
2 n p" 2 n p`

Suppose a p.d. V is set up between the inner cathode and the outer anode. This means a
potential function of the form

In p/b
cp = V b,
In a/b ' a>p>
as one can check by solving Laplace equation.

The electric field corresponding to this is,


tx
Vx '11
Ex = , E, = - 2 't/I , Ez = 0.
p2 ln aIb p ln a 1 b
The equations of motion are,
6
d I e IVz I e I go
— mv +
2 xz
dt p Inalb + 2np

e Ivy I eliA0I
yz
dt nivY p`' ln a 1 b+ 2 7t p2

[to 1 RO I
and Imvz = -lel 2 (X + y;) -
z It p I e I 2n Ida P

(- I e l) is the charge on the electron.


Integrating the last equation,

Rol
mvz- -(el — in p / a mi.
2n
419

since vz . 0 where p - a. We now substitute this i in the other two equations to get

15_1 ( 1 mv2 + 1 nwl


dt 2 x 2 Y

. [ leIV 1 el2 osoi)2


inp/b1 1-i+2361
lnalb m 23r
P

leIV_Iel2r/2 0 1 d 2
ln - - - 1 1 P 2p t
m 2x b
In 16-

(1
b

leIV_Iel2 ink clink


a m b dt b
In —
b

Integrating and using v2 = 0, at p = b,


22 it

we get,

2
1 2 eIV
— Mv = = I a lnf)--sln lel 2 Hii°11)
2
In —
b
The RHS must be positive, for all a > p > b. The condition for this- is,
2
1 I I (tto 1 a
V 2—— — in
2m 2n b
3.394 This differs from the previous problem in (a 4-,b) and the magnetic field is along the
z—direction. Thus Bx = By = 0, Bz = B
Assuming as usual the charge of the electron to be — I e I , we write the equation of motion
d lelVx d leIV
dt
mv .
x
p
2 b
In —
f
-IeIBy,
dt
mv = --L +I elB;
Y b
p2 ln ;
a

and m
vz
.0
=
>
z
=
0
The motion is confined to the plane z = 0. Eliminating B from the first two equations,
d (1 2). lejvxi+h
di riv ln b I a p2

—1 y2m leivi jtpzi


Or,
2 = I ln b/a
so, as expected, since magnetic forces do not work,

v= , at p = b.
-
-
420

On the other hand, eliminating V, we also get,

d
— m(xv —yv )= leIB(x.i+yj?)
dt Y x

ielB 2
i.e. (xvy — Yvx) p + constant
2m

The constant is easily evaluated, since v is zero at p a. Thus,

e IB
(xv 2m ( p2 — a2)> 0
Y — Yv1)

At p b, (xvy— yvx) s vb

Thus, vb a (b2 — a2)

2 mb
or, B s
b2 — a

2b {2771 B
or, B s
b2—a2 lel

3.395 The equations are as in 3.392.

dvx g dv qE qp_ dvz


cos wt — m v x and dt
v
dt m dt

211
co
m , vx + ivy, we get,
with

— co cos oat — i au
dr.i B

or multiplying by eiwt,

d • Ens 2i
— (getw ) i (e + 1)
dt 2B

icor E 2 iag ne
or integrating,
e e + — iwt
471 2B

or, (eiw +2icotei'`)+C eitg


4B

since

Thus,

a a
x 2 k ( s inn cot — cot cos wt), y t sin cot.
2 co
421

qE .
where a = , and we have used x. y 0, at t 0.

The trajectory is an unwinding spiral.

3396 We know that for a charged particle (proton) in a magnetic field,


mv2
Bev or my Ber
r
eB
But, —,
m
E. 1 2 1 2 r2
Thus — mv = — m
2 2
So, AE= mco2 r Ar = 4 n2 v2 mr At-
On the other hand AE = 2 eV, where V is the effective acceleration voltage, across the Dees,
there being two crossings per revolution. So,
V a 2 n2 v2 mr bole
2
my — Bev, or, my = Ber
3.397 (a) From —
r
(Ber)2 1
and T — my 2— 12 MeV
2m 2
2n r
(b) From --
w v

1 ..V-r
we get, 15 MHz
f z'min 2 —
2m
nr mr r
3.398 (a) The total time of acceleration is,
1
n, t=
2v
where n is the number of passages of the Dees.
2
B e2 r 2
But, T = neV —
2m

B2 e
or, n
2mV

B2 er2 n B r2 n2 my r2
So, t x - 30 µs
= eB 1 m 2mV 2V eV

(b) The distance covered is, s =


2
, 21;
12 7
But, Va = - ' v ,,
422

2
B e2 r2 2 it2 M V2 r2
But, n=
2 eV m = eV

Thus, s
4 n3 V2 mr2
= 1.24 km
3eV
2 n r„ n
3.399 In the nth orbit, vn — n To = — • We ignore the rest mass of the electron and write

vn= c. Also IV= cp = cBer„.

2 7tW.. n
Thus,
Bec2

2 n Wv
or, n 1 .1 9
Bec2
3.400 The basic condition is the relativistic equation,
2 mv
MV
Bqv, or, my — — Bqr.
r
— v2/c2

Or calling, 0)=
m '

030 co = Bq r
we get, — o
03(2). r2 m°
V 1 + 2

is the radius of the instantaneous orbit.


The time of acceleration is,
N N N

1
t = = NI Wn
2vn n q Bc2
n-1 n-1

N is the number of crossing of either Dee.


n AW
But, Tvn = mo c2 +—
2 , there being two crossings of the Dees per revolution.

So,

Also, r=r —— N
WN naN 2q Bc
423

wo
Hence fmally, w =

V
i 4. (72 B2 x N2
m20 c2 WW2
4 q2 B2 c2

wo
wo
_
ViTiTi t '
N i i ÷ AAIL)2. 4 qBc2
4 ni20 cal X a Aw t

q B AW
a=
n mo 2 c2

3.401 When the magnetic field is being set up in the solenoid, and electric field will be induced
in it, this will accelerate the charged particle. If B is the rate, at which the magnetic field
is increasing, then.
1 •
Tt r2 i si. 2 nrE or E. - rB
2

dv 1 • qBr
Thus, m - - Bq or =
dt 2r ' V 22 mm '

After the field is set up, the particle will execute a circular motion of radius p, where

1
my = B q p, or p = i r
3.402 The increment in energy per revolution is e 4), so the number of revolutions is,

N=W
e 4:b
The distance traversed is, s = 2 nrN

3.403 On the one hand,


r

ar_ . eE . e AI . e d
dt — — f 27.tr' B (r) dr'
2nr dt 2nr dt
0
On the other ,
p = B (r) er, r = constant.
dp d •
SO, = er — B (r) ■ er B (r)
dt - dt

Hence, er B (r) - e n r2 d < B >
2 ;tr. dt
So, i3 (r) = -Ili< B >

1
This equations is most easily satisfied by taking B (r0) - - < B >.
2
r
0

1 1
3.404 T he condition, B (r0) = - > = f 13• 2nr dr/nro
2 <B -2
0
424

'o
Or, B (rd.. -3 f Brdr
ro o
This gives ro.
In the present case,

3 ? 1
or, -ar- = - B or ro =
4 ° 2 ° 3a •
3.405 The induced electric field (or eddy current field) is given by,
r

E (r) .. ÷2rr /4 f 2 3tr' (r) B (r) dr'


0
Hence,
r
dE 1 d (r)
= - 2itr2 dt f 2tr' B (r) dr, +dB
dr dt
0
1d , dB (r)
=--—<L >+ i
2 dt dt
This vanishes for r = ro by the betatron condition, where r0 is the radius of the equilibrium
orbit
3.406 From the betatron condition,
1d dB B
B > = — (ro) ..
dt < dt At

2.13
Thus, 11 < B > =
dt At

A). nr2d<B>. 2icr2B


and
dt dt At
So, energy increment per revolution is,
d(1) 2 7t r2eB
e dt 12 At
3.407 (a) Even in the relativistic case, we know that : p = Ber
14, = Nic2p2 + m20 c4 c2 = mo 2 - /
Thus, m0 C ( V1 -1- (Ber I m0c)2 -1)
(b) The distance traversed is,
W W W At
2nr -e cf).= 2.7tr = Ber '
27tr2eB / At
on using the result of the previous problem.

You might also like